You are on page 1of 859

VERBAL:

1.

Choose the answer option which will correctly fill the blank.

He spilled ___________ milk all over the floor.

A. A

B. An

C. The

D. none of these

1) a

2) b

3) c

4) d

Correct Option
3
is:
Your Option is: 1
Result: Wrong
Timetaken: 0.0 secs
If we talk particularly about a thing there we have to use
Explanation:
'the'.So,the correct answer is 'the'

2.

Choose the word nearest in meaning to the word in UNDERLINED from the given options

He was punished for shirking his official work

A. Delegating

B. Slowing

C. Avoiding

D. Postponding
1) a

2) b

3) c

4) d

Correct Option is: 3


Your Option is: 3
Result: Correct
Timetaken: 0.0 secs
Explanation: shirking->avoid or neglect.So,option C is the correct answer.

3.

Read the passage and answer the questions that follow on the basis of the information

provided in the passage.

Disequilibrium at the interface of water and air is a factor on which the transfer of heat and

water vapor from the ocean to the air depends. The air within about a millimeter of the

water is almost saturated with water vapor and the temperature of the air is close to that of

the surface water. Irrespective of how small these differences mightbe, they are crucial, and

the disequilibrium is maintained by air near the surface mixing with air higher up, which is

typically appreciably cooler and lower in water vapor content. The turbulence, which takes

its energy from the wind mixes the air. As the speed of wind increases, so does the

turbulence, and consequently the rate of heat and moisture transfer. We can arrive at a

detailed understanding of this phenomenon after further study. The transfer of momentum

from wind to water, which occurs when waves are formed is an interacting -and complicated

phenomenon. When waves are made by the wind, it transfers important amounts of energy-

energy, which is consequently not available for the production of turbulence.

This passage principally intends to:

A. resolve a controversy
B. attempt a description of a phenomenon

C. sketch a theory

D. reinforce certain research findings

E. tabulate various observations

1) a

2) b

3) c

4) d

5) e

Correct Option
2
is:
Your Option is: 5
Result: Wrong
Timetaken: 0.0 secs
From the passage,it is very clear that,it principally intends to
Explanation:
attempt a description of a phenomenon.

4.

ERROR CORRECTION

My son crept into my lap and we hugged AND HAD TALKED for a while.

a) and had talked.

b) and have talked.

c) and had many talk.

d) and we were talking

e) and talked

1) a

2) b

3) c
4) d

5) e

Correct Option is: 5


Your Option is: 2
Result: Wrong
Timetaken: 0.0 secs
Explanation: The past tense is suggested by ?talked?.?Had? is not required.

5.

ERROR IDENTIFICATION

In the following question,two sentences are given.There may be an error in the

sentence(s).Mark as your answer

(a) if there is an error only in the first sentence.

(b) if there is an error only in the second sentence.

(c) if there are errors in both the sentences and

(d) if there is no error in either of the two sentences.

SENTENCES

(1)He informed me before he had posted the letter yesterday.

(2)No one is as happy as he.

1) a

2) b

3) c

4) d

Correct Option is: 1


Your Option is: 2
Result: Wrong
Timetaken: 0.0 secs
Explanation: He informed me before he posted the letter yesterday
6. Choose the option which will correctly fill the blank.

I will be here __________- Thursday and Friday.

A. During

B. for

C. until

D. after

1) a

2) b

3) c

4) d

Correct Option is: 1


Your Option is: 3
Result: Wrong
Timetaken: 0.0 secs
Explanation: during is used throughout the duration.So,option A is the correct.

7.

Choose the word nearest in meaning to the word in UNDERLINED from the given options.

Being a member of this club, he has certain rights.

A. status

B. Truth

C. Virtues

D. privileges

1) a

2) b

3) c

4) d
Correct Option is: 4
Your Option is: 4
Result: Correct
Timetaken: 0.0 secs
Option D is the correct answer.Privileges ->special
Explanation:
right,advantage

8.

ERROR IDENTIFICATION

In the following question,two sentences are given.There may be an error in the

sentence(s).Mark as your answer

(a) if there is an error only in the first sentence.

(b) if there is an error only in the second sentence.

(c) if there are errors in both the sentences and

(d) if there is no error in either of the two sentences.

SENTENCES

(1)There is a little truth in what we have heard.

(2)You are not going to the theatre,isn?t it?

1) a

2) b

3) c

4) d

Correct Option is: 2


Your Option is: 3
Result: Wrong
Timetaken: 0.0 secs
Explanation: You are not going to the theatre,are you?
9. Choose the answer option which will correctly fill the blank.

__________ awards ceremony at Kremlin would not normally have attracted so much

attention.

A. A

B. An

C. The

D. All the above

1) a

2) b

3) c

4) d

Correct Option
2
is:
Your Option is: 3
Result: Wrong
Timetaken: 0.0 secs
For the words which begin with vowel sound we have to
Explanation:
use'a'.So,the correct answer is option B.

10 Choose the option which will correctly fill the blank.

. The sun rose ______the horizon.

A. below

B. over

C. in

D. above

1) a

2) b
3) c

4) d

Correct Option
4
is:
Your Option is: 4
Result: Correct
Timetaken: 0.0 secs
Sun can only rises above the horizontal level.So,Option D is the
Explanation:
correct answer.

11

. Choose the option which will correctly fill the blank.

This train travels from London ______ Paris.

A. At

B. to

C. over

D. below

1) a

2) b

3) c

4) d

Correct Option
2
is:
Your Option is: 2
Result: Correct
Timetaken: 0.0 secs
The destination is Paris.It has started from one place and ends in
Explanation:
some other place.So,Option b is the correct answer.
12 Choose the answer option which will correctly fill the blank.

. Seiko is______ practicing Buddhist

A. an

B. The

C. A

D. none of these

1) a

2) b

3) c

4) d

Correct Option is: 3


Your Option is: 4
Result: Wrong
Timetaken: 0.0 secs
Explanation: Option c is the correct answer.

13 Choose the option which will correctly fill the blank.

. I have been here ______ three years

A. since

B. from

C. for

D. none of the above

1) a

2) b

3) c

4) d
Correct Option is: 3
Your Option is: 3
Result: Correct
Timetaken: 0.0 secs
Explanation: for->over a period of time.So,option C is the correct answer.

14

. Choose the word nearest in meaning to the word in UNDERLINED from the given options.

Now the fury of the demonstrators turned against the machines.

A. Rage

B. Acrimony

C. Asperity

D. Passion

1) a

2) b

3) c

4) d

Correct Option
1
is:
Your Option is: 1
Result: Correct
Timetaken: 0.0 secs
fury->violent anger rage->violent uncontrollable anger acrimony-
Explanation: >bitterness asperity->harshness of tone passion->emotions
So,option A is the correct answer.

15

. Read the passage and answer the questions that follow on the basis of the information

provided in the passage.


Roger Rosenblatt 's book Black Fiction, manages to alter the approach taken in many

previous studies by making an attempt to apply literary rather than sociopolitical criteria to

its subject. Rosenblatt points out that criticism of Black writing has very often served as a

pretext for an expounding on Blackhistory. The recent work of Addison Gayle's passes a

judgement on the value of Black fiction by clearly polit ical standards, rating each work

according to the ideas of Black identity, which it propounds. Though fiction results from

political circumstances, its author react not in ideological ways to those circumstances, and

talking about novels and stories primarily as instruments of ideology circumvents much of

the fictional enterprise. Affinities andconnections are revealed in the works of Black fiction in

Rosenblatt's literary analysis; these affinities and connections have been overlooked and

ignored by solely political studies. The writing of acceptable criticism of Black fiction,

however, presumes giving satisfactory answers to a quite a few questions. The most

important of all, is there a sufficient reason, apart from the racial identity of the authors, for

the grouping together of Black authors? Secondly, what is the distinction of Black fiction

from other modern fiction with which it is largely contemporaneous? In the work Rosenblatt

demonstrates that Black fiction is a distinct body of writing, which has an identifiable,

coherent literary tradition. He highlights recurring concerns and designs, which are

independent of chronology in Black fiction written over the past eighty years. These

concerns and designs are thematic, and they come form the central fact of the predominant

white culture, where the Black characters in the novel are situated irrespective of whether

they attempt to conform to that culture or they rebel against it. Rosenblatt's work does

leave certain aesthetic questions open. His thematic a nalysis allows considerable

objectivity; he even clearly states that he does not intend to judge the merit of the various

works yet his reluctance seems misplaced, especially since an attempt to appraise might

have led to interesting results. For example, certain novels have an appearance of structural

diffusion. Is this a defeat, or are the authors workingout of, or attempting to forge, a

different kind of aesthetic? Apart from this, the style of certain Black novels, like Jean
Toomer's Cane, verges on expressionism or surrealism; does this technique provide a

counterpoint to the prevalent theme that portrays the fate against which Black heroes are

pitted, a theme usually conveyed by more naturalistic modes of expressions? Irrespective of

such omissions, w hat Rosenblatt talks about in his work makes for an astute and

worthwhile study. His book very effectively surveys a variety of novels, highlighting certain

fascinating and little -known works like James Weldon Johnson's Autobiography of an Ex-

Coloured Man. Black Fiction is tightly constructed, and levelheaded and penetrating criticism

is exemplified in its forthright and lucid style.

The primary concern of the author in the above passage is:

A. Reviewing the validity of a work of criticism

B. Comparing various critical approaches to a subject

C. Talking of the limitations of a particular kind of criticism

D. Recapitulation of the major points in a work of criticism

E. Illustrating the theoretical background of a certain kind of criticism.

1) a

2) b

3) c

4) d

5) e

Correct Option
1
is:
Your Option is: 4
Result: Wrong
Timetaken: 0.0 secs
From the passage,it is very clear that the primary concern in this
Explanation:
passage is reviewing the validity of a work of criticism.

16

. Read the passage and answer the questions that follow on the basis of the information
provided in the passage.

Disequilibrium at the interface of water and air is a factor on which the transfer of heat and

water vapor from the ocean to the air depends. The air within about a millimeter of the

water is almost saturated with water vapor and the temperature of the air is close to that of

the surface water. Irrespective of how small these differences might be, they are crucial,

and the disequilibrium is maintained by air near the surface mixing with air higher up, which

is typically appreciably cooler and lower in water vapor content. The turbulence, which takes

its energy from the wind mixes the air. As the speed of wind increases, so does the

turbulence, and consequently the rate of heat and moisture transfer. We can arrive at a

detailed understanding of this phenomenon after further study. The transfer of momentum

from wind to water, which occurs when waves are formed is an interacting -and complicated

phenomenon. When waves are made by the wind, it transfers important amounts of energy-

energy, which is consequently not available for the production of turbulence.

The wind over the ocean usually does which of the following according to the given

passage?

I. Leads to cool, dry air coming in proximity with the ocean surface.

II. Maintains a steady rate of heat and moisture transfer between the ocean and the air.

III. Results in frequent changes in the ocean surface temperature.

A. I only

B. II only

C. I and II only

D. II and III only

E. I, II, and III

1) a

2) b

3) c
4) d

5) e

Correct Option
1
is:
Your Option is: 3
Result: Wrong
Timetaken: 0.0 secs
From the passage,it is clear that the wind over the ocean does only
Explanation:
I.So,the answer is Option A

17 Directions for Questions 15-20: Read the passage and answer the questions that follow on

. the basis of the information provided in the passage.

Roger Rosenblatt?s book Black Fiction, manages to alter the approach taken in many

previous studies by making an attempt to apply literary rather than sociopolitical criteria to

its subject. Rosenblatt points out that criticism of Black writing has very often served as a

pretext for an expounding on Black history. The recent work of Addison Gayle?s passes a

judgement on the value of Black fiction by clearly political standards, rating each work

according to the ideas of Black identity, which it propounds.

Though fiction results from political circumstances, its author react not in ideological ways to

those circumstances, and talking about novels and stories primarily as instruments of

ideology circumvents much of the fictional enterprise. Affinities and connections are

revealed in the works of Black fiction in Rosenblatt?s literary analysis; these affinities and

connections have been overlooked and ignored by solely political studies.

The writing of acceptable criticism of Black fiction, however, presumes giving satisfactory

answers to a quite a few questions. The most important of all, is there a sufficient reason,

apart from the racial identity of the authors, for the grouping together of Black authors?

Secondly, what is the distinction of Black fiction from other modern fiction with which it is

largely contemporaneous? In the work Rosenblatt demonstrates that Black fiction is a

distinct body of writing, which has an identifiable, coherent literary tradition. He highlights

recurring concerns and designs, which are independent of chronology in Black fiction written
over the past eighty years. These concerns and designs are thematic, and they come form

the central fact of the predominant white culture, where the Black characters in the novel

are situated irrespective of whether they attempt to conform to that culture or they rebel

against it.

Rosenblatt?s work does leave certain aesthetic questions open. His thematic analysis allows

considerable objectivity; he even clearly states that he does not intend to judge the merit of

the various works yet his reluctance seems misplaced, especially since an attempt to

appraise might have led to interesting results. For example, certain novels have an

appearance of structural diffusion. Is this a defeat, or are the authors working out of, or

attempting to forge, a different kind of aesthetic? Apart from this, the style of certain Black

novels, like Jean Toomer?s Cane, verges on expressionism or surrealism; does this

technique provide a counterpoint to the prevalent theme that portrays the fate against

which Black heroes are pitted, a theme usually conveyed by more naturalistic modes of

expressions?

Irrespective of such omissions, what Rosenblatt talks about in his work makes for an astute

and worthwhile study. His book very effectively surveys a variety of novels, highlighting

certain fascinating and little-known works like James Weldon Johnson?s Autobiography of an

Ex-Coloured Man. Black Fiction is tightly constructed, and levelheaded and penetrating

criticism is exemplified in its forthright and lucid style.

According to the given passage the author would be LEAST likely to approve of which among

the following?

1) Analyzing the influence of political events on the personal ideology of Black writers

2) Attempting a critical study, which applies sociopolitical criteria to the autobiographies of


Black authors

3) A literary study of Black poetry that appraises the merits of poems according to the
political acceptability of their themes

4) Studying the growth of a distinct Black literary tradition within the context of Black history

5) Undertaking a literary study, which attempts to isolate aesthetic qualities unique to Black
fiction.
Correct
3
Option is:
Your Option
2
is:
Result: Wrong
Timetaken: 0.0 secs
From the passage,it is clear that,author is LEAST likely to approve a
literary study of Black poetry that appraises the merits of poems
Explanation:
according to the political acceptability of their
themes.Therefore,option 3 is the correct answer.

18

. ERROR IDENTIFICATION

In the following question,two sentences are given.There may be an error in the

sentence(s).Mark as your answer

(a) if there is an error only in the first sentence.

(b) if there is an error only in the second sentence.

(c) if there are errors in both the sentences and

(d) if there is no error in either of the two sentences.

SENTENCES

(1)To take a pay and then not to work is dishonest.

(2)To respect both our parents are our duty.

1) a

2) b

3) c

4) d

Correct Option is: 3


Your Option is: 3
Result: Correct
Timetaken: 0.0 secs
Statement 1:?To take pay? not ?a pay?
Explanation:
Statement 2:To respect both our parents is our duty.

19 Directions for Questions 15-20: Read the passage and answer the questions that follow on

. the basis of the information provided in the passage.

Roger Rosenblatt?s book Black Fiction, manages to alter the approach taken in many

previous studies by making an attempt to apply literary rather than sociopolitical criteria to

its subject. Rosenblatt points out that criticism of Black writing has very often served as a

pretext for an expounding on Black history. The recent work of Addison Gayle?s passes a

judgement on the value of Black fiction by clearly political standards, rating each work

according to the ideas of Black identity, which it propounds.

Though fiction results from political circumstances, its author react not in ideological ways to

those circumstances, and talking about novels and stories primarily as instruments of

ideology circumvents much of the fictional enterprise. Affinities and connections are

revealed in the works of Black fiction in Rosenblatt?s literary analysis; these affinities and

connections have been overlooked and ignored by solely political studies.

The writing of acceptable criticism of Black fiction, however, presumes giving satisfactory

answers to a quite a few questions. The most important of all, is there a sufficient reason,

apart from the racial identity of the authors, for the grouping together of Black authors?

Secondly, what is the distinction of Black fiction from other modern fiction with which it is

largely contemporaneous? In the work Rosenblatt demonstrates that Black fiction is a

distinct body of writing, which has an identifiable, coherent literary tradition. He highlights

recurring concerns and designs, which are independent of chronology in Black fiction written

over the past eighty years. These concerns and designs are thematic, and they come form

the central fact of the predominant white culture, where the Black characters in the novel

are situated irrespective of whether they attempt to conform to that culture or they rebel

against it.

Rosenblatt?s work does leave certain aesthetic questions open. His thematic analysis allows

considerable objectivity; he even clearly states that he does not intend to judge the merit of
the various works yet his reluctance seems misplaced, especially since an attempt to

appraise might have led to interesting results. For example, certain novels have an

appearance of structural diffusion. Is this a defeat, or are the authors working out of, or

attempting to forge, a different kind of aesthetic? Apart from this, the style of certain Black

novels, like Jean Toomer?s Cane, verges on expressionism or surrealism; does this

technique provide a counterpoint to the prevalent theme that portrays the fate against

which Black heroes are pitted, a theme usually conveyed by more naturalistic modes of

expressions?

Irrespective of such omissions, what Rosenblatt talks about in his work makes for an astute

and worthwhile study. His book very effectively surveys a variety of novels, highlighting

certain fascinating and little-known works like James Weldon Johnson?s Autobiography of an

Ex-Coloured Man. Black Fiction is tightly constructed, and levelheaded and penetrating

criticism is exemplified in its forthright and lucid style.

From the following options, which does the author not make use of while discussing Black

Fiction?

1) Rhetorical questions

2) Specific examples

3) Comparison and contrast

4) Definition of terms

5) Personal opinion.

Correct Option
4
is:
Your Option is: 2
Result: Wrong
Timetaken: 0.0 secs
In the passage,author did not make use of definition of terms while
Explanation:
discussing Black Fiction.So, option 4 is the correct answer.

20
. Read the passage and answer the questions that follow on the basis of the information

provided in the passage.

Disequilibrium at the interface of water and air is a factor on which the transfer of heat and

water vapor from the ocean to the air depends. The air within about a millimeter of the

water is almost saturated with water vapor and the temperature of the air is close to that of

the surface water. Irrespective of how small these differences mightbe, they are crucial, and

the disequilibrium is maintained by air near the surface mixing with air higher up, which is

typically appreciably cooler and lower in water vapor content. The turbulence, which takes

its energy from the wind mixes the air. As the speed of wind increases, so does the

turbulence, and consequently the rate of heat and moisture transfer. We can arrive at a

detailed understanding of this phenomenon after further study. The transfer of momentum

from wind to water, which occurs when waves are formed is an interacting -and complicated

phenomenon. When waves are made by the wind, it transfers important amounts of energy-

energy, which is consequently not available for the production of turbulence.

According to the given passage, in case the wind was to decrease until there was no wind at

all, which of the following would occur?

A. The air, which is closest to the ocean surface would get saturated with water vapor.

B. The water would be cooler than the air closest to the ocean surface.

C. There would be a decrease in the amount of moisture in the air closest to the ocean

surface.

D. There would be an increase in the rate of heat and moisture-transfer.

E. The temperature of the air closest to the ocean and that of the air higher up would be the

same.

1) a

2) b

3) c
4) d

5) e

Correct Option
1
is:
Your Option is: 4
Result: Wrong
Timetaken: 0.0 secs
From the passage,one could get the answer.Option A is the
Explanation:
correct answer.

21

. ERROR CORRECTION

Being a recently divorced man with full custody of children, I was determined to give them

A NORMANL AND STABLE A HOME LIFE as possible.

a) a normal and stable a home life.

b) a normal as well as stable a home life.

c)as normal and stable a hoe life.

d) a home life which is normal and stable.

e) as a normal and a stable home life.

1) a

2) b

3) c

4) d

5) e

Correct Option is: 3


Your Option is: 3
Result: Correct
Timetaken: 0.0 secs
Explanation: ?as normal as possible ?is intended here
22

. Read the passage and answer the questions that follow on the basis of the information

provided in the passage.

Roger Rosenblatt 's book Black Fiction, manages to alter the approach taken in many

previous studies by making an attempt to apply literary rather than sociopolitical criteria to

its subject. Rosenblatt points out that criticism of Black writing has very often served as a

pretext for an expounding on Blackhistory. The recent work of Addison Gayle's passes a

judgement on the value of Black fiction by clearly polit ical standards, rating each work

according to the ideas of Black identity, which it propounds. Though fiction results from

political circumstances, its author react not in ideological ways to those circumstances, and

talking about novels and stories primarily as instruments of ideology circumvents much of

the fictional enterprise. Affinities andconnections are revealed in the works of Black fiction in

Rosenblatt's literary analysis; these affinities and connections have been overlooked and

ignored by solely political studies. The writing of acceptable criticism of Black fiction,

however, presumes giving satisfactory answers to a quite a few questions. The most

important of all, is there a sufficient reason, apart from the racial identity of the authors, for

the grouping together of Black authors? Secondly, what is the distinction of Black fiction

from other modern fiction with which it is largely contemporaneous? In the work Rosenblatt

demonstrates that Black fiction is a distinct body of writing, which has an identifiable,

coherent literary tradition. He highlights recurring concerns and designs, which are

independent of chronology in Black fiction written over the past eighty years. These

concerns and designs are thematic, and they come form the central fact of the predominant

white culture, where the Black characters in the novel are situated irrespective of whether

they attempt to conform to that culture or they rebel against it. Rosenblatt's work does

leave certain aesthetic questions open. His thematic a nalysis allows considerable

objectivity; he even clearly states that he does not intend to judge the merit of the various

works yet his reluctance seems misplaced, especially since an attempt to appraise might
have led to interesting results. For example, certain novels have an appearance of structural

diffusion. Is this a defeat, or are the authors workingout of, or attempting to forge, a

different kind of aesthetic? Apart from this, the style of certain Black novels, like Jean

Toomer's Cane, verges on expressionism or surrealism; does this technique provide a

counterpoint to the prevalent theme that portrays the fate against which Black heroes are

pitted, a theme usually conveyed by more naturalistic modes of expressions? Irrespective of

such omissions, w hat Rosenblatt talks about in his work makes for an astute and

worthwhile study. His book very effectively surveys a variety of novels, highlighting certain

fascinating and little -known works like James Weldon Johnson's Autobiography of an Ex-

Coloured Man. Black Fiction is tightly constructed, and levelheaded and penetrating criticism

is exemplified in its forthright and lucid style.

Rosenblatt's discussion of Black Fiction is :

A. Pedantic and contentious

B. Critical but admiring

C. Ironic and deprecating

D. Argumentative but unfocused

E. Stilted and insincere.

1) a

2) b

3) c

4) d

5) e

Correct Option
2
is:
Your Option is: 3
Result: Wrong
Timetaken: 0.0 secs
Explanation: From the passage,it is clear that Rosenblatt's discussion of Black
Fiction is critical but admiring.So option 2 is the correct answer.

23

. Read the passage and answer the questions that follow on the basis of the information

provided in the passage.

Roger Rosenblatt 's book Black Fiction, manages to alter the approach taken in many

previous studies by making an attempt to apply literary rather than sociopolitical criteria to

its subject. Rosenblatt points out that criticism of Black writing has very often served as a

pretext for an expounding on Blackhistory. The recent work of Addison Gayle's passes a

judgement on the value of Black fiction by clearly polit ical standards, rating each work

according to the ideas of Black identity, which it propounds. Though fiction results from

political circumstances, its author react not in ideological ways to those circumstances, and

talking about novels and stories primarily as instruments of ideology circumvents much of

the fictional enterprise. Affinities andconnections are revealed in the works of Black fiction in

Rosenblatt's literary analysis; these affinities and connections have been overlooked and

ignored by solely political studies. The writing of acceptable criticism of Black fiction,

however, presumes giving satisfactory answers to a quite a few questions. The most

important of all, is there a sufficient reason, apart from the racial identity of the authors, for

the grouping together of Black authors? Secondly, what is the distinction of Black fiction

from other modern fiction with which it is largely contemporaneous? In the work Rosenblatt

demonstrates that Black fiction is a distinct body of writing, which has an identifiable,

coherent literary tradition. He highlights recurring concerns and designs, which are

independent of chronology in Black fiction written over the past eighty years. These

concerns and designs are thematic, and they come form the central fact of the predominant

white culture, where the Black characters in the novel are situated irrespective of whether

they attempt to conform to that culture or they rebel against it. Rosenblatt's work does

leave certain aesthetic questions open. His thematic a nalysis allows considerable

objectivity; he even clearly states that he does not intend to judge the merit of the various
works yet his reluctance seems misplaced, especially since an attempt to appraise might

have led to interesting results. For example, certain novels have an appearance of structural

diffusion. Is this a defeat, or are the authors workingout of, or attempting to forge, a

different kind of aesthetic? Apart from this, the style of certain Black novels, like Jean

Toomer's Cane, verges on expressionism or surrealism; does this technique provide a

counterpoint to the prevalent theme that portrays the fate against which Black heroes are

pitted, a theme usually conveyed by more naturalistic modes of expressions? Irrespective of

such omissions, w hat Rosenblatt talks about in his work makes for an astute and

worthwhile study. His book very effectively surveys a variety of novels, highlighting certain

fascinating and little -known works like James Weldon Johnson's Autobiography of an Ex-

Coloured Man. Black Fiction is tightly constructed, and levelheaded and penetrating criticism

is exemplified in its forthright and lucid style.

The author of the passage raises and objection to criticism of Black fiction like that by

Addison Gayle as it:

A. Highlights only the purely literary aspects of such works

B. Misconceive the ideological content of such fiction

C. Miscalculate the notions of Black identity presented in such fiction

D. Replaces political for literary criteria in evaluating such fiction

E. Disregards the reciprocation between Black history and Black identity exhibited in such

fiction.

1) a

2) b

3) c

4) d

5) e

Correct
4
Option is:
Your Option 1
is:
Result: Wrong
Timetaken: 0.0 secs
According to the passage,it is very clear that author objets to
Explanation: criticism of Black fiction like that by Addison Gayle as it replaces
political for literary criteria in evaluating such fiction.

24

. ERROR CORRECTION

We HAD BEEN ALMOST halfway there when the left rear tyre blew out.

(a)had been almost

(b)were almost

(c)were going to be almost.

(d)almost were reaching

(e)have been almost.

1) a

2) b

3) c

4) d

5) e

Correct Option is: 2


Your Option is: 5
Result: Wrong
Timetaken: 0.0 secs
Explanation: To denote a completed action we use the simple past

25 Choose the answer option which will correctly fill the blank.

. I saw _________ movie last night. _____ movie was entertaining.

A. the,A

B. A,the
C. An,A

D. the,the

1) a

2) b

3) c

4) d

Correct Option is: 4


Your Option is: 2
Result: Wrong
Timetaken: 0.0 secs
Explanation: (the,the)is the correct answer.

26

. JUMBLED SENTENCES

1.Kamala Das's conversion to Islam has created a stir.

A. But religion and faith are personal issues.

B. We have no right to judge or lokk down on a person because of the fact that he or she

converted to Islam.

C. Many people have expressed shock and anger over her conversion.

D. However,she should have been more circumspect in her remarks and refrained from

criticizing Hinduism.

6. Certain myths which are integral to Hinduism, especially the Radha Krishna cycle of

stories, have been a rich source for her poetry.

a)CABD

b)ADBC

c)ABCD

d)DACB
e)BDAC

1) a

2) b

3) c

4) d

5) e

Correct Option
1
is:
Your Option is: 4
Result: Wrong
Timetaken: 0.0 secs
C follows 1- the link being ?conversion?. A follows linked by ?
Explanation: But?. Similarly B and D are
linked by ?However?

27

. JUMBLED SENTENCES

1.What compels kids to be insolent?

A. Often they do it to get attention or to test their skill at arguing or to try to dominate their

parents,friends or teachers.

B. Thus using words to make other people angry or sad gives youngsters a sense of power.

C.However not all rude behavior should be considered an act of defiance.

D. Children are bound to be disappointed when their wants clash with parental rules and

authority.

6.You should expect and allow a certain amount of grumbling when you are telling a child to

do something or enforcing limits.

a) DABC

b) ABCD

c) BACD
d) CABD

e) DCAB.

1) a

2) b

3) c

4) d

5) e

Correct Option
2
is:
Your Option is: 5
Result: Wrong
Timetaken: 0.0 secs
The words ?they do it? gives a reason to the given question.
Explanation: Therefore, A should come after the question(statement 1).B adds
on to the stated reason.

28

. Read the passage and answer the questions that follow on the basis of the information

provided in the passage.

Disequilibrium at the interface of water and air is a factor on which the transfer of heat and

water vapor from the ocean to the air depends. The air within about a millimeter of the

water is almost saturated with water vapor and the temperature of the air is close to that of

the surface water. Irrespective of how small these differences mightbe, they are crucial, and

the disequilibrium is maintained by air near the surface mixing with air higher up, which is

typically appreciably cooler and lower in water vapor content. The turbulence, which takes

its energy from the wind mixes the air. As the speed of wind increases, so does the

turbulence, and consequently the rate of heat and moisture transfer. We can arrive at a

detailed understanding of this phenomenon after further study. The transfer of momentum
from wind to water, which occurs when waves are formed is an interacting -and complicated

phenomenon. When waves are made by the wind, it transfers important amounts of energy-

energy, which is consequently not available for the production of turbulence.

According to the author the present knowledge regarding heat and moisture transfer from

the ocean to air as

A. revolutionary

B. inconsequential

C. outdated

D. derivative

E. incomplete

1) a

2) b

3) c

4) d

5) e

Correct Option
5
is:
Your Option is: 5
Result: Correct
Timetaken: 0.0 secs
According to the author, the present knowledge regarding heat and
Explanation:
moisture transfer from ocean to air in the passage is incomplete.

29 Choose the word nearest in meaning to the word in UNDERLINED from the given options.

. Because of a family feud, he never spoke to his wife's parents.

A. Crisis

B. Trouble

C. Problem

D. Quarrel
1) a

2) b

3) c

4) d

Correct Option is: 4


Your Option is: 4
Result: Correct
Timetaken: 0.0 secs
feud->prolonged or bitter quarrel.So,option D is the correct
Explanation:
answer.

30 DIRECTIONS : Find in the blanks with correct Prepositions.

. The Work was completed _______ sunset.

1) for

2) in

3) at

4) about

Correct Option is: 3


Your Option is: 4
Result: Wrong
Timetaken: 0.0 secs
Explanation: As the work is completed while sunset,only at must be used.

31

. JUMBLED SENTENCES

1. Blood tests can diagnose thyroid disorders.

A. The most sensitive test measures the thyroid-stimulating hormone (TSH), secreted by

the pituitary gland.


B. However, says Dr.Rao, one should also take two other tests.

C. When the thyroid is under active, TSH levels will be high; low TSH levels signal an

overactive thyroid.

D. Doctors often recommend them for patients who are constantly tired or unwell,

particularly the elderly and women over 40.

6. These measure the two important thyroid hormones, thyroxin and triiodothyronine.

(a) ABCD

(b) DACB

(c) CABD

(d) BCAD

(e) DCBA.

1) a

2) b

3) c

4) d

5) e

Correct Option
2
is:
Your Option is: 3
Result: Wrong
Timetaken: 0.0 secs
Sentence 1 talks of blodd tests in general.Statement D has a
Explanation: word ?them? which refers
to the blood tests.Also,sentence B should be precede the last line.

32

. Read the passage and answer the questions that follow on the basis of the information

provided in the passage.


Roger Rosenblatt 's book Black Fiction, manages to alter the approach taken in many

previous studies by making an attempt to apply literary rather than sociopolitical criteria to

its subject. Rosenblatt points out that criticism of Black writing has very often served as a

pretext for an expounding on Blackhistory. The recent work of Addison Gayle's passes a

judgement on the value of Black fiction by clearly polit ical standards, rating each work

according to the ideas of Black identity, which it propounds. Though fiction results from

political circumstances, its author react not in ideological ways to those circumstances, and

talking about novels and stories primarily as instruments of ideology circumvents much of

the fictional enterprise. Affinities andconnections are revealed in the works of Black fiction in

Rosenblatt's literary analysis; these affinities and connections have been overlooked and

ignored by solely political studies. The writing of acceptable criticism of Black fiction,

however, presumes giving satisfactory answers to a quite a few questions. The most

important of all, is there a sufficient reason, apart from the racial identity of the authors, for

the grouping together of Black authors? Secondly, what is the distinction of Black fiction

from other modern fiction with which it is largely contemporaneous? In the work Rosenblatt

demonstrates that Black fiction is a distinct body of writing, which has an identifiable,

coherent literary tradition. He highlights recurring concerns and designs, which are

independent of chronology in Black fiction written over the past eighty years. These

concerns and designs are thematic, and they come form the central fact of the predominant

white culture, where the Black characters in the novel are situated irrespective of whether

they attempt to conform to that culture or they rebel against it. Rosenblatt's work does

leave certain aesthetic questions open. His thematic a nalysis allows considerable

objectivity; he even clearly states that he does not intend to judge the merit of the various

works yet his reluctance seems misplaced, especially since an attempt to appraise might

have led to interesting results. For example, certain novels have an appearance of structural

diffusion. Is this a defeat, or are the authors workingout of, or attempting to forge, a

different kind of aesthetic? Apart from this, the style of certain Black novels, like Jean

Toomer's Cane, verges on expressionism or surrealism; does this technique provide a


counterpoint to the prevalent theme that portrays the fate against which Black heroes are

pitted, a theme usually conveyed by more naturalistic modes of expressions? Irrespective of

such omissions, w hat Rosenblatt talks about in his work makes for an astute and

worthwhile study. His book very effectively surveys a variety of novels, highlighting certain

fascinating and little -known works like James Weldon Johnson's Autobiography of an Ex-

Coloured Man. Black Fiction is tightly constructed, and levelheaded and penetrating criticism

is exemplified in its forthright and lucid style.

The author is of the opinion that Black Fiction would have been improved had Rosenblatt:

A. Undertaken a more careful evaluation of the ideological and historical aspects of Black

Fiction

B. Been more objective in his approach to novels and stories by Black authors

C. Attempted a more detailed exploration of the recurring themes in Black fiction throughout

its history

D. Established a basis for placing Black fiction within its own unique literary tradition

E. Calculated the relative literary merit of the novels he analyzed thematically.

1) a

2) b

3) c

4) d

5) e

Correct Option
5
is:
Your Option
4
is:
Result: Wrong
Timetaken: 0.0 secs
From the passage,it is clear that,Black Fiction would have been
Explanation: improved had Rosenblatt calculated the relative literary merit of the
novels he analyzed thematically.
33

. ERROR IDENTIFICATION

In the following question,two sentences are given.There may be an error in the

sentence(s).Mark as your answer

(a) if there is an error only in the first sentence.

(b) if there is an error only in the second sentence.

(c) if there are errors in both the sentences and

(d) if there is no error in either of the two sentences.

SENTENCES

(1)Being a very hot day I remained in my room.

(2)I hope you will excuse me leaving early today.

1) a

2) b

3) c

4) d

Correct Option is: 3


Your Option is: 3
Result: Correct
Timetaken: 0.0 secs
Statement 1:It being a very hot day I remained???.
Explanation:
Statement 2:I hope you will excuse my leaving early today.

34 Choose the answer option which will correctly fill the blank.

. Are you shopping for ________ health club to join so you can get in shape? Shop wisely!

You could end up choosing _______ wrong club and losing more money than pounds.

A. the, an

B. the, the

C. A, the
D. An, the

1) a

2) b

3) c

4) d

Correct Option is: 3


Your Option is: 1
Result: Wrong
Timetaken: 0.0 secs
Explanation: (a,the) is the correct answer.

35 Choose the word nearest in meaning to the word in UNDERLINED from the given options.

. Malice is a feeling that we should always avoid.

A. Spite

B. Envy

C. Hatred

D. Cruelty

1) a

2) b

3) c

4) d

Correct Option
1
is:
Your Option is: 1
Result: Correct
Timetaken: 0.0 secs
malice->desire to harm someone spite->desire to hurt envy-
Explanation: >jealousy hatred->intense dislike cruelty->cruel behaviour
So,Option A is the correct answer.

DIRECTIONS: Fill in the following blanks with appropriate articles. Question: Both of them
1.
entered _____ after finishing high school.

1) college

2) the college

3) a college

4) in the college

Correct Option
1
is:
Your Option is: 4
Result: Wrong
Timetaken: 0.0 secs
Sentence is not particular about which college,so only 'college'
Explanation:
must be used.
2.
Sixty years ago,on the evening of august 14,1947 a few hours before britain?s indian empire

was formally divided into the nation states of india and pakistan,lord lousie mountbatten and

his wife,edwina sat down in the vicargal mansion in new g\delhi to watch the katest bob hpe

movie.my favourite brunette.?Large parts of the subconstitnent were descending into

chaos.as the implications of partioning the indian empire along religious lines became clear

to the millions of hindus,muslims twleve million people would be uprooted and as many as a

million murdered but on that night In mid aguest the bloodbath and the fuller consequence

of hasty imperial retreat ?still lay in the future,and the mountbattens probably felt they had

earned their evening?s entertainment. Posterity has enriched this speech,as nehru clearly

intended but today his quaint phrase ?tryst with density?resonate ominously,so enduring

have been the political and psychological scarst of partition.The souls of the two new nation

states immediately found utterance in brutal enmity.In punjab armed vigligants


groups.organised along religious lines and incited by local politicians ,murdered countless

people,abducing nad raping thousands of women soon.india and pakistan were displacement

was shot dead in januray 1946.by a hindu extrement who believed that the father of the

indian nation was soft on muslims jinnah racked with apperantly burned with him.

Why was gandhi assassinated?

A)Because he was favouring the musilms.

B)He assian thoughts he was partially muslims.

C)He got killed in the violance after the partition.

D)None of these

1) A

2) B

3) C

4) D

Correct Option
1
is:
Your Option is: 2
Result: Wrong
Timetaken: 0.0 secs
From the passage it is clear that gandhi was assassinated because
Explanation:
he was favouring the muslims.
3.
DIRECTIONS: Read the passage and answer the questions asked below. Japanese firms have

achieved the highest levels of manufacturing efficiency in the world automobile industry.

Some observers of Japan have assumed that Japanese firms use the same manufacturing

equipment and techniques as United States firms but have benefited from the unique

characteristics of Japanese employees and the Japanese culture. However, if this were true,

then one would expect Japanese auto plants in the United States to perform no better than

factories run by United States companies. This is not the case, Japanese-run automobile
plants located in the United States and staffed by local workers have demonstrated higher

levels of productivity when compared with factories owned by United States companies.

Other observers link high Japanese productivity to higher levels of capital investment per

worker. But a historical perspective leads to a different conclusion. When the two top

Japanese automobile makers matched and then doubled United States productivity levels in

the mid-sixties, capital investment per employee was comparable to that of United States

firms. Furthermore, by the late seventies, the amount of fixed assets required to produce

one vehicle was roughly equivalent in Japan and in the United States. Since capital

investment was not higher in Japan, it had to be other factors that led to higher productivity.

A more fruitful explanation may lie with Japanese production techniques. Japanese

automobile producers did not simply implement conventional processes more effectively:

they made critical changes in United States procedures. For instance, the mass- production

philosophy of United States automakers encouraged the production of huge lots of cars in

order to utilize fully expensive, component-specific equipment and to occupy fully workers

who have been trained to execute one operation efficiently. Japanese automakers chose to

make small-lot production feasible by introducing several departures from United States

practices, including the use of flexible equipment that could be altered easily to do several

different production tasks and the training of workers in multiple jobs. Automakers could

schedule the production of different components or models on single machines, thereby

eliminating the need to store the buffer stocks of extra components that result when

specialized equipment and workers are kept constantly active.

Question: Which of the following statements concerning the productivity levels of

automakers can be inferred from the passage?

1) Prior to the 1960's, the productivity levels of the top Japanese automakers were exceeded
by those of United States automakers.

2) The culture of a country has a large effect on the productivity levels of its automakers.

3) During the late 1970?s and early 1980?s, productivity levels were comparable in Japan
and the United States.

4) The greater the number of cars that are produced in a single lot, the higher a plant?s
productivity level.

Correct
1
Option is:
Your Option
2
is:
Result: Wrong
Timetaken: 0.0 secs
According to the passage,Prior to the 1960's,the productivity levels
Explanation: of the top Japanese automakers were exceede by those of United
States automakers.So Correct answer is option 1

Directions: Use appropriate prepositions to fill in the blanks. Question: We can do this work
4.
____ any extra heep

1) without

2) withdraw

3) with

4) within

Correct Option
1
is:
Your Option is: 1
Result: Correct
Timetaken: 0.0 secs
As the sentence means that they complete the work soon,option 1
Explanation:
is the correct answer
5.
DIRECTIONS: Read the passage and answer the question asked below. Kazuko Nakane's

history of the early Japanese immigrants to central California's Pajaro Valley focuses on the

development of farming communities there from 1890 to 1940. The Issei (first-generation

immigrants)were brought into the Pajaro Valley to raise sugar beets. Like Issei laborers in

American cities, Japanese men in rural areas sought employment via the "boss" system. The
system comprised three elements: immigrant wage laborers; Issei boardinghouses where

laborers stayed; and labor contractors, who gathered workers for a particular job and then

negotiated a contract between workers and employer. This same system was originally

utilized by the Chinese laborers who had preceded the Japanese. A related institution was

the "labor club," which provided job information and negotiated employment contracts and

other legal matters, such as the rental of land, for Issei who chose to belong and paid an

annual fee to the cooperative for membership. When the local sugar beet industry collapsed

in 1902,the Issei began to lease land from the valley's strawberry farmers. The Japanese

provided the labor and the crop was divided between laborers and landowners. The Issei

began to operate farms, they began to marry and start families, forming an established

Japanese American community. Unfortunately, the Issei's efforts to attain agricultural

independence were hampered by government restrictions, such as the Alien Land Law of

1913. But immigrants could circumvent such exclusionary laws by leasing or purchasing land

in their American-born children's names. Nakane's case study of one rural Japanese

American community provides valuable information about the lives and experiences of the

Isseil. It is, however, too particularistic. This limitation derives from Nakane's methodology

that of oral history which cannot substitute for a broader theoretical or comparative

perspective. Future research might well consider two issues raised by her study: were the

Issei of the Pajaro Valley similar to or different from Issei in urban settings, and what

variations existed between rural Japanese American communities?

Question: The primary purpose of the passage is to

1) defend a controversial hypothesis presented in a history of early Japanese immigrants to


Califronia

2) dismiss a history of an early Japanese settlement in California as narrow and ill


constructed

3) summarize and critique a history of an early Japanese settlement in California

4) compare a history of one Japanese American community with studies of Japanese


settlements throughout California
Correct Option
3
is:
Your Option is: 4
Result: Wrong
Timetaken: 0.0 secs
From the passage it is clear that the primary purpose of the passage
Explanation: is to summarize and critique a history of an early Japanese
settlement in California

Directions : Fill in the following with appropriate words Question: Legislation has been
6.
introduced in some states that requires plastics be made of bio degradable material which

gradually ___ over a period of time when exposed to sun light.

1) proliferates

2) secretes

3) decomposes

4) ossifies

Correct Option
3
is:
Your Option is: 1
Result: Wrong
Timetaken: 0.0 secs
The sentence gives proper meaning only when using
Explanation:
'decomposes'.So option 3 is the correct answer.
7.
DIRECTIONS: Read the passage and answer the questions asked below. Japanese firms have

achieved the highest levels of manufacturing efficiency in the world automobile industry.

Some observers of Japan have assumed that Japanese firms use the same manufacturing

equipment and techniques as United States firms but have benefited from the unique

characteristics of Japanese employees and the Japanese culture. However, if this were true,

then one would expect Japanese auto plants in the United States to perform no better than

factories run by United States companies. This is not the case, Japanese-run automobile
plants located in the United States and staffed by local workers have demonstrated higher

levels of productivity when compared with factories owned by United States companies.

Other observers link high Japanese productivity to higher levels of capital investment per

worker. But a historical perspective leads to a different conclusion. When the two top

Japanese automobile makers matched and then doubled United States productivity levels in

the mid-sixties, capital investment per employee was comparable to that of United States

firms. Furthermore, by the late seventies, the amount of fixed assets required to produce

one vehicle was roughly equivalent in Japan and in the United States. Since capital

investment was not higher in Japan, it had to be other factors that led to higher productivity.

A more fruitful explanation may lie with Japanese production techniques. Japanese

automobile producers did not simply implement conventional processes more effectively:

they made critical changes in United States procedures. For instance, the mass- production

philosophy of United States automakers encouraged the production of huge lots of cars in

order to utilize fully expensive, component-specific equipment and to occupy fully workers

who have been trained to execute one operation efficiently. Japanese automakers chose to

make small-lot production feasible by introducing several departures from United States

practices, including the use of flexible equipment that could be altered easily to do several

different production tasks and the training of workers in multiple jobs. Automakers could

schedule the production of different components or models on single machines, thereby

eliminating the need to store the buffer stocks of extra components that result when

specialized equipment and workers are kept constantly active.

Question: Which of the following best describes the organization of the first paragraph?

1) A thesis is presented and supporting examples are provided.

2) Opposing views are presented, classified, and then reconciled.

3) A fact is stated, and an explanation is advanced and then refuted.

4) A theory is proposed, considered, and then amended.


Correct Option
3
is:
Your Option is: 4
Result: Wrong
Timetaken: 0.0 secs
In the first paragraph of the given passage,A fact is stated and an
Explanation: explanation is advanced and then refuted.Therefore option 3 is the
correct answer
8.
DIRECTIONS: Read the passage and answer the question asked below: It can be argued that

much consumer dissatisfaction with marketing strategies arises from an inability to aim

advertising at only the likely buyers of a given product. There are three groups of consumers

who are affected by the marketing process. First, there is the market segment people who

need the commodity in question. Second, there is the program target people in the market

segment with the "best fit" characteristics for a specific product. Lots of people may need

trousers, but only a few qualify as likely buyers of very expensive designer trousers. Finally,

there is the program audience all people who are actually exposed to the marketing program

without regard to whether they need or want the product. These three groups are rarely

identical. An exception occurs occasionally in cases where customers for a particular

industrial product may be few and easily identifiable. Such customers, all sharing a particular

need, are likely to form a meaningful target, for example, all companies with a particular

application of the product in question, such as high-speed fillers of bottles at breweries. In

such circumstances, direct selling (marketing that reaches only the program target) is likely

to be economically justified, and highly specialized trade media exist to expose members of

the program target and only members of the program target to the marketing program. Most

consumer-goods markets are significantly different. Typically, there are many rather than

few potential customers. Each represents a relatively small percentage of potential sales.

Rarely do members of a particular market segment group themselves neatly into a

meaningful program target. There are substantial differences among consumers with similar

demographic characteristics. Even with all the past decade's advances in information

technology, direct selling of consumer goods is rare, and mass marketing a marketing
approach that aims at a wide audience remains the only economically feasible mode.

Unfortunately, there are few media that allow the marketer to direct a marketing program

exclusively to the program target. Inevitably, people get exposed to a great deal of

marketing for products in which they have no interest and so they become annoyed.

Question: The passage suggests which of the following about direct selling?

1) It is used in the marketing of most industrial products.

2) It is often used in cases where there is a large program target.

3) It is not economically feasible for most marketing programs.

4) It is used only for products for which there are many potential customers.

Correct Option
3
is:
Your Option is: 2
Result: Wrong
Timetaken: 0.0 secs
According to the passage,direct selling is not economically
Explanation:
feasible for most marketing programs.
9.
DIRECTIONS: Read the passage and answer the question asked below: It can be argued that

much consumer dissatisfaction with marketing strategies arises from an inability to aim

advertising at only the likely buyers of a given product. There are three groups of consumers

who are affected by the marketing process. First, there is the market segment people who

need the commodity in question. Second, there is the program target people in the market

segment with the "best fit" characteristics for a specific product. Lots of people may need

trousers, but only a few qualify as likely buyers of very expensive designer trousers. Finally,

there is the program audience all people who are actually exposed to the marketing program

without regard to whether they need or want the product. These three groups are rarely

identical. An exception occurs occasionally in cases where customers for a particular

industrial product may be few and easily identifiable. Such customers, all sharing a particular
need, are likely to form a meaningful target, for example, all companies with a particular

application of the product in question, such as high-speed fillers of bottles at breweries. In

such circumstances, direct selling (marketing that reaches only the program target) is likely

to be economically justified, and highly specialized trade media exist to expose members of

the program target and only members of the program target to the marketing program. Most

consumer-goods markets are significantly different. Typically, there are many rather than

few potential customers. Each represents a relatively small percentage of potential sales.

Rarely do members of a particular market segment group themselves neatly into a

meaningful program target. There are substantial differences among consumers with similar

demographic characteristics. Even with all the past decade's advances in information

technology, direct selling of consumer goods is rare, and mass marketing a marketing

approach that aims at a wide audience remains the only economically feasible mode.

Unfortunately, there are few media that allow the marketer to direct a marketing program

exclusively to the program target. Inevitably, people get exposed to a great deal of

marketing for products in which they have no interest and so they become annoyed.

Question: According to the passage, most consumer-goods markets share which of the

following characteristics? (I) Customers who differ significantly from each other

(II) Large numbers of potential customers

(III) Customers who each represent a small percentage of potential sales

1) I only

2) II only

3) I and II only

4) I, II, and III

Correct Option
4
is:
Your Option is: 4
Result: Correct
Timetaken: 0.0 secs
According to the passage,most consumer0goods markets share all
Explanation:
the three in the options I,II and III
10.
DIRECTIONS: Read the passage and answer the question asked below. Kazuko Nakane's

history of the early Japanese immigrants to central California's Pajaro Valley focuses on the

development of farming communities there from 1890 to 1940. The Issei (first-generation

immigrants)were brought into the Pajaro Valley to raise sugar beets. Like Issei laborers in

American cities, Japanese men in rural areas sought employment via the "boss" system. The

system comprised three elements: immigrant wage laborers; Issei boardinghouses where

laborers stayed; and labor contractors, who gathered workers for a particular job and then

negotiated a contract between workers and employer. This same system was originally

utilized by the Chinese laborers who had preceded the Japanese. A related institution was

the"labor club," which provided job information and negotiated employment contracts and

other legal matters, such as the rental of land, for Issei who chose to belong and paid an

annual fee to the cooperative for membership. When the local sugar beet industry collapsed

in 1902,the Issei began to lease land from the valley's strawberry farmers. The Japanese

provided the labor and the crop was divided between laborers and landowners. The Issei

began to operate farms, they began to marry and start families, forming an established

Japanese American community. Unfortunately, the Issei's efforts to attain agricultural

independence were hampered by government restrictions, such as the Alien Land Law of

1913. But immigrants could circumvent such exclusionary laws by leasing or purchasing land

in their American-born children's names. Nakane's case study of one rural Japanese

American community provides valuable information about the lives and experiences of the

Isseil. It is, however, too particularistic. This limitation derives from Nakane's methodology

that of oral history which cannot substitute for a broader theoretical or comparative

perspective. Future research might well consider two issues raised by her study: were the

Issei of the Pajaro Valley similar to or different from Issei in urban settings, and what

variations existed between rural Japanese American communities?

Question: Several Issei families join together to purchase a strawberry field and the
necessary farming equipment.Such a situation best exemplifies which of the following, as it

is described in the passage?

1) A typical sharecropping agreement

2) A farming corporation

3) A "labor club"

4) The "boss?" system

Correct Option
2
is:
Your Option is: 1
Result: Wrong
Timetaken: 0.0 secs
According to the passage,Issei family join to purchase represents a
Explanation:
farming corporation
11.
DIRECTIONS: Read the passage and answer the questions asked below. Japanese firms have

achieved the highest levels of manufacturing efficiency in the world automobile industry.

Some observers of Japan have assumed that Japanese firms use the same manufacturing

equipment and techniques as United States firms but have benefited from the unique

characteristics of Japanese employees and the Japanese culture. However, if this were true,

then one would expect Japanese auto plants in the United States to perform no better than

factories run by United States companies. This is not the case, Japanese-run automobile

plants located in the United States and staffed by local workers have demonstrated higher

levels of productivity when compared with factories owned by United States companies.

Other observers link high Japanese productivity to higher levels of capital investment per

worker. But a historical perspective leads to a different conclusion. When the two top

Japanese automobile makers matched and then doubled United States productivity levels in

the mid-sixties, capital investment per employee was comparable to that of United States

firms. Furthermore, by the late seventies, the amount of fixed assets required to produce
one vehicle was roughly equivalent in Japan and in the United States. Since capital

investment was not higher in Japan, it had to be other factors that led to higher productivity.

A more fruitful explanation may lie with Japanese production techniques. Japanese

automobile producers did not simply implement conventional processes more effectively:

they made critical changes in United States procedures. For instance, the mass- production

philosophy of United States automakers encouraged the production of huge lots of cars in

order to utilize fully expensive, component-specific equipment and to occupy fully workers

who have been trained to execute one operation efficiently. Japanese automakers chose to

make small-lot production feasible by introducing several departures from United States

practices, including the use of flexible equipment that could be altered easily to do several

different production tasks and the training of workers in multiple jobs. Automakers could

schedule the production of different components or models on single machines, thereby

eliminating the need to store the buffer stocks of extra components that result when

specialized equipment and workers are kept constantly active.

Question: The author suggests that if the observers of Japan mentioned in line 3 were

correct, which of the following would be the case?

1) The equipment used in Japanese automobile plants would be different from the
equipment used in United States plants.

2) Japanese workers would be trained to do several different production jobs.

3) Culture would not have an influence on the productivity levels of workers.

4) The production levels of Japanese-run plants located in the United States would be equal
to those of plants run by United States companies.

Correct
4
Option is:
Your Option
2
is:
Result: Wrong
Timetaken: 0.0 secs
In the passage,If the observers of Japan mentioned in the line 3 were
correct,then the production levels of Japanese-run plants located in
Explanation:
the United States would be equal to those of plants run by United
States Companies
12.
DIRECTIONS: Read the passage and answer the question asked below: It can be argued that

much consumer dissatisfaction with marketing strategies arises from an inability to aim

advertising at only the likely buyers of a given product. There are three groups of consumers

who are affected by the marketing process. First, there is the market segment people who

need the commodity in question. Second, there is the program target people in the market

segment with the "best fit" characteristics for a specific product. Lots of people may need

trousers, but only a few qualify as likely buyers of very expensive designer trousers. Finally,

there is the program audience all people who are actually exposed to the marketing program

without regard to whether they need or want the product. These three groups are rarely

identical. An exception occurs occasionally in cases where customers for a particular

industrial product may be few and easily identifiable. Such customers, all sharing a particular

need, are likely to form a meaningful target, for example, all companies with a particular

application of the product in question, such as high-speed fillers of bottles at breweries. In

such circumstances, direct selling (marketing that reaches only the program target) is likely

to be economically justified, and highly specialized trade media exist to expose members of

the program target and only members of the program target to the marketing program. Most

consumer-goods markets are significantly different. Typically, there are many rather than

few potential customers. Each represents a relatively small percentage of potential sales.

Rarely do members of a particular market segment group themselves neatly into a

meaningful program target. There are substantial differences among consumers with similar

demographic characteristics. Even with all the past decade?s advances in information

technology, direct selling of consumer goods is rare, and mass marketing a marketing

approach that aims at a wide audience remains the only economically feasible mode.

Unfortunately, there are few media that allow the marketer to direct a marketing program

exclusively to the program target. Inevitably, people get exposed to a great deal of
marketing for products in which they have no interest and so they become annoyed.

Question: The passage suggests which of the following about highly specialized trade media?

1) They should be used only when direct selling is not economically feasible.

2) They can be used to exclude from the program audience people who are not part of the
program target.

3) They are used only for very expensive products.

4) They are rarely used in the implementation of marketing programs for industrial products.

Correct Option
2
is:
Your Option is: 3
Result: Wrong
Timetaken: 0.0 secs
The passage suggests that, the highly specialized trade media can be
Explanation: used to exclude from the program audience people who are not part
of the program target.
13.
DIRECTIONS: Read the passage and answer the question asked below: It can be argued that

much consumer dissatisfaction with marketing strategies arises from an inability to aim

advertising at only the likely buyers of a given product. There are three groups of consumers

who are affected by the marketing process. First, there is the market segment people who

need the commodity in question. Second, there is the program target people in the market

segment with the "best fit" characteristics for a specific product. Lots of people may need

trousers, but only a few qualify as likely buyers of very expensive designer trousers. Finally,

there is the program audience all people who are actually exposed to the marketing program

without regard to whether they need or want the product. These three groups are rarely

identical. An exception occurs occasionally in cases where customers for a particular

industrial product may be few and easily identifiable. Such customers, all sharing a particular

need, are likely to form a meaningful target, for example, all companies with a particular

application of the product in question, such as high-speed fillers of bottles at breweries. In


such circumstances, direct selling (marketing that reaches only the program target) is likely

to be economically justified, and highly specialized trade media exist to expose members of

the program target and only members of the program target to the marketing program. Most

consumer-goods markets are significantly different. Typically, there are many rather than

few potential customers. Each represents a relatively small percentage of potential sales.

Rarely do members of a particular market segment group themselves neatly into a

meaningful program target. There are substantial differences among consumers with similar

demographic characteristics. Even with all the past decade's advances in information

technology, direct selling of consumer goods is rare, and mass marketing a marketing

approach that aims at a wide audience remains the only economically feasible mode.

Unfortunately, there are few media that allow the marketer to direct a marketing program

exclusively to the program target. Inevitably, people get exposed to a great deal of

marketing for products in which they have no interest and so they become annoyed.

Question: The author mentions "trousers" (lines 9 and 11) most likely in order to

1) make a comparison between the program target and the program audience

2) emphasize the similarities between the market segment and the program target

3) provide an example of the way three groups of consumers are affected by a marketing
program

4) clarify the distinction between the market segment and the program target

Correct Option
4
is:
Your Option is: 2
Result: Wrong
Timetaken: 0.0 secs
In the line 9 and 11 in the passage,author mentions 'trousers' most
Explanation: likely in order to clarify the distinction between the market segment
and the program target.
14.
DIRECTIONS: Read the passage and answer the questions asked below. Protein synthesis
begins when the gene encoding a protein is activated. The gene?s sequence of nucleotides is

transcribed into a molecule of messenger RNA (mRNA), which reproduces the information

contained in that sequence. Transported outside the nucleus to the cytoplasm, the mRNA is

translated into the protein it encodes by an organelle known as a ribosome, which strings

together amino acids in the order specified by the sequence of elements in the mRNA

molecule. Since the amount of mRNA in a cell determines the amount of the corresponding

protein, factors affecting the abundance of mRNA?s play a major part in the normal

functioning of a cell by appropriately regulating protein synthesis. For example, an excess of

certain proteins can cause cells to proliferate abnormally and become cancerous; a lack of

the protein insulin results in diabetes. Biologists once assumed that the variable rates at

which cells synthesize different mRNA?s determine the quantities of mRNA?s and their

corresponding proteins in a cell. However, recent investigations have shown that the

concentrations of most mRNA?s correlate best, not with their synthesis rate, but rather with

the equally variable rates at which cells degrade the different mRNA?s in their cytoplasm. If

a cell degrades both a rapidly and a slowly synthesized mRNA slowly, both mRNA?s will

accumulate to high levels. An important example of this phenomenon is the development of

red blood cells from their unspecialized parent cells in bone marrow. For red blood cells to

accumulate sufficient concentrations of hemoglobin (which transports oxygen) to carry out

their main function, the cells? parent cells must simultaneously produce more of the

constituent proteins of hemoglobin and less of most other proteins. To do this, the parent

cells halt synthesis of nonhemoglobin mRNA?s in the nucleus and rapidly degrade copies of

the non-hemoglobin mRNA?s remaining in the cytoplasm. Halting synthesis of mRNA alone

would not affect the quantities of proteins synthesized by the mRNA?s still existing in the

cytoplasm. Biologists now believe that most cells can regulate protein production most

efficiently by varying both mRNA synthesis and degradation, as developing red cells do,

rather than by just varying one or the other.

Question: The passage suggests that a biologist who held the view described in the first

sentence of the second paragraph would most probably also have believed which of the
following?

1) The rate of degradation of specific mRNA's has little effect on protein concentrations.

2) The rate of degradation of specific mRNA?s should be studied intensively.

3) The rates of synthesis and degradation for any given mRNA are normally equal.

4) Different mRNA?s undergo degradation at widely varying rates.

Correct
1
Option is:
Your Option
4
is:
Result: Wrong
Timetaken: 0.0 secs
The passage suggests that a biologist who held the view described in
the first sentence of the second paragraph would most probably also
Explanation:
have believed the rate of degradation of specific mRNA's has little
effect on protein concentrations.Therefore,correct answer is option 1

Directions: Fill in the following blanks with appropriate articles Question: People elected
15.
Lincoln ______ President of the United States.

1) the

2) an

3) a

Correct Option
1
is:
Your Option is: 1
Result: Correct
Timetaken: 0.0 secs
As the president is specified particularly,so the article 'the' must
Explanation:
be used.
Directions: Fill in the following blanks with appropriate articles Question: He spent ___ of his
16.
spare time learning English

1) most

2) the most

3) a most

Correct Option
1
is:
Your Option is: 2
Result: Wrong
Timetaken: 0.0 secs
no articles should be used before most.So option 1 is the correct
Explanation:
answer
17.
DIRECTIONS: Read the passage and answer the question asked below. Kazuko Nakane's

history of the early Japanese immigrants to central California's Pajaro Valley focuses on the

development of farming communities there from 1890 to 1940. The Issei (first-generation

immigrants)were brought into the Pajaro Valley to raise sugar beets. Like Issei laborers in

American cities, Japanese men in rural areas sought employment via the "boss" system. The

system comprised three elements: immigrant wage laborers; Issei boardinghouses where

laborers stayed; and labor contractors, who gathered workers for a particular job and then

negotiated a contract between workers and employer. This same system was originally

utilized by the Chinese laborers who had preceded the Japanese. A related institution was

the "labor club," which provided job information and negotiated employment contracts and

other legal matters, such as the rental of land, for Issei who chose to belong and paid an

annual fee to the cooperative for membership. When the local sugar beet industry collapsed

in 1902,the Issei began to lease land from the valley's strawberry farmers. The Japanese

provided the labor and the crop was divided between laborers and landowners. The Issei
began to operate farms, they began to marry and start families, forming an established

Japanese American community. Unfortunately, the Issei's efforts to attain agricultural

independence were hampered by government restrictions, such as the Alien Land Law of

1913. But immigrants could circumvent such exclusionary laws by leasing or purchasing land

in their American-born children's names. Nakane's case study of one rural Japanese

American community provides valuable information about the lives and experiences of the

Isseil. It is, however, too particularistic. This limitation derives from Nakane's methodology

that of oral history which cannot substitute for a broader theoretical or comparative

perspective. Future research might well consider two issues raised by her study: were the

Issei of the Pajaro Valley similar to or different from Issei in urban settings, and what

variations existed between rural Japanese American communities?

Question: The passage suggests that which of the following was an indirect consequence of

the collapse of the sugar beet industry in the Pajaro Valley?

1) The Issei formed a permanent, family-based community.

2) Boardinghouses were built to accommodate the Issei.

3) The Issei began to lease land in their children?s names.

4) The Issei adopted a labor contract system similar to that used by Chinese immigrants.

Correct
1
Option is:
Your Option
4
is:
Result: Wrong
Timetaken: 0.0 secs
According to the passage,it suggests that the Issei formed a
permanent family based community,which was an indirect
Explanation:
consequence of the collapse of the sugar beet industry in the Pajaro
valley.
18.
Directions: Use appropriate prepositions to fill in the blanks. Question: I tried to argue him
out _______ his complacency

1) of

2) since

3) by

4) for

Correct Option is: 1


Your Option is: 2
Result: Wrong
Timetaken: 0.0 secs
As the given sentence is about complacency, only 'of' should be
Explanation:
used
19.
DIRECTIONS: Read the passage and answer the questions asked below. Japanese firms have

achieved the highest levels of manufacturing efficiency in the world automobile industry.

Some observers of Japan have assumed that Japanese firms use the same manufacturing

equipment and techniques as United States firms but have benefited from the unique

characteristics of Japanese employees and the Japanese culture. However, if this were true,

then one would expect Japanese auto plants in the United States to perform no better than

factories run by United States companies. This is not the case, Japanese-run automobile

plants located in the United States and staffed by local workers have demonstrated higher

levels of productivity when compared with factories owned by United States companies.

Other observers link high Japanese productivity to higher levels of capital investment per

worker. But a historical perspective leads to a different conclusion. When the two top

Japanese automobile makers matched and then doubled United States productivity levels in

the mid-sixties, capital investment per employee was comparable to that of United States

firms. Furthermore, by the late seventies, the amount of fixed assets required to produce

one vehicle was roughly equivalent in Japan and in the United States. Since capital

investment was not higher in Japan, it had to be other factors that led to higher productivity.
A more fruitful explanation may lie with Japanese production techniques. Japanese

automobile producers did not simply implement conventional processes more effectively:

they made critical changes in United States procedures. For instance, the mass- production

philosophy of United States automakers encouraged the production of huge lots of cars in

order to utilize fully expensive, component-specific equipment and to occupy fully workers

who have been trained to execute one operation efficiently. Japanese automakers chose to

make small-lot production feasible by introducing several departures from United States

practices, including the use of flexible equipment that could be altered easily to do several

different production tasks and the training of workers in multiple jobs. Automakers could

schedule the production of different components or models on single machines, thereby

eliminating the need to store the buffer stocks of extra components that result when

specialized equipment and workers are kept constantly active.

Question: The primary purpose of the passage is to

1) present the major steps of a process

2) clarify an ambiguity

3) chronicle a dispute

4) correct misconceptions

Correct Option
4
is:
Your Option is: 4
Result: Correct
Timetaken: 0.0 secs
From the passage it is very clear that the primary purpose of the
Explanation:
given passage is to correct the misconceptions

Directions: Use appropriate prepositions to fill in the blanks. Question: During my study time
20.
, I came _______ this beautiful poem.

1) past
2) over

3) onto

4) upon

Correct Option
4
is:
Your Option is: 1
Result: Wrong
Timetaken: 0.0 secs
sentence gives proper answer only when using upon,remaining
Explanation:
options are incorrect for this sentence
21.
DIRECTIONS: Read the passage and answer the question asked below. Kazuko Nakane's

history of the early Japanese immigrants to central California's Pajaro Valley focuses on the

development of farming communities there from 1890 to 1940. The Issei (first-generation

immigrants)were brought into the Pajaro Valley to raise sugar beets. Like Issei laborers in

American cities, Japanese men in rural areas sought employment via the "boss" system. The

system comprised three elements: immigrant wage laborers; Issei boardinghouses where

laborers stayed; and labor contractors, who gathered workers for a particular job and then

negotiated a contract between workers and employer. This same system was originally

utilized by the Chinese laborers who had preceded the Japanese. A related institution was

the "labor club," which provided job information and negotiated employment contracts and

other legal matters, such as the rental of land, for Issei who chose to belong and paid an

annual fee to the cooperative for membership. When the local sugar beet industry collapsed

in 1902,the Issei began to lease land from the valley's strawberry farmers. The Japanese

provided the labor and the crop was divided between laborers and landowners. The Issei

began to operate farms, they began to marry and start families, forming an established

Japanese American community. Unfortunately, the Issei's efforts to attain agricultural

independence were hampered by government restrictions, such as the Alien Land Law of

1913. But immigrants could circumvent such exclusionary laws by leasing or purchasing land
in their American-born children's names. Nakane's case study of one rural Japanese

American community provides valuable information about the lives and experiences of the

Isseil. It is, however, too particularistic. This limitation derives from Nakane's methodology

that of oral history which cannot substitute for a broader theoretical or comparative

perspective. Future research might well consider two issues raised by her study: were the

Issei of the Pajaro Valley similar to or different from Issei in urban settings, and what

variations existed between rural Japanese American communities?

Question: Which of the following best describes a "labor club," as defined in the passage?

1) An organization to which Issei were compelled to belong if they sought employment in the
Pajaro Valley

2) An association whose members included labor contractors and landowning ?bosses?

3) A type of farming corporation set up by Issei who had resided in the Pajaro Valley for
some time

4) A cooperative association whose members were dues-paying Japanese laborers

Correct Option
4
is:
Your Option is: 4
Result: Correct
Timetaken: 0.0 secs
According to the passage,'Labor Club' describes A cooperative
Explanation:
association whose members were dues paying japanese laborers

DIRECTIONS: Fill in the following blanks with appropriate articles. Question: ______ is
22.
different from that of the 19th century.

1) New York of the 18th century.

2) The New York of the 18th

3) New York of 18th century.


Correct Option
1
is:
Your Option is: 1
Result: Correct
Timetaken: 0.0 secs
Option 1 is the answer, which is similar to the phrase used in the
Explanation:
second half of the given sentence.
23.
DIRECTIONS: Read the passage and answer the questions asked below. Japanese firms have

achieved the highest levels of manufacturing efficiency in the world automobile industry.

Some observers of Japan have assumed that Japanese firms use the same manufacturing

equipment and techniques as United States firms but have benefited from the unique

characteristics of Japanese employees and the Japanese culture. However, if this were true,

then one would expect Japanese auto plants in the United States to perform no better than

factories run by United States companies. This is not the case, Japanese-run automobile

plants located in the United States and staffed by local workers have demonstrated higher

levels of productivity when compared with factories owned by United States companies.

Other observers link high Japanese productivity to higher levels of capital investment per

worker. But a historical perspective leads to a different conclusion. When the two top

Japanese automobile makers matched and then doubled United States productivity levels in

the mid-sixties, capital investment per employee was comparable to that of United States

firms. Furthermore, by the late seventies, the amount of fixed assets required to produce

one vehicle was roughly equivalent in Japan and in the United States. Since capital

investment was not higher in Japan, it had to be other factors that led to higher productivity.

A more fruitful explanation may lie with Japanese production techniques. Japanese

automobile producers did not simply implement conventional processes more effectively:

they made critical changes in United States procedures. For instance, the mass- production

philosophy of United States automakers encouraged the production of huge lots of cars in

order to utilize fully expensive, component-specific equipment and to occupy fully workers

who have been trained to execute one operation efficiently. Japanese automakers chose to
make small-lot production feasible by introducing several departures from United States

practices, including the use of flexible equipment that could be altered easily to do several

different production tasks and the training of workers in multiple jobs. Automakers could

schedule the production of different components or models on single machines, thereby

eliminating the need to store the buffer stocks of extra components that result when

specialized equipment and workers are kept constantly active.

Question: According to the passage, which of the following statements is true of Japanese

automobile workers?

1) Their productivity levels did not equal those of United States automobile workers until the
late seventies.

2) Their high efficiency levels are a direct result of cultural influences.

3) They operate component-specific machinery.

4) They are trained to do more than one job.

Correct Option
4
is:
Your Option is: 1
Result: Wrong
Timetaken: 0.0 secs
According to the passage,It is very clear that Japanese Automobile
Explanation: workers are trained to do more than one job. So correct answer is
option 4

Directions: Fill in the following blanks with appropriate articles Question: The British
24.
commander tried to make Nathan hale_____ ,but he failed

1) turn a traitor

2) turn traitor

3) turn the traitor

4) turn traitors
Correct Option
2
is:
Your Option is: 3
Result: Wrong
Timetaken: 0.0 secs
turn traitor is the correct phrase,should not use it as turn a traitor or
Explanation:
turn the traitor or turn traitors
25.
DIRECTIONS: Read the passage and answer the questions asked below. Protein synthesis

begins when the gene encoding a protein is activated. The gene?s sequence of nucleotides is

transcribed into a molecule of messenger RNA (mRNA), which reproduces the information

contained in that sequence. Transported outside the nucleus to the cytoplasm, the mRNA is

translated into the protein it encodes by an organelle known as a ribosome, which strings

together amino acids in the order specified by the sequence of elements in the mRNA

molecule. Since the amount of mRNA in a cell determines the amount of the corresponding

protein, factors affecting the abundance of mRNA?s play a major part in the normal

functioning of a cell by appropriately regulating protein synthesis. For example, an excess of

certain proteins can cause cells to proliferate abnormally and become cancerous; a lack of

the protein insulin results in diabetes. Biologists once assumed that the variable rates at

which cells synthesize different mRNA?s determine the quantities of mRNA?s and their

corresponding proteins in a cell. However, recent investigations have shown that the

concentrations of most mRNA?s correlate best, not with their synthesis rate, but rather with

the equally variable rates at which cells degrade the different mRNA?s in their cytoplasm. If

a cell degrades both a rapidly and a slowly synthesized mRNA slowly, both mRNA?s will

accumulate to high levels. An important example of this phenomenon is the development of

red blood cells from their unspecialized parent cells in bone marrow. For red blood cells to

accumulate sufficient concentrations of hemoglobin (which transports oxygen) to carry out

their main function, the cells? parent cells must simultaneously produce more of the

constituent proteins of hemoglobin and less of most other proteins. To do this, the parent
cells halt synthesis of non-hemoglobin mRNA?s in the nucleus and rapidly degrade copies of

the nonhemoglobin mRNA?s remaining in the cytoplasm. Halting synthesis of mRNA alone

would not affect the quantities of proteins synthesized by the mRNA?s still existing in the

cytoplasm. Biologists now believe that most cells can regulate protein production most

efficiently by varying both mRNA synthesis and degradation, as developing red cells do,

rather than by just varying one or the other.

Question: The accumulation of concentrations of hemoglobin in red blood cells is mentioned

in the passage as an example of which of the following?

1) The effectiveness of simultaneous variation of the rates of synthesis and degradation of


mRNA

2) The role of the ribosome in enabling a parent cell to develop properly into a more
specialized form

3) The importance of activating the genes for particular proteins at the correct moment

4) The abnormal proliferation of a protein that threatens to make the cell cancerous

Correct
1
Option is:
Your Option
2
is:
Result: Wrong
Timetaken: 0.0 secs
In the passage,the accumulation of concentrations of hemoglobin in
red blood cells is mentioned as an example of the effectiveness of
Explanation:
simultaneous variation of the rates of synthesis and degradation of
mRNA.
26.
DIRECTIONS: Read the passage and answer the questions asked below. Protein synthesis

begins when the gene encoding a protein is activated. The gene?s sequence of nucleotides is

transcribed into a molecule of messenger RNA (mRNA), which reproduces the information

contained in that sequence. Transported outside the nucleus to the cytoplasm, the mRNA is

translated into the protein it encodes by an organelle known as a ribosome, which strings

together amino acids in the order specified by the sequence of elements in the mRNA
molecule. Since the amount of mRNA in a cell determines the amount of the corresponding

protein, factors affecting the abundance of mRNA?s play a major part in the normal

functioning of a cell by appropriately regulating protein synthesis. For example, an excess of

certain proteins can cause cells to proliferate abnormally and become cancerous; a lack of

the protein insulin results in diabetes. Biologists once assumed that the variable rates at

which cells synthesize different mRNA?s determine the quantities of mRNA?s and their

corresponding proteins in a cell. However, recent investigations have shown that the

concentrations of most mRNA?s correlate best, not with their synthesis rate, but rather with

the equally variable rates at which cells degrade the different mRNA?s in their cytoplasm. If

a cell degrades both a rapidly and a slowly synthesized mRNA slowly, both mRNA?s will

accumulate to high levels. An important example of this phenomenon is the development of

red blood cells from their unspecialized parent cells in bone marrow. For red blood cells to

accumulate sufficient concentrations of hemoglobin (which transports oxygen) to carry out

their main function, the cells? parent cells must simultaneously produce more of the

constituent proteins of hemoglobin and less of most other proteins. To do this, the parent

cells halt synthesis of nonhemoglobin mRNA?s in the nucleus and rapidly degrade copies of

the nonhemoglobin mRNA?s remaining in the cytoplasm. Halting synthesis of mRNA alone

would not affect the quantities of proteins synthesized by the mRNA?s still existing in the

cytoplasm. Biologists now believe that most cells can regulate protein production most

efficiently by varying both mRNA synthesis and degradation, as developing red cells do,

rather than by just varying one or the other.

Question: To begin to control a disease caused by a protein deficiency, the passage suggests

that a promising experimental treatment would be to administer a drug that would reduce

1) only the degradation rate for the mRNA of the protein involved

2) only the synthesis rate for the mRNA of the protein involved

3) both the synthesis and degradation rates for the mRNA of the protein involved

4) the incidence of errors in the transcription of mRNA?s from genetic nucleotide sequences
Correct
1
Option is:
Your Option
4
is:
Result: Wrong
Timetaken: 0.0 secs
According to the passage,A promising experimental treatment would
Explanation: be used to administer a drug that would reduce only the degradation
rate for the mRNA of the protein involved.
27.
DIRECTIONS: Read the passage and answer the questions asked below. Protein synthesis

begins when the gene encoding a protein is activated. The gene?s sequence of nucleotides is

transcribed into a molecule of messenger RNA (mRNA), which reproduces the information

contained in that sequence. Transported outside the nucleus to the cytoplasm, the mRNA is

translated into the protein it encodes by an organelle known as a ribosome, which strings

together amino acids in the order specified by the sequence of elements in the mRNA

molecule. Since the amount of mRNA in a cell determines the amount of the corresponding

protein, factors affecting the abundance of mRNA?s play a major part in the normal

functioning of a cell by appropriately regulating protein synthesis. For example, an excess of

certain proteins can cause cells to proliferate abnormally and become cancerous; a lack of

the protein insulin results in diabetes. Biologists once assumed that the variable rates at

which cells synthesize different mRNA?s determine the quantities of mRNA?s and their

corresponding proteins in a cell. However, recent investigations have shown that the

concentrations of most mRNA?s correlate best, not with their synthesis rate, but rather with

the equally variable rates at which cells degrade the different mRNA?s in their cytoplasm. If

a cell degrades both a rapidly and a slowly synthesized mRNA slowly, both mRNA?s will

accumulate to high levels. An important example of this phenomenon is the development of

red blood cells from their unspecialized parent cells in bone marrow. For red blood cells to

accumulate sufficient concentrations of hemoglobin (which transports oxygen) to carry out

their main function, the cells? parent cells must simultaneously produce more of the
constituent proteins of hemoglobin and less of most other proteins. To do this, the parent

cells halt synthesis of nonhemoglobin mRNA?s in the nucleus and rapidly degrade copies of

the nonhemoglobin mRNA?s remaining in the cytoplasm. Halting synthesis of mRNA alone

would not affect the quantities of proteins synthesized by the mRNA?s still existing in the

cytoplasm. Biologists now believe that most cells can regulate protein production most

efficiently by varying both mRNA synthesis and degradation, as developing red cells do,

rather than by just varying one or the other.

Question: The passage is primarily concerned with discussing the

1) influence of mRNA concentrations on the development of red blood cells

2) role of the synthesis and degradation of mRNA in cell functioning

3) mechanism by which genes are transcribed into mRNA

4) differences in mRNA concentrations in cell nuclei and in the cytoplasm

Correct Option
2
is:
Your Option is: 1
Result: Wrong
Timetaken: 0.0 secs
From the passage,it is primarily concerned with the role of
Explanation:
synthesis and degradation of mRNA in cell functioning

DIRECTIONS: Fill in the following with appropriate words. Question: Nature has an
28.
apparently _______ source of plants that can be fruitfully exploited for the herbal cure of

many diseases.

1) indefatigable

2) insatiable

3) inexhaustible

4) iniquitous
Correct Option
3
is:
Your Option is: 4
Result: Wrong
Timetaken: 0.0 secs
Nature has an apparently unlimited source of plants.The word
Explanation: unlimited is meant by 'inexhaustible'.So option 3 is the correct
answer.

Directions : Fill in the following with appropriate words Question: If this dictionary is not
29.
yours,_______ can it be ?

1) what else

2) who else

3) which else?s

4) who else?s

Correct Option
4
is:
Your Option is: 2
Result: Wrong
Timetaken: 0.0 secs
In the options,only 'who else's' is the correct phrase.remaining
Explanation:
options are incorrect for this sentence
30.
DIRECTIONS: Read the passage and answer the questions asked below. Protein synthesis

begins when the gene encoding a protein is activated. The gene?s sequence of nucleotides is

transcribed into a molecule of messenger RNA (mRNA), which reproduces the information

contained in that sequence. Transported outside the nucleus to the cytoplasm, the mRNA is

translated into the protein it encodes by an organelle known as a ribosome, which strings

together amino acids in the order specified by the sequence of elements in the mRNA

molecule. Since the amount of mRNA in a cell determines the amount of the corresponding

protein, factors affecting the abundance of mRNA?s play a major part in the normal
functioning of a cell by appropriately regulating protein synthesis. For example, an excess of

certain proteins can cause cells to proliferate abnormally and become cancerous; a lack of

the protein insulin results in diabetes. Biologists once assumed that the variable rates at

which cells synthesize different mRNA?s determine the quantities of mRNA?s and their

corresponding proteins in a cell. However, recent investigations have shown that the

concentrations of most mRNA?s correlate best, not with their synthesis rate, but rather with

the equally variable rates at which cells degrade the different mRNA?s in their cytoplasm. If

a cell degrades both a rapidly and a slowly synthesized mRNA slowly, both mRNA?s will

accumulate to high levels. An important example of this phenomenon is the development of

red blood cells from their unspecialized parent cells in bone marrow. For red blood cells to

accumulate sufficient concentrations of hemoglobin (which transports oxygen) to carry out

their main function, the cells? parent cells must simultaneously produce more of the

constituent proteins of hemoglobin and less of most other proteins. To do this, the parent

cells halt synthesis of nonhemoglobin mRNA?s in the nucleus and rapidly degrade copies of

the nonhemoglobin mRNA?s remaining in the cytoplasm. Halting synthesis of mRNA alone

would not affect the quantities of proteins synthesized by the mRNA?s still existing in the

cytoplasm. Biologists now believe that most cells can regulate protein production most

efficiently by varying both mRNA synthesis and degradation, as developing red cells do,

rather than by just varying one or the other.

Question: Which of the following best describes the relationship between the second and

third paragraphs of the passage?

1) The second paragraph presents arguments in support of a new theory and the third
paragraph presents arguments against that same theory.

2) The second paragraph describes a traditional view and the third paragraph describes the
view that has replaced it on the basis of recent investigations.

3) The third paragraph describes a specific case of a phenomenon that is described generally
in the second paragraph.

4) The third paragraph describes an investigation that was undertaken to resolve problems
raised by phenomena described in the second paragraph.
Correct Option
3
is:
Your Option is: 1
Result: Wrong
Timetaken: 0.0 secs
In the passage, the third paragraph describes a specific case of a
Explanation:
phenomenon that is described generally in the second paragraph.

DIRECTIONS: Fill in the following blanks with appropriate articles. Question: This
31.
is________ book I want to buy

1) a

2) an

3) the

Correct Option is: 3


Your Option is: 2
Result: Wrong
Timetaken: 0.0 secs
Explanation: As the sentence is about a particular book,'the' must be used

DIRECTIONS: Fill in the following with appropriate words. Question: Although there was
32.
hardly any scope to find fault with the minister's cautiously worded response, several

members of the opposition took ________ it.

1) liking for

2) liberties with

3) exception to

4) advice from
Correct Option
3
is:
Your Option is: 4
Result: Wrong
Timetaken: 0.0 secs
Given sentence gives proper meaning only by using 'exception to'
Explanation:
as answer.so option 3 is the correct answer

Sixty years ago,on the evening of august 14,1947 a few hours before britain's indian empire

was formally divided into the nation states of india and pakistan,lord lousie mountbatten and

his wife,edwina sat down in the vicargal mansion in new g\delhi to watch the katest bob hpe

movie.my favourite brunette.?Large parts of the subconstitnent were descending into

chaos.as the implications of partioning the indian empire along religious lines became clear

to the millions of hindus,muslims twleve million people would be uprooted and as many as a

million murdered but on that night In mid aguest the bloodbath and the fuller consequence

of hasty imperial retreat ?still lay in the future,and the mountbattens probably felt they had

earned their evening?s entertainment. Posterity has enriched this speech,as nehru clearly

intended but today his quaint phrase ?tryst with density?resonate ominously,so enduring

have been the political and psychological scarst of partition.The souls of the two new nation
33.
states immediately found utterance in brutal enmity.In punjab armed vigligants

groups.organised along religious lines and incited by local politicians ,murdered countless

people,abducing nad raping thousands of women soon.india and pakistan were displacement

was shot dead in januray 1946.by a hindu extrement who believed that the father of the

indian nation was soft on muslims jinnah racked with apperantly burned with him.

In the view of the author persists on talking the 'Bob Hope movie' in the article,why?

A)Because the movie was a classic of 1947.

B)He thinks it caused the partition of the sub cnstitnent.

C)He uses it to show the apathy of the britishers towards constituent.

D)It was mountbatten?s favourite movie.


1) A

2) B

3) C

4) D

Correct Option
1
is:
Your Option is: 3
Result: Wrong
Timetaken: 0.0 secs
From the passage it is clear that author persists on talking the 'bob
Explanation:
hope movie' in the article.because the music was a classic of 1947.

Directions : Fill in the following with appropriate words Question: Would you slow down a
34.
bit ? I can't ____ you

1) keep up with

2) make up to

3) put up with

4) hold on to

Correct Option
1
is:
Your Option is: 3
Result: Wrong
Timetaken: 0.0 secs
Given sentence gives the proper meaning only when using 'keep
Explanation:
up with u'
35.
DIRECTIONS: Read the passage and answer the question asked below. Kazuko Nakane's

history of the early Japanese immigrants to central California's Pajaro Valley focuses on the

development of farming communities there from 1890 to 1940. The Issei (first-generation
immigrants)were brought into the Pajaro Valley to raise sugar beets. Like Issei laborers in

American cities, Japanese men in rural areas sought employment via the "boss" system. The

system comprised three elements: immigrant wage laborers; Issei boardinghouses where

laborers stayed; and labor contractors, who gathered workers for a particular job and then

negotiated a contract between workers and employer. This same system was originally

utilized by the Chinese laborers who had preceded the Japanese. A related institution was

the "labor club," which provided job information and negotiated employment contracts and

other legal matters, such as the rental of land, for Issei who chose to belong and paid an

annual fee to the cooperative for membership. When the local sugar beet industry collapsed

in 1902,the Issei began to lease land from the valley's strawberry farmers. The Japanese

provided the labor and the crop was divided between laborers and landowners. The Issei

began to operate farms, they began to marry and start families, forming an established

Japanese American community. Unfortunately, the Issei's efforts to attain agricultural

independence were hampered by government restrictions, such as the Alien Land Law of

1913. But immigrants could circumvent such exclusionary laws by leasing or purchasing land

in their American-born children's names. Nakane's case study of one rural Japanese

American community provides valuable information about the lives and experiences of the

Isseil. It is, however, too particularistic. This limitation derives from Nakane's methodology

that of oral history which cannot substitute for a broader theoretical or comparative

perspective. Future research might well consider two issues raised by her study: were the

Issei of the Pajaro Valley similar to or different from Issei in urban settings, and what

variations existed between rural Japanese American communities?

Question: Based on information in the passage, which of the following statements

concerning the Alien Land Law of 1913 is most accurate?

1) It excluded American-born citizens of Japanese ancestry from landownership.

2) It sought to restrict the number of foreign immigrants to California.

3) It successfully prevented Issei from ever purchasing farmland.


4) It was applicable to first-generation immigrants but not to their American-born children.

Correct Option
4
is:
Your Option is: 4
Result: Correct
Timetaken: 0.0 secs
According to the passage,it is clear that the Alien Land Law of 1913
Explanation: was applicable to first-generation immigrants but not to their
American-Born children

Directions: Use appropriate prepositions to fill in the blanks. Question: We ran five laps
36.
________the gym and then practiced our jump shots.

1) against

2) across

3) around

4) among

Correct Option is: 3


Your Option is: 1
Result: Wrong
Timetaken: 0.0 secs
they can run only around the gym,other three options are
Explanation:
incorrect
37.
DIRECTIONS: Read the passage and answer the question asked below: It can be argued that

much consumer dissatisfaction with marketing strategies arises from an inability to aim

advertising at only the likely buyers of a given product. There are three groups of consumers

who are affected by the marketing process. First, there is the market segment people who

need the commodity in question. Second, there is the program target people in the market

segment with the "best fit" characteristics for a specific product. Lots of people may need
trousers, but only a few qualify as likely buyers of very expensive designer trousers. Finally,

there is the program audience all people who are actually exposed to the marketing program

without regard to whether they need or want the product. These three groups are rarely

identical. An exception occurs occasionally in cases where customers for a particular

industrial product may be few and easily identifiable. Such customers, all sharing a particular

need, are likely to form a meaningful target, for example, all companies with a particular

application of the product in question, such as high-speed fillers of bottles at breweries. In

such circumstances, direct selling (marketing that reaches only the program target) is likely

to be economically justified, and highly specialized trade media exist to expose members of

the program target and only members of the program target to the marketing program. Most

consumer-goods markets are significantly different. Typically, there are many rather than

few potential customers. Each represents a relatively small percentage of potential sales.

Rarely do members of a particular market segment group themselves neatly into a

meaningful program target. There are substantial differences among consumers with similar

demographic characteristics. Even with all the past decade?s advances in information

technology, direct selling of consumer goods is rare, and mass marketing a marketing

approach that aims at a wide audience remains the only economically feasible mode.

Unfortunately, there are few media that allow the marketer to direct a marketing program

exclusively to the program target. Inevitably, people get exposed to a great deal of

marketing for products in which they have no interest and so they become annoyed.

Question: Which of the following best exemplifies the situation described in the last two

sentences of the passage?

1) A product suitable for women age 21-30 is marketed at meetings attended only by
potential customers.

2) A company develops a new product and must develop an advertising campaign to create
a market for it.

3) An idea for a specialized product remains unexplored because media exposure of the
product to its few potential customers would be too expensive.

4) A product suitable for men age 60 and over is advertised in a magazine read by adults of
all ages.
Correct Option
4
is:
Your Option is: 4
Result: Correct
Timetaken: 0.0 secs
From the last two sentences it is very clear that option 4,'A product
Explanation: suitable for men age 60 and over is advertised in a magazine read
by adults of all ages.
1.
Read the passage and answer the questions that follow on the basis of the information

provided in the passage.

Further more, insofar as any conclusion about its author can be drawn from five or six plays

attributed to him, the Wakefield Master is without exception considered to be a man of sharp

contemporary observation. He was, probably clerically educated, as indicated by his Latin

and music, his Biblical and patristic lore. Even today he is remembered for his his quick

sympathy for the oppressed and forgotten man, his sharp eye for character, a ready ear for

colloquial, vernacular turns of speech and a humor alternately rude and boisterous, coarse

and happy. Therefore in spite of his conscious artistry as can be seen in his feeling for

intricate metrical and stanza forms, he is regarded as a kind of medieval Steinbeck,

indignantly angry at, uncompromisingly and even brutally realistic in presenting the plight of

the agricultural poor. It is now fairly accepted to regard the play as a kind of ultimate point

in the secularization of the medieval drama. Therefore more stress has been laid on it as

depicting realistically humble manners and pastoral life in the bleak of the west riding of

Yorkshire on a typically cold night of December 24th. After what are often regarded as

almost "documentaries" given in the three successive monologues of the three shepherds,

critics go on to affirm that the realism is then intensified into a burlesque mock-treatment of

the Nativity. Finally as a sort of epilogue or after-thought in deference to the Biblical origins

of the materials, the play slides back into an atavistic mood of early innocent reverence. In

actuality, the final scene is the culminating scene and also the raison d'etre of the
introductory "realism."Superficially the present play supports the conventional view of its

mood of secular realism. At the same time, the "realism" of the Wakefield Master is of a

paradoxical turn. His wide knowledge of people, as well as books indicates no cloistered

contemplative but one in close relation to his times. Still, that life was after all a

predominantly religious one, a time which never neglected the belief that man was a

rebellious and sinful creature in need of redemption. So deeply (one can hardly say "naively"

of so sophisticated a writer) and implicitly religious is the Master that he is less able (or less

willing) to present actual history realistically than is the author of the Brome Abraham and

Isaac. His historical sense is even less realistic than that of Chaucer who just a few years

before had done for his own time "costume romances," such as The Knight's Tele, Troilus and

Cressida, etc. Furthermore, used highly romantic materials, which could excuse his taking

liberties with history.

Guilt : Past : : Hope : ?

1) Present

2) Sorrow

3) Past

4) Future

Correct Option is: 4


Your Option is: 2
Result: Wrong
Timetaken: 0.0 secs
Explanation: As guilt is to past,Hope is to future

Choose the option which will correctly fill the blank.


2.

This train travels from London ______ Paris.


1) at

2) to

3) over

4) below

Correct Option
2
is:
Your Option is: 4
Result: Wrong
Timetaken: 0.0 secs
As the sentence is used with source and destination,option 2 is the
Explanation:
correct answer

Choose the word nearest in meaning to the word in ITALICS from the given options.
3.
The antidote to these problems is hard to find

1) Cause for

2) Result of

3) Remedy for

4) Consequence of

5) None of these

Correct Option is: 3


Your Option is: 2
Result: Wrong
Timetaken: 0.0 secs
Explanation: Direct meaning of 'Antidote' is 'Remedy for'
4.

Directions:A Sentence is given in five different forms.Only one of them is correct

grammatically.Mark the alphabet of the correct one as the answer: .

a)None of the soldiers have escaped unhurt. .


b)None of the soldiers has been escaping unhurt. .

C)None of the soldiers escape without a hurt. .

d)None of the soldiers have an escape without a hurt. .

e)None of the soldiers has escaped unhurt.

1) a

2) b

3) c

4) d

5) e

Correct Option
5
is:
Your Option is: 4
Result: Wrong
Timetaken: 0.0 secs
'None' indicates 'not even one'.The focus is on each
Explanation:
individual,therefore,'none'will be followed by a singular verb 'has'.

Choose the answer option which will correctly fill the blank. <br<

Are you shopping for ________ health club to join so you can get in shape? Shop wisely! You
5.
could end up choosing _______ wrong club and losing more money than pounds.

</br<

1) the, an

2) the, the

3) A, the

4) An, the

Correct Option
3
is:
Your Option is: 2
Result: Wrong
Timetaken: 0.0 secs
In the sentence,a person is joining in a health club,So 'A' must be
Explanation: used and the remaining sentence is about that particular health
club,So answer is 'the'
6.
Read the passage and answer the questions that follow on the basis of the information

provided in the passage.

Further more, insofar as any conclusion about its author can be drawn from five or six plays

attributed to him, the Wakefield Master is without exception considered to be a man of sharp

contemporary observation. He was, probably clerically educated, as indicated by his Latin

and music, his Biblical and patristic lore. Even today he is remembered for his his quick

sympathy for the oppressed and forgotten man, his sharp eye for character, a ready ear for

colloquial, vernacular turns of speech and a humor alternately rude and boisterous, coarse

and happy. Therefore in spite of his conscious artistry as can be seen in his feeling for

intricate metrical and stanza forms, he is regarded as a kind of medieval Steinbeck,

indignantly angry at, uncompromisingly and even brutally realistic in presenting the plight of

the agricultural poor. It is now fairly accepted to regard the play as a kind of ultimate point

in the secularization of the medieval drama. Therefore more stress has been laid on it as

depicting realistically humble manners and pastoral life in the bleak of the west riding of

Yorkshire on a typically cold night of December 24th. After what are often regarded as

almost "documentaries" given in the three successive monologues of the three shepherds,

critics go on to affirm that the realism is then intensified into a burlesque mock-treatment of

the Nativity. Finally as a sort of epilogue or after-thought in deference to the Biblical origins

of the materials, the play slides back into an atavistic mood of early innocent reverence. In

actuality, the final scene is the culminating scene and also the raison d'etre of the

introductory "realism."Superficially the present play supports the conventional view of its

mood of secular realism. At the same time, the "realism" of the Wakefield Master is of a
paradoxical turn. His wide knowledge of people, as well as books indicates no cloistered

contemplative but one in close relation to his times. Still, that life was after all a

predominantly religious one, a time which never neglected the belief that man was a

rebellious and sinful creature in need of redemption. So deeply (one can hardly say "naively"

of so sophisticated a writer) and implicitly religious is the Master that he is less able (or less

willing) to present actual history realistically than is the author of the Brome Abraham and

Isaac. His historical sense is even less realistic than that of Chaucer who just a few years

before had done for his own time "costume romances," such as The Knight's Tele, Troilus and

Cressida, etc. Furthermore, used highly romantic materials, which could excuse his taking

liberties with history.

The statement about the "secularization of the medieval drama" (opening sentence of the

second paragraph) refers to the

1) Introduction of religious themes in the early days

2) Presentation of erudite material

3) Use of contemporary materials

4) Return to early innocent reverence at the end of the play

5) Introduction of mundane matters in religious plays

Correct Option
5
is:
Your Option is: 1
Result: Wrong
Timetaken: 0.0 secs
The statement "secularization of the medieval drama" refers to
Explanation:
'Introduction to mundane matters in religious plays'
7.
Choose the answer option which will correctly fill the blank. <br<

_________ man ran into the street. A car hit ____ man.
</br<

1) A, the

2) An, the

3) the, the

4) A, An

Correct Option
1
is:
Your Option is: 3
Result: Wrong
Timetaken: 0.0 secs
only one man is running,so 'A' is the correct answer and the car hit
Explanation:
that particula man,so answer is 'the'
8.
Read the passage and answer the questions that follow on the basis of the information

provided in the passage.

The pioneers of the teaching of science imagined that its introduction into education would

remove the conventionality, artificiality, and backward-lookingness which were

characteristic;of classical studies, but they were gravely disappointed. So, too, in their time

had the humanists thought that the study of the classical authors in the original would

banish at once the dull pedantry and superstition of mediaeval scholasticism. The

professional schoolmaster was a match for both of them, and has almost managed to make

the understanding of chemical reactions as dull and as dogmatic an affair as the reading of

Virgil's Aeneid. The chief claim for the use of science in education is that it teaches a child

something about the actual universe in which he is living, in making him acquainted with the

results of scientific discovery, and at the same time teaches him how to think logically and

inductively by studying scientific method. A certain limited success has been reached in the

first of these aims, but practically none at all in the second. Those privileged members of the
community who have been through a secondary or public school education may be expected

to know something about the elementary physics and chemistry of a hundred years ago, but

they probably know hardly more than any bright boy can pick up from an interest in wireless

or scientific hobbies out of school hours. As to the learning of scientific method, the whole

thing is palpably a farce. Actually, for the convenience of teachers and the requirements of

the examination system, it is necessary that the pupils not only do not learn scientific

method but learn precisely the reverse, that is, to believe exactly what they are told and to

reproduce it when asked, whether it seems nonsense to them or not. The way in which

educated people respond to such quackeries as spiritualism or astrology, not to say more

dangerous ones such as racial theories or currency myths, shows that fifty years of

education in the method of science in Britain or Germany has produced no visible effect

whatever. The only way of learning the method of science is the long and bitter way of

personal experience, and, until the educational or social systems are altered to make this

possible, the best we can expect is the production of a minority of people who are able to

acquire some of the techniques of science and a still smaller minority who are able to use

and develop them.

The author's attitude to secondary and public school education in the sciences is

1) ambivalent

2) neutra

3) supportive

4) satirical

5) contemptuous

Correct Option
5
is:
Your Option is: 4
Result: Wrong
Timetaken: 0.0 secs
Author's attitude to secondary and public school education in the
Explanation:
science is proud or disrespectful,which is meant by contemptuous.

Choose the option which will correctly fill the blank.


9.

I will work _________ five o'clock.

1) until

2) up

3) in

4) to

Correct Option is: 1


Your Option is: 1
Result: Correct
Timetaken: 0.0 secs
Explanation: As the sentence is used with time limit,until is the correct answer

Directions:A Sentence is given in five different forms.Only one of them is correct

grammatically.Mark the alphabet of the correct one as the answer:

a)The youngster will benefit from the experience.


10.
b)The youngster will benefit by the experience.

c)The youngster will benefit of the experience.

d)The youngster will benefit out of the experience.

e)The youngster will be benefiting out of the experience.

1) a

2) b

3) c

4) d
5) e

Correct Option is: 1


Your Option is: 3
Result: Wrong
Timetaken: 0.0 secs
Explanation: You benefit from something.Correct preposition.

Directions:A Sentence is given in five different forms.Only one of them is correct

grammatically.Mark the alphabet of the correct one as the answer:

a)My sister likes painting,dancing and to cooking .


11.
b)My sister likes painting,dancing and to cook .

c)My sister like painting,dancing and cooking .

d) My sister likes painting,dancing and cooking.

e)My sister likes to paint,dancing and cooking.

1) a

2) b

3) c

4) d

5) e

Correct Option is: 4


Your Option is: 4
Result: Correct
Timetaken: 0.0 secs
Explanation: The gerund form must be maintained throughout a sentence.
12.
Read the passage and answer the questions that follow on the basis of the information

provided in the passage.

The pioneers of the teaching of science imagined that its introduction into education would
remove the conventionality, artificiality, and backward-lookingness which were

characteristic;of classical studies, but they were gravely disappointed. So, too, in their time

had the humanists thought that the study of the classical authors in the original would

banish at once the dull pedantry and superstition of mediaeval scholasticism. The

professional schoolmaster was a match for both of them, and has almost managed to make

the understanding of chemical reactions as dull and as dogmatic an affair as the reading of

Virgil's Aeneid. The chief claim for the use of science in education is that it teaches a child

something about the actual universe in which he is living, in making him acquainted with the

results of scientific discovery, and at the same time teaches him how to think logically and

inductively by studying scientific method. A certain limited success has been reached in the

first of these aims, but practically none at all in the second. Those privileged members of the

community who have been through a secondary or public school education may be expected

to know something about the elementary physics and chemistry of a hundred years ago, but

they probably know hardly more than any bright boy can pick up from an interest in wireless

or scientific hobbies out of school hours. As to the learning of scientific method, the whole

thing is palpably a farce. Actually, for the convenience of teachers and the requirements of

the examination system, it is necessary that the pupils not only do not learn scientific

method but learn precisely the reverse, that is, to believe exactly what they are told and to

reproduce it when asked, whether it seems nonsense to them or not. The way in which

educated people respond to such quackeries as spiritualism or astrology, not to say more

dangerous ones such as racial theories or currency myths, shows that fifty years of

education in the method of science in Britain or Germany has produced no visible effect

whatever. The only way of learning the method of science is the long and bitter way of

personal experience, and, until the educational or social systems are altered to make this

possible, the best we can expect is the production of a minority of people who are able to

acquire some of the techniques of science and a still smaller minority who are able to use

and develop them.


If the author were to study current education in science to see how things have changed

since he wrote the piece, he would probably be most interested in the answer to which of the

following questions?

1) Do students know more about the world about them?

2) Do students spend more time in laboratories?

3) Can students apply their knowledge logically?

4) Have textbooks improved?

5) Do they respect their teachers

Correct Option
3
is:
Your Option is: 1
Result: Wrong
Timetaken: 0.0 secs
Author would be more interested in the answer for the question
Explanation:
'Can students apply their knowledge logically?'

Choose the answer option which will correctly fill the blank. <br<

Albert Einstein was _____ famous scientist. Einstein won _______ Nobel Prize in Physics in
13.
1921.Einstein left his country and lived in _______ States until he died in 1955.

</br<

1) A, the, an

2) A, the, the

3) A, an, the

4) An, an, the

Correct Option
2
is:
Your Option is: 1
Result: Wrong
Timetaken: 0.0 secs
Einstein is one of the scientists,So 'A' is correct.Remaining sentence
Explanation: is about some particular award and place.So 'the' is the correct
answer.
14.
Read the passage and answer the questions that follow on the basis of the information

provided in the passage.

Further more, insofar as any conclusion about its author can be drawn from five or six plays

attributed to him, the Wakefield Master is without exception considered to be a man of sharp

contemporary observation. He was, probably clerically educated, as indicated by his Latin

and music, his Biblical and patristic lore. Even today he is remembered for his his quick

sympathy for the oppressed and forgotten man, his sharp eye for character, a ready ear for

colloquial, vernacular turns of speech and a humor alternately rude and boisterous, coarse

and happy. Therefore in spite of his conscious artistry as can be seen in his feeling for

intricate metrical and stanza forms, he is regarded as a kind of medieval Steinbeck,

indignantly angry at, uncompromisingly and even brutally realistic in presenting the plight of

the agricultural poor. It is now fairly accepted to regard the play as a kind of ultimate point

in the secularization of the medieval drama. Therefore more stress has been laid on it as

depicting realistically humble manners and pastoral life in the bleak of the west riding of

Yorkshire on a typically cold night of December 24th. After what are often regarded as

almost "documentaries" given in the three successive monologues of the three shepherds,

critics go on to affirm that the realism is then intensified into a burlesque mock-treatment of

the Nativity. Finally as a sort of epilogue or after-thought in deference to the Biblical origins

of the materials, the play slides back into an atavistic mood of early innocent reverence. In

actuality, the final scene is the culminating scene and also the raison d'etre of the

introductory "realism."Superficially the present play supports the conventional view of its

mood of secular realism. At the same time, the "realism" of the Wakefield Master is of a

paradoxical turn. His wide knowledge of people, as well as books indicates no cloistered
contemplative but one in close relation to his times. Still, that life was after all a

predominantly religious one, a time which never neglected the belief that man was a

rebellious and sinful creature in need of redemption. So deeply (one can hardly say "naively"

of so sophisticated a writer) and implicitly religious is the Master that he is less able (or less

willing) to present actual history realistically than is the author of the Brome Abraham and

Isaac. His historical sense is even less realistic than that of Chaucer who just a few years

before had done for his own time "costume romances," such as The Knight's Tele, Troilus and

Cressida, etc. Furthermore, used highly romantic materials, which could excuse his taking

liberties with history.

The word 'patristic' in the first paragraph is used to mean:

1) patriotic

2) superstitious

3) folk

4) relating to the Christian Fathers

5) realistic

Correct Option
4
is:
Your Option is: 2
Result: Wrong
Timetaken: 0.0 secs
The word 'patristic' in the first paragraph means that 'it is related
Explanation:
to the christian fathers'
15.
Read the passage and answer the questions that follow on the basis of the information

provided in the passage.

The pioneers of the teaching of science imagined that its introduction into education would

remove the conventionality, artificiality, and backward-lookingness which were


characteristic;of classical studies, but they were gravely disappointed. So, too, in their time

had the humanists thought that the study of the classical authors in the original would

banish at once the dull pedantry and superstition of mediaeval scholasticism. The

professional schoolmaster was a match for both of them, and has almost managed to make

the understanding of chemical reactions as dull and as dogmatic an affair as the reading of

Virgil's Aeneid. The chief claim for the use of science in education is that it teaches a child

something about the actual universe in which he is living, in making him acquainted with the

results of scientific discovery, and at the same time teaches him how to think logically and

inductively by studying scientific method. A certain limited success has been reached in the

first of these aims, but practically none at all in the second. Those privileged members of the

community who have been through a secondary or public school education may be expected

to know something about the elementary physics and chemistry of a hundred years ago, but

they probably know hardly more than any bright boy can pick up from an interest in wireless

or scientific hobbies out of school hours. As to the learning of scientific method, the whole

thing is palpably a farce. Actually, for the convenience of teachers and the requirements of

the examination system, it is necessary that the pupils not only do not learn scientific

method but learn precisely the reverse, that is, to believe exactly what they are told and to

reproduce it when asked, whether it seems nonsense to them or not. The way in which

educated people respond to such quackeries as spiritualism or astrology, not to say more

dangerous ones such as racial theories or currency myths, shows that fifty years of

education in the method of science in Britain or Germany has produced no visible effect

whatever. The only way of learning the method of science is the long and bitter way of

personal experience, and, until the educational or social systems are altered to make this

possible, the best we can expect is the production of a minority of people who are able to

acquire some of the techniques of science and a still smaller minority who are able to use

and develop them.

The word 'palpably' most nearly means


1) empirically

2) obviously

3) tentatively

4) markedly

5) ridiculously

Correct Option is: 2


Your Option is: 3
Result: Wrong
Timetaken: 0.0 secs
Explanation: Meaning of 'palpably' is real,clear,obvious,etc.

The students agitation was kept in ABEYANCE by the police action

a)post poned
16.
b)kept suspended

c)alleviated

d)invigorated

e)check

1) a

2) b

3) c

4) d

5) e

Correct Option is: 2


Your Option is: 2
Result: Correct
Timetaken: 0.0 secs
Explanation: meaning of "Abeyance' is suspension,reserve,pending,etc.
Choose the word nearest in meaning to the word in ITALICS from the given options.
17.
The jacket is impervious to water.

1) Dirty

2) Pure

3) Impenetrable

4) Favorable

Correct Option
3
is:
Your Option is: 3
Result: Correct
Timetaken: 0.0 secs
Meaning of 'impervious' is waterproof,impenetrable,water-
Explanation:
resistant,etc.

18.
(1)No sooner did the train start (2)when there was (3)a great rush (4)into my compartment

(5)No Error.

1) a

2) b

3) c

4) d

5) e

Correct Option is: 2


Your Option is: 2
Result: Correct
Timetaken: 0.0 secs
Explanation: 'No Sooner'is followed by 'Than' and not 'when'
19.
Choose the word nearest in meaning to the word in ITALICS from the given options.
Because of a family feud, he never spoke to his wife's parents.

1) Crisis

2) Trouble

3) Problem

4) Quarrel

5) None of these

Correct Option is: 4


Your Option is: Not Attempted
Result: Not Attempted
Timetaken: 0.0 secs
Explanation: Direct meaning of 'feud' is 'Quarrel'

20.
(1)The child picked up (2)a burned paper (3)from the debris (4)left behind by the inferno

(5)No Error

1) a

2) b

3) c

4) d

5) e

Correct Option is: 2


Your Option is: 3
Result: Wrong
Timetaken: 0.0 secs
...........picked up a burnt paper...............
Explanation:
'burnt'is the right adjectival form.'Burned' is past tense.
21.
Choose the option which will correctly fill the blank.
The microscopic animals are the primary food for larval cod and their decline has meant that

fewer fish are making it to adulthood to be caught_________ trawler men.

1) in

2) into

3) by

4) with

Correct Option
3
is:
Your Option is: 2
Result: Wrong
Timetaken: 0.0 secs
As the trawlers used to catch the fishes,it can be written as 'caught
Explanation:
by'.So option 3 is the correct answer

Choose the answer option which will correctly fill the blank. <br<

The interesting thing about _____ Romans is all the roads that they built in Britain.
22.

</br<

1) A

2) An

3) none of these

4) The

Correct Option
4
is:
Your Option is: 4
Result: Correct
Timetaken: 0.0 secs
In the given sentence,it gives the information about some
Explanation:
particular people.So answer is 'the'

Directions:A Sentence is given in five different forms.Only one of them is correct

grammatically.Mark the alphabet of the correct one as the answer: .

a)The captain with all his crew have reached the shore
23.
b)The capatain and all his crew has reached the shore

c)The captain as well as his crew have reached the shore

d)The captain and his crew have reached the shore

e)Both the captain as well as his crew have reached the shore.

1) a

2) b

3) c

4) d

5) e

Correct Option
4
is:
Your Option is: 2
Result: Wrong
Timetaken: 0.0 secs
When two nouns are joined by 'and' the verb that follows would
Explanation:
be of the plural form.
24.
Read the passage and answer the questions that follow on the basis of the information

provided in the passage.

The pioneers of the teaching of science imagined that its introduction into education would

remove the conventionality, artificiality, and backward-lookingness which were

characteristic;of classical studies, but they were gravely disappointed. So, too, in their time

had the humanists thought that the study of the classical authors in the original would
banish at once the dull pedantry and superstition of mediaeval scholasticism. The

professional schoolmaster was a match for both of them, and has almost managed to make

the understanding of chemical reactions as dull and as dogmatic an affair as the reading of

Virgil's Aeneid. The chief claim for the use of science in education is that it teaches a child

something about the actual universe in which he is living, in making him acquainted with the

results of scientific discovery, and at the same time teaches him how to think logically and

inductively by studying scientific method. A certain limited success has been reached in the

first of these aims, but practically none at all in the second. Those privileged members of the

community who have been through a secondary or public school education may be expected

to know something about the elementary physics and chemistry of a hundred years ago, but

they probably know hardly more than any bright boy can pick up from an interest in wireless

or scientific hobbies out of school hours. As to the learning of scientific method, the whole

thing is palpably a farce. Actually, for the convenience of teachers and the requirements of

the examination system, it is necessary that the pupils not only do not learn scientific

method but learn precisely the reverse, that is, to believe exactly what they are told and to

reproduce it when asked, whether it seems nonsense to them or not. The way in which

educated people respond to such quackeries as spiritualism or astrology, not to say more

dangerous ones such as racial theories or currency myths, shows that fifty years of

education in the method of science in Britain or Germany has produced no visible effect

whatever. The only way of learning the method of science is the long and bitter way of

personal experience, and, until the educational or social systems are altered to make this

possible, the best we can expect is the production of a minority of people who are able to

acquire some of the techniques of science and a still smaller minority who are able to use

and develop them.

The author blames all of the following for the failure to impart scientific method through the

education system except

1) poor teaching
2) examination methods

3) lack of direct experience

4) the social and education systems

5) lack of interest on the part of students

Correct Option
5
is:
Your Option is: 1
Result: Wrong
Timetaken: 0.0 secs
Author blames everything in the options except option 5,that is
Explanation:
'lack of interest on the part of students'
25.
Read the passage and answer the questions that follow on the basis of the information

provided in the passage.

Further more, insofar as any conclusion about its author can be drawn from five or six plays

attributed to him, the Wakefield Master is without exception considered to be a man of sharp

contemporary observation. He was, probably clerically educated, as indicated by his Latin

and music, his Biblical and patristic lore. Even today he is remembered for his his quick

sympathy for the oppressed and forgotten man, his sharp eye for character, a ready ear for

colloquial, vernacular turns of speech and a humor alternately rude and boisterous, coarse

and happy. Therefore in spite of his conscious artistry as can be seen in his feeling for

intricate metrical and stanza forms, he is regarded as a kind of medieval Steinbeck,

indignantly angry at, uncompromisingly and even brutally realistic in presenting the plight of

the agricultural poor. It is now fairly accepted to regard the play as a kind of ultimate point

in the secularization of the medieval drama. Therefore more stress has been laid on it as

depicting realistically humble manners and pastoral life in the bleak of the west riding of

Yorkshire on a typically cold night of December 24th. After what are often regarded as

almost "documentaries" given in the three successive monologues of the three shepherds,

critics go on to affirm that the realism is then intensified into a burlesque mock-treatment of
the Nativity. Finally as a sort of epilogue or after-thought in deference to the Biblical origins

of the materials, the play slides back into an atavistic mood of early innocent reverence. In

actuality, the final scene is the culminating scene and also the raison d'etre of the

introductory "realism."Superficially the present play supports the conventional view of its

mood of secular realism. At the same time, the "realism" of the Wakefield Master is of a

paradoxical turn. His wide knowledge of people, as well as books indicates no cloistered

contemplative but one in close relation to his times. Still, that life was after all a

predominantly religious one, a time which never neglected the belief that man was a

rebellious and sinful creature in need of redemption. So deeply (one can hardly say "naively"

of so sophisticated a writer) and implicitly religious is the Master that he is less able (or less

willing) to present actual history realistically than is the author of the Brome Abraham and

Isaac. His historical sense is even less realistic than that of Chaucer who just a few years

before had done for his own time "costume romances," such as The Knight's Tele, Troilus and

Cressida, etc. Furthermore, used highly romantic materials, which could excuse his taking

liberties with history.

Of the following statements, which is not true of Wakefield Master?

1) He and Chaucer were contemporaries

2) Wakefield Master is remembered as having written five or six realistic plays

3) His plays realistically portray the plight of the country folk of his day

4) His writing was similar to that of John Steinbeck

5) He was an accomplished artist

Correct Option
4
is:
Your Option is: 4
Result: Correct
Timetaken: 0.0 secs
Explanation: In the options,'His writing was similar to the John Steinbeck' is not
true of Wakefield Master.
26.
Read the passage and answer the questions that follow on the basis of the information

provided in the passage.

The pioneers of the teaching of science imagined that its introduction into education would

remove the conventionality, artificiality, and backward-lookingness which were

characteristic;of classical studies, but they were gravely disappointed. So, too, in their time

had the humanists thought that the study of the classical authors in the original would

banish at once the dull pedantry and superstition of mediaeval scholasticism. The

professional schoolmaster was a match for both of them, and has almost managed to make

the understanding of chemical reactions as dull and as dogmatic an affair as the reading of

Virgil's Aeneid. The chief claim for the use of science in education is that it teaches a child

something about the actual universe in which he is living, in making him acquainted with the

results of scientific discovery, and at the same time teaches him how to think logically and

inductively by studying scientific method. A certain limited success has been reached in the

first of these aims, but practically none at all in the second. Those privileged members of the

community who have been through a secondary or public school education may be expected

to know something about the elementary physics and chemistry of a hundred years ago, but

they probably know hardly more than any bright boy can pick up from an interest in wireless

or scientific hobbies out of school hours. As to the learning of scientific method, the whole

thing is palpably a farce. Actually, for the convenience of teachers and the requirements of

the examination system, it is necessary that the pupils not only do not learn scientific

method but learn precisely the reverse, that is, to believe exactly what they are told and to

reproduce it when asked, whether it seems nonsense to them or not. The way in which

educated people respond to such quackeries as spiritualism or astrology, not to say more

dangerous ones such as racial theories or currency myths, shows that fifty years of

education in the method of science in Britain or Germany has produced no visible effect

whatever. The only way of learning the method of science is the long and bitter way of
personal experience, and, until the educational or social systems are altered to make this

possible, the best we can expect is the production of a minority of people who are able to

acquire some of the techniques of science and a still smaller minority who are able to use

and develop them.

Astrology is mentioned as an example of

1) a science that needs to be better understood

2) a belief which no educated people hold

3) something unsupportable to those who have absorbed the methods of science

4) the gravest danger to society

5) an acknowledged failure of science

Correct Option
3
is:
Your Option is: 3
Result: Correct
Timetaken: 0.0 secs
From the passage it is clear that, Astrology is mentioned as an
Explanation: example of something unsupportable to those who have absorbed
the methods of science.
27.
Read the passage and answer the questions that follow on the basis of the information

provided in the passage.

Further more, insofar as any conclusion about its author can be drawn from five or six plays

attributed to him, the Wakefield Master is without exception considered to be a man of sharp

contemporary observation. He was, probably clerically educated, as indicated by his Latin

and music, his Biblical and patristic lore. Even today he is remembered for his his quick

sympathy for the oppressed and forgotten man, his sharp eye for character, a ready ear for

colloquial, vernacular turns of speech and a humor alternately rude and boisterous, coarse

and happy. Therefore in spite of his conscious artistry as can be seen in his feeling for
intricate metrical and stanza forms, he is regarded as a kind of medieval Steinbeck,

indignantly angry at, uncompromisingly and even brutally realistic in presenting the plight of

the agricultural poor. It is now fairly accepted to regard the play as a kind of ultimate point

in the secularization of the medieval drama. Therefore more stress has been laid on it as

depicting realistically humble manners and pastoral life in the bleak of the west riding of

Yorkshire on a typically cold night of December 24th. After what are often regarded as

almost "documentaries" given in the three successive monologues of the three shepherds,

critics go on to affirm that the realism is then intensified into a burlesque mock-treatment of

the Nativity. Finally as a sort of epilogue or after-thought in deference to the Biblical origins

of the materials, the play slides back into an atavistic mood of early innocent reverence. In

actuality, the final scene is the culminating scene and also the raison d'etre of the

introductory "realism."Superficially the present play supports the conventional view of its

mood of secular realism. At the same time, the "realism" of the Wakefield Master is of a

paradoxical turn. His wide knowledge of people, as well as books indicates no cloistered

contemplative but one in close relation to his times. Still, that life was after all a

predominantly religious one, a time which never neglected the belief that man was a

rebellious and sinful creature in need of redemption. So deeply (one can hardly say "naively"

of so sophisticated a writer) and implicitly religious is the Master that he is less able (or less

willing) to present actual history realistically than is the author of the Brome Abraham and

Isaac. His historical sense is even less realistic than that of Chaucer who just a few years

before had done for his own time "costume romances," such as The Knight's Tele, Troilus and

Cressida, etc. Furthermore, used highly romantic materials, which could excuse his taking

liberties with history.

From the following what would the writer be expected to do in the subsequent paragraphs:

1) Make a justification for his comparison with Steinbeck

2) Put forth a view point, which would take up the thought of the second paragraph

3) Point out the anachronisms in the play


4) Discuss the works of Chaucer

5) Talk about the lack of realism in the works of the Wakefield Master

Correct Option
2
is:
Your Option is: 1
Result: Wrong
Timetaken: 0.0 secs
From the passage it is clear that,the writer expects to put forth a
Explanation: view point,which would take up the thought of the second
paragraph,in the subsequent paragraphs.

Choose the option which will correctly fill the blank.

28.
I am writing to enquire _________the possibility of hiring a conference room at the hotel on

the 2nd of September.

1) of

2) about

3) into

4) after

Correct Option
2
is:
Your Option is: 3
Result: Wrong
Timetaken: 0.0 secs
In the given sentence,a person is writing to know the possibility of
Explanation:
hiring a conference.So option 2 is the correct answer.

Choose the option which will correctly fill the blank.


29.

We stood at the back ______ the theater.


1) of

2) on

3) in

4) for

Correct Option is: 1


Your Option is: 3
Result: Wrong
Timetaken: 0.0 secs
As they are standing at the back side,option 1 is the correct
Explanation:
answer

Choose the word nearest in meaning to the word in ITALICS from the given options.
30.
Instead of putting up a united front against on common enemy, the medieval states frittered

away their energy in internecine warfare.

1) mutually destructive

2) baneful

3) pernicious

4) detrimental

Correct Option
1
is:
Your Option is: 4
Result: Wrong
Timetaken: 0.0 secs
Direct meaning of 'internecine' is mutually
Explanation:
destructive,bloody,violent,deadly,etc.
31.
Read the passage and answer the questions that follow on the basis of the information

provided in the passage.


The pioneers of the teaching of science imagined that its introduction into education would

remove the conventionality, artificiality, and backward-lookingness which were

characteristic;of classical studies, but they were gravely disappointed. So, too, in their time

had the humanists thought that the study of the classical authors in the original would

banish at once the dull pedantry and superstition of mediaeval scholasticism. The

professional schoolmaster was a match for both of them, and has almost managed to make

the understanding of chemical reactions as dull and as dogmatic an affair as the reading of

Virgil's Aeneid. The chief claim for the use of science in education is that it teaches a child

something about the actual universe in which he is living, in making him acquainted with the

results of scientific discovery, and at the same time teaches him how to think logically and

inductively by studying scientific method. A certain limited success has been reached in the

first of these aims, but practically none at all in the second. Those privileged members of the

community who have been through a secondary or public school education may be expected

to know something about the elementary physics and chemistry of a hundred years ago, but

they probably know hardly more than any bright boy can pick up from an interest in wireless

or scientific hobbies out of school hours. As to the learning of scientific method, the whole

thing is palpably a farce. Actually, for the convenience of teachers and the requirements of

the examination system, it is necessary that the pupils not only do not learn scientific

method but learn precisely the reverse, that is, to believe exactly what they are told and to

reproduce it when asked, whether it seems nonsense to them or not. The way in which

educated people respond to such quackeries as spiritualism or astrology, not to say more

dangerous ones such as racial theories or currency myths, shows that fifty years of

education in the method of science in Britain or Germany has produced no visible effect

whatever. The only way of learning the method of science is the long and bitter way of

personal experience, and, until the educational or social systems are altered to make this

possible, the best we can expect is the production of a minority of people who are able to

acquire some of the techniques of science and a still smaller minority who are able to use

and develop them.


The author implies that the 'professional schoolmaster' has

1) no interest in teaching science

2) thwarted attempts to enliven education

3) aided true learning

4) supported the humanists

5) been a pioneer in both science and humanities

Correct Option
2
is:
Your Option is: 1
Result: Wrong
Timetaken: 0.0 secs
From the passage it is clear that, the 'professional schoolmaster' has
Explanation:
thwarted attempts to enliven education.

Directions:A Sentence is given in five different forms.Only one of them is correct

grammatically.Mark the alphabet of the correct one as the answer: .

a)Little money shall I spent on costly dress. .


32.
b)A little money I shall spend on costly dress. .

c)The little money I have spent on costly dress. .

d)Whatever money I have,I shall spend on costly dress. .

e)I shall spend little money I have on a costly dress

1) a

2) b

3) c

4) d

5) e
Correct Option
4
is:
Your Option is: 3
Result: Wrong
Timetaken: 0.0 secs
Whatever money i have,i will spend that on costly dress.This
Explanation:
conveyed by choice 4 only.

33.
(1)Hardly (2)had i left the school (3)than it began (4)to rain (5)No error

1) a

2) b

3) c

4) d

5) e

Correct Option is: 3


Your Option is: 2
Result: Wrong
Timetaken: 0.0 secs
Hardly.......When.
Explanation:
Corelative conjunction.

34.
(1)Her ability to (2)talk to strangers (3)is one of her (4)string points (5)No error

1) a

2) b

3) c

4) d

5) e

Correct Option is: 5


Your Option is: 4
Result: Wrong
Timetaken: 0.0 secs
Explanation: The sentence is grammatically correct.

Choose the word nearest in meaning to the word in ITALICS from the given options.
35.
The article is written in a very lucid style.

1) Elaborate

2) Clear

3) Intricate

4) Noble

5) None of these

Correct Option is: 2


Your Option is: 3
Result: Wrong
Timetaken: 0.0 secs
Explanation: Direct meaning of 'lucid' is Clear,Simple,Direct,etc.

Directions: For each of the following questions, there are two words and five alternative

answers. In each of the alternative answers there are two words. Then find out the

alternative whose two words have the same relation as in the two words given in question
1.
Explore : Discover

(a) read : skim

(b) research : learn

(c) write : print

(d) think : relate

1) a

2) b
3) c

4) d

Correct Option is: 4


Your Option is: 2
Result: Wrong
Timetaken: 0.0 secs
Explanation: think : relate

HACKNEYED

(a) trite
2.
(b) equine

(c) serrated

(d) jointed

1) a

2) b

3) c

4) d

Correct Option is: 1


Your Option is: 4
Result: Wrong
Timetaken: 0.0 secs
Explanation: (of a phrase or idea) having been overused; unoriginal and trite.
3.
br>DIRECTIONS:

Each sentence has five parts.One of them has a mistake.Identify that part as your answer.

(1)I ordered for a cup of tea

(2)but the bearer

(3)brought it only
(4)at the end of the tea-session

(5)No Error.

1) a

2) b

3) c

4) d

5) e

Correct Option
1
is:
Your Option is: 1
Result: Correct
Timetaken: 0.0 secs
'Ordered' means 'called for',therefore,'ordered for' is not correct.No
Explanation:
preposition is required after 'ordered'.

DIRECTIONS: In the following questions choose the word which best expresses the meaning

of the given word.

ACQUIESCE
4.
(a) acknowledge

(b) restate

(c) consent

(d) interfere

1) a

2) b

3) c

4) d

Correct Option is: 1


Your Option is: 4
Result: Wrong
Timetaken: 0.0 secs
Explanation: accept something reluctantly but without protest.

DIRECTIONS: In the following questions choose the word which the exact opposite of the

given words.

PSYCHOTIC

(a) holy
5.
(b) earthly

(c) sane

(d) physical

1) a

2) b

3) c

4) d

Correct Option is: 4


Your Option is: 2
Result: Wrong
Timetaken: 0.0 secs
psychotic means relating to, denoting, or suffering from a
Explanation:
psychosis.
6.
DIRECTIONS: In the following questions choose the word which the exact opposite of the

given words.

PULCHRITUDE

(a) austerity

(b) ugliness

(c) inerrancy
(d) beatitude

1) a

2) b

3) c

4) d

Correct Option is: 2


Your Option is: 1
Result: Wrong
Timetaken: 0.0 secs
Explanation: Meaning:beauty.

DIRECTIONS

Order the sentences to form a coherent passage

A)The carribean crisis which had the potential of starting the 3rd world war was successfully

averted.

B)The U.N.O has succeeded in preventing war and maintaining peace in most cases.

C)War was ended between India and Pakistan and peace was finally established in this part
7.
of the world.

D)It deployed its forces to maintain peace in Korea and succeeded in its mission.

a)BACD

b)ADCB

c)ADBC

d)BDAC

e)ACBD.

1) a
2) b

3) c

4) d

5) e

Correct
4
Option is:
Your Option
4
is:
Result: Correct
Timetaken: 0.0 secs
Look at the choices and notice that either A or B is the opening
sentence.
Explanation: We can rule out D as the concluding statement.Therefore,options 1
and 5 are ruled out.D cannot follow A.So options 2 and 3 are also
ruled out.
8.

READING COMPREHENSION.

BRITISH MANAGEMENT

According to an article in Management Today, the British manager still stops work for tea.

The French managing director of Novotel, the hotel group, tells what happened at his first

management meeting when he took over its English operation in 1991:

"The meeting was in the afternoon, and it got to about 3.30 and everyone started looking at

their watches. I didn't know what was going on. I turned to my secretary, who was English,

and she said that they probably wanted to break for tea. I couldn't understand it. In France

we just carry on until we have finished. Now we always break for tea."

I am not sure how many English managers now break for tea. Not that many, I imagine.

Twenty years ago it was commonplace. I will never forget a Canadian sales director, pink

with fury, telling me at that time how he had been offered tea and biscuits at three in the

afternoon in Northampton: "I come three thousand miles, and take three taxis and a damn

steam engine into the sticks, and I wanna do business, and this guy gives me a `nice cup of

China tea and a biccy'. What IS he?" Mind you, this was the same man who in Rome told his
Italian agent that the Colosseum would make a nice parking lot.

It is not so much the cup of tea as the pint of bitter that managers from outside Britain find

a bar to efficiency. In the article, a German manager tells how when he first took over a job

in Britain, he discovered that at lunchtime and especially on Fridays, the majority of his

management team left for the pub. He says: "I stopped that right away. Now they are not

allowed off the premises. It didn't make me very popular at the time but it is not good for

efficiency. There is no way we would do that in Germany. No way."

According to European managers the British are still too concerned with class and status. A

German says: "People say that the class system is a hindrance to progress and then two

weeks later you overhear them discussing a colleague and saying, `Well, he is not very well-

spoken, is he?'" And another says: "The class gaps translate into big gulfs in the pay league,

too. In Germany, I might earn three times more than my secretary. Here it is five times."

Some criticisms are rather like those levelled at Japanese management. For example, talking

about status, a Dutch manager says: "A director is God here. They respect him and think

that he is right even when he is wrong. It's quite difficult to have an open conversation.

People will not say `I disagree'."

Too great an interest in money is also criticised. A Dutch manager says: "My first impression

on coming to Britain was that profit seemed to be the most important thing." And a

Frenchman points out the difference between the UK and France: "In France there is no

pressure on the bottom line."

The French manager also points out a difference in educational standards: "In France all the

secretaries, or personal assistants as we call them, would have degrees. You wouldn't

consider recruiting one without a degree. So, that means you can delegate much more to

the secretaries in France. In the UK you cannot do that so much."

In the middle of the afternoon

A. everyone checked their watches.

B. hinted that it was teatime.

C. took a break.
D. noticed it was 3.30.

1) a

2) b

3) c

4) d

5) e

Correct Option is: 1


Your Option is: 1
Result: Correct
Timetaken: 0.0 secs
From para 2,one could the answer.The correct answer is option
Explanation:
A.

DIRECTIONS: Choose the best option which should replace the phrase printed in ITALICS to

make the sentence Grammatically correct.

The capacity of embryonic stem cells to develop into most type of human tissue which

makes them potentially valuable for medical applications, but the genetic program that
9.
underlies this quality is not yet known.

(a) tissue, which makes them

(b) tissue, makes them

(c) tissues, which make them

(d) tissue, making them

1) a

2) b

3) c

4) d
Correct Option is: 2
Your Option is: 2
Result: Correct
Timetaken: 0.0 secs
Explanation: tissue, makes them

Directions: Choose the best option which should replace the phrase printed in ITALICS to

make the sentence

One of every two new business fail within two years.


10.
(a) has failed

(b) fails

(c) may have failed

(d) should fail

1) a

2) b

3) c

4) d

Correct Option is: 1


Your Option is: 1
Result: Correct
Timetaken: 0.0 secs
Explanation: has failed
11.
DIRECTIONS: In the following questions choose the word which the exact opposite of the

given words.

PUSILLANIMOUS

(a) fragrant

(b) masculine
(c) inopportune

(d) courageous

1) a

2) b

3) c

4) d

Correct Option is: 4


Your Option is: 4
Result: Correct
Timetaken: 0.0 secs
Explanation: Meaning : showing a lack of courage or determination; timid.

DIRECTIONS: In the questions below have been given in active/passive voice, from the

given alternatives ,choose the one which best expresses the given sentence in passive/active

voice.26) The wind blow down the trees.


12.
(a)The trees were blow down by the wind.

(b)The trees were blown down by the wind.

(c)The wind were blowing down the trees.

(d)The trees are being blown down by the wind.

1) a

2) b

3) c

4) d
Correct Option is: 2
Your Option is: 1
Result: Wrong
Timetaken: 0.0 secs
Explanation: The trees were blown down by the wind.

DIRECTIONS: Each of the following questions has four alternative responses. Select the

correct response

Which one of the following is same as cork, plug, dam?


13.
(a) Obstruction

(b) Drain

(c) Pipe

(d) Tap

1) a

2) b

3) c

4) d

Correct Option is: 1


Your Option is: 4
Result: Wrong
Timetaken: 0.0 secs
Explanation: Obstruction
14.

DIRECTIONS: In the questions below have been given in active/passive voice, from the

given alternatives ,choose the one which best expresses the given sentence in passive/active

voice

This house was built by John Mathews in 1991. (a)In 1991, John Mathews built this house.
(b)John Mathews ,in 1991 built this house.

(c)John Mathews built this house in 1991.

(d)John Mathews in 1991 built this house.

1) a

2) b

3) c

4) d

Correct Option is: 3


Your Option is: 1
Result: Wrong
Timetaken: 0.0 secs
Explanation: John Mathews built this house in 1991.

Directions: For each of the following questions, there are two words and five alternative

answers. In each of the alternative answers there are two words. Then find out the

alternative whose two words have the same relation as in the two words given in question.
15.
Resignation : Office :

(a) Competition: Game

(b) Illness: Hospital

(c) Abdication : Throne

(d) Moisture : Rain

1) a

2) b

3) c

4) d
Correct Option is: 3
Your Option is: 4
Result: Wrong
Timetaken: 0.0 secs
Explanation: Abdication : Throne

br>DIRECTIONS:

Each sentence has five parts.One of them has a mistake.Identify that part as your answer.

(1)Shakespeare was
16.
(2)better than any poet

(3)of his days

(4)according to critics

(5)No Error.

1) a

2) b

3) c

4) d

5) e

Correct Option
2
is:
Your Option is: 3
Result: Wrong
Timetaken: 0.0 secs
'.........better than any other poet.......'correct form of
Explanation:
comparision.'than any other'
17.

Which one of the following is same as Jute, Cotton and Wool?

(a) Nylon
(b) Rayon

(c) Silk

(d) Terylene

1) a

2) b

3) c

4) d

Correct Option is: 3


Your Option is: 4
Result: Wrong
Timetaken: 0.0 secs
Explanation: Silk

DIRECTIONS: In the questions below the sentences are been given in the direct/indirect

speech from the given alternatives , choose the one which best expresses the given

sentence in indirect/direct speech. The king was impressed with the magician and asked ,?
18.
what can I do for you??

(a) The king was impressed with the magician and asked what he can do for him.

(b) The king being impressed with the magician and asked what he could do for him.

(c)The king was impressed with the magician and asked what he could do for him.

1) a

2) b

3) c
Correct Option
2
is:
Your Option is: 1
Result: Wrong
Timetaken: 0.0 secs
The king being impressed with the magician and asked what he
Explanation:
could do for him.

DIRECTIONS

Select the correct option of words from the choices that completed the given sentence as

your answer.

Peace of mind is not something which you can ___________.


19.
a)find easily when you are troubled.

b)understand as a mature human being.

c)overrule with any stupid excuse.

d)find and take for yourself.

e)even in chaotic surroundings.

1) a

2) b

3) c

4) d

5) e

Correct
1
Option is:
Your Option
4
is:
Result: Wrong
Timetaken: 0.0 secs
Explanation: The narrator intends to say that peace of mind is not easy to
find.Option (1),which maintains the tone of peace of mind not being
a thing which can be got easily',is the right answer

Choose the best option which should replace the phrase printed in ITALICS to make the

sentence

Female sparrows and immature are quite non-descript compared to the adult male

sparrow's distinctive and conspicuous markings.


20.

(a) those of adult males, their

(b) the adult male, which has

(c) adult male sparrow's

(d) adult males and their

1) a

2) b

3) c

4) d

Correct Option is: 2


Your Option is: 3
Result: Wrong
Timetaken: 0.0 secs
Explanation: the adult male, which has
21.

BRITISH MANAGEMENT.

According to an article in Management Today, the British manager still stops work for tea.

The French managing director of Novotel, the hotel group, tells what happened at his first

management meeting when he took over its English operation in 1991: .

"The meeting was in the afternoon, and it got to about 3.30 and everyone started looking at

their watches. I didn't know what was going on. I turned to my secretary, who was English,
and she said that they probably wanted to break for tea. I couldn't understand it. In France

we just carry on until we have finished. Now we always break for tea." .

I am not sure how many English managers now break for tea. Not that many, I imagine.

Twenty years ago it was commonplace. I will never forget a Canadian sales director, pink

with fury, telling me at that time how he had been offered tea and biscuits at three in the

afternoon in Northampton: "I come three thousand miles, and take three taxis and a damn

steam engine into the sticks, and I wanna do business, and this guy gives me a `nice cup of

China tea and a biccy'. What IS he?" Mind you, this was the same man who in Rome told his

Italian agent that the Colosseum would make a nice parking lot.

It is not so much the cup of tea as the pint of bitter that managers from outside Britain find

a bar to efficiency. In the article, a German manager tells how when he first took over a job

in Britain, he discovered that at lunchtime and especially on Fridays, the majority of his

management team left for the pub. He says: "I stopped that right away. Now they are not

allowed off the premises. It didn't make me very popular at the time but it is not good for

efficiency. There is no way we would do that in Germany. No way." .

According to European managers the British are still too concerned with class and status. A

German says: "People say that the class system is a hindrance to progress and then two

weeks later you overhear them discussing a colleague and saying, `Well, he is not very well-

spoken, is he?'" And another says: "The class gaps translate into big gulfs in the pay league,

too. In Germany, I might earn three times more than my secretary. Here it is five times." .

Some criticisms are rather like those levelled at Japanese management. For example, talking

about status, a Dutch manager says: "A director is God here. They respect him and think

that he is right even when he is wrong. It's quite difficult to have an open conversation.

People will not say `I disagree'." .

Too great an interest in money is also criticised. A Dutch manager says: "My first impression

on coming to Britain was that profit seemed to be the most important thing." And a

Frenchman points out the difference between the UK and France: "In France there is no

pressure on the bottom line." .


The French manager also points out a difference in educational standards: "In France all the

secretaries, or personal assistants as we call them, would have degrees. You wouldn't

consider recruiting one without a degree. So, that means you can delegate much more to

the secretaries in France. In the UK you cannot do that so much." .

Why did the Canadian say the Colosseum would make a nice parking lot? .

A. He had nowhere to park.

B. He was joking.

C. He was worried about Rome's traffic problems.

D. He was envious of Italy's glorious past.

1) a

2) b

3) c

4) d

5) e

Correct Option
2
is:
Your Option is: 2
Result: Correct
Timetaken: 0.0 secs
From para 2 we can answer to this question.He was just
Explanation:
joking.So,Option B is the correct answer.
22.

DIRECTIONS: Each of the following questions has four alternative responses. Select the

correct response

Which one of the following is same as uncle, aunt, brother?

(a) Child

(b) Boy

(c) Adult
(d) Nephew

1) a

2) b

3) c

4) d

Correct Option is: 4


Your Option is: 2
Result: Wrong
Timetaken: 0.0 secs
Explanation: Nephew

DIRECTIONS

Order the sentences to form a coherent passage

A)If we want to make our democracy stable there should be some checks upon the people in

power.

B)We do not have a healthy opposition and this gap can be covered only by a free and bold

press.

C)In India democracy is still passing through its infantile stage suffering from a lot of ups
23.
and downs.

D)The leaders generally become very ambitious and so they try to gain power as much as is

possible.

a)CABD

b)CADB

c)DACB

d)DBCA

e)ACDB.
1) a

2) b

3) c

4) d

5) e

Correct Option
2
is:
Your Option is: 3
Result: Wrong
Timetaken: 0.0 secs
Statement C can precede A but C cannot follow A as A explains
what is stated in C.The remaining choices indicate that either C or D
is the first sentence.
Explanation: However the word 'the' in D indicates that this is not the first time
that leaders are being mentionrd.
So only C can start a sentence.A follows C.Also Dfollows A
because of the 'check upon the people in power'

br>DIRECTIONS:

Each sentence has five parts.One of them has a mistake.Identify that part as your answer.

(1)The first lesson I got


24.
(2)in my military days was that

(3)one should love his country

(4)better than anything else

(5)No Error.

1) a

2) b

3) c

4) d

5) e
Correct Option
3
is:
Your Option is: 2
Result: Wrong
Timetaken: 0.0 secs
'One' should be followed by one or one's pnly and not by
Explanation:
'his'..........one should love one's country.

DIRECTIONS

Order the sentences to form a coherent passage

A)Mahatma Gandhi rightly said that the prosperity of India depends upon the prosperity of

villages.

B)He exhorted the social workers that emanicipation of India lay in the emanicipation of

villages.

C)If we are really interested in national development,villages are to be given the first
25.
priority.

D)India is predominantly a rural society with about 70% of the people living in rural areas

and if things are determined as per majority,it may well be said that India lives in villages.

a)DABC

b)CABD

c)CADB

d)DBAC

e)DACB.

1) a

2) b

3) c

4) d

5) e
Correct Option is: 1
Your Option is: 3
Result: Wrong
Timetaken: 0.0 secs
The choice indicates C or D as the first sentence.
B has to follow A.Therefore either 1 or 2 is the answer.
Explanation:
D is a better opening statement while C is a good closing
statement.

DIRECTIONS:

Each sentence has five parts.One of them has a mistake.Identify that part as your answer.

(1)Before he started out


26.
(2)he had collected

(3)all the informations about the country

(4)he was going to visit

(5)No Error.

1) a

2) b

3) c

4) d

5) e

Correct Option
3
is:
Your Option is: 4
Result: Wrong
Timetaken: 0.0 secs
Information is a collective noun
which means items of knowledge.Collective nouns do not take a
Explanation:
plural form.
Hence,informations is incorrect.
DIRECTIONS

Select the correct option of words from the choices that completed the given sentence as

your answer.

Sam and I were walking along the road when a bus hit _____________.
27.
a)a car coming round the bend.

b)a bucket full of water.

c)two trams together.

d)four of the rickshaws that were sold.

e)a herd cattle grazing in the distant fields.

1) a

2) b

3) c

4) d

5) e

Correct Option
1
is:
Your Option
1
is:
Result: Correct
Timetaken: 0.0 secs
Look at the context of the sentence.A person is narrating an
Explanation: incident.Hence,logically option (1) is correct".......were
walking.......when a bus hit a car.
28.

DIRECTIONS: In the questions below the sentences are been given in the direct/indirect

speech from the given alternatives , choose the one which best expresses the given

sentence in indirect/direct speech.

They asked who he was and what he wanted.


(a)"who do you want ?" They asked him.

(b)"who are you ,sir , and what do you want"? They asked.

(c)"what and when do you want"? They asked.

1) a

2) b

3) c

4) d

Correct Option is: 2


Your Option is: 4
Result: Wrong
Timetaken: 0.0 secs
Explanation: "who are you ,sir , and what do you want"? They asked.

DIRECTIONS: In the questions below have been given in active/passive voice, from the

given alternatives ,choose the one which best expresses the given sentence in passive/active

voice.28)

My friend sent me a letter.


29.
(a)A letter was sent to me by my friend.

(b)I was sent a letter by my friend.

(c)I sent my friend a letter.

(d)My friend sent a letter to me.

1) a

2) b

3) c
4) d

Correct Option is: 2


Your Option is: 3
Result: Wrong
Timetaken: 0.0 secs
Explanation: I was sent a letter by my friend.

DIRECTIONS

Select the correct option of words from the choices that completed the given sentence as

your answer.

The Productive hours of the employees are _____________.


30.
a)wasted in meaningless chatter.

b)as good as any other.

c)designed for creativity.

d)all nearly spent in an office.

e)highly stressful.

1) a

2) b

3) c

4) d

5) e

Correct Option
4
is:
Your Option is: 2
Result: Wrong
Timetaken: 0.0 secs
Explanation: Since we are talking of 'productive hours'we can rule out
'wasting'(choice 1).
choice 2 and 3 do not make any sense.
31.

BRITISH MANAGEMENT

According to an article in Management Today, the British manager still stops work for tea.

The French managing director of Novotel, the hotel group, tells what happened at his first

management meeting when he took over its English operation in 1991:

"The meeting was in the afternoon, and it got to about 3.30 and everyone started looking at

their watches. I didn't know what was going on. I turned to my secretary, who was English,

and she said that they probably wanted to break for tea. I couldn't understand it. In France

we just carry on until we have finished. Now we always break for tea."

I am not sure how many English managers now break for tea. Not that many, I imagine.

Twenty years ago it was commonplace. I will never forget a Canadian sales director, pink

with fury, telling me at that time how he had been offered tea and biscuits at three in the

afternoon in Northampton: "I come three thousand miles, and take three taxis and a damn

steam engine into the sticks, and I wanna do business, and this guy gives me a `nice cup of

China tea and a biccy'. What IS he?" Mind you, this was the same man who in Rome told his

Italian agent that the Colosseum would make a nice parking lot.

It is not so much the cup of tea as the pint of bitter that managers from outside Britain find

a bar to efficiency. In the article, a German manager tells how when he first took over a job

in Britain, he discovered that at lunchtime and especially on Fridays, the majority of his

management team left for the pub. He says: "I stopped that right away. Now they are not

allowed off the premises. It didn't make me very popular at the time but it is not good for

efficiency. There is no way we would do that in Germany. No way."

According to European managers the British are still too concerned with class and status. A

German says: "People say that the class system is a hindrance to progress and then two

weeks later you overhear them discussing a colleague and saying, `Well, he is not very well-

spoken, is he?'" And another says: "The class gaps translate into big gulfs in the pay league,

too. In Germany, I might earn three times more than my secretary. Here it is five times."
Some criticisms are rather like those levelled at Japanese management. For example, talking

about status, a Dutch manager says: "A director is God here. They respect him and think

that he is right even when he is wrong. It's quite difficult to have an open conversation.

People will not say `I disagree'."

Too great an interest in money is also criticised. A Dutch manager says: "My first impression

on coming to Britain was that profit seemed to be the most important thing." And a

Frenchman points out the difference between the UK and France: "In France there is no

pressure on the bottom line."

The French manager also points out a difference in educational standards: "In France all the

secretaries, or personal assistants as we call them, would have degrees. You wouldn't

consider recruiting one without a degree. So, that means you can delegate much more to

the secretaries in France. In the UK you cannot do that so much."

The Canadian

A. was impressed by the Englishman's good manners.

B. had travelled a long way.

C. liked to do business.

D. considered his time was being wasted.

1) a

2) b

3) c

4) d

5) e

Correct Option
4
is:
Your Option is: 3
Result: Wrong
Timetaken: 0.0 secs
We could answer to this from the details collected from para
Explanation:
2.Option D is the correct answer.

DIRECTIONS: In the questions below have been given in active/passive voice, from the

given alternatives ,choose the one which best expresses the given sentence in passive/active

voice.29)

A lot of goods have been stolen by the customers.


32.
(a)The customers stole a lot of goods.

(b)The customers steal a lot of goods.

(c)The customers have been stealing a lot of goods.

(d)The customers will steal a lot of goods

1) a

2) b

3) c

4) d

Correct Option is: 1


Your Option is: 4
Result: Wrong
Timetaken: 0.0 secs
Explanation: The customers stole a lot of goods
33.

BRITISH MANAGEMENT

According to an article in Management Today, the British manager still stops work for tea.

The French managing director of Novotel, the hotel group, tells what happened at his first

management meeting when he took over its English operation in 1991:

"The meeting was in the afternoon, and it got to about 3.30 and everyone started looking at
their watches. I didn't know what was going on. I turned to my secretary, who was English,

and she said that they probably wanted to break for tea. I couldn't understand it. In France

we just carry on until we have finished. Now we always break for tea."

I am not sure how many English managers now break for tea. Not that many, I imagine.

Twenty years ago it was commonplace. I will never forget a Canadian sales director, pink

with fury, telling me at that time how he had been offered tea and biscuits at three in the

afternoon in Northampton: "I come three thousand miles, and take three taxis and a damn

steam engine into the sticks, and I wanna do business, and this guy gives me a `nice cup of

China tea and a biccy'. What IS he?" Mind you, this was the same man who in Rome told his

Italian agent that the Colosseum would make a nice parking lot.

It is not so much the cup of tea as the pint of bitter that managers from outside Britain find

a bar to efficiency. In the article, a German manager tells how when he first took over a job

in Britain, he discovered that at lunchtime and especially on Fridays, the majority of his

management team left for the pub. He says: "I stopped that right away. Now they are not

allowed off the premises. It didn't make me very popular at the time but it is not good for

efficiency. There is no way we would do that in Germany. No way."

According to European managers the British are still too concerned with class and status. A

German says: "People say that the class system is a hindrance to progress and then two

weeks later you overhear them discussing a colleague and saying, `Well, he is not very well-

spoken, is he?'" And another says: "The class gaps translate into big gulfs in the pay league,

too. In Germany, I might earn three times more than my secretary. Here it is five times."

Some criticisms are rather like those levelled at Japanese management. For example, talking

about status, a Dutch manager says: "A director is God here. They respect him and think

that he is right even when he is wrong. It's quite difficult to have an open conversation.

People will not say `I disagree'."

Too great an interest in money is also criticised. A Dutch manager says: "My first impression

on coming to Britain was that profit seemed to be the most important thing." And a

Frenchman points out the difference between the UK and France: "In France there is no
pressure on the bottom line."

The French manager also points out a difference in educational standards: "In France all the

secretaries, or personal assistants as we call them, would have degrees. You wouldn't

consider recruiting one without a degree. So, that means you can delegate much more to

the secretaries in France. In the UK you cannot do that so much."

In the past

A. managers usually had a teabreak.

B. offered visitors China tea.

C. wasted a lot of time drinking tea.

D. drank a lot of tea.

1) a

2) b

3) c

4) d

5) e

Correct Option
1
is:
Your Option is: 1
Result: Correct
Timetaken: 0.0 secs
From para 2,they have clearly mentioned that managers usually had
Explanation:
a tea break.So,option A is the correct answer.
34.
DIRECTIONS: In the following questions choose the word which best expresses the meaning

of the given word. INSOLENT

(a) audacious

(b) generous

(c) deferential

(d) detrimental
1) a

2) b

3) c

4) d

Correct Option is: 1


Your Option is: 4
Result: Wrong
Timetaken: 0.0 secs
Explanation: showing a rude and arrogant lack of respect.

DIRECTIONS

Order the sentences to form a coherent passage

A)The girl gets a new home and the boy gets a wife,a human being to keep his home;both

get a family without which they cannot have self-fulfilment.

B)The argument of the supporters of dowry-that dowry is the means by which the daughter

gets a share of her father?s property-is meaningless and hallow.


35.
C)The system of dowry is definitely an evil.

D)The needs of both are equal and are equally supplied by both.

a)CDBA

b)BADC

C)CBAD

d)BACD

e)BCAD.

1) a

2) b
3) c

4) d

5) e

Correct Option
3
is:
Your Option is: 3
Result: Correct
Timetaken: 0.0 secs
The choice start with B or C.
However,omly C can start the argument as it states the subject-the
Explanation:
system of dowry.
B follows C.Only,choice 3 has this combination.

Directions: For each of the following questions, there are two words and five alternative

answers. In each of the alternative answers there are two words. Then find out the

alternative whose two words have the same relation as in the two words given in question
36.
Explore : Discover

(a) read : skim

(b) research : learn

(c) write : print

(d) think : relate

1) a

2) b

3) c

4) d

Correct Option is: 4


Your Option is: 2
Result: Wrong
Timetaken: 0.0 secs
Explanation: think : relate

HACKNEYED

(a) trite
37.
(b) equine

(c) serrated

(d) jointed

1) a

2) b

3) c

4) d

Correct Option is: 1


Your Option is: 4
Result: Wrong
Timetaken: 0.0 secs
Explanation: (of a phrase or idea) having been overused; unoriginal and trite.

br>DIRECTIONS:

Each sentence has five parts.One of them has a mistake.Identify that part as your answer.

(1)I ordered for a cup of tea


38.
(2)but the bearer

(3)brought it only

(4)at the end of the tea-session

(5)No Error.

1) a

2) b

3) c
4) d

5) e

Correct Option
1
is:
Your Option is: 1
Result: Correct
Timetaken: 0.0 secs
'Ordered' means 'called for',therefore,'ordered for' is not correct.No
Explanation:
preposition is required after 'ordered'.

DIRECTIONS: In the following questions choose the word which best expresses the meaning

of the given word.

ACQUIESCE
39.
(a) acknowledge

(b) restate

(c) consent

(d) interfere

1) a

2) b

3) c

4) d

Correct Option is: 1


Your Option is: 4
Result: Wrong
Timetaken: 0.0 secs
Explanation: accept something reluctantly but without protest.
40.
DIRECTIONS: In the following questions choose the word which the exact opposite of the

given words.
PSYCHOTIC

(a) holy

(b) earthly

(c) sane

(d) physical

1) a

2) b

3) c

4) d

Correct Option is: 4


Your Option is: 2
Result: Wrong
Timetaken: 0.0 secs
psychotic means relating to, denoting, or suffering from a
Explanation:
psychosis.

DIRECTIONS: In the following questions choose the word which the exact opposite of the

given words.

PULCHRITUDE

(a) austerity
41.
(b) ugliness

(c) inerrancy

(d) beatitude

1) a

2) b
3) c

4) d

Correct Option is: 2


Your Option is: 1
Result: Wrong
Timetaken: 0.0 secs
Explanation: Meaning:beauty.

DIRECTIONS

Order the sentences to form a coherent passage

A)The carribean crisis which had the potential of starting the 3rd world war was successfully

averted.

B)The U.N.O has succeeded in preventing war and maintaining peace in most cases.

C)War was ended between India and Pakistan and peace was finally established in this part
42.
of the world.

D)It deployed its forces to maintain peace in Korea and succeeded in its mission.

a)BACD

b)ADCB

c)ADBC

d)BDAC

e)ACBD.

1) a

2) b

3) c

4) d

5) e
Correct
4
Option is:
Your Option
4
is:
Result: Correct
Timetaken: 0.0 secs
Look at the choices and notice that either A or B is the opening
sentence.
Explanation: We can rule out D as the concluding statement.Therefore,options 1
and 5 are ruled out.D cannot follow A.So options 2 and 3 are also
ruled out.
43.

READING COMPREHENSION.

BRITISH MANAGEMENT

According to an article in Management Today, the British manager still stops work for tea.

The French managing director of Novotel, the hotel group, tells what happened at his first

management meeting when he took over its English operation in 1991:

"The meeting was in the afternoon, and it got to about 3.30 and everyone started looking at

their watches. I didn't know what was going on. I turned to my secretary, who was English,

and she said that they probably wanted to break for tea. I couldn't understand it. In France

we just carry on until we have finished. Now we always break for tea."

I am not sure how many English managers now break for tea. Not that many, I imagine.

Twenty years ago it was commonplace. I will never forget a Canadian sales director, pink

with fury, telling me at that time how he had been offered tea and biscuits at three in the

afternoon in Northampton: "I come three thousand miles, and take three taxis and a damn

steam engine into the sticks, and I wanna do business, and this guy gives me a `nice cup of

China tea and a biccy'. What IS he?" Mind you, this was the same man who in Rome told his

Italian agent that the Colosseum would make a nice parking lot.

It is not so much the cup of tea as the pint of bitter that managers from outside Britain find

a bar to efficiency. In the article, a German manager tells how when he first took over a job

in Britain, he discovered that at lunchtime and especially on Fridays, the majority of his
management team left for the pub. He says: "I stopped that right away. Now they are not

allowed off the premises. It didn't make me very popular at the time but it is not good for

efficiency. There is no way we would do that in Germany. No way."

According to European managers the British are still too concerned with class and status. A

German says: "People say that the class system is a hindrance to progress and then two

weeks later you overhear them discussing a colleague and saying, `Well, he is not very well-

spoken, is he?'" And another says: "The class gaps translate into big gulfs in the pay league,

too. In Germany, I might earn three times more than my secretary. Here it is five times."

Some criticisms are rather like those levelled at Japanese management. For example, talking

about status, a Dutch manager says: "A director is God here. They respect him and think

that he is right even when he is wrong. It's quite difficult to have an open conversation.

People will not say `I disagree'."

Too great an interest in money is also criticised. A Dutch manager says: "My first impression

on coming to Britain was that profit seemed to be the most important thing." And a

Frenchman points out the difference between the UK and France: "In France there is no

pressure on the bottom line."

The French manager also points out a difference in educational standards: "In France all the

secretaries, or personal assistants as we call them, would have degrees. You wouldn't

consider recruiting one without a degree. So, that means you can delegate much more to

the secretaries in France. In the UK you cannot do that so much."

In the middle of the afternoon

A. everyone checked their watches.

B. hinted that it was teatime.

C. took a break.

D. noticed it was 3.30.

1) a

2) b

3) c
4) d

5) e

Correct Option is: 1


Your Option is: 1
Result: Correct
Timetaken: 0.0 secs
From para 2,one could the answer.The correct answer is option
Explanation:
A.

DIRECTIONS: Choose the best option which should replace the phrase printed in ITALICS to

make the sentence Grammatically correct.

The capacity of embryonic stem cells to develop into most type of human tissue which

makes them potentially valuable for medical applications, but the genetic program that
44.
underlies this quality is not yet known.

(a) tissue, which makes them

(b) tissue, makes them

(c) tissues, which make them

(d) tissue, making them

1) a

2) b

3) c

4) d

Correct Option is: 2


Your Option is: 2
Result: Correct
Timetaken: 0.0 secs
Explanation: tissue, makes them

Directions: Choose the best option which should replace the phrase printed in ITALICS to

make the sentence

One of every two new business fail within two years.


45.
(a) has failed

(b) fails

(c) may have failed

(d) should fail

1) a

2) b

3) c

4) d

Correct Option is: 1


Your Option is: 1
Result: Correct
Timetaken: 0.0 secs
Explanation: has failed

DIRECTIONS: In the following questions choose the word which the exact opposite of the

given words.

PUSILLANIMOUS

(a) fragrant
46.
(b) masculine

(c) inopportune

(d) courageous

1) a
2) b

3) c

4) d

Correct Option is: 4


Your Option is: 4
Result: Correct
Timetaken: 0.0 secs
Explanation: Meaning : showing a lack of courage or determination; timid.

DIRECTIONS: In the questions below have been given in active/passive voice, from the

given alternatives ,choose the one which best expresses the given sentence in passive/active

voice.26) The wind blow down the trees.


47.
(a)The trees were blow down by the wind.

(b)The trees were blown down by the wind.

(c)The wind were blowing down the trees.

(d)The trees are being blown down by the wind.

1) a

2) b

3) c

4) d

Correct Option is: 2


Your Option is: 1
Result: Wrong
Timetaken: 0.0 secs
Explanation: The trees were blown down by the wind.
DIRECTIONS: Each of the following questions has four alternative responses. Select the

correct response

Which one of the following is same as cork, plug, dam?


48.
(a) Obstruction

(b) Drain

(c) Pipe

(d) Tap

1) a

2) b

3) c

4) d

Correct Option is: 1


Your Option is: 4
Result: Wrong
Timetaken: 0.0 secs
Explanation: Obstruction

DIRECTIONS: In the questions below have been given in active/passive voice, from the

given alternatives ,choose the one which best expresses the given sentence in passive/active

voice
49.
This house was built by John Mathews in 1991. (a)In 1991, John Mathews built this house.

(b)John Mathews ,in 1991 built this house.

(c)John Mathews built this house in 1991.

(d)John Mathews in 1991 built this house.


1) a

2) b

3) c

4) d

Correct Option is: 3


Your Option is: 1
Result: Wrong
Timetaken: 0.0 secs
Explanation: John Mathews built this house in 1991.

Directions: For each of the following questions, there are two words and five alternative

answers. In each of the alternative answers there are two words. Then find out the

alternative whose two words have the same relation as in the two words given in question.
50.
Resignation : Office :

(a) Competition: Game

(b) Illness: Hospital

(c) Abdication : Throne

(d) Moisture : Rain

1) a

2) b

3) c

4) d

Correct Option is: 3


Your Option is: 4
Result: Wrong
Timetaken: 0.0 secs
Explanation: Abdication : Throne

br>DIRECTIONS:

Each sentence has five parts.One of them has a mistake.Identify that part as your answer.

(1)Shakespeare was
51.
(2)better than any poet

(3)of his days

(4)according to critics

(5)No Error.

1) a

2) b

3) c

4) d

5) e

Correct Option
2
is:
Your Option is: 3
Result: Wrong
Timetaken: 0.0 secs
'.........better than any other poet.......'correct form of
Explanation:
comparision.'than any other'

Which one of the following is same as Jute, Cotton and Wool?

(a) Nylon
52.
(b) Rayon

(c) Silk

(d) Terylene

1) a

2) b
3) c

4) d

Correct Option is: 3


Your Option is: 4
Result: Wrong
Timetaken: 0.0 secs
Explanation: Silk

DIRECTIONS: In the questions below the sentences are been given in the direct/indirect

speech from the given alternatives , choose the one which best expresses the given

sentence in indirect/direct speech. The king was impressed with the magician and asked ,?
53.
what can I do for you??

(a) The king was impressed with the magician and asked what he can do for him.

(b) The king being impressed with the magician and asked what he could do for him.

(c)The king was impressed with the magician and asked what he could do for him.

1) a

2) b

3) c

Correct Option
2
is:
Your Option is: 1
Result: Wrong
Timetaken: 0.0 secs
Explanation: The king being impressed with the magician and asked what he
could do for him.

DIRECTIONS

Select the correct option of words from the choices that completed the given sentence as

your answer.

Peace of mind is not something which you can ___________.


54.
a)find easily when you are troubled.

b)understand as a mature human being.

c)overrule with any stupid excuse.

d)find and take for yourself.

e)even in chaotic surroundings.

1) a

2) b

3) c

4) d

5) e

Correct
1
Option is:
Your Option
4
is:
Result: Wrong
Timetaken: 0.0 secs
The narrator intends to say that peace of mind is not easy to
Explanation: find.Option (1),which maintains the tone of peace of mind not being
a thing which can be got easily',is the right answer
55.
Choose the best option which should replace the phrase printed in ITALICS to make the

sentence

Female sparrows and immature are quite non-descript compared to the adult male
sparrow's distinctive and conspicuous markings.

(a) those of adult males, their

(b) the adult male, which has

(c) adult male sparrow's

(d) adult males and their

1) a

2) b

3) c

4) d

Correct Option is: 2


Your Option is: 3
Result: Wrong
Timetaken: 0.0 secs
Explanation: the adult male, which has
56.

BRITISH MANAGEMENT.

According to an article in Management Today, the British manager still stops work for tea.

The French managing director of Novotel, the hotel group, tells what happened at his first

management meeting when he took over its English operation in 1991: .

"The meeting was in the afternoon, and it got to about 3.30 and everyone started looking at

their watches. I didn't know what was going on. I turned to my secretary, who was English,

and she said that they probably wanted to break for tea. I couldn't understand it. In France

we just carry on until we have finished. Now we always break for tea." .

I am not sure how many English managers now break for tea. Not that many, I imagine.

Twenty years ago it was commonplace. I will never forget a Canadian sales director, pink

with fury, telling me at that time how he had been offered tea and biscuits at three in the
afternoon in Northampton: "I come three thousand miles, and take three taxis and a damn

steam engine into the sticks, and I wanna do business, and this guy gives me a `nice cup of

China tea and a biccy'. What IS he?" Mind you, this was the same man who in Rome told his

Italian agent that the Colosseum would make a nice parking lot.

It is not so much the cup of tea as the pint of bitter that managers from outside Britain find

a bar to efficiency. In the article, a German manager tells how when he first took over a job

in Britain, he discovered that at lunchtime and especially on Fridays, the majority of his

management team left for the pub. He says: "I stopped that right away. Now they are not

allowed off the premises. It didn't make me very popular at the time but it is not good for

efficiency. There is no way we would do that in Germany. No way." .

According to European managers the British are still too concerned with class and status. A

German says: "People say that the class system is a hindrance to progress and then two

weeks later you overhear them discussing a colleague and saying, `Well, he is not very well-

spoken, is he?'" And another says: "The class gaps translate into big gulfs in the pay league,

too. In Germany, I might earn three times more than my secretary. Here it is five times." .

Some criticisms are rather like those levelled at Japanese management. For example, talking

about status, a Dutch manager says: "A director is God here. They respect him and think

that he is right even when he is wrong. It's quite difficult to have an open conversation.

People will not say `I disagree'." .

Too great an interest in money is also criticised. A Dutch manager says: "My first impression

on coming to Britain was that profit seemed to be the most important thing." And a

Frenchman points out the difference between the UK and France: "In France there is no

pressure on the bottom line." .

The French manager also points out a difference in educational standards: "In France all the

secretaries, or personal assistants as we call them, would have degrees. You wouldn't

consider recruiting one without a degree. So, that means you can delegate much more to

the secretaries in France. In the UK you cannot do that so much." .

Why did the Canadian say the Colosseum would make a nice parking lot? .
A. He had nowhere to park.

B. He was joking.

C. He was worried about Rome's traffic problems.

D. He was envious of Italy's glorious past.

1) a

2) b

3) c

4) d

5) e

Correct Option
2
is:
Your Option is: 2
Result: Correct
Timetaken: 0.0 secs
From para 2 we can answer to this question.He was just
Explanation:
joking.So,Option B is the correct answer.

DIRECTIONS: Each of the following questions has four alternative responses. Select the

correct response

Which one of the following is same as uncle, aunt, brother?


57.
(a) Child

(b) Boy

(c) Adult

(d) Nephew

1) a

2) b

3) c

4) d
Correct Option is: 4
Your Option is: 2
Result: Wrong
Timetaken: 0.0 secs
Explanation: Nephew

DIRECTIONS

Order the sentences to form a coherent passage

A)If we want to make our democracy stable there should be some checks upon the people in

power.

B)We do not have a healthy opposition and this gap can be covered only by a free and bold

press.

C)In India democracy is still passing through its infantile stage suffering from a lot of ups
58.
and downs.

D)The leaders generally become very ambitious and so they try to gain power as much as is

possible.

a)CABD

b)CADB

c)DACB

d)DBCA

e)ACDB.

1) a

2) b

3) c

4) d

5) e
Correct Option
2
is:
Your Option is: 3
Result: Wrong
Timetaken: 0.0 secs
Statement C can precede A but C cannot follow A as A explains
what is stated in C.The remaining choices indicate that either C or D
is the first sentence.
Explanation: However the word 'the' in D indicates that this is not the first time
that leaders are being mentionrd.
So only C can start a sentence.A follows C.Also Dfollows A
because of the 'check upon the people in power'

br>DIRECTIONS:

Each sentence has five parts.One of them has a mistake.Identify that part as your answer.

(1)The first lesson I got


59.
(2)in my military days was that

(3)one should love his country

(4)better than anything else

(5)No Error.

1) a

2) b

3) c

4) d

5) e

Correct Option
3
is:
Your Option is: 2
Result: Wrong
Timetaken: 0.0 secs
'One' should be followed by one or one's pnly and not by
Explanation:
'his'..........one should love one's country.
DIRECTIONS

Order the sentences to form a coherent passage

A)Mahatma Gandhi rightly said that the prosperity of India depends upon the prosperity of

villages.

B)He exhorted the social workers that emanicipation of India lay in the emanicipation of

villages.

C)If we are really interested in national development,villages are to be given the first
60.
priority.

D)India is predominantly a rural society with about 70% of the people living in rural areas

and if things are determined as per majority,it may well be said that India lives in villages.

a)DABC

b)CABD

c)CADB

d)DBAC

e)DACB.

1) a

2) b

3) c

4) d

5) e

Correct Option is: 1


Your Option is: 3
Result: Wrong
Timetaken: 0.0 secs
The choice indicates C or D as the first sentence.
B has to follow A.Therefore either 1 or 2 is the answer.
Explanation:
D is a better opening statement while C is a good closing
statement.
DIRECTIONS:

Each sentence has five parts.One of them has a mistake.Identify that part as your answer.

(1)Before he started out


61.
(2)he had collected

(3)all the informations about the country

(4)he was going to visit

(5)No Error.

1) a

2) b

3) c

4) d

5) e

Correct Option
3
is:
Your Option is: 4
Result: Wrong
Timetaken: 0.0 secs
Information is a collective noun
which means items of knowledge.Collective nouns do not take a
Explanation:
plural form.
Hence,informations is incorrect.
62.

DIRECTIONS

Select the correct option of words from the choices that completed the given sentence as

your answer.

Sam and I were walking along the road when a bus hit _____________.

a)a car coming round the bend.

b)a bucket full of water.


c)two trams together.

d)four of the rickshaws that were sold.

e)a herd cattle grazing in the distant fields.

1) a

2) b

3) c

4) d

5) e

Correct Option
1
is:
Your Option
1
is:
Result: Correct
Timetaken: 0.0 secs
Look at the context of the sentence.A person is narrating an
Explanation: incident.Hence,logically option (1) is correct".......were
walking.......when a bus hit a car.

DIRECTIONS: In the questions below the sentences are been given in the direct/indirect

speech from the given alternatives , choose the one which best expresses the given

sentence in indirect/direct speech.


63.
They asked who he was and what he wanted.

(a)"who do you want ?" They asked him.

(b)"who are you ,sir , and what do you want"? They asked.

(c)"what and when do you want"? They asked.

1) a

2) b
3) c

4) d

Correct Option is: 2


Your Option is: 4
Result: Wrong
Timetaken: 0.0 secs
Explanation: "who are you ,sir , and what do you want"? They asked.

DIRECTIONS: In the questions below have been given in active/passive voice, from the

given alternatives ,choose the one which best expresses the given sentence in passive/active

voice.28)

My friend sent me a letter.


64.
(a)A letter was sent to me by my friend.

(b)I was sent a letter by my friend.

(c)I sent my friend a letter.

(d)My friend sent a letter to me.

1) a

2) b

3) c

4) d

Correct Option is: 2


Your Option is: 3
Result: Wrong
Timetaken: 0.0 secs
Explanation: I was sent a letter by my friend.
DIRECTIONS

Select the correct option of words from the choices that completed the given sentence as

your answer.

The Productive hours of the employees are _____________.


65.
a)wasted in meaningless chatter.

b)as good as any other.

c)designed for creativity.

d)all nearly spent in an office.

e)highly stressful.

1) a

2) b

3) c

4) d

5) e

Correct Option
4
is:
Your Option is: 2
Result: Wrong
Timetaken: 0.0 secs
Since we are talking of 'productive hours'we can rule out
Explanation: 'wasting'(choice 1).
choice 2 and 3 do not make any sense.
66.

BRITISH MANAGEMENT

According to an article in Management Today, the British manager still stops work for tea.

The French managing director of Novotel, the hotel group, tells what happened at his first

management meeting when he took over its English operation in 1991:

"The meeting was in the afternoon, and it got to about 3.30 and everyone started looking at
their watches. I didn't know what was going on. I turned to my secretary, who was English,

and she said that they probably wanted to break for tea. I couldn't understand it. In France

we just carry on until we have finished. Now we always break for tea."

I am not sure how many English managers now break for tea. Not that many, I imagine.

Twenty years ago it was commonplace. I will never forget a Canadian sales director, pink

with fury, telling me at that time how he had been offered tea and biscuits at three in the

afternoon in Northampton: "I come three thousand miles, and take three taxis and a damn

steam engine into the sticks, and I wanna do business, and this guy gives me a `nice cup of

China tea and a biccy'. What IS he?" Mind you, this was the same man who in Rome told his

Italian agent that the Colosseum would make a nice parking lot.

It is not so much the cup of tea as the pint of bitter that managers from outside Britain find

a bar to efficiency. In the article, a German manager tells how when he first took over a job

in Britain, he discovered that at lunchtime and especially on Fridays, the majority of his

management team left for the pub. He says: "I stopped that right away. Now they are not

allowed off the premises. It didn't make me very popular at the time but it is not good for

efficiency. There is no way we would do that in Germany. No way."

According to European managers the British are still too concerned with class and status. A

German says: "People say that the class system is a hindrance to progress and then two

weeks later you overhear them discussing a colleague and saying, `Well, he is not very well-

spoken, is he?'" And another says: "The class gaps translate into big gulfs in the pay league,

too. In Germany, I might earn three times more than my secretary. Here it is five times."

Some criticisms are rather like those levelled at Japanese management. For example, talking

about status, a Dutch manager says: "A director is God here. They respect him and think

that he is right even when he is wrong. It's quite difficult to have an open conversation.

People will not say `I disagree'."

Too great an interest in money is also criticised. A Dutch manager says: "My first impression

on coming to Britain was that profit seemed to be the most important thing." And a

Frenchman points out the difference between the UK and France: "In France there is no
pressure on the bottom line."

The French manager also points out a difference in educational standards: "In France all the

secretaries, or personal assistants as we call them, would have degrees. You wouldn't

consider recruiting one without a degree. So, that means you can delegate much more to

the secretaries in France. In the UK you cannot do that so much."

The Canadian

A. was impressed by the Englishman's good manners.

B. had travelled a long way.

C. liked to do business.

D. considered his time was being wasted.

1) a

2) b

3) c

4) d

5) e

Correct Option
4
is:
Your Option is: 3
Result: Wrong
Timetaken: 0.0 secs
We could answer to this from the details collected from para
Explanation:
2.Option D is the correct answer.
67.

DIRECTIONS: In the questions below have been given in active/passive voice, from the

given alternatives ,choose the one which best expresses the given sentence in passive/active

voice.29)

A lot of goods have been stolen by the customers.


(a)The customers stole a lot of goods.

(b)The customers steal a lot of goods.

(c)The customers have been stealing a lot of goods.

(d)The customers will steal a lot of goods

1) a

2) b

3) c

4) d

Correct Option is: 1


Your Option is: 4
Result: Wrong
Timetaken: 0.0 secs
Explanation: The customers stole a lot of goods
68.

BRITISH MANAGEMENT

According to an article in Management Today, the British manager still stops work for tea.

The French managing director of Novotel, the hotel group, tells what happened at his first

management meeting when he took over its English operation in 1991:

"The meeting was in the afternoon, and it got to about 3.30 and everyone started looking at

their watches. I didn't know what was going on. I turned to my secretary, who was English,

and she said that they probably wanted to break for tea. I couldn't understand it. In France

we just carry on until we have finished. Now we always break for tea."

I am not sure how many English managers now break for tea. Not that many, I imagine.

Twenty years ago it was commonplace. I will never forget a Canadian sales director, pink

with fury, telling me at that time how he had been offered tea and biscuits at three in the

afternoon in Northampton: "I come three thousand miles, and take three taxis and a damn
steam engine into the sticks, and I wanna do business, and this guy gives me a `nice cup of

China tea and a biccy'. What IS he?" Mind you, this was the same man who in Rome told his

Italian agent that the Colosseum would make a nice parking lot.

It is not so much the cup of tea as the pint of bitter that managers from outside Britain find

a bar to efficiency. In the article, a German manager tells how when he first took over a job

in Britain, he discovered that at lunchtime and especially on Fridays, the majority of his

management team left for the pub. He says: "I stopped that right away. Now they are not

allowed off the premises. It didn't make me very popular at the time but it is not good for

efficiency. There is no way we would do that in Germany. No way."

According to European managers the British are still too concerned with class and status. A

German says: "People say that the class system is a hindrance to progress and then two

weeks later you overhear them discussing a colleague and saying, `Well, he is not very well-

spoken, is he?'" And another says: "The class gaps translate into big gulfs in the pay league,

too. In Germany, I might earn three times more than my secretary. Here it is five times."

Some criticisms are rather like those levelled at Japanese management. For example, talking

about status, a Dutch manager says: "A director is God here. They respect him and think

that he is right even when he is wrong. It's quite difficult to have an open conversation.

People will not say `I disagree'."

Too great an interest in money is also criticised. A Dutch manager says: "My first impression

on coming to Britain was that profit seemed to be the most important thing." And a

Frenchman points out the difference between the UK and France: "In France there is no

pressure on the bottom line."

The French manager also points out a difference in educational standards: "In France all the

secretaries, or personal assistants as we call them, would have degrees. You wouldn't

consider recruiting one without a degree. So, that means you can delegate much more to

the secretaries in France. In the UK you cannot do that so much."

In the past

A. managers usually had a teabreak.


B. offered visitors China tea.

C. wasted a lot of time drinking tea.

D. drank a lot of tea.

1) a

2) b

3) c

4) d

5) e

Correct Option
1
is:
Your Option is: 1
Result: Correct
Timetaken: 0.0 secs
From para 2,they have clearly mentioned that managers usually had
Explanation:
a tea break.So,option A is the correct answer.

DIRECTIONS: In the following questions choose the word which best expresses the meaning

of the given word. INSOLENT

(a) audacious
69.
(b) generous

(c) deferential

(d) detrimental

1) a

2) b

3) c

4) d
Correct Option is: 1
Your Option is: 4
Result: Wrong
Timetaken: 0.0 secs
Explanation: showing a rude and arrogant lack of respect.

DIRECTIONS

Order the sentences to form a coherent passage

A)The girl gets a new home and the boy gets a wife,a human being to keep his home;both

get a family without which they cannot have self-fulfilment.

B)The argument of the supporters of dowry-that dowry is the means by which the daughter

gets a share of her father?s property-is meaningless and hallow.


70.
C)The system of dowry is definitely an evil.

D)The needs of both are equal and are equally supplied by both.

a)CDBA

b)BADC

C)CBAD

d)BACD

e)BCAD.

1) a

2) b

3) c

4) d

5) e

Correct Option
3
is:
Your Option is: 3
Result: Correct
Timetaken: 0.0 secs
The choice start with B or C.
However,omly C can start the argument as it states the subject-the
Explanation:
system of dowry.
B follows C.Only,choice 3 has this combination.

1.

Directions : The following questions, consist of two words each that have a certain

relationship to each other, followed by four lettered pairs of words. Select the lettered

pair that has the same relationship as the original pair of words.

POLTERGEIST : APPARITION::

A. dwarf : stature

B. witch : familiar

C. ogre : monster

D. sorcerer : spell

1) a

2) b

3) c

4) d

Correct
3
Option is:
Your Option
2
is:
Result: Wrong
Timetaken: 0.0 secs
poltergeist-ghost;apparition-ghost dwarf-short creatures ;stature-
a person's natural height witch-women having evil
Explanation: powers;familiar-well known ogre-giant;monster-giant sorcerer-
witch ;spell-pronounce 'poltergeist','apparition'have the same
meaning. So,the correct answer is option C
2.

Directions : Pick out the most effective word from the given words to fill in the blank to

make the sentence meaningfully complete.

This custom seems to have originated .....................and East European country

A. in

B. from

C. by

D. with

1) a

2) b

3) c

4) d

Correct Option
1
is:
Your Option is: 4
Result: Wrong
Timetaken: 0.0 secs
Tradition has originated from certain place.So,'in' will be the
Explanation:
correct answer.

3.

DIRECTIONS

Rearrange sentences in the correct order to form coherent passage:

A.The use as well as the disposal of chemicals has posed a very serious problem to the

government.

B.But in order to enforce it legally,it must be given legislative sanction.

C.Despite clear printed instructions,these are likely to be misused owing to the lack of

training and proper apparatus.


D.However,a code of conduct regulating the use of chemicals has been evolved and it is

pleaded that it should be observed strictly by those who pursue agriculture for profit.

a)ACBD

b)CADB

c)CBDA

d)ACDB

e)ADBC.

1) a

2) b

3) c

4) d

Correct Option
4
is:
Your Option is: 1
Result: Wrong
Timetaken: 0.0 secs
C cannot start the argument because of the word 'these'.
Explanation: Therefore,options 2 and 3 are ruled out.AlsoD must precede
B.Option 5 is ruled out because D cannot follow A.

4.

DIRECTIONS

Each question gives a sentence with a part of the sentence in bold.Five alternatives for

the bold part are given.One of them is correct.identify it.

Centuries of servility HAS BEEN MAKING HIM tame and passive,incapable of any

resentment or revolt.

A_has been making him.

B.have made him.


C.has made him.

D.has been made him.

E.will have made him.

1) a

2) b

3) c

4) d

5) e

Correct Option is: 3


Your Option is: 4
Result: Wrong
Timetaken: 0.0 secs
Explanation: Centuries of sevility has (already)made him tame

5.

If I was you / I would have / terminated his services then and there / No error

1) a

2) b

3) c

4) d

Correct Option is: 1


Your Option is: 3
Result: Wrong
Timetaken: 0.0 secs
Explanation: It should be as'If I were you'

6.

As soon as I will reach Bombay / I will send / you the books / you have asked for / No

error

1) a

2) b

3) c

4) d

5) e

Correct Option is: 1


Your Option is: 2
Result: Wrong
Timetaken: 0.0 secs
Explanation: The correct answer is'As soon as I reach Bombay'.

7.

DIRECTIONS

Read the passage and answer the questions according to the passage:

NOISE The world ought to be a quieter place. Anti-noise has hit the marketplace. It

means, for example, that refrigerators will no longer whine, vacuum cleaners no longer

roar, and washing machines no longer rumble. It may even soon be possible to cancel

out the low-frequency throb of the teenage stereo blasting its bass through the wall
from the flat next door. Already Toshiba has introduced a silent refrigerator. Built into

the fridge is a system that silences the hum of the motor by firing anti-noise at it.

What is anti-noise? It is the exact opposite of a particular sound. It peaks when the

other sound dips, and dips when the other sound wave peaks. The anti-sound is fired at

the unwanted sound through a loudspeaker. The two waves cancel each other out, like

ripples meeting each other on a pond. The rest is silence.

There must, however, be a perfect match, otherwise you end up with double the din.

Thus anti-noise has had to wait for the microprocessor in order to be effective. The

microprocessor can monitor the unwanted sound and respond in milliseconds.

A typical active noise cancellation system has been sold to a railway company in the

United States. The company unloads grain from trains by using giant vacuum tubes.

These tubes produce as much noise a jet taking off. After installing the system, the

noise was reduced to the level of an air-conditioner's hum. Forty were installed, to the

relief of Americans living near railway unloading points.

In the aircraft industry, active noise cancellation is contributing to a revival in propeller-

driven passenger aircraft, which passengers disliked because of their noise. The first

test flight with an anti-noise system took off in 1989. The system has now move on

from research to commercial development, with passengers likely to hear the benefits

in a year or two.

In the car industry, conventional mufflers generate back-pressure, which forces the

engine to work harder. An active noise canceller removes the need for sound-absorbing

chambers, and improves fuel consumption by as much as six per cent. From the point

of view of saving fuel, car makers are keen to install the technology.

It won't be long before silent washing machines, fridges and vacuum cleaners are in the

High Street shops. Electrolux has signed an agreement to manufacture the technology

which should be on the market by 1994.

The trouble is, once you remove one noise, you become more aware of all the others.

How will we be able to shut up the sparrows at five in the morning?


What anti-noise does is

A. compensate for noise.

B. react with noise.

C. camouflage noise.

D. neutralise noise.

1) a

2) b

3) c

4) d

Correct Option
4
is:
Your Option is: 3
Result: Wrong
Timetaken: 0.0 secs
In the second paragraph it is clear that anti noise will
Explanation:
neutralise noises.

8.

Directions : In each of the following questions, a sentence has been given in Active (or

passive) voice. Out of the four alternatives suggested select the one which best express

the same sentence in Passive (or Active) Voice.

Do you imitate others ?

A.Are others imitated by you ?

B. Are others being imitated by you ?

C. Were others being imitated by you ?

D. Have others been imitated by you ?

1) a
2) b

3) c

4) d

Correct
1
Option is:
Your Option
4
is:
Result: Wrong
Timetaken: 0.0 secs
While changing the voice,we should not change the type of
tense.Options B,C are in the continuous tense.So,eliminate
Explanation:
it.Option D is in perfect tense.Hence,the correct answer is option
A.

9.

Directions : In each of the following questions, a sentence has been given in Active (or

passive) voice. Out of the four alternatives suggested select the one which best express

the same sentence in Passive (or Active) Voice

I saw him conducting the rehearsal.

A. He was seen conducting the rehearsal

B. I saw the rehearsal to be conducted by him

C. He was seen by me to conduct the rehearsal

D. I saw the rehearsal being conducted by him

1) a

2) b

3) c

4) d

Correct Option is: 4


Your Option is: 3
Result: Wrong
Timetaken: 0.0 secs
Explanation: option D is the correct answer.

10

. DIRECTIONS

Select the correct option of words from the choices that completed the given sentence

as your answer.

There was not a single reason why the judge should _____________ A.have adjourned

the court for the day

B.called of the meet for another time

C.continue to be in parliament.

D.take care of law and order

E.take a biased stand.

1) a

2) b

3) c

4) d

5) e

Correct Option
1
is:
Your Option is: 2
Result: Wrong
Timetaken: 0.0 secs
A judge adjourns the court for the day.The words
Explanation:
'Why.......judge'indicate that choice (1)is the best answer.

11

. Directions : Pick out the most effective word from the given words to fill in the blank to
make the sentence meaningfully complete.

Please distribute these sweets........................the children

A. in

B.between

C. amid

D. among

1) a

2) b

3) c

4) d

Correct
4
Option is:
Your Option
1
is:
Result: Wrong
Timetaken: 0.0 secs
Among is the correct answer.Because,in the above sentence they
are distributing the sweets within a group of people.Between can
Explanation:
be used in the case of two persons.So,eliminate B.Amid means
in the centre or middle of.

12

. Directions: Pick out the most effective word from the given words to fill in the blank to

make the sentence meaningfully complete.

While facts are ________ and data hard to come by, even scientists occasionally throw

side the

professional pretense of ______ and tear into each other with shameless appeals to

authority and

arguments that shameless appeals to authority and arguments that are unabashedly ad
hominid.

A. elusive...objectivity

B. establish...courtesy

C. demonstrate .. neutrality

D. ineluctable...cooperation

1) a

2) b

3) c

4) d

5) e

Correct Option
1
is:
Your Option is: 4
Result: Wrong
Timetaken: 0.0 secs
elusive-difficult to find;objectivity-the quality of being
Explanation: objective from the options,one could infer that option A as the
correct answer.Because,

13

. DIRECTIONS

Read the passage and answer the questions according to the passage:

NOISE The world ought to be a quieter place. Anti-noise has hit the marketplace. It

means, for example, that refrigerators will no longer whine, vacuum cleaners no longer

roar, and washing machines no longer rumble. It may even soon be possible to cancel

out the low-frequency throb of the teenage stereo blasting its bass through the wall

from the flat next door. Already Toshiba has introduced a silent refrigerator. Built into

the fridge is a system that silences the hum of the motor by firing anti-noise at it.

What is anti-noise? It is the exact opposite of a particular sound. It peaks when the
other sound dips, and dips when the other sound wave peaks. The anti-sound is fired at

the unwanted sound through a loudspeaker. The two waves cancel each other out, like

ripples meeting each other on a pond. The rest is silence.

There must, however, be a perfect match, otherwise you end up with double the din.

Thus anti-noise has had to wait for the microprocessor in order to be effective. The

microprocessor can monitor the unwanted sound and respond in milliseconds.

A typical active noise cancellation system has been sold to a railway company in the

United States. The company unloads grain from trains by using giant vacuum tubes.

These tubes produce as much noise a jet taking off. After installing the system, the

noise was reduced to the level of an air-conditioner's hum. Forty were installed, to the

relief of Americans living near railway unloading points.

In the aircraft industry, active noise cancellation is contributing to a revival in propeller-

driven passenger aircraft, which passengers disliked because of their noise. The first

test flight with an anti-noise system took off in 1989. The system has now move on

from research to commercial development, with passengers likely to hear the benefits

in a year or two.

In the car industry, conventional mufflers generate back-pressure, which forces the

engine to work harder. An active noise canceller removes the need for sound-absorbing

chambers, and improves fuel consumption by as much as six per cent. From the point

of view of saving fuel, car makers are keen to install the technology.

It won't be long before silent washing machines, fridges and vacuum cleaners are in the

High Street shops. Electrolux has signed an agreement to manufacture the technology

which should be on the market by 1994.

The trouble is, once you remove one noise, you become more aware of all the others.

How will we be able to shut up the sparrows at five in the morning?

On a pop recording, the bass emits a loud low-frequency

A. beat.

B. blast.
C. boom.

D. bang.

1) a

2) b

3) c

4) d

Correct Option
1
is:
Your Option is: 3
Result: Wrong
Timetaken: 0.0 secs
From the para 1,we could get the answer for this
Explanation:
question.Hence option A is the correct answer.

14 AS THEY APPROACHING THE HOUSE the colonel's wife calls him for tea

. a)As they approaching the house.

b)As they will be approaching the house.

c)As they approach the house.

d)As they approach to the house.

e)As they will approach to the house

1) a

2) b

3) c

4) d

5) e

Correct
3
Option is:
Your Option 2
is:
Result: Wrong
Timetaken: 0.0 secs
The given sentence should be in simple present or present
Explanation: continuous.We don't have present continuous.So,the answer
should be in Simple present tense.

15

. Directions : The following questions, consist of two words each that have a certain

relationship to each other, followed by four lettered pairs of words. Select the lettered

pair that has the same relationship as the original pair of words.

AVER : AFFIRMATION : :

A. proclaim: objection

B. denounce : defiance

C. refuse : distress

D. demur : protest

1) a

2) b

3) c

4) d

Correct
4
Option is:
Your Option
3
is:
Result: Wrong
Timetaken: 0.0 secs
aver-state; affirmation-statement proclaim-announce officially;
objection-feeling of disapproval denounce-publicly declare to be
Explanation: wrong; defiance-open resistance refuse-reject; distress-sorrow
demur-raise objections ;protest-objection 'Aver' and 'affirmation'
have similar meaning.So,the correct answer is option D
16

. Directions : In each of the following questions, a sentence has been given in Active (or

passive) voice. Out of the four alternatives suggested select the one which best express

the same sentence in Passive (or Active) Voice.

His pocket has been picked.

A. They have his pocket picked

B. Picking has been done to his pocket

C. Picked has been his pocket

D. Someone has picked his pocket

1) a

2) b

3) c

4) d

Correct Option
4
is:
Your Option
4
is:
Result: Correct
Timetaken: 0.0 secs
In voice conversion,we should not change the tense.Question is
Explanation: in passive voice,while converting this into active.The correct
answer will be option D.

17

. Directions : The following questions, consist of two words each that have a certain

relationship to each other, followed by four lettered pairs of words. Select the lettered

pair that has the same relationship as the original pair of words.

OFFHAND : PREMEDITATION : :

A. upright : integrity
B. aboveboard : guile

C. cutthroat : competition

D. backward :direction

1) a

2) b

3) c

4) d

Correct
2
Option is:
Your Option
2
is:
Result: Correct
Timetaken: 0.0 secs
offhand-without trying or planning; premeditation-planning
upright-;standing with the back straight; integrity-being honest
aboveboard-honest; guile-cunning intelligence cut-throat-
Explanation:
murderer; competition-an event backward-reverse; direction-
guidance 'offhand','premeditation'are opposite in nature.So,the
correct answer is option B

18

. DIRECTIONS

Read the passage and answer the questions according to the passage:

NOISE The world ought to be a quieter place. Anti-noise has hit the marketplace. It

means, for example, that refrigerators will no longer whine, vacuum cleaners no longer

roar, and washing machines no longer rumble. It may even soon be possible to cancel

out the low-frequency throb of the teenage stereo blasting its bass through the wall

from the flat next door. Already Toshiba has introduced a silent refrigerator. Built into

the fridge is a system that silences the hum of the motor by firing anti-noise at it.

What is anti-noise? It is the exact opposite of a particular sound. It peaks when the
other sound dips, and dips when the other sound wave peaks. The anti-sound is fired at

the unwanted sound through a loudspeaker. The two waves cancel each other out, like

ripples meeting each other on a pond. The rest is silence.

There must, however, be a perfect match, otherwise you end up with double the din.

Thus anti-noise has had to wait for the microprocessor in order to be effective. The

microprocessor can monitor the unwanted sound and respond in milliseconds.

A typical active noise cancellation system has been sold to a railway company in the

United States. The company unloads grain from trains by using giant vacuum tubes.

These tubes produce as much noise a jet taking off. After installing the system, the

noise was reduced to the level of an air-conditioner's hum. Forty were installed, to the

relief of Americans living near railway unloading points.

In the aircraft industry, active noise cancellation is contributing to a revival in propeller-

driven passenger aircraft, which passengers disliked because of their noise. The first

test flight with an anti-noise system took off in 1989. The system has now move on

from research to commercial development, with passengers likely to hear the benefits

in a year or two.

In the car industry, conventional mufflers generate back-pressure, which forces the

engine to work harder. An active noise canceller removes the need for sound-absorbing

chambers, and improves fuel consumption by as much as six per cent. From the point

of view of saving fuel, car makers are keen to install the technology.

It won't be long before silent washing machines, fridges and vacuum cleaners are in the

High Street shops. Electrolux has signed an agreement to manufacture the technology

which should be on the market by 1994.

The trouble is, once you remove one noise, you become more aware of all the others.

How will we be able to shut up the sparrows at five in the morning?

Because of anti-noise, propeller-driven aircraft are

A. making a come-back.

B. cropping up.
C. being renovated.

D. becoming fashionable.

1) a

2) b

3) c

4) d

Correct
1
Option is:
Your Option
1
is:
Result: Correct
Timetaken: 0.0 secs
In para 5,they have mentioned about propeller-driven
Explanation: aircraft.Because of anti-noise in the aircraft system,there is a
come-back in that industry.Answer is option A.

19

. DIRECTIONS

Select the correct option of words from the choices that completed the given sentence

as your answer.

In a bid to globalize chinese handset makers have been_____________

A.imitating their counterparts in Japan

B.trying to build a strong product-base in third-generation technology

C.taking recourse to the traditional ideas of the past

D.catering to the interest of the vast chinese population

E.using inferior quality products.

1) a

2) b
3) c

4) d

5) e

Correct Option
2
is:
Your Option is: 2
Result: Correct
Timetaken: 0.0 secs
The key word in the given sentence is 'globalize'.Only choice 2
Explanation:
fits with this.'Imitating<="" td="">

20

. DIRECTIONS

Read the passage and answer the questions according to the passage:

NOISE The world ought to be a quieter place. Anti-noise has hit the marketplace. It

means, for example, that refrigerators will no longer whine, vacuum cleaners no longer

roar, and washing machines no longer rumble. It may even soon be possible to cancel

out the low-frequency throb of the teenage stereo blasting its bass through the wall

from the flat next door. Already Toshiba has introduced a silent refrigerator. Built into

the fridge is a system that silences the hum of the motor by firing anti-noise at it.

What is anti-noise? It is the exact opposite of a particular sound. It peaks when the

other sound dips, and dips when the other sound wave peaks. The anti-sound is fired at

the unwanted sound through a loudspeaker. The two waves cancel each other out, like

ripples meeting each other on a pond. The rest is silence.

There must, however, be a perfect match, otherwise you end up with double the din.

Thus anti-noise has had to wait for the microprocessor in order to be effective. The

microprocessor can monitor the unwanted sound and respond in milliseconds.

A typical active noise cancellation system has been sold to a railway company in the

United States. The company unloads grain from trains by using giant vacuum tubes.

These tubes produce as much noise a jet taking off. After installing the system, the
noise was reduced to the level of an air-conditioner's hum. Forty were installed, to the

relief of Americans living near railway unloading points.

In the aircraft industry, active noise cancellation is contributing to a revival in propeller-

driven passenger aircraft, which passengers disliked because of their noise. The first

test flight with an anti-noise system took off in 1989. The system has now move on

from research to commercial development, with passengers likely to hear the benefits

in a year or two.

In the car industry, conventional mufflers generate back-pressure, which forces the

engine to work harder. An active noise canceller removes the need for sound-absorbing

chambers, and improves fuel consumption by as much as six per cent. From the point

of view of saving fuel, car makers are keen to install the technology.

It won't be long before silent washing machines, fridges and vacuum cleaners are in the

High Street shops. Electrolux has signed an agreement to manufacture the technology

which should be on the market by 1994.

The trouble is, once you remove one noise, you become more aware of all the others.

How will we be able to shut up the sparrows at five in the morning?

After the anti-noise devices were installed, the life of the people living near the railway

was

A. comforted.

B. consoled.

C. enlivened.

D. eased.

1) a

2) b

3) c

4) d
Correct Option
4
is:
Your Option is: 3
Result: Wrong
Timetaken: 0.0 secs
From the information provided in para 4.We could conclude
Explanation: that after installing the anti-noise in the railway.It has made the
life of people ease.

21 'Match' is related to 'Victory' in the same way as 'Examination' is related to

. A.Write

B.Appear

C.Sucess

D.Attempt

E.Prepare

1) a

2) b

3) c

4) d

Correct
3
Option is:
Your Option
4
is:
Result: Wrong
Timetaken: 0.0 secs
The result of match is 'victory',similarly the result of
examination is 'success'.whereas 'write,appear,prepare can be
Explanation:
done but these are not the results.So,we can eliminate theses
options

22

. Directions : Pick out the most effective word from the given words to fill in the blank to
make the sentence meaningfully complete.

While the disease is in ______ state it is almost impossible to determine its existence

by_____

A. a dormant ..postulate

B. a critical...examination

C. an acute ...analysis

D. a latent...observation

1) a

2) b

3) c

4) d

Correct Option
4
is:
Your Option is: 3
Result: Wrong
Timetaken: 0.0 secs
latent-existing but not developed ;observation-
Explanation:
monitoring.Option D is the correct answer.

23

. Directions : Pick out the most effective word from the given words to fill in the blank to

make the sentence meaningfully complete.

The storehouse was infested.........................rats.

A. by

B. of

C. with

D. in
1) a

2) b

3) c

4) d

Correct Option
3
is:
Your Option is: 2
Result: Wrong
Timetaken: 0.0 secs
infest-present in large number.'with' is the correct preposition
Explanation:
for the above question.

24

. Directions : The following questions, consist of two words each that have a certain

relationship to each other, followed by four lettered pairs of words. Select the lettered

pair that has the same relationship as the original pair of words.

Cool : Frigid

A.Livid :Lurid

B.Pool : Placid

C.Tepid : Torrid

D. Lack : Abundant

1) a

2) b

3) c

4) d

Correct
3
Option is:
Your Option
4
is:
Result: Wrong
Timetaken: 0.0 secs
cool-chill;frigid-very cold livid-furiously anger;lurid-unpleasant
pool-a small place of still water; placid-calm tepid-slightly
hot;torrid-very hot lack-not having enough of
Explanation:
something;abundant-plenty The relationship between cool and
frigid is extreme of cold is only frigid.So,Option C is the right
answer.

25

. DIRECTIONS

Select the correct option of words from the choices that completed the given sentence

as your answer.

_______________ was a breathtaking experience for each of us.

A.The movie made for the blind.

B.The scene in the forest

C.Struggling to come out of the crowd.

D.The view from the peak.

E.Standing in the serpentine queue.

1) a

2) b

3) c

4) d

5) e

Correct Option
4
is:
Your Option is: 3
Result: Wrong
Timetaken: 0.0 secs
The expression 'breath-taking experience' indicates amazement
Explanation:
and excitement.Hence Choice (4) is the best fit.

26

. Our conception of / what should a science of mental life be / has changed considerably

since James' time / No error

1) a

2) b

3) c

4) d

Correct Option is: 2


Your Option is: 4
Result: Wrong
Timetaken: 0.0 secs
Explanation: It should be as'what a science of mental life should be'

27

. DIRECTIONS

Read the passage and answer the questions according to the passage:

NOISE The world ought to be a quieter place. Anti-noise has hit the marketplace. It

means, for example, that refrigerators will no longer whine, vacuum cleaners no longer

roar, and washing machines no longer rumble. It may even soon be possible to cancel

out the low-frequency throb of the teenage stereo blasting its bass through the wall

from the flat next door. Already Toshiba has introduced a silent refrigerator. Built into

the fridge is a system that silences the hum of the motor by firing anti-noise at it.
What is anti-noise? It is the exact opposite of a particular sound. It peaks when the

other sound dips, and dips when the other sound wave peaks. The anti-sound is fired at

the unwanted sound through a loudspeaker. The two waves cancel each other out, like

ripples meeting each other on a pond. The rest is silence.

There must, however, be a perfect match, otherwise you end up with double the din.

Thus anti-noise has had to wait for the microprocessor in order to be effective. The

microprocessor can monitor the unwanted sound and respond in milliseconds.

A typical active noise cancellation system has been sold to a railway company in the

United States. The company unloads grain from trains by using giant vacuum tubes.

These tubes produce as much noise a jet taking off. After installing the system, the

noise was reduced to the level of an air-conditioner's hum. Forty were installed, to the

relief of Americans living near railway unloading points.

In the aircraft industry, active noise cancellation is contributing to a revival in propeller-

driven passenger aircraft, which passengers disliked because of their noise. The first

test flight with an anti-noise system took off in 1989. The system has now move on

from research to commercial development, with passengers likely to hear the benefits

in a year or two.

In the car industry, conventional mufflers generate back-pressure, which forces the

engine to work harder. An active noise canceller removes the need for sound-absorbing

chambers, and improves fuel consumption by as much as six per cent. From the point

of view of saving fuel, car makers are keen to install the technology.

It won't be long before silent washing machines, fridges and vacuum cleaners are in the

High Street shops. Electrolux has signed an agreement to manufacture the technology

which should be on the market by 1994.

The trouble is, once you remove one noise, you become more aware of all the others.

How will we be able to shut up the sparrows at five in the morning?

Anti-noise works by being on

A. the same wavelength.


B. a completely different wavelength.

C. a higher-frequency wavelength.

D. a diametrically opposed wavelength.

1) a

2) b

3) c

4) d

Correct Option
4
is:
Your Option is: 3
Result: Wrong
Timetaken: 0.0 secs
From paragraph 2,it is clear that anti-noise neutralises the other
Explanation: noises.So,to neutralise noise the other nose should be of
opposite wavelength.

28

. DIRECTIONS

Read the passage and answer the questions according to the passage:

NOISE The world ought to be a quieter place. Anti-noise has hit the marketplace. It

means, for example, that refrigerators will no longer whine, vacuum cleaners no longer

roar, and washing machines no longer rumble. It may even soon be possible to cancel

out the low-frequency throb of the teenage stereo blasting its bass through the wall

from the flat next door. Already Toshiba has introduced a silent refrigerator. Built into

the fridge is a system that silences the hum of the motor by firing anti-noise at it.

What is anti-noise? It is the exact opposite of a particular sound. It peaks when the

other sound dips, and dips when the other sound wave peaks. The anti-sound is fired at

the unwanted sound through a loudspeaker. The two waves cancel each other out, like
ripples meeting each other on a pond. The rest is silence.

There must, however, be a perfect match, otherwise you end up with double the din.

Thus anti-noise has had to wait for the microprocessor in order to be effective. The

microprocessor can monitor the unwanted sound and respond in milliseconds.

A typical active noise cancellation system has been sold to a railway company in the

United States. The company unloads grain from trains by using giant vacuum tubes.

These tubes produce as much noise a jet taking off. After installing the system, the

noise was reduced to the level of an air-conditioner's hum. Forty were installed, to the

relief of Americans living near railway unloading points.

In the aircraft industry, active noise cancellation is contributing to a revival in propeller-

driven passenger aircraft, which passengers disliked because of their noise. The first

test flight with an anti-noise system took off in 1989. The system has now move on

from research to commercial development, with passengers likely to hear the benefits

in a year or two.

In the car industry, conventional mufflers generate back-pressure, which forces the

engine to work harder. An active noise canceller removes the need for sound-absorbing

chambers, and improves fuel consumption by as much as six per cent. From the point

of view of saving fuel, car makers are keen to install the technology.

It won't be long before silent washing machines, fridges and vacuum cleaners are in the

High Street shops. Electrolux has signed an agreement to manufacture the technology

which should be on the market by 1994.

The trouble is, once you remove one noise, you become more aware of all the others.

How will we be able to shut up the sparrows at five in the morning?

What does the microprocessor do?

A. It checks the sound.

B. It records the sound.

C. It responds to sound.

D. It reacts to low frequencies.


1) a

2) b

3) c

4) d

Correct Option
1
is:
Your Option is: 2
Result: Wrong
Timetaken: 0.0 secs
In para 3,they have insisted that microprocessor reacts to low
Explanation:
frequency.

29

. The Cabinet Ministers and the Prime Minister / was at the airport / to receive the

foreign dignitary / No error

1) a

2) b

3) c

4) d

Correct Option is: 2


Your Option is: 3
Result: Wrong
Timetaken: 0.0 secs
option B is the correct answer.It should be as 'were at the
Explanation:
airport'.

30

. While proceeding on leave / he had orally committed that / he will resume after two

days/ No error

1) a

2) b

3) c

4) d

Correct Option is: 3


Your Option is: 4
Result: Wrong
Timetaken: 0.0 secs
Explanation: The correct answer is 'he would resume after two days'

31

. DIRECTIONS

Read the passage and answer the questions according to the passage:

NOISE The world ought to be a quieter place. Anti-noise has hit the marketplace. It

means, for example, that refrigerators will no longer whine, vacuum cleaners no longer

roar, and washing machines no longer rumble. It may even soon be possible to cancel

out the low-frequency throb of the teenage stereo blasting its bass through the wall

from the flat next door. Already Toshiba has introduced a silent refrigerator. Built into

the fridge is a system that silences the hum of the motor by firing anti-noise at it.
What is anti-noise? It is the exact opposite of a particular sound. It peaks when the

other sound dips, and dips when the other sound wave peaks. The anti-sound is fired at

the unwanted sound through a loudspeaker. The two waves cancel each other out, like

ripples meeting each other on a pond. The rest is silence.

There must, however, be a perfect match, otherwise you end up with double the din.

Thus anti-noise has had to wait for the microprocessor in order to be effective. The

microprocessor can monitor the unwanted sound and respond in milliseconds.

A typical active noise cancellation system has been sold to a railway company in the

United States. The company unloads grain from trains by using giant vacuum tubes.

These tubes produce as much noise a jet taking off. After installing the system, the

noise was reduced to the level of an air-conditioner's hum. Forty were installed, to the

relief of Americans living near railway unloading points.

In the aircraft industry, active noise cancellation is contributing to a revival in propeller-

driven passenger aircraft, which passengers disliked because of their noise. The first

test flight with an anti-noise system took off in 1989. The system has now move on

from research to commercial development, with passengers likely to hear the benefits

in a year or two.

In the car industry, conventional mufflers generate back-pressure, which forces the

engine to work harder. An active noise canceller removes the need for sound-absorbing

chambers, and improves fuel consumption by as much as six per cent. From the point

of view of saving fuel, car makers are keen to install the technology.

It won't be long before silent washing machines, fridges and vacuum cleaners are in the

High Street shops. Electrolux has signed an agreement to manufacture the technology

which should be on the market by 1994.

The trouble is, once you remove one noise, you become more aware of all the others.

How will we be able to shut up the sparrows at five in the morning?

Which title best suits the passage?

A. Towards a Quieter World


B. The Noise-Busters

C. A Better Life

D. Anti-Noise.

1) a

2) b

3) c

4) d

Correct Option
4
is:
Your Option is: 3
Result: Wrong
Timetaken: 0.0 secs
Throughout the passage,the author has talked about reducing
Explanation:
the noise.So,"Anti Noise" will be the suitable title.

32

. DIRECTIONS

Rearrange sentences in the correct order to form coherent passage:

A.They have made progress possible,otherwise there would have been little to

distinguish man from ape.

B.That is why men use tools which make one pair of hands do the work of five or ten

pairs.

C.In the competition of life he,who can do the largest amount of work in the shortest

span of time,wins.

D.The savage who scratches his land with his bare hands has to give way to the man

who makes use of various tools like the plough,the loom and the vehicles.

a)CBDA

b)CBAD
c)BCDA

d)BADC

e)CABD.

1) a

2) b

3) c

4) d

5) e

Correct Option
1
is:
Your Option is: 2
Result: Wrong
Timetaken: 0.0 secs
The choices indicate that we have to look at B or C as the
opening statement.
Explanation:
B cannot start the argument.The words 'that is why' show it is
an explanation.B follows C.D must precede A.

33

. DIRECTIONS

Rearrange sentences in the correct order to form coherent passage:

A.What is called the industrial area is entirely the monopoly of the cities.

B.An important development that has,of late taken place in the countryside is the

phenomenal success of the green revolution.

C.Another aspect of the problem is the confinement of industry to cities and towns.

D.For the expansion of education and literacy in the rural areas it is necessary that

industry should be dispersed in villages also.

a)CBDA

b)CDBA

c)CADB
d)BDAC

e)BDCA.

1) a

2) b

3) c

4) d

5) e

Correct Option
3
is:
Your Option is: 3
Result: Correct
Timetaken: 0.0 secs
The choices start with B or C,D cannot follow B.Therefore
Explanation: option 4 is ruled out.
Also,A has to follow C.D follows A.

34

. Directions : The following questions, consist of two words each that have a certain

relationship to each other, followed by four lettered pairs of words. Select the lettered

pair that has the same relationship as the original pair of words.

Duralumin : Aircraft

A. Brass : Alloy

B. Stone : Sculptor

C.Iron : Steel

D. Bronze : Statue

1) a

2) b

3) c

4) d
Correct
4
Option is:
Your Option
4
is:
Result: Correct
Timetaken: 0.0 secs
duralumin is used in the manufacture of aircraft. brass is an alloy
of copper and zinc. sculptor uses stone to make sculptors. iron-
Explanation:
strong metal; steel-strong metal mixture of iron and carbon.
statues are made up of bronze.So,the correct answer is option D

35

. DIRECTIONS

Read the passage and answer the questions according to the passage:

NOISE The world ought to be a quieter place. Anti-noise has hit the marketplace. It

means, for example, that refrigerators will no longer whine, vacuum cleaners no longer

roar, and washing machines no longer rumble. It may even soon be possible to cancel

out the low-frequency throb of the teenage stereo blasting its bass through the wall

from the flat next door. Already Toshiba has introduced a silent refrigerator. Built into

the fridge is a system that silences the hum of the motor by firing anti-noise at it.

What is anti-noise? It is the exact opposite of a particular sound. It peaks when the

other sound dips, and dips when the other sound wave peaks. The anti-sound is fired at

the unwanted sound through a loudspeaker. The two waves cancel each other out, like

ripples meeting each other on a pond. The rest is silence.

There must, however, be a perfect match, otherwise you end up with double the din.

Thus anti-noise has had to wait for the microprocessor in order to be effective. The

microprocessor can monitor the unwanted sound and respond in milliseconds.

A typical active noise cancellation system has been sold to a railway company in the

United States. The company unloads grain from trains by using giant vacuum tubes.

These tubes produce as much noise a jet taking off. After installing the system, the
noise was reduced to the level of an air-conditioner's hum. Forty were installed, to the

relief of Americans living near railway unloading points.

In the aircraft industry, active noise cancellation is contributing to a revival in propeller-

driven passenger aircraft, which passengers disliked because of their noise. The first

test flight with an anti-noise system took off in 1989. The system has now move on

from research to commercial development, with passengers likely to hear the benefits

in a year or two.

In the car industry, conventional mufflers generate back-pressure, which forces the

engine to work harder. An active noise canceller removes the need for sound-absorbing

chambers, and improves fuel consumption by as much as six per cent. From the point

of view of saving fuel, car makers are keen to install the technology.

It won't be long before silent washing machines, fridges and vacuum cleaners are in the

High Street shops. Electrolux has signed an agreement to manufacture the technology

which should be on the market by 1994.

The trouble is, once you remove one noise, you become more aware of all the others.

How will we be able to shut up the sparrows at five in the morning?

Next year, silent devices will be

A. sold.

B. installed.

C. available.

D. consumed.

1) a

2) b

3) c

4) d

Correct Option
3
is:
Your Option is: 3
Result: Correct
Timetaken: 0.0 secs
From the passage,one could infer that that the silent devices
Explanation:
will be available in next coming years.

1. READING COMPREHENSION

FAT

One day, 43 years old, Walter Hudson walked out of the front door of his home on Long

Island for the first time in 17 years. He sat down on a specially made concrete chair and

told the reporters and TV cameras assembled there, "This is the best day of my life." A bit

unsteady on his feet, having spent the past 28 years in bed, he looked half the man he used

to be: that is, he had cut himself back from 600 kilo to just 300.

Although few Americans aspire to Walter's size, 34 million of them are heavier than they

ought to be. So, a few years ago, when Walter fell out of bed and was stuck on the floor

until the fire brigade came to help him up, there was some general sympathy with his

predicament. Walter explained: "As long as I lay in bed, my size never bothered me. It

never even dawned upon me that I couldn't do what I wanted to until the day I fell and

couldn't get up. That was the complete turn-around that made me want to change my life."

The publicity which resulted changed his life. More than one thousand people contacted him.

He set up a hotline to his home, and now spends a lot of time talking to fellow-sufferers. A

true American, he has marketed his own brand of powdered food formula called "Bio-

Nutrition".

In the case of the severely obese, drastic answers may be necessary. In America, obesity is

seen as an illness, a disease, and treated accordingly. In extreme cases, surgery could be

recommended. Surgery was first used in America during the 50s with an operation to short-

circuit the small bowel in order to prevent the absorption of fats. Weight was lost but some

of the side-effects of the operation turned out to be fatal. In 1981, in Iowa, the first
gastroplasty operation was performed. The stomach was stitched across, horizontally, so

that only half of it could be used. The latest development, an alternative to surgery, is to

have a balloon put in the stomach. There are 20,000 people in America walking around with

balloons in their stomachs. The balloon is made of specially prepared rubber, is inflated in

the stomach and left there for three months. The big danger is deflation and blockage of the

bowels. Otherwise, it's like having permanently just eaten a ham sandwich.

All her life, Barbara Quelch had weighed 140 kilo. She was successful, the director of an

advertising agency and the mother of four children. She had made several attempts to lose

weight, even going to the lengths of having her jaws wired. She explained: "It was very

antisocial and over nine months I cut my weight down by half. However, when my jaws were

unwired, I soon returned to my usual weight. I didn't suffer as much as other fat people. I

had a full and happy life. But I got out of breath very quickly, and was tired and irritable a

lot of the time. Then I had the gastroplasty operation. I lost weight within days. I could only

take fluids for the first two months after the operation, and in the following five months I

lost three kilo a week. I still can't eat certain foods, such as meat and potatoes. Most of my

meals wouldn't fill a saucer. I am often sick after meals, but I don't regret a thing. I have

always wanted to wear modern clothes and now I can go to a shop and choose what I want

instead of having to take something because it fits. I am more assertive now. I used to be

quieter and didn't want to do anything in case it drew attention to my figure. Now I say

what I like. I don't care what people think any more. It's not a miracle. The stomach can

stretch again, usually after three years, so it is important to eat small amounts of food,

often. If this fails there is nothing else. It is a last resort."

Surgery was first used to help people lose weight by

A. removing the small bowel.

B. reducing the size of the stomach.

C. inflating balloons.

D. sending food direct to the large bowel.

1) a
2) b

3) c

4) d

Correct Option
4
is:
Your Option is: 2
Result: Wrong
Timetaken: 0.0 secs
From the third paragraph it is very clear that surgery was first used
Explanation: to help people lose weight by sending food directly to the large
bowel.

2. In each of the following questions, find out which part of the sentence has an error. if there

is no mistake the answer is 'no error' Neither the plans / suits him and therefore / he

decided not to / go out yesterday / No error

1) A

2) B

3) C

4) D

5) E

Correct Option is: 2


Your Option is: 4
Result: Wrong
Timetaken: 0.0 secs
Explanation: It should be suited him and therefore, not suits him.

3. br>READING COMPREHENSION

OZONE

In 1928, Thomas Midgley, a scientist and engineer in America, found a way to use
chlorofluorcarbons (CFCs) as a refrigerant. They proved to be a valuable compound, stable,

non-poisonous, non-corrosive, non-flammable.

Their low thermal conductivity made them ideal for coolants in refrigerators and air-

conditioners. In the Second World War, CFCs were widely used as cleaning solvents and in

plastic foam for food and drink containers, and the insulation of buildings.

These are the products which, doubling in output every ten years, have contributed to the

destruction of the ozone layer, the thin veil in the stratosphere which protects animals and

plants from disease and, possibly, extinction.

Other man-made chemicals, apart from CFCs, are eating ozone molecules. Among them are

halon gases used in fire extinguishers, aerosols, and refrigerators, and two compounds

widely used as solvents: methyl chloroform and carbon tetrachloride. Their combined

reaction on the ozone layer is devastating, allowing ultraviolet rays from the sun, known as

UV-Bs, to bombard the earth.

UV-Bs cause skin cancer. Medical journals in Australia say two-thirds of the population alive

today will develop some form of skin cancer. More than 250,000 of the continent's 16 million

inhabitants will develop the deadliest of all, malignant melanoma.

UV-Bs can damage the immune system and leave you open to infectious diseases. They

damage your eyes, burning the cornea, injuring the retina and generating cataracts. In

southern Chile, blindness has begun to strike humans, sheep, rabbits and horses. The

radiation kills off the plankton on which larger sea creatures depend, and in southern Chile a

12 per cent reduction in plankton has been measured.

The scientists who know about the ozone layer are worried. They give three reasons for

their pessimism:

1. ozone depletion is now general over the globe, and occurring twice as fast as had been

predicted: according to reports, in the first two months of this year, parts of the ozone layer

shrank by 20 per cent, and levels of chlorine, the ozone-eating chemical, were 70 times

higher than normal;

2. depletion is now being caused by the CFCs released in the mid-1970s, and so the
chemicals being released now will endanger our children;

3. no one knows what the cumulative effects of the depletion of the ozone layer will be, but

they do know that things from now on are going to change very quickly indeed.

As CFCs are non-corrosive, they don't

A. poison food.

B. cause explosions.

C. eat metals.

D. damage plastics.

1) a

2) b

3) c

4) d

Correct Option is: 3


Your Option is: 4
Result: Wrong
Timetaken: 0.0 secs
Explanation: As they are non-corrosive,they don't eat metals.

4.

Directions 15-18 : In each of the following questions, a paragraph or a sentence has been

broken up into different parts. The parts have been scrambled and numbered as given

below. Choose the correct order of these parts from the given alternatives.

1) is decidedly harmful

2) disregarding other equally important aspects,

3) to the total neglect of others

4) in the life of a man or a woman

5) is not wisdom but


6) cultivating only one quality

7) giving all attention and energy to one aspect of national life only,

8) folly

9) similarly in the life of a nation.

A) 4,6,2,5,8,9,7,1,3 B) 4,6,3,1,9,7,2,5,8

C) 6,2,4,5,1,9,7,3,8 D) 6,4,2,1,9,7,3,5,8

1) a

2) b

3) c

4) d

Correct
2
Option is:
Your Option
1
is:
Result: Wrong
Timetaken: 0.0 secs
Correct and meaningful sentence is,"In the life of a man or a woman
cultivation only one quality to the total neglect of others is decidedly
Explanation: harmful.Similarly in the life of a nation,giving all attention and
energy to one aspect of national life only, disregarding other equally
important aspects,is not wisdom but folly".So correct option is 2

5. In each of the following questions, find out which part of the sentence has an error. if there

is no mistake the answer is 'no error' The number of people / applying were so large / that

the college had to / stop issuing application forms / No error

1) A

2) B

3) C

4) D
5) E

Correct Option is: 2


Your Option is: 4
Result: Wrong
Timetaken: 0.0 secs
Explanation: It should be were applying, not appplying were

6.

ERROR CORRECTION.

Read the sentence and replace the words in bold with any of the 5 options given.

The sun WAS JUST SETTING over the distant horizon when I heard the roar of an

unexpected motor in the driveway.

a)was just setting.

b)was to be setting.

c)was to just set.

d)just set.

e)just was setting.

1) a

2) b

3) c

4) d

Correct Option is: 1


Your Option is: 2
Result: Wrong
Timetaken: 0.0 secs
Explanation: Past continous tense 'was just setting' is appropriate.

7.
READING COMPREHENSION

OZONE

In 1928, Thomas Midgley, a scientist and engineer in America, found a way to use

chlorofluorcarbons (CFCs) as a refrigerant. They proved to be a valuable compound, stable,

non-poisonous, non-corrosive, non-flammable.

Their low thermal conductivity made them ideal for coolants in refrigerators and air-

conditioners. In the Second World War, CFCs were widely used as cleaning solvents and in

plastic foam for food and drink containers, and the insulation of buildings.

These are the products which, doubling in output every ten years, have contributed to the

destruction of the ozone layer, the thin veil in the stratosphere which protects animals and

plants from disease and, possibly, extinction.

Other man-made chemicals, apart from CFCs, are eating ozone molecules. Among them are

halon gases used in fire extinguishers, aerosols, and refrigerators, and two compounds

widely used as solvents: methyl chloroform and carbon tetrachloride. Their combined

reaction on the ozone layer is devastating, allowing ultraviolet rays from the sun, known as

UV-Bs, to bombard the earth.

UV-Bs cause skin cancer. Medical journals in Australia say two-thirds of the population alive

today will develop some form of skin cancer. More than 250,000 of the continent's 16 million

inhabitants will develop the deadliest of all, malignant melanoma.

UV-Bs can damage the immune system and leave you open to infectious diseases. They

damage your eyes, burning the cornea, injuring the retina and generating cataracts. In

southern Chile, blindness has begun to strike humans, sheep, rabbits and horses. The

radiation kills off the plankton on which larger sea creatures depend, and in southern Chile a

12 per cent reduction in plankton has been measured.

The scientists who know about the ozone layer are worried. They give three reasons for

their pessimism:

1. ozone depletion is now general over the globe, and occurring twice as fast as had been

predicted: according to reports, in the first two months of this year, parts of the ozone layer
shrank by 20 per cent, and levels of chlorine, the ozone-eating chemical, were 70 times

higher than normal;

2. depletion is now being caused by the CFCs released in the mid-1970s, and so the

chemicals being released now will endanger our children;

3. no one knows what the cumulative effects of the depletion of the ozone layer will be, but

they do know that things from now on are going to change very quickly indeed.

A solvent is used in

A. cooking.

B. lubricating.

C. cleaning.

D. sterilising.

1) a

2) b

3) c

4) d

Correct Option
3
is:
Your Option is: 1
Result: Wrong
Timetaken: 0.0 secs
From the second paragraph,it is very clear that the solvents is
Explanation:
used for cleaning.

8.

ERROR CORRECTION

Read the sentence and replace the words in bold with any of the 5 options given.

Looking back, I CAN RARELY REMEMBER the names of the children who shared my

birthday.
a)can rarely remember.

b)can lately remember.

c)can shortly remember.

d)can scarcely remember.

e)can rightly remember.

1) a

2) b

3) c

4) d

5) e

Correct
4
Option is:
Your Option
4
is:
Result: Correct
Timetaken: 0.0 secs
'I can scarcely remember'.'I can hardly remember' indicate that the
person finds it difficult or impossible to recall."Rarely' has the tone
Explanation:
of 'occasionally' which is not intended.Rightly' means he can
remember,which is not implied.

9. each of the following questions, find out which part of the sentence has an error. if there is

no mistake the answer is 'no error' The Cabinet Ministers and the Prime Minister / was at

the airport / to receive the foreign dignitary / No error

1) A

2) B

3) C

4) D

5) E
Correct Option is: 2
Your Option is: 3
Result: Wrong
Timetaken: 0.0 secs
Explanation: It should be were at the airport, not was

10

. Overlook : Aberration

A) Mitigate : Penitence

B) Condone : Offence

C) Error : Omission

D) Conviction : Criminal

1) a

2) b

3) c

4) d

Correct Option is: 2


Your Option is: 4
Result: Wrong
Timetaken: 0.0 secs
Explanation: First is the act of neglecting the second.

11 READING COMPREHENSION

. FAT

One day, 43 years old, Walter Hudson walked out of the front door of his home on Long

Island for the first time in 17 years. He sat down on a specially made concrete chair and

told the reporters and TV cameras assembled there, "This is the best day of my life." A bit

unsteady on his feet, having spent the past 28 years in bed, he looked half the man he used
to be: that is, he had cut himself back from 600 kilo to just 300.

Although few Americans aspire to Walter's size, 34 million of them are heavier than they

ought to be. So, a few years ago, when Walter fell out of bed and was stuck on the floor

until the fire brigade came to help him up, there was some general sympathy with his

predicament. Walter explained: "As long as I lay in bed, my size never bothered me. It

never even dawned upon me that I couldn't do what I wanted to until the day I fell and

couldn't get up. That was the complete turn-around that made me want to change my life."

The publicity which resulted changed his life. More than one thousand people contacted him.

He set up a hotline to his home, and now spends a lot of time talking to fellow-sufferers. A

true American, he has marketed his own brand of powdered food formula called "Bio-

Nutrition".

In the case of the severely obese, drastic answers may be necessary. In America, obesity is

seen as an illness, a disease, and treated accordingly. In extreme cases, surgery could be

recommended. Surgery was first used in America during the 50s with an operation to short-

circuit the small bowel in order to prevent the absorption of fats. Weight was lost but some

of the side-effects of the operation turned out to be fatal. In 1981, in Iowa, the first

gastroplasty operation was performed. The stomach was stitched across, horizontally, so

that only half of it could be used. The latest development, an alternative to surgery, is to

have a balloon put in the stomach. There are 20,000 people in America walking around with

balloons in their stomachs. The balloon is made of specially prepared rubber, is inflated in

the stomach and left there for three months. The big danger is deflation and blockage of the

bowels. Otherwise, it's like having permanently just eaten a ham sandwich.

All her life, Barbara Quelch had weighed 140 kilo. She was successful, the director of an

advertising agency and the mother of four children. She had made several attempts to lose

weight, even going to the lengths of having her jaws wired. She explained: "It was very

antisocial and over nine months I cut my weight down by half. However, when my jaws were

unwired, I soon returned to my usual weight. I didn't suffer as much as other fat people. I

had a full and happy life. But I got out of breath very quickly, and was tired and irritable a
lot of the time. Then I had the gastroplasty operation. I lost weight within days. I could only

take fluids for the first two months after the operation, and in the following five months I

lost three kilo a week. I still can't eat certain foods, such as meat and potatoes. Most of my

meals wouldn't fill a saucer. I am often sick after meals, but I don't regret a thing. I have

always wanted to wear modern clothes and now I can go to a shop and choose what I want

instead of having to take something because it fits. I am more assertive now. I used to be

quieter and didn't want to do anything in case it drew attention to my figure. Now I say

what I like. I don't care what people think any more. It's not a miracle. The stomach can

stretch again, usually after three years, so it is important to eat small amounts of food,

often. If this fails there is nothing else. It is a last resort."

Which do you think is the most appropriate title 'Problems of

A. Diet

B. Obesity

C. Health

D. Fat

1) a

2) b

3) c

4) d

Correct Option
2
is:
Your Option is: 1
Result: Wrong
Timetaken: 0.0 secs
As the problem occurs due to the obesity is discussed in the
Explanation:
passage,Option 2 is the correct answer

12
. Money : Transaction

A) Life : Death

B) Water : Drink

C) Ideas : Exchange

D) Language : Conversation

1) a

2) b

3) c

4) d

Correct Option
4
is:
Your Option is: 1
Result: Wrong
Timetaken: 0.0 secs
Money is transacted.Similarly,we transact through conversation in
Explanation:
a language.

13

. ERROR CORRECTION

Read the sentence and replace the words in bold with any of the 5 options given.

HE ATE NOTHING and leaves his cottage to go to work.

a) He ate nothing.

b) He eats nothing.

c) He is eating something.

d) He eats anything.

e) He has eaten anything.

1) a
2) b

3) c

4) d

Correct Option is: 2


Your Option is: 4
Result: Wrong
Timetaken: 0.0 secs
Explanation: A habitual action should be in the simple present tense.

14

. Directions 11-12 : In each of the following questions, a sentence has been given in Active

(or passive) voice. Out of the four alternatives suggested select the one which best express

the same sentence in Passive (or Active) Voice.

I know him.

a. He is known by me

b. He was known to me

c. He has been known by me

d. He is known to me

1) a

2) b

3) c

4) d

Correct Option
4
is:
Your Option is: 3
Result: Wrong
Timetaken: 0.0 secs
'He is known to me' is the correct answer which gives the same
Explanation:
meaning of the given sentence.

15

. There is an exception............................every rule.

A) on

B) to

C) in

D) for

1) a

2) b

3) c

4) d

Correct Option is: 2


Your Option is: 2
Result: Correct
Timetaken: 0.0 secs
Explanation: As exception is apart from the rule,option is the correct answer.

16

. FILL IN THE BLANKS

Select the most logical word/words that complete the sentence.

The truck hit twenty people who were _____________.

a)in the middle of a voyage.

b)available at that point of time.

c)crossing the road at the zebra crossing.

d)spending their time leisurely.


e)dining in a restaurant.

1) a

2) b

3) c

4) d

5) e

Correct
3
Option is:
Your Option
4
is:
Result: Wrong
Timetaken: 0.0 secs
The truck hit twenty people who were ........The words who were talk
of the twenty people who were doing something when the truck hit
Explanation:
them.They were crossing the road is more logical than any other
options.

17

. Directions 6-10: The following questions, consist of two words each that have a certain

relationship to each other, followed by four lettered pairs of words. Select the lettered pair

that has the same relationship as the original pair of words.

Illiterate : Uneducated

A) Country : State

B) City : Village

C) Palace : Hut

D) Vision : Sight

1) a

2) b

3) c
4) d

Correct Option
4
is:
Your Option is: 1
Result: Wrong
Timetaken: 0.0 secs
Illiterate is also called as uneducated.Similarly,Vision is also
Explanation:
called as sight.

18

. FILL IN THE BLANKS

Select the most logical word/words that complete the sentence.

The President,in his speech delivered on the occasion of the republic day,promised to

tighten ______________

a)the security of the country,especially on its borders.

b)the economic and political situation of our nation.

c)any amendment of the constitution proposed in the houses.

d)the facilities made available to develop mentally challenged children.

e)stringent rules to improve the law and order situation of the country

1) a

2) b

3) c

4) d

5) e

Correct Option
1
is:
Your Option is: 2
Result: Wrong
Timetaken: 0.0 secs
Option 1 is the most logical because of the words 'promised to
tighten'.
'Tighten the security of the country makes good sense'.
Explanation:
we don't tighten a situation(option 2),tighten an amendment(option
3) or tighten facilities (option 4) or tighten rules to improve law and
order(option 5)

19

. Utopia : English

A) Odyssey : Greek

B) Tulsidas : Sanskrit

C) Monalisa : English

D) Dante : Latin

1) a

2) b

3) c

4) d

Correct Option
1
is:
Your Option is: 4
Result: Wrong
Timetaken: 0.0 secs
Utopia is the famous work of English literature. Odyssey is a
Explanation:
famous work of greek literature.

20

. 1) Pentium 4

2) any

3) conflicts

4) handle
5) It seems

6)can

7)that

8)without

9) it

A) 5, 7, 1, 4, 6, 9, 8, 3, 2

B) 5, 7, 2, 4, 6, 8, 9, 1, 3

C) 5, 7, 1, 4, 6, 9, 8, 2, 3

D) 5, 7, 1, 6, 4, 9, 8, 2, 3

1) a

2) b

3) c

4) d

Correct Option
4
is:
Your Option is: 1
Result: Wrong
Timetaken: 0.0 secs
Correct sentence is, " It seems that Pentium 4 can handle it
Explanation:
without any conflicts".

21

. READING COMPREHENSION

FAT

One day, 43 years old, Walter Hudson walked out of the front door of his home on Long

Island for the first time in 17 years. He sat down on a specially made concrete chair and

told the reporters and TV cameras assembled there, "This is the best day of my life." A bit
unsteady on his feet, having spent the past 28 years in bed, he looked half the man he used

to be: that is, he had cut himself back from 600 kilo to just 300.

Although few Americans aspire to Walter's size, 34 million of them are heavier than they

ought to be. So, a few years ago, when Walter fell out of bed and was stuck on the floor

until the fire brigade came to help him up, there was some general sympathy with his

predicament. Walter explained: "As long as I lay in bed, my size never bothered me. It

never even dawned upon me that I couldn't do what I wanted to until the day I fell and

couldn't get up. That was the complete turn-around that made me want to change my life."

The publicity which resulted changed his life. More than one thousand people contacted him.

He set up a hotline to his home, and now spends a lot of time talking to fellow-sufferers. A

true American, he has marketed his own brand of powdered food formula called "Bio-

Nutrition".

In the case of the severely obese, drastic answers may be necessary. In America, obesity is

seen as an illness, a disease, and treated accordingly. In extreme cases, surgery could be

recommended. Surgery was first used in America during the 50s with an operation to short-

circuit the small bowel in order to prevent the absorption of fats. Weight was lost but some

of the side-effects of the operation turned out to be fatal. In 1981, in Iowa, the first

gastroplasty operation was performed. The stomach was stitched across, horizontally, so

that only half of it could be used. The latest development, an alternative to surgery, is to

have a balloon put in the stomach. There are 20,000 people in America walking around with

balloons in their stomachs. The balloon is made of specially prepared rubber, is inflated in

the stomach and left there for three months. The big danger is deflation and blockage of the

bowels. Otherwise, it's like having permanently just eaten a ham sandwich.

All her life, Barbara Quelch had weighed 140 kilo. She was successful, the director of an

advertising agency and the mother of four children. She had made several attempts to lose

weight, even going to the lengths of having her jaws wired. She explained: "It was very

antisocial and over nine months I cut my weight down by half. However, when my jaws were

unwired, I soon returned to my usual weight. I didn't suffer as much as other fat people. I
had a full and happy life. But I got out of breath very quickly, and was tired and irritable a

lot of the time. Then I had the gastroplasty operation. I lost weight within days. I could only

take fluids for the first two months after the operation, and in the following five months I

lost three kilo a week. I still can't eat certain foods, such as meat and potatoes. Most of my

meals wouldn't fill a saucer. I am often sick after meals, but I don't regret a thing. I have

always wanted to wear modern clothes and now I can go to a shop and choose what I want

instead of having to take something because it fits. I am more assertive now. I used to be

quieter and didn't want to do anything in case it drew attention to my figure. Now I say

what I like. I don't care what people think any more. It's not a miracle. The stomach can

stretch again, usually after three years, so it is important to eat small amounts of food,

often. If this fails there is nothing else. It is a last resort."

Walter Hudson

A. was slim at last.

B. sat in a wheelchair.

C. seemed depressed.

D. wasn't able to walk properly.

1) a

2) b

3) c

4) d

Correct Option
4
is:
Your Option is: 2
Result: Wrong
Timetaken: 0.0 secs
In the first paragraph,given that he walked a bit unsteady on his
Explanation:
feet.SO option 4 is the correct answer
22

. ERROR CORRECTION

Read the sentence and replace the words in bold with any of the 5 options given.

With only two weeks before the party,the last place I WOULD WANTED TO BE was in the

hospital recovering from surgery.

a)would wanted to be.

b)wanted to be.

c)would want myself to be.

d)wanted me to be.

e)will be waning myself to be.

1) a

2) b

3) c

4) d

5) e

Correct Option is: 3


Your Option is: 5
Result: Wrong
Timetaken: 0.0 secs
Explanation: Out of the three choices option (3) is structurally correct.

23 In each of the following questions, find out which part of the sentence has an error. if there

. is no mistake the answer is 'no error' While proceeding on leave / he had orally committed

that / he will resume after two days/No error

1) A

2) B

3) C

4) D
5) E

Correct Option is: 3


Your Option is: 4
Result: Wrong
Timetaken: 0.0 secs
Explanation: It should be would instead of will

24

. READING COMPREHENSION

OZONE

In 1928, Thomas Midgley, a scientist and engineer in America, found a way to use

chlorofluorcarbons (CFCs) as a refrigerant. They proved to be a valuable compound, stable,

non-poisonous, non-corrosive, non-flammable.

Their low thermal conductivity made them ideal for coolants in refrigerators and air-

conditioners. In the Second World War, CFCs were widely used as cleaning solvents and in

plastic foam for food and drink containers, and the insulation of buildings.

These are the products which, doubling in output every ten years, have contributed to the

destruction of the ozone layer, the thin veil in the stratosphere which protects animals and

plants from disease and, possibly, extinction.

Other man-made chemicals, apart from CFCs, are eating ozone molecules. Among them are

halon gases used in fire extinguishers, aerosols, and refrigerators, and two compounds

widely used as solvents: methyl chloroform and carbon tetrachloride. Their combined

reaction on the ozone layer is devastating, allowing ultraviolet rays from the sun, known as

UV-Bs, to bombard the earth.

UV-Bs cause skin cancer. Medical journals in Australia say two-thirds of the population alive

today will develop some form of skin cancer. More than 250,000 of the continent's 16 million

inhabitants will develop the deadliest of all, malignant melanoma.

UV-Bs can damage the immune system and leave you open to infectious diseases. They

damage your eyes, burning the cornea, injuring the retina and generating cataracts. In
southern Chile, blindness has begun to strike humans, sheep, rabbits and horses. The

radiation kills off the plankton on which larger sea creatures depend, and in southern Chile a

12 per cent reduction in plankton has been measured.

The scientists who know about the ozone layer are worried. They give three reasons for

their pessimism:

1. ozone depletion is now general over the globe, and occurring twice as fast as had been

predicted: according to reports, in the first two months of this year, parts of the ozone layer

shrank by 20 per cent, and levels of chlorine, the ozone-eating chemical, were 70 times

higher than normal;

2. depletion is now being caused by the CFCs released in the mid-1970s, and so the

chemicals being released now will endanger our children;

3. no one knows what the cumulative effects of the depletion of the ozone layer will be, but

they do know that things from now on are going to change very quickly indeed.

How rapidly was the use of CFCs increasing?

A. Doubling every decade.

B. Doubling every year.

C. Twice as much each year.

D. Ten per cent every year.

1) a

2) b

3) c

4) d

Correct Option
1
is:
Your Option is: 2
Result: Wrong
Timetaken: 0.0 secs
In the second paragraph,it is given that,use of CFCs increasing
Explanation:
every ten years.So the correct answer is option 1

25 br>READING COMPREHENSION

. FAT

One day, 43 years old, Walter Hudson walked out of the front door of his home on Long

Island for the first time in 17 years. He sat down on a specially made concrete chair and

told the reporters and TV cameras assembled there, "This is the best day of my life." A bit

unsteady on his feet, having spent the past 28 years in bed, he looked half the man he used

to be: that is, he had cut himself back from 600 kilo to just 300.

Although few Americans aspire to Walter's size, 34 million of them are heavier than they

ought to be. So, a few years ago, when Walter fell out of bed and was stuck on the floor

until the fire brigade came to help him up, there was some general sympathy with his

predicament. Walter explained: "As long as I lay in bed, my size never bothered me. It

never even dawned upon me that I couldn't do what I wanted to until the day I fell and

couldn't get up. That was the complete turn-around that made me want to change my life."

The publicity which resulted changed his life. More than one thousand people contacted him.

He set up a hotline to his home, and now spends a lot of time talking to fellow-sufferers. A

true American, he has marketed his own brand of powdered food formula called "Bio-

Nutrition".

In the case of the severely obese, drastic answers may be necessary. In America, obesity is

seen as an illness, a disease, and treated accordingly. In extreme cases, surgery could be

recommended. Surgery was first used in America during the 50s with an operation to short-

circuit the small bowel in order to prevent the absorption of fats. Weight was lost but some

of the side-effects of the operation turned out to be fatal. In 1981, in Iowa, the first

gastroplasty operation was performed. The stomach was stitched across, horizontally, so

that only half of it could be used. The latest development, an alternative to surgery, is to

have a balloon put in the stomach. There are 20,000 people in America walking around with

balloons in their stomachs. The balloon is made of specially prepared rubber, is inflated in
the stomach and left there for three months. The big danger is deflation and blockage of the

bowels. Otherwise, it's like having permanently just eaten a ham sandwich.

All her life, Barbara Quelch had weighed 140 kilo. She was successful, the director of an

advertising agency and the mother of four children. She had made several attempts to lose

weight, even going to the lengths of having her jaws wired. She explained: "It was very

antisocial and over nine months I cut my weight down by half. However, when my jaws were

unwired, I soon returned to my usual weight. I didn't suffer as much as other fat people. I

had a full and happy life. But I got out of breath very quickly, and was tired and irritable a

lot of the time. Then I had the gastroplasty operation. I lost weight within days. I could only

take fluids for the first two months after the operation, and in the following five months I

lost three kilo a week. I still can't eat certain foods, such as meat and potatoes. Most of my

meals wouldn't fill a saucer. I am often sick after meals, but I don't regret a thing. I have

always wanted to wear modern clothes and now I can go to a shop and choose what I want

instead of having to take something because it fits. I am more assertive now. I used to be

quieter and didn't want to do anything in case it drew attention to my figure. Now I say

what I like. I don't care what people think any more. It's not a miracle. The stomach can

stretch again, usually after three years, so it is important to eat small amounts of food,

often. If this fails there is nothing else. It is a last resort."

His life changed because

A. he had a telephone installed.

B. he went into business.

C. people wrote to him.

D. he received a lot of publicity.

1) a

2) b

3) c

4) d
Correct Option
4
is:
Your Option is: 1
Result: Wrong
Timetaken: 0.0 secs
From the second paragraph,given that his life changed because he
Explanation:
received a lot of publicity.So Correct answer is option 4

26 In each of the following questions, find out which part of the sentence has an error. if there

. is no mistake the answer is 'no error' A series of lectures / of Indian philosophy / are

arranged by the University / No error

1) A

2) B

3) C

4) D

5) E

Correct Option is: 3


Your Option is: 4
Result: Wrong
Timetaken: 0.0 secs
Explanation: It should be were instead of are

27

. READING COMPREHENSION

OZONE

In 1928, Thomas Midgley, a scientist and engineer in America, found a way to use

chlorofluorcarbons (CFCs) as a refrigerant. They proved to be a valuable compound, stable,

non-poisonous, non-corrosive, non-flammable.

Their low thermal conductivity made them ideal for coolants in refrigerators and air-
conditioners. In the Second World War, CFCs were widely used as cleaning solvents and in

plastic foam for food and drink containers, and the insulation of buildings.

These are the products which, doubling in output every ten years, have contributed to the

destruction of the ozone layer, the thin veil in the stratosphere which protects animals and

plants from disease and, possibly, extinction.

Other man-made chemicals, apart from CFCs, are eating ozone molecules. Among them are

halon gases used in fire extinguishers, aerosols, and refrigerators, and two compounds

widely used as solvents: methyl chloroform and carbon tetrachloride. Their combined

reaction on the ozone layer is devastating, allowing ultraviolet rays from the sun, known as

UV-Bs, to bombard the earth.

UV-Bs cause skin cancer. Medical journals in Australia say two-thirds of the population alive

today will develop some form of skin cancer. More than 250,000 of the continent's 16 million

inhabitants will develop the deadliest of all, malignant melanoma.

UV-Bs can damage the immune system and leave you open to infectious diseases. They

damage your eyes, burning the cornea, injuring the retina and generating cataracts. In

southern Chile, blindness has begun to strike humans, sheep, rabbits and horses. The

radiation kills off the plankton on which larger sea creatures depend, and in southern Chile a

12 per cent reduction in plankton has been measured.

The scientists who know about the ozone layer are worried. They give three reasons for

their pessimism:

1. ozone depletion is now general over the globe, and occurring twice as fast as had been

predicted: according to reports, in the first two months of this year, parts of the ozone layer

shrank by 20 per cent, and levels of chlorine, the ozone-eating chemical, were 70 times

higher than normal;

2. depletion is now being caused by the CFCs released in the mid-1970s, and so the

chemicals being released now will endanger our children;

3. no one knows what the cumulative effects of the depletion of the ozone layer will be, but

they do know that things from now on are going to change very quickly indeed.
CFCs were used in drink containers because they

A. are easily moulded.

B. are light-weight.

C. don't conduct heat.

D. are stable compounds.

1) a

2) b

3) c

4) d

Correct Option
3
is:
Your Option is: 4
Result: Wrong
Timetaken: 0.0 secs
In the second paragraph,"Their low thermal conductivity .......... and
Explanation:
drink containers".It is clear that correct answer is option 3.

28

. READING COMPREHENSION

OZONE

In 1928, Thomas Midgley, a scientist and engineer in America, found a way to use

chlorofluorcarbons (CFCs) as a refrigerant. They proved to be a valuable compound, stable,

non-poisonous, non-corrosive, non-flammable.

Their low thermal conductivity made them ideal for coolants in refrigerators and air-

conditioners. In the Second World War, CFCs were widely used as cleaning solvents and in

plastic foam for food and drink containers, and the insulation of buildings.

These are the products which, doubling in output every ten years, have contributed to the

destruction of the ozone layer, the thin veil in the stratosphere which protects animals and
plants from disease and, possibly, extinction.

Other man-made chemicals, apart from CFCs, are eating ozone molecules. Among them are

halon gases used in fire extinguishers, aerosols, and refrigerators, and two compounds

widely used as solvents: methyl chloroform and carbon tetrachloride. Their combined

reaction on the ozone layer is devastating, allowing ultraviolet rays from the sun, known as

UV-Bs, to bombard the earth.

UV-Bs cause skin cancer. Medical journals in Australia say two-thirds of the population alive

today will develop some form of skin cancer. More than 250,000 of the continent's 16 million

inhabitants will develop the deadliest of all, malignant melanoma.

UV-Bs can damage the immune system and leave you open to infectious diseases. They

damage your eyes, burning the cornea, injuring the retina and generating cataracts. In

southern Chile, blindness has begun to strike humans, sheep, rabbits and horses. The

radiation kills off the plankton on which larger sea creatures depend, and in southern Chile a

12 per cent reduction in plankton has been measured.

The scientists who know about the ozone layer are worried. They give three reasons for

their pessimism:

1. ozone depletion is now general over the globe, and occurring twice as fast as had been

predicted: according to reports, in the first two months of this year, parts of the ozone layer

shrank by 20 per cent, and levels of chlorine, the ozone-eating chemical, were 70 times

higher than normal;

2. depletion is now being caused by the CFCs released in the mid-1970s, and so the

chemicals being released now will endanger our children;

3. no one knows what the cumulative effects of the depletion of the ozone layer will be, but

they do know that things from now on are going to change very quickly indeed.

Which title would best suit the passage?

A. CFCs: from Coolers to Cancers.

B. The Ozone Layer.

C. UV-B and Skin Cancer.


D. Worries about the Ozone Layer.

1) a

2) b

3) c

4) d

Correct Option
1
is:
Your Option is: 1
Result: Correct
Timetaken: 0.0 secs
As the part of CFCs from coolers to cancers is discussed in the
Explanation:
passage.Option A is the best title for this passage.

29

. FILL IN THE BLANKS

Select the most logical word/words that complete the sentence.

She looked at me __________ and gently put forth her question.

a)with a blank vision.

b)quite terribly.

c)helplessly.

d)independently.

e)mercilessly.

1) a

2) b

3) c

4) d
5) e

Correct
3
Option is:
Your Option
4
is:
Result: Wrong
Timetaken: 0.0 secs
The expression 'looked at me helplessly' is syntactically correct
whereas the other options are not.We do not look at someone with a
Explanation: blank vision(we could have a blank expression).
Also we don't look at someone quite terribly independently or
mercilessly.

30

. Help yourself....................whatever you can use without wasting

A)with

B) to

C) in

D) for

1) a

2) b

3) c

4) d

Correct Option is: 2


Your Option is: 2
Result: Correct
Timetaken: 0.0 secs
Explanation: to is the logically correct answer.

31

. Sailor : Pirate
A) Police : Robbers

B) Lion : Lamb

C) Plant : Fungus

D) Major : Sepoy

1) a

2) b

3) c

4) d

Correct Option
1
is:
Your Option is: 3
Result: Wrong
Timetaken: 0.0 secs
Both belongs to same species, but first is useful while second is
Explanation:
harmful.

32

. The residents celebrated the Independence Day

The Independence Day celebrated by the residents

The Independence Day was celebrated by the residents

The Independence Day has been celebrated by the residents

Celebration of Independence Day was done by the residents

1) a

2) b

3) c

4) d
Correct Option is: 2
Your Option is: 1
Result: Wrong
Timetaken: 0.0 secs
Explanation: nil

33

. Directions 13-15 : Pick out the most effective word from the given words to fill in the blank

to make the sentence meaningfully complete

Even if rains all day I will not be able to .........................my journey

A)put by

B)put out

C) put off

D) put away

1) a

2) b

3) c

4) d

Correct Option
3
is:
Your Option is: 4
Result: Wrong
Timetaken: 0.0 secs
Put off is the logically correct answer,which means 'not able
Explanation:
avoid the journey'

34 READING COMPREHENSION

. FAT

One day, 43 years old, Walter Hudson walked out of the front door of his home on Long
Island for the first time in 17 years. He sat down on a specially made concrete chair and

told the reporters and TV cameras assembled there, "This is the best day of my life." A bit

unsteady on his feet, having spent the past 28 years in bed, he looked half the man he used

to be: that is, he had cut himself back from 600 kilo to just 300.

Although few Americans aspire to Walter's size, 34 million of them are heavier than they

ought to be. So, a few years ago, when Walter fell out of bed and was stuck on the floor

until the fire brigade came to help him up, there was some general sympathy with his

predicament. Walter explained: "As long as I lay in bed, my size never bothered me. It

never even dawned upon me that I couldn't do what I wanted to until the day I fell and

couldn't get up. That was the complete turn-around that made me want to change my life."

The publicity which resulted changed his life. More than one thousand people contacted him.

He set up a hotline to his home, and now spends a lot of time talking to fellow-sufferers. A

true American, he has marketed his own brand of powdered food formula called "Bio-

Nutrition".

In the case of the severely obese, drastic answers may be necessary. In America, obesity is

seen as an illness, a disease, and treated accordingly. In extreme cases, surgery could be

recommended. Surgery was first used in America during the 50s with an operation to short-

circuit the small bowel in order to prevent the absorption of fats. Weight was lost but some

of the side-effects of the operation turned out to be fatal. In 1981, in Iowa, the first

gastroplasty operation was performed. The stomach was stitched across, horizontally, so

that only half of it could be used. The latest development, an alternative to surgery, is to

have a balloon put in the stomach. There are 20,000 people in America walking around with

balloons in their stomachs. The balloon is made of specially prepared rubber, is inflated in

the stomach and left there for three months. The big danger is deflation and blockage of the

bowels. Otherwise, it's like having permanently just eaten a ham sandwich.

All her life, Barbara Quelch had weighed 140 kilo. She was successful, the director of an

advertising agency and the mother of four children. She had made several attempts to lose

weight, even going to the lengths of having her jaws wired. She explained: "It was very
antisocial and over nine months I cut my weight down by half. However, when my jaws were

unwired, I soon returned to my usual weight. I didn't suffer as much as other fat people. I

had a full and happy life. But I got out of breath very quickly, and was tired and irritable a

lot of the time. Then I had the gastroplasty operation. I lost weight within days. I could only

take fluids for the first two months after the operation, and in the following five months I

lost three kilo a week. I still can't eat certain foods, such as meat and potatoes. Most of my

meals wouldn't fill a saucer. I am often sick after meals, but I don't regret a thing. I have

always wanted to wear modern clothes and now I can go to a shop and choose what I want

instead of having to take something because it fits. I am more assertive now. I used to be

quieter and didn't want to do anything in case it drew attention to my figure. Now I say

what I like. I don't care what people think any more. It's not a miracle. The stomach can

stretch again, usually after three years, so it is important to eat small amounts of food,

often. If this fails there is nothing else. It is a last resort."

After Walter fell out of bed,

A. people felt sorry for him.

B. the police were called.

C. the neighbours helped him up.

D. he decided life was better in bed.

1) a

2) b

3) c

4) d

Correct Option
1
is:
Your Option is: 4
Result: Wrong
Timetaken: 0.0 secs
From the second paragraph,it is clear that option 1 is the correct
Explanation:
answer

35 1) Zealand

. 2) islands

3) Australia

4) of

5) new

6) consist

7) both

8) and

9) two

A) 2,4,3,6,5,7,1,8,9

B) 5,1, 8 3,7,6,9,2,4

C) 5,1,8,3,7,6,4,9,2

D) 5,1,8,2,3,7,6,4,9

1) a

2) b

3) c

4) d

Correct Option
3
is:
Your Option is: 3
Result: Correct
Timetaken: 0.0 secs
Correct sentence is,"New Zealand and Australia both consist of
Explanation:
two islands
1.
JUMBLED SENTENCE

Rearrange the following sentences from P to S to form a paragraph.S1 is the first sentence in

the paragraph and S6 is the last sentence.

S1: As a dramatist Rabindranath was not what might be called a success.

P : His dramas were moulded on the lines of the traditional Indian village dramas than the

dramas of modern world.

Q : His plays were more a catalogue of ideas than a vehicle of the expression of action.

R : Actually the drama has always been the life of Indian people, as it deals with legends of

gods and goddesses.

S : Although in his short stories and novels he was able to create living and well defined

characters, he did not seem to be able to do so in dramas.

S6: Therefor, drama forms the essential part of the traditional Indian Culture.

The Proper sequence should be:

A. SRQP

B.QPSR

C. QSPR

D.RSQP

1) a

2) b

3) c

4) d

Correct Option
3
is:
Your Option is: 3
Result: Correct
Timetaken: 0.0 secs
Explanation: S1 and Q is a pair of sentences.The sentence S gives the information
which should follow Q.P and R is a pair of sentences. So correct
answer is option 3.

In the following questions choose the word which the exact opposite of the given words.

PULCHRITUDE
2.
(a) austerity

(b) ugliness

(c) inerrancy

(d) beatitude

1) a

2) b

3) c

4) d

Correct Option is: 2


Your Option is: 2
Result: Correct
Timetaken: 0.0 secs
Pulchritude means beauty
Explanation:
So answer is option 2
3.

In the questions below have been given in active/passive voice, from the given

alternatives ,choose the one which best expresses the given sentence in passive/active

voice.

This house was built by John Mathews in 1991.

(a)In 1991, John Mathews built this house.

(b)John Mathews ,in 1991 built this house.

(c)John Mathews built this house in 1991.


(d)John Mathews in 1991 built this house.

1) a

2) b

3) c

4) d

Correct Option
3
is:
Your Option is: 4
Result: Wrong
Timetaken: 0.0 secs
Only option 3 gives the same meaning of the given sentence,in
Explanation:
active voice
4.

READING COMPREHENSION

There are two theories that have often been used to explain ancient and modern tragedy.

Neither quite explains the complexity of the tragic process or the tragic hero, but each

explains important elements of tragedy, and, because their conclusions are contradictory,

they represent extreme views.

But this theory of tragedy is an oversimplification, primarily because it confuses the tragic

condition with the tragic process: the theory does not acknowledge that fate, in a tragedy,

normally becomes external to the hero only after the tragic process has as a heroism that

creates the splendor and exhilaration that is unique to tragedy.The tragic hero quality of an

honest person, but the external antagonist of the criminal. Secondarily, this theory of

tragedy does not distinguish tragedy from irony. Irony does not need an exceptional central

figure: the original destiny never quite fades out of the tragedy.

As a rule, the more ignoble the hero the sharper the irony, when irony alone is the objective.

It is heroism that creates the splendor and exhilaration that is unique to tragedy. The tragic
hero normally has an extraordinary, often a nearly divine, destiny almost within grasp, and

the glory of the original destiny never quite fades out of the tragedy.

The second theory of tragedy states that the act that sets the tragic process in motion must

be primarily a violation of normal law, whether human or divine; in short, that the tragic

hero must have a flaw that has an essential connection with sin. Again it is true that the

great majority of tragic heroes do possess hubris, or a proud and passionate mind that

seems to make the hero?s downfall morally explicable. But such hubris is only the

precipitating agent of catastrophe, just as in comedy the cause )f the happy ending is

usually some act of humility often performed by a noble character who is meanly disguised.

This theory of tragedy as morally explicable runs into the question of whether an innocent

sufferer in a tragedy, such as Iphigenia, or Socrates in Plato Apology, is a tragic figure. They

are, of course, even though it is not very easy to find crucial moral flaws in them. Cordelia

shows sincerity and high spirit in refusing to flatter her faber, and Cordelia is 30 hanged.

Tragedy, in short, is ambiguous and cannot be reduced to the opposition between human

effort. and external fate, just as it cannot be reduced to the opposition between good and

evil.

Which of the following comparisons of the tragic with the ironic hero is best supported by

information contained in the? passage?

a) A tragic hero's fate is an external condition, but an ironic hero?s fate is an internal one.

b) A tragic hero must be controlled by fate, but an ironic hero cannot be.

c) A tragic hero's moral flaw surprises the, audience, but an ironic hero?s sin does not.

d) A tragic hero and an ironic hero cannot both be virtuous figures in the same tragedy.

e) A tragic hero is usually extraordinary, but an ironic hero may be cowardly or even

villainous

1) a

2) b

3) c
4) d

5) e

Correct Option
5
is:
Your Option
2
is:
Result: Wrong
Timetaken: 0.0 secs
See the last three sentences of para 2.It clearly states that 'irony'does
Explanation: not need an exceptional character.It is heroism that qualifies
tragedy.Hence the answer is option E.

ERROR CORRECTION

Read the sentence and replace the words in bold with any of the 5 options given.

Looking back, I CAN RARELY REMEMBER the names of the children who shared my birthday.
5.
a)can rarely remember.

b)can lately remember.

c)can shortly remember.

d)can scarcely remember.

e)can rightly remember.

1) a

2) b

3) c

4) d

5) e

Correct
4
Option is:
Your Option
2
is:
Result: Wrong
Timetaken: 0.0 secs
'I can scarcely remember','I can hardly remember'indicate that the
person finds it difficult or impossible to recall."Rarely'has the tone of
Explanation:
'occasionaly' which is not intended.'Rightly'means he can
remember,whcih is not implied.

JUMBLED SENTENCE

Rearrange the following sentences from P to S to form a paragraph.S1 is the first sentence in

the paragraph and S6 is the last sentence.

S1: In the middle of one side of the square sits the Chairman of the committee, the most

important person in the room.

P : For a committee is not just a mere collection of individuals.

Q : On him rests much of the responsibility for the success or failure of the committee.
6.
R : While this is happening we have an opportunity to get the 'feel' of this committe.

S : As the meeting opens, he runs briskly through a number of formalities.

S6: From the moment its members meet, it begins to have a sort nebulous life of its own.

The Proper sequence should be:

A. RSQP

B.PQRS

C. SQPR

D.QSRP

1) a

2) b

3) c

4) d

Correct Option
4
is:
Your Option is: 1
Result: Wrong
Timetaken: 0.0 secs
the sentence Q gives the information about the chairman,so it
Explanation: follows S1, sentence R follows S and S6 follows P.So the correct
option is 4

In the following questions choose the word which best expresses the meaning of the given

word.

INSOLENT
7.
(a) audacious

(b) generous

(c) deferential

(d) detrimental

1) a

2) b

3) c

4) d

Correct Option is: 1


Your Option is: 3
Result: Wrong
Timetaken: 0.0 secs
Insolent means unmannerly,rude,insubordinate, disrespectful,etc
Explanation: audacious-unmannerly,rude,etc.
detrimental-harmful,dangerous,bad,etc.
8.

READING COMPREHENSION

Democratic institutions are devices for reconciling social order with individual freedom and

initiative, and for making the immediate power of a country's rulers subject to the ultimate
power of the ruled. The fact that, in Western Europe and America, these devices have

worked, all things considered, ot too badly is proof enough that the eighteenth century

optimists were not entirely wrong. Given a fair chance, I repeat; for the fair chance is an

indispensable prerequisite. No people that pass abruptly from a state of subservience under

the rule of a despot to the completely unfamiliar state of political independence can be said

to have a fair chance of being able to govern itself democratically. Liberalism flourishes in an

atmosphere of prosperity and declines as declining prosperity makes it necessary for the

government to intervene ever more frequently and drastically in the affairs of its subjects.

Over-population and over-organization are two conditions which ... deprive a society of a fair

chance of making democratic institutions work effectively.

We see, then, that there are certain historical, economic, demographic and technological

conditions which make it very hard for Jefferson's rational animals, endowed by nature with

inalienable rights and an innate sense of justice, to exercise their reason, claim their rights

and act justly within a democratically organized society.

We in the West have been supremely fortunate in having been given a fair chance of making

the great experiment in self-government. Unfortunately, it now looks as though, owing to

recent changes in our circumstances, this infinitely precious fair chance were being, little by

little, taken away from us.

The author's attitude to the way democratic institutions have functioned in Western Europe

and America can best be described as

a. Deliberate neutrality

b. Cautious approval

c. Qualified disapproval

d. Wholehearted endorsement

e. Mocking disdain

1) a

2) b

3) c
4) d

5) e

Correct
2
Option is:
Your Option
2
is:
Result: Correct
Timetaken: 0.0 secs
The author thinks that democracy has worked 'not too badly';this
Explanation: suggests that he approves but not strongly enough to make D
correct,and the best,mildly positive answer is B.

In the following questions choose the word which the exact opposite of the given words.

PSYCHOTIC
9.
(a) holy

(b) earthly

(c) sane

(d) healthy

1) a

2) b

3) c

4) d

Correct Option
4
is:
Your Option is: 1
Result: Wrong
Timetaken: 0.0 secs
Psychotic means mad,unstable,mental,etc. SO option 4 is the
Explanation:
correct answer.
Choose the best option which should replace the phrase printed in ITALICS to make the

sentence Grammatically correct.

Female sparrows and immature are quite non-descript compared to "the adult male
10.
sparrow's" distinctive and conspicuous markings.

(a) those of adult males, their

(b) the adult male, which has

(c) adult male sparrow?s

(d) adult males and their

1) a

2) b

3) c

4) d

Correct Option
3
is:
Your Option is: 3
Result: Correct
Timetaken: 0.0 secs
The phrase after the male sparrows give the infomation about the
Explanation: sparrows,So the given phrase must be replaced by "the adult
male,which has".
11.

JUMBLED SENTENCE

Rearrange the following sentences from P to S to form a paragraph.S1 is the first sentence in

the paragraph and S6 is the last sentence.

S1: There is difference between Gandhiji's concept of secularism and that of Nehru's.

P : Nehru's idea of secularism was equal indifference to all religions and bothering about

none of them.
Q : According to Gandhiji, all religions are equally true and each scripture is worthy of

respect.

R : Such secularism which means the rejection of all religions is contrary to our culture and

tradition.

S : In Gandhiji's view, secularism stands for equal respect for all religions.

S6: Instead of doing any good, such secularism can do harm instead of good.

The Proper sequence should be:

A. SQPR

B.PSQR.

C. QSPR

D.PRSQ

1) a

2) b

3) c

4) d

Correct Option
1
is:
Your Option is: 2
Result: Wrong
Timetaken: 0.0 secs
S and Q is a pair of sentences which should follow S1.And P and R
Explanation: is a pair of sentences,should follow S and Q.So the correct answer is
option 1
12.

For each of the following questions, there are two words and five alternative answers. In

each of the alternative answers there are two words. Then find out the alternative whose two

words have the same relation as in the two words given in question.

Resignation : Office :
(a) Competition: Game

(b) Illness: Hospital

(c) Abdication : Throne

(d) Moisture : Rain

1) a

2) b

3) c

4) d

Correct Option
3
is:
Your Option is: 3
Result: Correct
Timetaken: 0.0 secs
In the resignation office is to be left.In the same manner,when
Explanation:
abdication is done,throne is to be left.

In the following questions choose the word which the exact opposite of the given words.

PUSILLANIMOUS
13.
(a) fragrant

(b) masculine

(c) inopportune

(d) courageous

1) a

2) b

3) c

4) d
Correct Option
4
is:
Your Option is: 4
Result: Correct
Timetaken: 0.0 secs
Pusillanimous means lack of courage or determination,timid.
Explanation:
Therefore,correct answer is option 4.

JUMBLED SENTENCE

Rearrange the following sentences from P to S to form a paragraph.S1 is the first sentence in

the paragraph and S6 is the last sentence.

S1: You know my wife, Madhavi, always urged me to give up smoking.

P : I really gave it up.

Q : And so When I went to jail I said to myself I really must give it up, if for no other reason

than of being self-reliant.


14.
R : When I emerged from jail, I wanted to tell her of my great triumph.

S : But when I met her, there she was with a packet of cigarettes.

S6: poor girl!.

The Proper sequence should be:

A. PSRQ

B.SPQR

C. QPRS

D.RSPQ

1) a

2) b

3) c

4) d
Correct
3
Option is:
Your Option
3
is:
Result: Correct
Timetaken: 0.0 secs
Effect of the sentence S1 is discussed in the sentence Q,so Q follows
Explanation: S1.And Q and P is a pair of sentences.R and S is a pair of sentences
which follows P.So answer option is 3

Each of the following questions has four alternative responses. Select the correct response.

Which one of the following is same as Jute, Cotton and Wool?


15.
(a) Nylon

(b) Rayon

(c) Silk

(d) Terylene

1) a

2) b

3) c

4) d

Correct Option is: 3


Your Option is: 1
Result: Wrong
Timetaken: 0.0 secs
Explanation: Jute,cotton,wool and silk all are natural yarns
16.

READING COMPREHENSION

There are two theories that have often been used to explain ancient and modern tragedy.
Neither quite explains the complexity of the tragic process or the tragic hero, but each

explains important elements of tragedy, and, because their conclusions are contradictory,

they represent extreme views.

But this theory of tragedy is an oversimplification, primarily because it confuses the tragic

condition with the tragic process: the theory does not acknowledge that fate, in a tragedy,

normally becomes external to the hero only after the tragic process has as a heroism that

creates the splendor and exhilaration that is unique to tragedy.The tragic hero quality of an

honest person, but the external antagonist of the criminal. Secondarily, this theory of

tragedy does not distinguish tragedy from irony. Irony does not need an exceptional central

figure: the original destiny never quite fades out of the tragedy.

As a rule, the more ignoble the hero the sharper the irony, when irony alone is the objective.

It is heroism that creates the splendor and exhilaration that is unique to tragedy. The tragic

hero normally has an extraordinary, often a nearly divine, destiny almost within grasp, and

the glory of the original destiny never quite fades out of the tragedy.

The second theory of tragedy states that the act that sets the tragic process in motion must

be primarily a violation of normal law, whether human or divine; in short, that the tragic

hero must have a flaw that has an essential connection with sin. Again it is true that the

great majority of tragic heroes do possess hubris, or a proud and passionate mind that

seems to make the hero?s downfall morally explicable. But such hubris is only the

precipitating agent of catastrophe, just as in comedy the cause )f the happy ending is

usually some act of humility often performed by a noble character who is meanly disguised.

This theory of tragedy as morally explicable runs into the question of whether an innocent

sufferer in a tragedy, such as Iphigenia, or Socrates in Plato Apology, is a tragic figure. They

are, of course, even though it is not very easy to find crucial moral flaws in them. Cordelia

shows sincerity and high spirit in refusing to flatter her faber, and Cordelia is 30 hanged.

Tragedy, in short, is ambiguous and cannot be reduced to the opposition between human

effort. and external fate, just as it cannot be reduced to the opposition between good and

evil.
The author contrasts an honest person and a criminal primarily in order to

a) prove that fate cannot be external to the tragic hero.

b) establish a criterion that allows a distinction to be made between irony and tragedy.

c) develop the distinction between the tragic condition and the tragic process.

d) introduce the concept of sin as the cause of tragic action.

e) argue that the theme of omnipotent external fate is shared by comedy and tragedy.

1) a

2) b

3) c

4) d

5) e

Correct Option
3
is:
Your Option is: 1
Result: Wrong
Timetaken: 0.0 secs
The argument laid forth in sentences 2-8 of para 2 is supported by
Explanation:
this illustration.Hence the answer is option C.

In the questions below the sentences are been given in the direct/indirect speech from the

given alternatives , choose the one which best expresses the given sentence in

indirect/direct speech.
17.
The king was impressed with the magician and asked ,'what can I do for you?'

(a) The king was impressed with the magician and asked what he can do for him.

(b) The king being impressed with the magician and asked what he could do for him.

(c)The king was impressed with the magician and asked what he could do for him.

1) a

2) b
3) c

4) d

Correct Option
3
is:
Your Option is: 3
Result: Correct
Timetaken: 0.0 secs
Only option 3 gives the correct meaning and proper sentence
Explanation:
formation.
18.

READING COMPREHENSION

There are two theories that have often been used to explain ancient and modern tragedy.

Neither quite explains the complexity of the tragic process or the tragic hero, but each

explains important elements of tragedy, and, because their conclusions are contradictory,

they represent extreme views.

But this theory of tragedy is an oversimplification, primarily because it confuses the tragic

condition with the tragic process: the theory does not acknowledge that fate, in a tragedy,

normally becomes external to the hero only after the tragic process has as a heroism that

creates the splendor and exhilaration that is unique to tragedy.The tragic hero quality of an

honest person, but the external antagonist of the criminal. Secondarily, this theory of

tragedy does not distinguish tragedy from irony. Irony does not need an exceptional central

figure: the original destiny never quite fades out of the tragedy.

As a rule, the more ignoble the hero the sharper the irony, when irony alone is the objective.

It is heroism that creates the splendor and exhilaration that is unique to tragedy. The tragic

hero normally has an extraordinary, often a nearly divine, destiny almost within grasp, and

the glory of the original destiny never quite fades out of the tragedy.

The second theory of tragedy states that the act that sets the tragic process in motion must

be primarily a violation of normal law, whether human or divine; in short, that the tragic
hero must have a flaw that has an essential connection with sin. Again it is true that the

great majority of tragic heroes do possess hubris, or a proud and passionate mind that

seems to make the hero?s downfall morally explicable. But such hubris is only the

precipitating agent of catastrophe, just as in comedy the cause )f the happy ending is

usually some act of humility often performed by a noble character who is meanly disguised.

This theory of tragedy as morally explicable runs into the question of whether an innocent

sufferer in a tragedy, such as Iphigenia, or Socrates in Plato Apology, is a tragic figure. They

are, of course, even though it is not very easy to find crucial moral flaws in them. Cordelia

shows sincerity and high spirit in refusing to flatter her faber, and Cordelia is 30 hanged.

Tragedy, in short, is ambiguous and cannot be reduced to the opposition between human

effort. and external fate, just as it cannot be reduced to the opposition between good and

evil.

The author states that the theories discussed in the passage ?represent extreme views?

because their conclusions are

(a) unpopular

(b) complex

(c) paradoxical

(d) contradictory.

(e) imaginative

1) a

2) b

3) c

4) d

5) e

Correct Option
4
is:
Your Option is: 4
Result: Correct
Timetaken: 0.0 secs
No tenable middle ground can be sought when two disparate
Explanation:
theories juxtaposed.Refer para 1.Hence the correct option is D.
19.

READING COMPREHENSION

Democratic institutions are devices for reconciling social order with individual freedom and

initiative, and for making the immediate power of a country's rulers subject to the ultimate

power of the ruled. The fact that, in Western Europe and America, these devices have

worked, all things considered, ot too badly is proof enough that the eighteenth century

optimists were not entirely wrong. Given a fair chance, I repeat; for the fair chance is an

indispensable prerequisite. No people that pass abruptly from a state of subservience under

the rule of a despot to the completely unfamiliar state of political independence can be said

to have a fair chance of being able to govern itself democratically. Liberalism flourishes in an

atmosphere of prosperity and declines as declining prosperity makes it necessary for the

government to intervene ever more frequently and drastically in the affairs of its subjects.

Over-population and over-organization are two conditions which ... deprive a society of a fair

chance of making democratic institutions work effectively.

We see, then, that there are certain historical, economic, demographic and technological

conditions which make it very hard for Jefferson's rational animals, endowed by nature with

inalienable rights and an innate sense of justice, to exercise their reason, claim their rights

and act justly within a democratically organized society.

We in the West have been supremely fortunate in having been given a fair chance of making

the great experiment in self-government. Unfortunately, it now looks as though, owing to

recent changes in our circumstances, this infinitely precious fair chance were being, little by

little, taken away from us.

The 'infinitely precious fair chance' highlighted in the last sentence, according to the author

is

a. Unlikely to emerge in an atmosphere of liberalism

b. Incompatible with Jefferson?s views


c. Vitiated in an atmosphere of prosperity

d. An essential precondition for the success of democracy

e. Only possible in a large, advanced and highly organised society

1) a

2) b

3) c

4) d

5) e

Correct
4
Option is:
Your Option
2
is:
Result: Wrong
Timetaken: 0.0 secs
The 'fair chance' is the opportunity for democracy to flourish, and
this is clearly expressed in D, the correct choice. Since the author
mentions liberalism in a way that indicates he equates it with
democracy, we can eliminate A. Jefferson?s views are of people who
are free to exercise their rights and are not ?incompatible with a fair
Explanation:
chance for democracy so B cannot be correct. We are told that
declining prosperity is one reason for governments to intervene more
and is thus not favourable for democracy. Thus, since 'vitiated' means
weakened, answer C cannot be correct. Answer E is worded far too
strongly to be correct.
20.

READING COMPREHENSION

There are two theories that have often been used to explain ancient and modern tragedy.

Neither quite explains the complexity of the tragic process or the tragic hero, but each

explains important elements of tragedy, and, because their conclusions are contradictory,

they represent extreme views.

But this theory of tragedy is an oversimplification, primarily because it confuses the tragic

condition with the tragic process: the theory does not acknowledge that fate, in a tragedy,
normally becomes external to the hero only after the tragic process has as a heroism that

creates the splendor and exhilaration that is unique to tragedy.The tragic hero quality of an

honest person, but the external antagonist of the criminal. Secondarily, this theory of

tragedy does not distinguish tragedy from irony. Irony does not need an exceptional central

figure: the original destiny never quite fades out of the tragedy.

As a rule, the more ignoble the hero the sharper the irony, when irony alone is the objective.

It is heroism that creates the splendor and exhilaration that is unique to tragedy. The tragic

hero normally has an extraordinary, often a nearly divine, destiny almost within grasp, and

the glory of the original destiny never quite fades out of the tragedy.

The second theory of tragedy states that the act that sets the tragic process in motion must

be primarily a violation of normal law, whether human or divine; in short, that the tragic

hero must have a flaw that has an essential connection with sin. Again it is true that the

great majority of tragic heroes do possess hubris, or a proud and passionate mind that

seems to make the hero?s downfall morally explicable. But such hubris is only the

precipitating agent of catastrophe, just as in comedy the cause )f the happy ending is

usually some act of humility often performed by a noble character who is meanly disguised.

This theory of tragedy as morally explicable runs into the question of whether an innocent

sufferer in a tragedy, such as Iphigenia, or Socrates in Plato Apology, is a tragic figure. They

are, of course, even though it is not very easy to find crucial moral flaws in them. Cordelia

shows sincerity and high spirit in refusing to flatter her faber, and Cordelia is 30 hanged.

Tragedy, in short, is ambiguous and cannot be reduced to the opposition between human

effort. and external fate, just as it cannot be reduced to the opposition between good and

evil.

In the author's opinion, an act of humility in comedy is most analogous to

a) a catastrophe in tragedy.

b) an ironic action in tragedy.

c) a tragic hero's pride and passion.

d) a tragic hero's aversion to sin.


e) a tragic hero's pursuit of an unusual destiny.

1) a

2) b

3) c

4) d

5) e

Correct
3
Option is:
Your Option
4
is:
Result: Wrong
Timetaken: 0.0 secs
Refer para 3.The author states the proud and passionate mind of a
Explanation: tragic hero as hubris.he further states that the hubris in a comedy is
'usually some act of humility'.Hence the answer is option C.
21.

JUMBLED SENTENCE

Rearrange the following sentences from P to S to form a paragraph.S1 is the first sentence in

the paragraph and S6 is the last sentence.

S1: For decades, American society has been calling a melting pot.

P : Differences remained - in appearence, mannerisms, customs, speech, religion and more.

Q : The term has long been a cliche and half-truth.

R : But homogenisation was never acheived.

S : Yes, immigrants from diverse cultures and traditions did cast off vestiges of their native

lands and become almost imperceptibly woven in to the American fabric.

S6: In recent years, such differences accentuated by the arrival of immigrants from Asia and

other parts of the world in the United States - have become something to celebrate and to

nurture.

The Proper sequence should be:


A. QRSP

B.SQRP.

C. SQPR

D.QSRP

1) a

2) b

3) c

4) d

Correct Option
2
is:
Your Option is: 1
Result: Wrong
Timetaken: 0.0 secs
S and Q is a pair of sentences and R and P is a pair of sentences.S
Explanation:
follows S1,therefore R follows Q.SO the Correct option is 2

Choose the best option which should replace the phrase printed in ITALICS to make the

sentence Grammatically correct.

The capacity of embryonic stem cells to develop into most type of human "tissue which

makes them" potentially valuable for medical applications, but the genetic program that
22.
underlies this quality is not yet known.

(a) tissue, which makes them

(b) tissue, makes them

(c)tissues, which make them

(d)tissue, making them

1) a

2) b
3) c

4) d

Correct Option
2
is:
Your Option is: 3
Result: Wrong
Timetaken: 0.0 secs
In the given sentence,which should not be used before 'makes
Explanation:
them'.So correct option is 2
23.

READING COMPREHENSION

There are two theories that have often been used to explain ancient and modern tragedy.

Neither quite explains the complexity of the tragic process or the tragic hero, but each

explains important elements of tragedy, and, because their conclusions are contradictory,

they represent extreme views.

But this theory of tragedy is an oversimplification, primarily because it confuses the tragic

condition with the tragic process: the theory does not acknowledge that fate, in a tragedy,

normally becomes external to the hero only after the tragic process has as a heroism that

creates the splendor and exhilaration that is unique to tragedy.The tragic hero quality of an

honest person, but the external antagonist of the criminal. Secondarily, this theory of

tragedy does not distinguish tragedy from irony. Irony does not need an exceptional central

figure: the original destiny never quite fades out of the tragedy.

As a rule, the more ignoble the hero the sharper the irony, when irony alone is the objective.

It is heroism that creates the splendor and exhilaration that is unique to tragedy. The tragic

hero normally has an extraordinary, often a nearly divine, destiny almost within grasp, and

the glory of the original destiny never quite fades out of the tragedy.

The second theory of tragedy states that the act that sets the tragic process in motion must

be primarily a violation of normal law, whether human or divine; in short, that the tragic
hero must have a flaw that has an essential connection with sin. Again it is true that the

great majority of tragic heroes do possess hubris, or a proud and passionate mind that

seems to make the hero?s downfall morally explicable. But such hubris is only the

precipitating agent of catastrophe, just as in comedy the cause )f the happy ending is

usually some act of humility often performed by a noble character who is meanly disguised.

This theory of tragedy as morally explicable runs into the question of whether an innocent

sufferer in a tragedy, such as Iphigenia, or Socrates in Plato Apology, is a tragic figure. They

are, of course, even though it is not very easy to find crucial moral flaws in them. Cordelia

shows sincerity and high spirit in refusing to flatter her faber, and Cordelia is 30 hanged.

Tragedy, in short, is ambiguous and cannot be reduced to the opposition between human

effort. and external fate, just as it cannot be reduced to the opposition between good and

evil.

According to the. author, Cordellia is an example of a figure who

a) transcended both the laws of ?fate and the laws of society.

b) sinned, but whose sin did not set the tragic process in motion.

c) disobeyed a moral law, but was not punished for doing so.

d) submitted willingly to fate, even though her submission caused her death.

e) did not set the tragic process in motion, but is still a tragic figure

1) a

2) b

3) c

4) d

5) e

Correct Option
5
is:
Your Option is: 1
Result: Wrong
Timetaken: 0.0 secs
Cordellia is presented as an innocent victim of fate.Hence the
Explanation:
answer is option E.

Choose the best option which should replace the phrase printed in ITALICS to make the

sentence Grammatically correct.

One of every two new business fail within two years.


24.
(a) has failed

(b) fails

(c) may have failed

(d) should fail

1) a

2) b

3) c

4) d

Correct Option is: 2


Your Option is: 4
Result: Wrong
Timetaken: 0.0 secs
Explanation: As the object is in singular form,only fails must be used
25.

In the questions below have been given in active/passive voice, from the given

alternatives ,choose the one which best expresses the given sentence in passive/active

voice.

The wind blow down the trees.

(a)The trees were blow down by the wind.

(b)The trees were blown down by the wind.

(c)The wind were blowing down the trees.


(d)The trees are being blown down by the wind.

1) a

2) b

3) c

4) d

Correct Option
2
is:
Your Option is: 1
Result: Wrong
Timetaken: 0.0 secs
Only option 2 gives the correct sentence and same meaning of the
Explanation:
given sentence, in passive voice.

In the questions below the sentences are been given in the direct/indirect speech from the

given alternatives , choose the one which best expresses the given sentence in

indirect/direct speech.
26.
They asked who he was and what he wanted.

(a)'who do you want ?' They asked him.

(b)'who are you ,sir , and what do you want?' They asked.

(c)'what and when do you want ?' They asked.

1) a

2) b

3) c

4) d

Correct Option is: 2


Your Option is: 3
Result: Wrong
Timetaken: 0.0 secs
Only option 2 gives the correct meaning of the complete
Explanation:
sentence.

For each of the following questions, there are two words and five alternative answers. In

each of the alternative answers there are two words. Then find out the alternative whose two

words have the same relation as in the two words given in question.
27.
Explore : Discover

(a) read : skim

(b) research : learn

(c) write : print

(d) think : relate

1) a

2) b

3) c

4) d

Correct Option is: 2


Your Option is: 2
Result: Correct
Timetaken: 0.0 secs
Explanation: One explores to discover,one researches to learn.
28.

In the questions below have been given in active/passive voice, from the given

alternatives ,choose the one which best expresses the given sentence in passive/active

voice.

A lot of goods have been stolen by the customers.


(a)The customers stole a lot of goods.

(b)The customers steal a lot of goods.

(c)The customers have been stealing a lot of goods.

(d)The customers will steal a lot of goods .

1) a

2) b

3) c

4) d

Correct Option
1
is:
Your Option is: Not Attempted
Result: Not Attempted
Timetaken: 0.0 secs
Only option 1 gives the same meaning of the given sentence.SO
Explanation:
option 1 is the correct answer.

In the following questions choose the word which best expresses the meaning of the given

word.

HACKNEYED
29.
(a) trite

(b) equine

(c) serrated

(d) jointed

1) a

2) b

3) c

4) d
Correct Option is: 1
Your Option is: 4
Result: Wrong
Timetaken: 0.0 secs
Hackneyed means tired, dull, boring,routine,etc.
Trite- dull,boring,routine,etc
Explanation:
equine-member of horse family
serrated-jagged,sawtooth,zig-zag,etc

In the questions below have been given in active/passive voice, from the given

alternatives ,choose the one which best expresses the given sentence in passive/active

voice.
30.
My friend sent me a letter.

(a)A letter was sent to me by my friend.

(b)I was sent a letter by my friend.

(c)I sent my friend a letter.

(d)My friend sent a letter to me.

1) a

2) b

3) c

4) d

Correct Option
2
is:
Your Option is: 1
Result: Wrong
Timetaken: 0.0 secs
Only option 2 gives the same meaning of the given sntence in the
Explanation:
passive voice.
Each of the following questions has four alternative responses. Select the correct response.

Which one of the following is same as cork, plug, dam?


31.
(a) Obstruction

(b) Drain

(c) Pipe

(d) Tap

1) a

2) b

3) c

4) d

Correct Option is: 1


Your Option is: 4
Result: Wrong
Timetaken: 0.0 secs
Explanation: Obstruction is caused due to cork,plug and dam

In the following questions choose the word which best expresses the meaning of the given

word.

ACQUIESCE
32.
(a) acknowledge

(b) restate

(c) consent

(d) interfere

1) a

2) b
3) c

4) d

Correct Option is: 1


Your Option is: 1
Result: Correct
Timetaken: 0.0 secs
Acquiesce means accept something reluctantly but without
Explanation:
protest.
33.

READING COMPREHENSION

Democratic institutions are devices for reconciling social order with individual freedom and

initiative, and for making the immediate power of a country's rulers subject to the ultimate

power of the ruled. The fact that, in Western Europe and America, these devices have

worked, all things considered, ot too badly is proof enough that the eighteenth century

optimists were not entirely wrong. Given a fair chance, I repeat; for the fair chance is an

indispensable prerequisite. No people that pass abruptly from a state of subservience under

the rule of a despot to the completely unfamiliar state of political independence can be said

to have a fair chance of being able to govern itself democratically. Liberalism flourishes in an

atmosphere of prosperity and declines as declining prosperity makes it necessary for the

government to intervene ever more frequently and drastically in the affairs of its subjects.

Over-population and over-organization are two conditions which ... deprive a society of a fair

chance of making democratic institutions work effectively.

We see, then, that there are certain historical, economic, demographic and technological

conditions which make it very hard for Jefferson's rational animals, endowed by nature with

inalienable rights and an innate sense of justice, to exercise their reason, claim their rights

and act justly within a democratically organized society.

We in the West have been supremely fortunate in having been given a fair chance of making

the great experiment in self-government. Unfortunately, it now looks as though, owing to


recent changes in our circumstances, this infinitely precious fair chance were being, little by

little, taken away from us.

The author's primary purpose is apparently to

a. Explain a requirement and introduce a warning about that requirement.

b. Argue for the limitation of a certain form of government .

c. Define the conditions for social order .

d. Advocate liberalism in government of a certain era

e. Credit certain thinkers with foresight

1) a

2) b

3) c

4) d

5) e

Correct
1
Option is:
Your Option
3
is:
Result: Wrong
Timetaken: 0.0 secs
The 'requirement' in answer A is the need for democracy to be given a
'fair chance' which is what the author keeps reiterating.In the last
sentence he warns that the chance may be being taken away.He is not
Explanation:
arguing to limit democracy(Eliminate B)nor he is defining conditions
for social order in general(Eliminate C).Answers D and E are too
narrow to the main point of the whole extract.
34.

READING COMPREHENSION

There are two theories that have often been used to explain ancient and modern tragedy.

Neither quite explains the complexity of the tragic process or the tragic hero, but each

explains important elements of tragedy, and, because their conclusions are contradictory,
they represent extreme views.

But this theory of tragedy is an oversimplification, primarily because it confuses the tragic

condition with the tragic process: the theory does not acknowledge that fate, in a tragedy,

normally becomes external to the hero only after the tragic process has as a heroism that

creates the splendor and exhilaration that is unique to tragedy.The tragic hero quality of an

honest person, but the external antagonist of the criminal. Secondarily, this theory of

tragedy does not distinguish tragedy from irony. Irony does not need an exceptional central

figure: the original destiny never quite fades out of the tragedy.

As a rule, the more ignoble the hero the sharper the irony, when irony alone is the objective.

It is heroism that creates the splendor and exhilaration that is unique to tragedy. The tragic

hero normally has an extraordinary, often a nearly divine, destiny almost within grasp, and

the glory of the original destiny never quite fades out of the tragedy.

The second theory of tragedy states that the act that sets the tragic process in motion must

be primarily a violation of normal law, whether human or divine; in short, that the tragic

hero must have a flaw that has an essential connection with sin. Again it is true that the

great majority of tragic heroes do possess hubris, or a proud and passionate mind that

seems to make the hero?s downfall morally explicable. But such hubris is only the

precipitating agent of catastrophe, just as in comedy the cause )f the happy ending is

usually some act of humility often performed by a noble character who is meanly disguised.

This theory of tragedy as morally explicable runs into the question of whether an innocent

sufferer in a tragedy, such as Iphigenia, or Socrates in Plato Apology, is a tragic figure. They

are, of course, even though it is not very easy to find crucial moral flaws in them. Cordelia

shows sincerity and high spirit in refusing to flatter her faber, and Cordelia is 30 hanged.

Tragedy, in short, is ambiguous and cannot be reduced to the opposition between human

effort. and external fate, just as it cannot be reduced to the opposition between good and

evil.

The primary purpose of the passage is to

a) compare and criticize two theories of tragedy.


b) develop a new theory of tragedy.

c) summarize the thematic content of tragedy.

d) reject one theory of tragedy and offer another theory in its place.

e) distinguish between tragedy and iron

1) a

2) b

3) c

4) d

5) e

Correct Option
1
is:
Your Option is: 2
Result: Wrong
Timetaken: 0.0 secs
The passage juxtaposes two contradictory theories of tragedy.See
Explanation:
the first sentence of para 1.Hence the correct answer is option A.

Each of the following questions has four alternative responses. Select the correct response.

Which one of the following is same as uncle, aunt, brother?


35.
(a) Child

(b) Boy

(c) Adult

(d) Nephew

1) a

2) b

3) c

4) d
Correct Option
4
is:
Your Option is: 4
Result: Correct
Timetaken: 0.0 secs
Like uncle, aunt and brother, the term nephew is also used to
Explanation:
show relationship.
1.

READING COMPREHENSION

Most people can remember a phone number for up to thirty seconds. When this short

amount of time elapses, however, the numbers are erased from the memory. How did the

information get there in the first place? Information that makes its way to the short term

memory (STM) does so via the sensory storage area. The brain has a filter which only allows

stimuli that is of immediate interest to pass on to the STM, also known as the working

memory.

There is much debate about the capacity and duration of the short term memory. The most

accepted theory comes from George A. Miller, a cognitive psychologist who suggested that

humans can remember approximately seven chunks of information. A chunk is defined as a

meaningful unit of information, such as a word or name rather than just a letter or number.

Modern theorists suggest that one can increase the capacity of the short term memory by

chunking, or classifying similar information together. By organizing information, one can

optimize the STM, and improve the chances of a memory being passed on to long term

storage.

When making a conscious effort to memorize something, such as information for an exam,

many people engage in "rote rehearsal". By repeating something over and over again, one is

able to keep a memory alive. Unfortunately, this type of memory maintenance only succeeds

if there are no interruptions. As soon as a person stops rehearsing the information, it has the

tendency to disappear. When a pen and paper are not handy, people often attempt to

remember a phone number by repeating it aloud. If the doorbell rings or the dog barks to
come in before a person has the opportunity to make a phone call, he will likely forget the

number instantly.* Therefore, rote rehearsal is not an efficient way to pass information from

the short term to long term memory.* A better way is to practice "elaborate rehearsal".

*This involves assigning semantic meaning to a piece of information so that it can be filed

along with other pre-existing long term memories.*

Encoding information semantically also makes it more retrievable. Retrieving information can

be done by recognition or recall. Humans can easily recall memories that are stored in the

long term memory and used often; however, if a memory seems to be forgotten, it may

eventually be retrieved by prompting. The more cues a person is given (such as pictures),

the more likely a memory can be retrieved. This is why multiple choice tests are often used

for subjects that require a lot of memorization.

The author believes that rote rotation is:

A) the best way to remember something

B) more efficient than chunking

C) ineffective in the long run

D) an unnecessary interruption

1) a

2) b

3) c

4) d

Correct
3
Option is:
Your Option
2
is:
Result: Wrong
Timetaken: 0.0 secs
Explanation: ? Choice A is contradicted by "not an efficient way". ? Choice B is
incorrect because these two terms are not compared. ? Choice D is
illogical. The correct answer is C. This is a factual question

Sentence Correction

He admired the speed with which he completed the work and APPRECIATING THE METHOD

ADOPTED by him

A.appreciate the method being adopted


2.
B.appreciated the method adopted

C.appreciate the method of adoption

D.appreciated the method adopting method

E.No correction required

1) a

2) b

3) c

4) d

5) e

Correct Option is: 2


Your Option is: 4
Result: Wrong
Timetaken: 0.0 secs
Explanation: The given sentence is in past tense.So,the correct answer is B.

Synonym

RANT
3.
A.Praise inordinately

B.Formalise

C.To preach noisily

D.Treat with screen

1) a
2) b

3) c

4) d

5) e

Correct Option is: 3


Your Option is: 3
Result: Correct
Timetaken: 0.0 secs
Explanation: rant- speak or shout at length in angry

Synonym

DEBACLE
4.
A.Collapse

B.Decline

C.Defeat

D.Disgrace

1) a

2) b

3) c

4) d

5) e

Correct Option is: 1


Your Option is: 4
Result: Wrong
Timetaken: 0.0 secs
Explanation: Vocabulary question.
5.
Directions: Use appropriate prepositions to fill the blanks. Question: The factory was
destroyed ______ fire

1) in

2) of

3) by

4) over

Correct
3
Option is:
Your Option
3
is:
Result: Correct
Timetaken: 0.0 secs
in- expressing the situation of something of- expressing the
relationship between a part and a whole by- identifying the agent
Explanation:
performing an action over-extending directly upwards from In these
prepositions,Option C is the correct answer.

Error Identification

Read the each sentence to find out whether there is any grammatical error in it. The error, if

any will be in one part of the sentence. The letter of that part is the answer. If there is no
6.
error, the answer is 'D'. (Ignore the errors of punctuation, if any).

A. To facilitate exports and improve sales in the domestic market

B. some of the improvised fabrics and garments fabricated out from them

C. are displayed in the main pavilion.

D. No error.

1) a

2) b

3) c

4) d
5) e

Correct Option is: 2


Your Option is: 4
Result: Wrong
Timetaken: 0.0 secs
Explanation: NIL

Directions: Use appropriate prepositions to fill the blanks. Question: You can get extra
7.
supplies _____ the stationery store.

1) with

2) from

3) about

4) during

Correct
2
Option is:
Your Option
2
is:
Result: Correct
Timetaken: 0.0 secs
with means 'accompanied by' from means 'indicating the point in
space or action starts' about means 'on the subject of' during means
Explanation:
'throughout the course' From the given options,'from'is the correct
answer.
8.

Error Identification

Read the each sentence to find out whether there is any grammatical error in it. The error, if

any will be in one part of the sentence. The letter of that part is the answer. If there is no

error, the answer is 'D'. (Ignore the errors of punctuation, if any)

A. If you listen with

B. the question carefully


C. you will be able to answer them easily

D. No error.

1) a

2) b

3) c

4) d

5) e

Correct Option is: 1


Your Option is: 4
Result: Wrong
Timetaken: 0.0 secs
Explanation: NIL

ERROR IDENTIFICATION

In the following question,two sentences are given.There may be an error in the

sentence(s).Mark as your answer

(a) if there is an error only in the first sentence.

(b) if there is an error only in the second sentence.


9.
(c) if there are errors in both the sentences and

(d) if there is no error in either of the two sentences.

SENTENCES

(1) I cannot see anything wrong with the plan.

(2) You may read the book if you have enough time.

1) a

2) b
3) c

4) d

Correct Option is: 4


Your Option is: 1
Result: Wrong
Timetaken: 0.0 secs
Explanation: No error in the above sentence.Both sentences are correct.

Sentence Correction

They have a scheme of rewarding THE BEST OF THE PERFORMERS every year.

A.a best performer


10.
B.the best among the performer

C.a best among performer

D.the best of the performer

E.No correction required

1) a

2) b

3) c

4) d

5) e

Correct Option
5
is:
Your Option is: 5
Result: Correct
Timetaken: 0.0 secs
The given sentence itself represents the idea in a best
Explanation:
manner.So,no correction is required.
11.
READING COMPREHENSION

Most people can remember a phone number for up to thirty seconds. When this short

amount of time elapses, however, the numbers are erased from the memory. How did the

information get there in the first place? Information that makes its way to the short term

memory (STM) does so via the sensory storage area. The brain has a filter which only allows

stimuli that is of immediate interest to pass on to the STM, also known as the working

memory.

There is much debate about the capacity and duration of the short term memory. The most

accepted theory comes from George A. Miller, a cognitive psychologist who suggested that

humans can remember approximately seven chunks of information. A chunk is defined as a

meaningful unit of information, such as a word or name rather than just a letter or number.

Modern theorists suggest that one can increase the capacity of the short term memory by

chunking, or classifying similar information together. By organizing information, one can

optimize the STM, and improve the chances of a memory being passed on to long term

storage.

When making a conscious effort to memorize something, such as information for an exam,

many people engage in "rote rehearsal". By repeating something over and over again, one is

able to keep a memory alive. Unfortunately, this type of memory maintenance only succeeds

if there are no interruptions. As soon as a person stops rehearsing the information, it has the

tendency to disappear. When a pen and paper are not handy, people often attempt to

remember a phone number by repeating it aloud. If the doorbell rings or the dog barks to

come in before a person has the opportunity to make a phone call, he will likely forget the

number instantly.* Therefore, rote rehearsal is not an efficient way to pass information from

the short term to long term memory.* A better way is to practice "elaborate rehearsal".

*This involves assigning semantic meaning to a piece of information so that it can be filed

along with other pre-existing long term memories.*

Encoding information semantically also makes it more retrievable. Retrieving information can

be done by recognition or recall. Humans can easily recall memories that are stored in the
long term memory and used often; however, if a memory seems to be forgotten, it may

eventually be retrieved by prompting. The more cues a person is given (such as pictures),

the more likely a memory can be retrieved. This is why multiple choice tests are often used

for subjects that require a lot of memorization.

The word elaborate in paragraph 3 is closest in meaning to:

A) complex

B) efficient

C) pretty

D) regular

1) a

2) b

3) c

4) d

Correct Option is: 1


Your Option is: 1
Result: Correct
Timetaken: 0.0 secs
Explanation: The correct answer is A. This is a vocabulary question

Sentence Correction

Anand has the guts TO RISE FROM the occasion and come out successfully.

A.in rising from


12.
B.to raise with

C.to rise to

D.to rise against

E.No correction required

1) a
2) b

3) c

4) d

5) e

Correct Option is: 3


Your Option is: 2
Result: Wrong
Timetaken: 0.0 secs
Explanation: NIL

Sentence Correction

Can you tell me WHY DID YOU NOT SPEAK the truth?

A.why did not you speak


13.
B.that why did you not speak

C.why you did not speak

D.why did you not spoke

E.No correction required

1) a

2) b

3) c

4) d

5) e

Correct Option is: 3


Your Option is: 4
Result: Wrong
Timetaken: 0.0 secs
Explanation: NIL
14.
.

Prior to the work of Heckel, illustrations of fish were often beautiful but rarely __________;
this cat, combined with the__________ nature of most nineteenth-century taxonomic

descriptions, often kept scientists from recognizing differences between species.

A. impressive . . inaccurate

B. realistic . . detailed

C. traditional . . progressive

D. precise . . inexact

E. distinctive . . sophisticated

1) a

2) b

3) c

4) d

5) e

Correct Option is: 4


Your Option is: 1
Result: Wrong
Timetaken: 0.0 secs
Explanation: From the given options,option D only suits well.
15.
In the following choose the word which express the meaning of the same word SACROSANCT

1) too important

2) worship

3) sacrifice

4) best

Correct Option
1
is:
Your Option is: 4
Result: Wrong
Timetaken: 0.0 secs
SACROSANCT-regarded as too important So,the correct answer
Explanation:
is option A
16.

READING COMPREHENSION

Most people can remember a phone number for up to thirty seconds. When this short

amount of time elapses, however, the numbers are erased from the memory. How did the

information get there in the first place? Information that makes its way to the short term

memory (STM) does so via the sensory storage area. The brain has a filter which only allows

stimuli that is of immediate interest to pass on to the STM, also known as the working

memory.

There is much debate about the capacity and duration of the short term memory. The most

accepted theory comes from George A. Miller, a cognitive psychologist who suggested that

humans can remember approximately seven chunks of information. A chunk is defined as a

meaningful unit of information, such as a word or name rather than just a letter or number.

Modern theorists suggest that one can increase the capacity of the short term memory by

chunking, or classifying similar information together. By organizing information, one can

optimize the STM, and improve the chances of a memory being passed on to long term

storage.

When making a conscious effort to memorize something, such as information for an exam,

many people engage in "rote rehearsal". By repeating something over and over again, one is

able to keep a memory alive. Unfortunately, this type of memory maintenance only succeeds

if there are no interruptions. As soon as a person stops rehearsing the information, it has the

tendency to disappear. When a pen and paper are not handy, people often attempt to

remember a phone number by repeating it aloud. If the doorbell rings or the dog barks to

come in before a person has the opportunity to make a phone call, he will likely forget the

number instantly.* Therefore, rote rehearsal is not an efficient way to pass information from

the short term to long term memory.* A better way is to practice "elaborate rehearsal".

*This involves assigning semantic meaning to a piece of information so that it can be filed
along with other pre-existing long term memories.*

Encoding information semantically also makes it more retrievable. Retrieving information can

be done by recognition or recall. Humans can easily recall memories that are stored in the

long term memory and used often; however, if a memory seems to be forgotten, it may

eventually be retrieved by prompting. The more cues a person is given (such as pictures),

the more likely a memory can be retrieved. This is why multiple choice tests are often used

for subjects that require a lot of memorization.

According to the passage, how do memories get transferred to the STM?

A) They revert from the long term memory.

B) They are filtered from the sensory storage area.

C) They get chunked when they enter the brain.

D) They enter via the nervous system.

1) a

2) b

3) c

4) d

Correct
2
Option is:
Your Option
2
is:
Result: Correct
Timetaken: 0.0 secs
? Choice A is the opposite of what happens. ? Choice C is what a
Explanation: person should try to do when memorizing something. ? Choice D is
not mentioned. The correct answer is B. This is a factual question
17.

ERROR IDENTIFICATION

In the following question,two sentences are given.There may be an error in the


sentence(s).Mark as your answer

(a) if there is an error only in the first sentence.

(b) if there is an error only in the second sentence.

(c) if there are errors in both the sentences and

(d) if there is no error in either of the two sentences.

SENTENCES

(1) I and my friend like to play tennis in grass court. (2) Each of the mistakes have to be

corrected before printing

1) a

2) b

3) c

4) d

Correct Option
3
is:
Your Option is: 1
Result: Wrong
Timetaken: 0.0 secs
The second person must come first. Statement 1 should be .
Explanation: Statement 2 Each of the mistakes has to be corrected before
printing.
18.

READING COMPREHENSION

Most people can remember a phone number for up to thirty seconds. When this short

amount of time elapses, however, the numbers are erased from the memory. How did the

information get there in the first place? Information that makes its way to the short term

memory (STM) does so via the sensory storage area. The brain has a filter which only allows

stimuli that is of immediate interest to pass on to the STM, also known as the working
memory.

There is much debate about the capacity and duration of the short term memory. The most

accepted theory comes from George A. Miller, a cognitive psychologist who suggested that

humans can remember approximately seven chunks of information. A chunk is defined as a

meaningful unit of information, such as a word or name rather than just a letter or number.

Modern theorists suggest that one can increase the capacity of the short term memory by

chunking, or classifying similar information together. By organizing information, one can

optimize the STM, and improve the chances of a memory being passed on to long term

storage.

When making a conscious effort to memorize something, such as information for an exam,

many people engage in "rote rehearsal". By repeating something over and over again, one is

able to keep a memory alive. Unfortunately, this type of memory maintenance only succeeds

if there are no interruptions. As soon as a person stops rehearsing the information, it has the

tendency to disappear. When a pen and paper are not handy, people often attempt to

remember a phone number by repeating it aloud. If the doorbell rings or the dog barks to

come in before a person has the opportunity to make a phone call, he will likely forget the

number instantly.* Therefore, rote rehearsal is not an efficient way to pass information from

the short term to long term memory.* A better way is to practice "elaborate rehearsal".

*This involves assigning semantic meaning to a piece of information so that it can be filed

along with other pre-existing long term memories.*

Encoding information semantically also makes it more retrievable. Retrieving information can

be done by recognition or recall. Humans can easily recall memories that are stored in the

long term memory and used often; however, if a memory seems to be forgotten, it may

eventually be retrieved by prompting. The more cues a person is given (such as pictures),

the more likely a memory can be retrieved. This is why multiple choice tests are often used

for subjects that require a lot of memorization.

The word cues in the passage is closest in meaning to

A) questions
B) clues

C) images

D) tests

1) a

2) b

3) c

4) d

Correct Option is: 2


Your Option is: 4
Result: Wrong
Timetaken: 0.0 secs
Explanation: The correct answer is B. This is a vocabulary question

Statements:

All cars are cats.

All fans are cats.

Conclusions:

All cars are fans.


19.
Some fans are cars.

A.Only (1)conclusion follows

B.Only (2)conclusion follows

C.Either (1) or (2) follows

D.Neither (1) nor (2) follows

E.Both (1) and (2) follow

1) A

2) B
3) C

4) D

5) E

Correct
4
Option is:
Your Option
2
is:
Result: Wrong
Timetaken: 0.0 secs
From the given statements,we can conclude that the given
Explanation: conclusions didn't follow with the given statements.Neither (1)nor
(2) follows.So,the answer is option D

ANTONYM

Culpable
20.
A.Defendable

B.Blameless

C.Careless

D.Irresponsible.

1) a

2) b

3) c

4) d

Correct Option is: 2


Your Option is: 1
Result: Wrong
Timetaken: 0.0 secs
Meaning of 'culpable'is deserving blame So,the antonym is
Explanation:
blameless.
ERROR IDENTIFICATION

In the following question,two sentences are given.There may be an error in the

sentence(s).Mark as your answer

(a) if there is an error only in the first sentence.

(b) if there is an error only in the second sentence.


21.
(c) if there are errors in both the sentences and

(d) if there is no error in either of the two sentences.

SENTENCES

(1) He said that he will coe,but he didn't.

(2) I admit,'No news is good news now-a-days'

1) a

2) b

3) c

4) d

Correct Option
1
is:
Your Option is: 2
Result: Wrong
Timetaken: 0.0 secs
The past tense should be used.The first sentence should be 'he said
Explanation:
that he would come,but he didn't'
22.

READING COMPREHENSION

Most people can remember a phone number for up to thirty seconds. When this short

amount of time elapses, however, the numbers are erased from the memory. How did the
information get there in the first place? Information that makes its way to the short term

memory (STM) does so via the sensory storage area. The brain has a filter which only allows

stimuli that is of immediate interest to pass on to the STM, also known as the working

memory.

There is much debate about the capacity and duration of the short term memory. The most

accepted theory comes from George A. Miller, a cognitive psychologist who suggested that

humans can remember approximately seven chunks of information. A chunk is defined as a

meaningful unit of information, such as a word or name rather than just a letter or number.

Modern theorists suggest that one can increase the capacity of the short term memory by

chunking, or classifying similar information together. By organizing information, one can

optimize the STM, and improve the chances of a memory being passed on to long term

storage.

When making a conscious effort to memorize something, such as information for an exam,

many people engage in "rote rehearsal". By repeating something over and over again, one is

able to keep a memory alive. Unfortunately, this type of memory maintenance only succeeds

if there are no interruptions. As soon as a person stops rehearsing the information, it has the

tendency to disappear. When a pen and paper are not handy, people often attempt to

remember a phone number by repeating it aloud. If the doorbell rings or the dog barks to

come in before a person has the opportunity to make a phone call, he will likely forget the

number instantly.* Therefore, rote rehearsal is not an efficient way to pass information from

the short term to long term memory.* A better way is to practice "elaborate rehearsal".

*This involves assigning semantic meaning to a piece of information so that it can be filed

along with other pre-existing long term memories.*

Encoding information semantically also makes it more retrievable. Retrieving information can

be done by recognition or recall. Humans can easily recall memories that are stored in the

long term memory and used often; however, if a memory seems to be forgotten, it may

eventually be retrieved by prompting. The more cues a person is given (such as pictures),

the more likely a memory can be retrieved. This is why multiple choice tests are often used
for subjects that require a lot of memorization.

Why does the author mention a dog's bark?

A) To give an example of a type of memory

B) To provide a type of interruption

C) To prove that dogs have better memories than humans

D) To compare another sound that is loud like a doorbell

1) a

2) b

3) c

4) d

Correct
2
Option is:
Your Option
1
is:
Result: Wrong
Timetaken: 0.0 secs
? Choice A is incorrect because it is not the "reason" the author
mentions it. ? Choice C is not mentioned. ? Choice D distracts you
Explanation:
because both are mentioned as examples. The correct answer is B.
This is an author purpose question
23.

READING COMPREHENSION

Most people can remember a phone number for up to thirty seconds. When this short

amount of time elapses, however, the numbers are erased from the memory. How did the

information get there in the first place? Information that makes its way to the short term

memory (STM) does so via the sensory storage area. The brain has a filter which only allows

stimuli that is of immediate interest to pass on to the STM, also known as the working

memory.
There is much debate about the capacity and duration of the short term memory. The most

accepted theory comes from George A. Miller, a cognitive psychologist who suggested that

humans can remember approximately seven chunks of information. A chunk is defined as a

meaningful unit of information, such as a word or name rather than just a letter or number.

Modern theorists suggest that one can increase the capacity of the short term memory by

chunking, or classifying similar information together. By organizing information, one can

optimize the STM, and improve the chances of a memory being passed on to long term

storage.

When making a conscious effort to memorize something, such as information for an exam,

many people engage in "rote rehearsal". By repeating something over and over again, one is

able to keep a memory alive. Unfortunately, this type of memory maintenance only succeeds

if there are no interruptions. As soon as a person stops rehearsing the information, it has the

tendency to disappear. When a pen and paper are not handy, people often attempt to

remember a phone number by repeating it aloud. If the doorbell rings or the dog barks to

come in before a person has the opportunity to make a phone call, he will likely forget the

number instantly.* Therefore, rote rehearsal is not an efficient way to pass information from

the short term to long term memory.* A better way is to practice "elaborate rehearsal".

*This involves assigning semantic meaning to a piece of information so that it can be filed

along with other pre-existing long term memories.*

Encoding information semantically also makes it more retrievable. Retrieving information can

be done by recognition or recall. Humans can easily recall memories that are stored in the

long term memory and used often; however, if a memory seems to be forgotten, it may

eventually be retrieved by prompting. The more cues a person is given (such as pictures),

the more likely a memory can be retrieved. This is why multiple choice tests are often used

for subjects that require a lot of memorization.

Which of the following is NOT supported by the passage?

A) The working memory is the same as the short term memory.

B) A memory is kept alive through constant repetition.


C) Cues help people to recognize information.

D) Multiple choice exams are the most difficult.

1) a

2) b

3) c

4) d

Correct
4
Option is:
Your Option
4
is:
Result: Correct
Timetaken: 0.0 secs
? Choice A is mentioned in paragraph one. ? Choice B is mentioned
in paragraph three (though an interruption will destroy it). ? Choice
Explanation:
C is mentioned in the last paragraph. The correct answer is D. This is
a negative factual question

Although frequent air travelers remain unconvinced, researchers have found that,

paradoxically, the __________ orientation inherent in jet lag also may yield some mental

health __________
24.
A. temporal . . benefits

B. acquired . . hazards

C. somatic . . disorders

D. random . . deficiencies

E. typical . . standards

1) a

2) b

3) c
4) d

5) e

Correct Option is: 1


Your Option is: 1
Result: Correct
Timetaken: 0.0 secs
Explanation: From the given options,Option A only suits well.

Although skeptics say financial problems will probably __________ our establishing a base

on the Moon, supporters of the project remain __________, saying that human curiosity

should overcome such pragmatic constraints.


25.
A. beset . . disillusioned

B. hasten . . hopeful

C. postpone . . pessimistic

D. prevent . . enthusiastic

E. allow . . unconvinced

1) a

2) b

3) c

4) d

5) e

Correct Option is: 4


Your Option is: 2
Result: Wrong
Timetaken: 0.0 secs
Explanation: NIL
26.
Directions: Use appropriate prepositions to fill the blanks.Question: Please place the
umbrella stand __________ the door

1) across

2) beside

3) through

4) before

Correct
2
Option is:
Your Option
4
is:
Result: Wrong
Timetaken: 0.0 secs
across means 'from one side to the other' beside means 'near' through
Explanation: means'moving in one side and out of the other side' before means'in
front of' In these prepositions,'beside' is the correct answer.

______________, the Grammys are the highest rated.

A) There are many music awards shows in the US


27.
B) Although Grammys are considered to be highly prestigious

C) Because there is only one big music awards show in the US

D) Much as people are looking forward to next Grammy awards show

E) Of the "big three" music awards shows

1) a

2) b

3) c

4) d

5) e

Correct Option is: 5


Your Option is: 3
Result: Wrong
Timetaken: 0.0 secs
Explanation: From the given options,Option E only suits well than the other.

Directions: Use appropriate prepositions to fill the blanks. Question: Leneen and Miranda
28.
have been best friends ______ third grade.

1) after

2) during

3) while

4) since

Correct
4
Option is:
Your Option
2
is:
Result: Wrong
Timetaken: 0.0 secs
after-in the time following during-throughout the course while-a
Explanation: period of time since-in the intervening period between and the time
under consideration From the options,Option D suits well.
29.

READING COMPREHENSION

Most people can remember a phone number for up to thirty seconds. When this short

amount of time elapses, however, the numbers are erased from the memory. How did the

information get there in the first place? Information that makes its way to the short term

memory (STM) does so via the sensory storage area. The brain has a filter which only allows

stimuli that is of immediate interest to pass on to the STM, also known as the working

memory.

There is much debate about the capacity and duration of the short term memory. The most
accepted theory comes from George A. Miller, a cognitive psychologist who suggested that

humans can remember approximately seven chunks of information. A chunk is defined as a

meaningful unit of information, such as a word or name rather than just a letter or number.

Modern theorists suggest that one can increase the capacity of the short term memory by

chunking, or classifying similar information together. By organizing information, one can

optimize the STM, and improve the chances of a memory being passed on to long term

storage.

When making a conscious effort to memorize something, such as information for an exam,

many people engage in "rote rehearsal". By repeating something over and over again, one is

able to keep a memory alive. Unfortunately, this type of memory maintenance only succeeds

if there are no interruptions. As soon as a person stops rehearsing the information, it has the

tendency to disappear. When a pen and paper are not handy, people often attempt to

remember a phone number by repeating it aloud. If the doorbell rings or the dog barks to

come in before a person has the opportunity to make a phone call, he will likely forget the

number instantly.* Therefore, rote rehearsal is not an efficient way to pass information from

the short term to long term memory.* A better way is to practice "elaborate rehearsal".

*This involves assigning semantic meaning to a piece of information so that it can be filed

along with other pre-existing long term memories.*

Encoding information semantically also makes it more retrievable. Retrieving information can

be done by recognition or recall. Humans can easily recall memories that are stored in the

long term memory and used often; however, if a memory seems to be forgotten, it may

eventually be retrieved by prompting. The more cues a person is given (such as pictures),

the more likely a memory can be retrieved. This is why multiple choice tests are often used

for subjects that require a lot of memorization.

How do theorists believe a person can remember more information in a short time?

A) By organizing it

B) By repeating it

C) By giving it a name
D) By drawing it

1) a

2) b

3) c

4) d

Correct Option
1
is:
Your Option
4
is:
Result: Wrong
Timetaken: 0.0 secs
? Choice B is what regular people think is true. ? Choice C is not
Explanation: mentioned. ? Choice D is a type of cue for retrieval. The correct
answer is A. This is a factual question

Sentence Correction

The courts ARE ACTIVELY TO SAFEGAURD the interests and the rights of the poor.

A.are actively to safeguarding.


30.
B.have been actively safeguarding.

C.have to active in safeguarding.

D.are actively in safeguarding.

E.No correction required

1) a

2) b

3) c

4) d

5) e

Correct
2
Option is:
Your Option
2
is:
Result: Correct
Timetaken: 0.0 secs
This action has started in the past and it continues and also continues
Explanation: in the future also.So,this should be in the present perfect
continuous.Hence, the correct answer is Option B
31.

READING COMPREHENSION

Most people can remember a phone number for up to thirty seconds. When this short

amount of time elapses, however, the numbers are erased from the memory. How did the

information get there in the first place? Information that makes its way to the short term

memory (STM) does so via the sensory storage area. The brain has a filter which only allows

stimuli that is of immediate interest to pass on to the STM, also known as the working

memory.

There is much debate about the capacity and duration of the short term memory. The most

accepted theory comes from George A. Miller, a cognitive psychologist who suggested that

humans can remember approximately seven chunks of information. A chunk is defined as a

meaningful unit of information, such as a word or name rather than just a letter or number.

Modern theorists suggest that one can increase the capacity of the short term memory by

chunking, or classifying similar information together. By organizing information, one can

optimize the STM, and improve the chances of a memory being passed on to long term

storage.

When making a conscious effort to memorize something, such as information for an exam,

many people engage in "rote rehearsal". By repeating something over and over again, one is

able to keep a memory alive. Unfortunately, this type of memory maintenance only succeeds

if there are no interruptions. As soon as a person stops rehearsing the information, it has the

tendency to disappear. When a pen and paper are not handy, people often attempt to

remember a phone number by repeating it aloud. If the doorbell rings or the dog barks to

come in before a person has the opportunity to make a phone call, he will likely forget the
number instantly.* Therefore, rote rehearsal is not an efficient way to pass information from

the short term to long term memory.* A better way is to practice "elaborate rehearsal".

*This involves assigning semantic meaning to a piece of information so that it can be filed

along with other pre-existing long term memories.*

Encoding information semantically also makes it more retrievable. Retrieving information can

be done by recognition or recall. Humans can easily recall memories that are stored in the

long term memory and used often; however, if a memory seems to be forgotten, it may

eventually be retrieved by prompting. The more cues a person is given (such as pictures),

the more likely a memory can be retrieved. This is why multiple choice tests are often used

for subjects that require a lot of memorization.

The word elapses in paragraph 1 is closest in meaning to:

A) passes

B) adds up

C) appears

D) continues

1) a

2) b

3) c

4) d

Correct Option is: 1


Your Option is: 4
Result: Wrong
Timetaken: 0.0 secs
Explanation: The correct answer is A.This is a vocabulary question
32.

Most histories of science are success stories that conclude on __________ note with the final

__________ of a theory that is the basis of subsequent inquiries by later researchers.


A. a retrospective . .extrapolation

B. an analytic . . rebuttal

C. an objective . . rationalization

D. a positive . . refutation

E. a triumphal . . ascendancy

1) a

2) b

3) c

4) d

5) e

Correct Option is: 5


Your Option is: 2
Result: Wrong
Timetaken: 0.0 secs
Explanation: NIL
33.

READING COMPREHENSION

Most people can remember a phone number for up to thirty seconds. When this short

amount of time elapses, however, the numbers are erased from the memory. How did the

information get there in the first place? Information that makes its way to the short term

memory (STM) does so via the sensory storage area. The brain has a filter which only allows

stimuli that is of immediate interest to pass on to the STM, also known as the working

memory.

There is much debate about the capacity and duration of the short term memory. The most

accepted theory comes from George A. Miller, a cognitive psychologist who suggested that

humans can remember approximately seven chunks of information. A chunk is defined as a

meaningful unit of information, such as a word or name rather than just a letter or number.

Modern theorists suggest that one can increase the capacity of the short term memory by
chunking, or classifying similar information together. By organizing information, one can

optimize the STM, and improve the chances of a memory being passed on to long term

storage.

When making a conscious effort to memorize something, such as information for an exam,

many people engage in "rote rehearsal". By repeating something over and over again, one is

able to keep a memory alive. Unfortunately, this type of memory maintenance only succeeds

if there are no interruptions. As soon as a person stops rehearsing the information, it has the

tendency to disappear. When a pen and paper are not handy, people often attempt to

remember a phone number by repeating it aloud. If the doorbell rings or the dog barks to

come in before a person has the opportunity to make a phone call, he will likely forget the

number instantly.* Therefore, rote rehearsal is not an efficient way to pass information from

the short term to long term memory.* A better way is to practice "elaborate rehearsal".

*This involves assigning semantic meaning to a piece of information so that it can be filed

along with other pre-existing long term memories.*

Encoding information semantically also makes it more retrievable. Retrieving information can

be done by recognition or recall. Humans can easily recall memories that are stored in the

long term memory and used often; however, if a memory seems to be forgotten, it may

eventually be retrieved by prompting. The more cues a person is given (such as pictures),

the more likely a memory can be retrieved. This is why multiple choice tests are often used

for subjects that require a lot of memorization.

The word it in the last paragraph refers to:

A) encoding

B) STM

C) semantics

D) information

1) a

2) b

3) c
4) d

Correct Option is: 4


Your Option is: 1
Result: Wrong
Timetaken: 0.0 secs
Explanation: The correct answer is D. This is a reference question

ANTONYM

Quiescent
34.
A.Active

B.Dormant

C.Weak

D.Unconcerned

1) a

2) b

3) c

4) d

Correct Option is: 1


Your Option is: Not Attempted
Result: Not Attempted
Timetaken: 0.0 secs
Explanation: Quiescent means inactive.So,the antonym of this is 'active'
35.

READING COMPREHENSION

Most people can remember a phone number for up to thirty seconds. When this short

amount of time elapses, however, the numbers are erased from the memory. How did the
information get there in the first place? Information that makes its way to the short term

memory (STM) does so via the sensory storage area. The brain has a filter which only allows

stimuli that is of immediate interest to pass on to the STM, also known as the working

memory.

There is much debate about the capacity and duration of the short term memory. The most

accepted theory comes from George A. Miller, a cognitive psychologist who suggested that

humans can remember approximately seven chunks of information. A chunk is defined as a

meaningful unit of information, such as a word or name rather than just a letter or number.

Modern theorists suggest that one can increase the capacity of the short term memory by

chunking, or classifying similar information together. By organizing information, one can

optimize the STM, and improve the chances of a memory being passed on to long term

storage.

When making a conscious effort to memorize something, such as information for an exam,

many people engage in "rote rehearsal". By repeating something over and over again, one is

able to keep a memory alive. Unfortunately, this type of memory maintenance only succeeds

if there are no interruptions. As soon as a person stops rehearsing the information, it has the

tendency to disappear. When a pen and paper are not handy, people often attempt to

remember a phone number by repeating it aloud. If the doorbell rings or the dog barks to

come in before a person has the opportunity to make a phone call, he will likely forget the

number instantly.* Therefore, rote rehearsal is not an efficient way to pass information from

the short term to long term memory.* A better way is to practice "elaborate rehearsal".

*This involves assigning semantic meaning to a piece of information so that it can be filed

along with other pre-existing long term memories.*

Encoding information semantically also makes it more retrievable. Retrieving information can

be done by recognition or recall. Humans can easily recall memories that are stored in the

long term memory and used often; however, if a memory seems to be forgotten, it may

eventually be retrieved by prompting. The more cues a person is given (such as pictures),

the more likely a memory can be retrieved. This is why multiple choice tests are often used
for subjects that require a lot of memorization.

. All of the following are mentioned as places in which memories are stored EXCEPT the:

A) STM

B) long term memory

C) sensory storage area

D) maintenance area

1) a

2) b

3) c

4) d

Correct
4
Option is:
Your Option
2
is:
Result: Wrong
Timetaken: 0.0 secs
? Choice A is mentioned in the first paragraph. ? Choice B is
mentioned in the second paragraph. ? Choice C is mentioned in the
Explanation:
first paragraph. The correct answer is D. This is a negative factual
question

Read the sentence to find out whether there is any grammatical error it. The error , if any,

will be in one part of the sentence . The letter of the that part is the answer . If there is no
1.
error , The answer is D I am worrying (A) /about if he finds (B) /his room comfortable

enough.(C)

1) A

2) B

3) C

4) NONE OF THESE
Correct Option is: 2
Your Option is: 1
Result: Wrong
Timetaken: 0.0 secs
?If? does not follow a preposition. ?Whether? is used instead of ?
Explanation:
if ?.
2.
Answer the questions based on the given passage I remember being taught in school about

his landmark experiment in which he recreated the atmosphere of the primitive earth in a

laboratory and shot electricity through it to simulate the effects of lightning. Before long, he

found that amino acids-the building blocks of life-had been created. Sagan called it the

single most significant step in convincing many scientists that life is likely to be abundant in

the cosmos. Chemist William Day said "the experiment showed that this first step in the

creation of life was not a chance event, but it was inevitable". Astronomer Harlow Shapley

said Miller had proven that `the appearance of life is essentially an automatic biochemical

development that comes along naturally when physical conditions are right?. Only two

percent of the material he produced was composed of amino acids. But there was a major

problem with the experiment that has invalidated its results. "Miller and Oparin did not have

any real proof that the earth?s early atmosphere was composed of ammonia, methane, and

hydrogen, which Miller used in his experiment. They based their theory on physical

chemistry. They wanted to get a chemical reaction that would be favorable, and so they

proposed that the atmosphere was rich in those gases. Oparin was smart enough to know

that if you start with inert gases like nitrogen and carbon dioxide, they won?t react." From

1980 on, NASA scientists have shown that the primitive earth never had any methane,

ammonia, or hydrogen to amount to anything. Instead, it was composed of water, carbon

dioxide, and nitrogen and you absolutely cannot get the same experimental results with that

mixture. It just won?t work. More recent experiments have confirmed this to be the case.

Now new discoveries have changed everything-and yet there are generations of former

students still living under the impression that the origin of life issue has been resolved. When
textbooks present the Miller experiment, they should be honest enough to say it was

interesting historically but not terribly relevant to how life actually developed. Which of the

following does not represent any of the author?s key arguments?

1) Miller?s experiment was not germane to the question about the origin of life on earth.

2) Miller?s theory was based on physical chemistry

3) It was Miller and Oparin who proposed that atmosphere was rich in methane, ammonia
and hydrogen.

4) The appearance of life is essentially an automatic biochemical development that comes


along naturally when physical conditions are right

Correct
4
Option is:
Your Option
2
is:
Result: Wrong
Timetaken: 0.0 secs
n the last line of paragraph 1, it is mentioned "Astronomer Harlow
Shapley said Miller had proven that `the appearance of life is
Explanation:
essentially an automatic biochemical development that comes along
naturally when physical condition s are right?".

Choose the contextual meaning of the given word The little Dudley needs to be coddled all
3.
the time.

1) Moved or arranged in a comfortable and cosy position

2) Held close, as for affection, comfort, or warmth

3) Treated with excessive indulgence

4) Treated with tenderness

Correct Option is: 3


Your Option is: 1
Result: Wrong
Timetaken: 0.0 secs
Explanation: Coddle - Treat with excessive indulgence, overprotective way.

Fill in the blanks Is there any lecturer in the room? No there ___ no one. Are there any
4.
students in the room? No, there ____ none.

1) are, are

2) are, is

3) is, are

4) is, is

Correct Option
3
is:
Your Option is: 4
Result: Wrong
Timetaken: 0.0 secs
In negative sentences, ?no one? is used as singular and ?none? as
Explanation:
plural

The given sentences when properly sequenced form a coherent paragraph. Each sentence is

labelled with a number. Choose the most logical order of sentences from among the four

given choices to construct a coherent paragraph. A.This kind of birth creates a kind of
5.
contentment in the concerned person, B. discontent that ought to activate one?s mind and

trigger a fire within one to do hard work. C. and this kind of contentment kills motivation. D.

There is a saying that ?Mr X was born with a silver spoon in his mouth?. E. While one who

was born in a poor family without a silver or a golden spoon, develops a kind of

1) DECAB

2) ACBED

3) BECAD

4) DACEB
Correct
4
Option is:
Your Option
3
is:
Result: Wrong
Timetaken: 0.0 secs
In the given question, statement D marks the beginning of the
statement as th is is the only statement which can serve as the first
statement and this state ment discusses some Mr X born with a silver
spoon in his mouth. Further stat ement A refers to statement D and
Explanation:
tells about the kind of birth that has been discussed in statement D.
Hence, statement D comes just before statement A. EB link also is
missing in all the options except DACEB. Hence, DACEB f orms
coherent paragraph.

Read the sentence to find out whether there is any error in it.The error if any will be in one
6.
part of the sentence.The letter of that is the answer.If there is no error , Answer D Either he

or you (A) /are the needle (B) /of suspicion.(C)

1) A

2) B

3) C

4) NONE OF THESE

Correct
4
Option is:
Your Option
1
is:
Result: Wrong
Timetaken: 0.0 secs
Either he or you are the needle of suspicion. When the subjects
Explanation: joined by either or are different in person, the verb agrees with the
latter one. ?You? is the latter noun which assumes a plural verb.
7.
Choose the contextual meaning of the given word. The magician conjured a ball out of the
boy? sleeve.

1) Engage in plotting or enter into a conspiracy

2) Request earnestly

3) Bring into existence

4) Summon into action

Correct Option is: 3


Your Option is: 2
Result: Wrong
Timetaken: 0.0 secs
Explanation: Conjure - bring into existence, often as if by magic
8.
Change the speech of the sentence. She said, ?My daughter wants to be an actress?

1) She said that her daughter wanted to be an actress

2) She said that her daughter wants to be an actress.

3) She said him/her that her daughter wants to be an actress.

4) She said him/her that her daughter wants to be an actress.

Correct
1
Option is:
Your Option
3
is:
Result: Wrong
Timetaken: 0.0 secs
In indirect speech some changes are made in original words of the
person be cause these words have been uttered in past so the tense
Explanation: will change accordi ngly and pronoun may also be changed
accordingly. Hence here answer will be - She said that her daughter
wanted to be an actre ss.
9.
In the question a part of these sentence is italicised.Alteranatively to the italicised part are
given which may improve the sentence.Choose the correct alternative . King David

pretended madness to deceive the Achish, King of Gath as he was afraid of him

1) Dissemble

2) Camouflage

3) Dissolute

4) Disguise Tags: Verbal, Sentence Correction, italics part, A

Correct
1
Option is:
Your Option
2
is:
Result: Wrong
Timetaken: 0.0 secs
Dissemble is to make believe with the intent to deceive. When you
dissemble you disguise your true intentions or feelings behind a fal se
appearance Camouflage is the the act of concealing the identity of
Explanation:
something by modifyin g its appearance Dissolute is to be
unrestrained by convention or morality Disguise-Hold back; keep
from being perceived by others("She disguises her anger well

The given sentences when properly sequenced form a coherent paragraph. Each sentence is

labelled with a number. Choose the most logical order of sentences from among the four

given choices to construct a coherent paragraph. A. I find that the one common factor that

connects them all is their tendency to complain, B. Only remarkably few people are grateful
10.
or feel they are lucky or ecstatic about what is happening in their lives C. Having travelled

widely and met a wide cross section of people from all over the world, D. nonstop! If people

are asked to say a few words on their life, it will be lumps, bumps, what is missing, who and

how got hurt and the unfairness of life.

1) CBDA

2) ACDB

3) BDAC
4) CADB

Correct Option is: 4


Your Option is: 3
Result: Wrong
Timetaken: 0.0 secs
Explanation: CA link is missing in all the other options other than CADB.
11.
If Suresh _________, he would have never let her go

1) If Suresh were present here, he would have never let her go.

2) If Suresh was present here, he would have never let her go

3) If Suresh is present here, he would have never let her go

4) If Suresh had been present here, he would have never let her go

Correct
4
Option is:
Your Option
2
is:
Result: Wrong
Timetaken: 0.0 secs
Since this is a conditional tense and there are two connected events
that did not take place, the simple past has turned into had + past
Explanation:
participle, ?had bee n?. The correct sentence is ?If Suresh had been
present here, he would have never let her go?.

Read the sentence to find out whether there is any grammatical error in it. The error, if any,

will be in one part of the sentence. The letter of that part is the answer. If there is no error,
12.
the answer is 'D'. (Ignore - the errors of punctuation, if any) Krish realized that(A) /a friend

in need(B) /was a friend indeed.(C)

1) A

2) B
3) C

4) NONE OF THESE

Correct
3
Option is:
Your Option
3
is:
Result: Correct
Timetaken: 0.0 secs
If the subordinate clause states a universal truth, ?a friend in need is
Explanation: a friend in deed?, it assumes a present tense. The correct sentence is,
?Ravi realized that a friend in need is a friend indeed?.

In the question a part of the sentence is italicised. Alternatives to the italicised part are

given which may improve the sentence. Choose the correct alternative. In case no
13.
improvement is needed. Option ?E? is the answer. The politicians were solemnly ordered by

the judge to make a fair decision.

1) Dictated

2) Commanded

3) Notified

4) Adjured

Correct
4
Option is:
Your Option
2
is:
Result: Wrong
Timetaken: 0.0 secs
Dictated - Determined or decided upon as by an authority
Commanded - Be in command of Notified - Inform (somebody) of
Explanation:
something Adjured - Command solemnly (in a grave/serious
manner) Adjured is the right word here.
In the following question, select the word or phrase that is similar in meaning to the given
14.
word. Chutzpah

1) Temerity

2) Effulgent

3) Eclectic

4) Impetuous

Correct
1
Option is:
Your Option
1
is:
Result: Correct
Timetaken: 0.0 secs
Chutzpah -extreme self-confidence or audacity. Temerity - fearless
daring. Effulgent - radiating or as if radiating light Eclectic -
Explanation: selecting what seems best of various styles or ideas. Impetuous
-characterized by undue haste and lack of thought or deliberation.
Temerity is the synonym of chutzpah
15.
Answer the questions based on the given passage I remember being taught in school about

his landmark experiment in which he recreated the atmosphere of the primitive earth in a

laboratory and shot electricity through it to simulate the effects of lightning. Before long, he

found that amino acids-the building blocks of life-had been created. Sagan called it the

single most significant step in convincing many scientists that life is likely to be abundant in

the cosmos. Chemist William Day said "the experiment showed that this first step in the

creation of life was not a chance event, but it was inevitable". Astronomer Harlow Shapley

said Miller had proven that `the appearance of life is essentially an automatic biochemical

development that comes along naturally when physical conditions are right?. Only two

percent of the material he produced was composed of amino acids. But there was a major

problem with the experiment that has invalidated its results. "Miller and Oparin did not have

any real proof that the earth?s early atmosphere was composed of ammonia, methane, and
hydrogen, which Miller used in his experiment. They based their theory on physical

chemistry. They wanted to get a chemical reaction that would be favorable, and so they

proposed that the atmosphere was rich in those gases. Oparin was smart enough to know

that if you start with inert gases like nitrogen and carbon dioxide, they won?t react." From

1980 on, NASA scientists have shown that the primitive earth never had any methane,

ammonia, or hydrogen to amount to anything. Instead, it was composed of water, carbon

dioxide, and nitrogen and you absolutely cannot get the same experimental results with that

mixture. It just won?t work. More recent experiments have confirmed this to be the case.

Now new discoveries have changed everything-and yet there are generations of former

students still living under the impression that the origin of life issue has been resolved. When

textbooks present the Miller experiment, they should be honest enough to say it was

interesting historically but not terribly relevant to how life actually developed. Which of the

following are true? I. the primitive earth never had any methane, ammonia and hydrogen II.

the primitive earth was composed of water, carbon dioxide and nitrogen III. the primitive

earth may not have had traces of methane, ammonia and hydrogen IV. Miller?s experiment

has failed because the primitive atmosphere is not made of the gases which were used to

create primitive earth?s atmosphere in the lab during the experiment

1) I and II

2) II and III

3) I, II and IV

4) II, III and IV

Correct
3
Option is:
Your Option
3
is:
Result: Correct
Timetaken: 0.0 secs
I- from para 3 " the primitive earth never had any methane, ammonia,
or hydro gen to amount to anything." II " Instead, it was composed of
Explanation: water, carbon dioxide, and nitrogen and you absolutely cannot get the
same experimental results with that mixture. It just w on't work.
More recent experiments have confirmed this to be the case."
16.
Answer the questions based on the given passage I remember being taught in school about

his landmark experiment in which he recreated the atmosphere of the primitive earth in a

laboratory and shot electricity through it to simulate the effects of lightning. Before long, he

found that amino acids-the building blocks of life-had been created. Sagan called it the

single most significant step in convincing many scientists that life is likely to be abundant in

the cosmos. Chemist William Day said "the experiment showed that this first step in the

creation of life was not a chance event, but it was inevitable". Astronomer Harlow Shapley

said Miller had proven that `the appearance of life is essentially an automatic biochemical

development that comes along naturally when physical conditions are right?. Only two

percent of the material he produced was composed of amino acids. But there was a major

problem with the experiment that has invalidated its results. "Miller and Oparin did not have

any real proof that the earth?s early atmosphere was composed of ammonia, methane, and

hydrogen, which Miller used in his experiment. They based their theory on physical

chemistry. They wanted to get a chemical reaction that would be favorable, and so they

proposed that the atmosphere was rich in those gases. Oparin was smart enough to know

that if you start with inert gases like nitrogen and carbon dioxide, they won?t react." From

1980 on, NASA scientists have shown that the primitive earth never had any methane,

ammonia, or hydrogen to amount to anything. Instead, it was composed of water, carbon

dioxide, and nitrogen and you absolutely cannot get the same experimental results with that

mixture. It just won?t work. More recent experiments have confirmed this to be the case.

Now new discoveries have changed everything-and yet there are generations of former

students still living under the impression that the origin of life issue has been resolved. When

textbooks present the Miller experiment, they should be honest enough to say it was

interesting historically but not terribly relevant to how life actually developed. Miller?s
experiment was actually a

1) Chemical reaction

2) Theory on physical chemistry

3) Biochemical experiment

4) Method of creating amino acid

Correct
1
Option is:
Your Option
4
is:
Result: Wrong
Timetaken: 0.0 secs
In the passage from the second paragraph, there is a statement "They
wanted to get a chemical reaction that would be favourable, and so
Explanation: they proposed tha t the atmosphere was rich in those gases." From
this statement, we can conc lude that Miller?s experiment was
actually a Chemical Reaction.

Read the sentence to find out whether there is any grammatical error in it. The error , if any,
17.
will b in one part of the sentence.The letter of that part is the answer. If there is no error,

The answer is D He promised his dad that(A) /he will (B)/be a good boy.(C)

1) A

2) B

3) C

4) NONE OF THESE

Correct
2
Option is:
Your Option
1
is:
Result: Wrong
Timetaken: 0.0 secs
would If the main clause is stated in the past tense, the following
subordinate clause is conditional. ?Will be? is future tense which has
Explanation:
to be turned into a conditiona l ?would be?. The correct sentence is, ?
He promised his mother that he would be a good bo y?.

Fill in the following with appropriate words.


18.
If this dictionary is not yours, _______ can it be?

1) what else

2) who else

3) which else's

4) who else's

Correct
4
Option is:
Your Option
2
is:
Result: Wrong
Timetaken: 0.0 secs
sol:?Who else? is a pronoun used in interrogatives. Here the
Explanation: question is about wh om the dictionary belongs to . Hence
possessive pronoun ?who else?s? should be used.
19.
Change the voice of the sentence. Priya wrote a letter to me.

1) A letter was written to me by Priya.

2) I was written a letter by Priya.

3) Both (1) and (2)

4) A letter had been written by Priya to me

Correct
3
Option is:
Your Option 1
is:
Result: Wrong
Timetaken: 0.0 secs
A letter was written to me by Priya. I was written a letter by Priya.
While writing an active sentence with two objects in passive voice
Explanation: means that, on e of the two objects becomes the subject and the other
one remains as object. Th e object to transform into a subject
depends on what you want to put the focus o n.
20.
Choose the word which is farthest in meaning to the given word Doughty

1) Egregious

2) Effete

3) Potentate

4) Craven

Correct
4
Option is:
Your Option
4
is:
Result: Correct
Timetaken: 0.0 secs
Doughty - Resolute and without fear Egregious - Conspicuously and
outrageously bad or reprehensible Effete - Marked by excessive self-
Explanation: indulgence and moral decay Potentate - A ruler who is unconstrained
by law Craven - Lacking even the rudiments of courage; abjectly
fearful Craven is the right word here

In the following questions, select the word or phrase that is similar in meaning to the given
21.
word. Culpable

1) Harangue

2) Censurable

3) irreproachable

4) Castigate
Correct
2
Option is:
Your Option
2
is:
Result: Correct
Timetaken: 0.0 secs
Culpable - deserving blame or censure as being wrong or evil or
injurious. Harangue - a loud bombastic declamation expressed with
Explanation: strong emotion. irreproachable - beyond criticism; faultless.
Castigate - inflict severe punishment on. Censurable is the synonym
of Culpable
22.
The pilot told the passengers

1) The pilot told the passengers that they need to wait for some more time.

2) The pilot told the passengers about their need to wait for some more time.

3) The pilot told the passengers it was needed to wait for some more time.

4) The pilot told the passengers that they needed to wait for some more time.

Correct
4
Option is:
Your Option
1
is:
Result: Wrong
Timetaken: 0.0 secs
If the main clause is stated in the past tense, the following
subordinate clause also assumes a past tense. ?Told? is the past tense,
Explanation: so the subordinate claus e must have a past tense ?needed?. The
correct sentence is, ?The driver told the passengers that they needed
to w ait for some more time?.
23.
The given sentences when properly sequenced form a coherent paragraph. Each sentence is

labelled with a number. Choose the most logical order of sentences from among the four

given choices to construct a coherent paragraph. A. and Harry Seeley in the 19th century,
when little was known about the diversity of the group, B. resulting in the description of

dozens of species, C. However, more recent findings of pterosaur fossils have challenged

views on their diversity. D. Pterosaurs from the Cretaceous of England were first described

by British naturalists Richard Owen E. all based on very fragmentary remains, represented

mostly by the tips of the snouts of these animals.

1) DCABE

2) DABEC

3) CEBAD

4) DCEBA

Correct
2
Option is:
Your Option
3
is:
Result: Wrong
Timetaken: 0.0 secs
In the given question, statement D marks the beginning of the
paragraph as it is the only introductory topic and introduces
Pterosaurs from the Cretaceous of England. Further statement A
serves as the continuation of statement D. A gain if we check the
Explanation:
ending statement, A&D cannot be an ending statement a nd among
C&E statement C will be the proper ending statement. Looking into
the options, DABEC is the only one such option and hence is the
correct ans wer.
24.
Choose the word which is farthest in meaning to the given word Humongous

1) Expiate

2) Illustrious

3) Hoyden

4) Miniscule
Correct
4
Option is:
Your Option
1
is:
Result: Wrong
Timetaken: 0.0 secs
Humongous - very large Expiate - Make up for, compensate or
causing offence Illustrious - Widely known and esteemed Hoyden -
Explanation:
A girl who behaves in a boyish manner Miniscule - Very small
Miniscule is the right word here.
25.
Choose the word which is farthest in meaning to the given word Compendious

1) Inchoate

2) Windy

3) Condign

4) Compilation

Correct
2
Option is:
Your Option
4
is:
Result: Wrong
Timetaken: 0.0 secs
Compendious - Briefly giving the gist of something, brief. Inchoate -
Only partly in existence; imperfectly formed Windy - Using or
Explanation: containing too many words Condign - Fitting or appropriate and
deserved; used especially of punishment Compilation - Something
that is compiled, Collection.windy is the right word here

Read the sentence to find out whether there is any grammatical error it. The error , if any,

will be in one part of the sentence . The letter of the that part is the answer . If there is no
26.
error , The answer is D I am worrying (A) /about if he finds (B) /his room comfortable

enough.(C)
1) A

2) B

3) C

4) NONE OF THESE

Correct Option is: 2


Your Option is: 1
Result: Wrong
Timetaken: 0.0 secs
?If? does not follow a preposition. ?Whether? is used instead of ?
Explanation:
if ?.
27.
Answer the questions based on the given passage I remember being taught in school about

his landmark experiment in which he recreated the atmosphere of the primitive earth in a

laboratory and shot electricity through it to simulate the effects of lightning. Before long, he

found that amino acids-the building blocks of life-had been created. Sagan called it the

single most significant step in convincing many scientists that life is likely to be abundant in

the cosmos. Chemist William Day said "the experiment showed that this first step in the

creation of life was not a chance event, but it was inevitable". Astronomer Harlow Shapley

said Miller had proven that `the appearance of life is essentially an automatic biochemical

development that comes along naturally when physical conditions are right?. Only two

percent of the material he produced was composed of amino acids. But there was a major

problem with the experiment that has invalidated its results. "Miller and Oparin did not have

any real proof that the earth?s early atmosphere was composed of ammonia, methane, and

hydrogen, which Miller used in his experiment. They based their theory on physical

chemistry. They wanted to get a chemical reaction that would be favorable, and so they

proposed that the atmosphere was rich in those gases. Oparin was smart enough to know

that if you start with inert gases like nitrogen and carbon dioxide, they won?t react." From

1980 on, NASA scientists have shown that the primitive earth never had any methane,

ammonia, or hydrogen to amount to anything. Instead, it was composed of water, carbon


dioxide, and nitrogen and you absolutely cannot get the same experimental results with that

mixture. It just won?t work. More recent experiments have confirmed this to be the case.

Now new discoveries have changed everything-and yet there are generations of former

students still living under the impression that the origin of life issue has been resolved. When

textbooks present the Miller experiment, they should be honest enough to say it was

interesting historically but not terribly relevant to how life actually developed. Which of the

following does not represent any of the author?s key arguments?

1) Miller?s experiment was not germane to the question about the origin of life on earth.

2) Miller?s theory was based on physical chemistry

3) It was Miller and Oparin who proposed that atmosphere was rich in methane, ammonia
and hydrogen.

4) The appearance of life is essentially an automatic biochemical development that comes


along naturally when physical conditions are right

Correct
4
Option is:
Your Option
2
is:
Result: Wrong
Timetaken: 0.0 secs
n the last line of paragraph 1, it is mentioned "Astronomer Harlow
Shapley said Miller had proven that `the appearance of life is
Explanation:
essentially an automatic biochemical development that comes along
naturally when physical condition s are right?".

Choose the contextual meaning of the given word The little Dudley needs to be coddled all
28.
the time.

1) Moved or arranged in a comfortable and cosy position

2) Held close, as for affection, comfort, or warmth

3) Treated with excessive indulgence

4) Treated with tenderness


Correct Option is: 3
Your Option is: 1
Result: Wrong
Timetaken: 0.0 secs
Explanation: Coddle - Treat with excessive indulgence, overprotective way.

Fill in the blanks Is there any lecturer in the room? No there ___ no one. Are there any
29.
students in the room? No, there ____ none.

1) are, are

2) are, is

3) is, are

4) is, is

Correct Option
3
is:
Your Option is: 4
Result: Wrong
Timetaken: 0.0 secs
In negative sentences, ?no one? is used as singular and ?none? as
Explanation:
plural

The given sentences when properly sequenced form a coherent paragraph. Each sentence is

labelled with a number. Choose the most logical order of sentences from among the four

given choices to construct a coherent paragraph. A.This kind of birth creates a kind of
30.
contentment in the concerned person, B. discontent that ought to activate one?s mind and

trigger a fire within one to do hard work. C. and this kind of contentment kills motivation. D.

There is a saying that ?Mr X was born with a silver spoon in his mouth?. E. While one who

was born in a poor family without a silver or a golden spoon, develops a kind of

1) DECAB
2) ACBED

3) BECAD

4) DACEB

Correct
4
Option is:
Your Option
3
is:
Result: Wrong
Timetaken: 0.0 secs
In the given question, statement D marks the beginning of the
statement as th is is the only statement which can serve as the first
statement and this state ment discusses some Mr X born with a silver
spoon in his mouth. Further stat ement A refers to statement D and
Explanation:
tells about the kind of birth that has been discussed in statement D.
Hence, statement D comes just before statement A. EB link also is
missing in all the options except DACEB. Hence, DACEB f orms
coherent paragraph.

Read the sentence to find out whether there is any error in it.The error if any will be in one
31.
part of the sentence.The letter of that is the answer.If there is no error , Answer D Either he

or you (A) /are the needle (B) /of suspicion.(C)

1) A

2) B

3) C

4) NONE OF THESE

Correct
4
Option is:
Your Option
1
is:
Result: Wrong
Timetaken: 0.0 secs
Either he or you are the needle of suspicion. When the subjects
Explanation: joined by either or are different in person, the verb agrees with the
latter one. ?You? is the latter noun which assumes a plural verb.

Choose the contextual meaning of the given word. The magician conjured a ball out of the
32.
boy? sleeve.

1) Engage in plotting or enter into a conspiracy

2) Request earnestly

3) Bring into existence

4) Summon into action

Correct Option is: 3


Your Option is: 2
Result: Wrong
Timetaken: 0.0 secs
Explanation: Conjure - bring into existence, often as if by magic
33.
Change the speech of the sentence. She said, ?My daughter wants to be an actress?

1) She said that her daughter wanted to be an actress

2) She said that her daughter wants to be an actress.

3) She said him/her that her daughter wants to be an actress.

4) She said him/her that her daughter wants to be an actress.

Correct
1
Option is:
Your Option
3
is:
Result: Wrong
Timetaken: 0.0 secs
Explanation: In indirect speech some changes are made in original words of the
person be cause these words have been uttered in past so the tense
will change accordi ngly and pronoun may also be changed
accordingly. Hence here answer will be - She said that her daughter
wanted to be an actre ss.

In the question a part of these sentence is italicised.Alteranatively to the italicised part are
34.
given which may improve the sentence.Choose the correct alternative . King David

pretended madness to deceive the Achish, King of Gath as he was afraid of him

1) Dissemble

2) Camouflage

3) Dissolute

4) Disguise Tags: Verbal, Sentence Correction, italics part, A

Correct
1
Option is:
Your Option
2
is:
Result: Wrong
Timetaken: 0.0 secs
Dissemble is to make believe with the intent to deceive. When you
dissemble you disguise your true intentions or feelings behind a fal se
appearance Camouflage is the the act of concealing the identity of
Explanation:
something by modifyin g its appearance Dissolute is to be
unrestrained by convention or morality Disguise-Hold back; keep
from being perceived by others("She disguises her anger well
35.
The given sentences when properly sequenced form a coherent paragraph. Each sentence is

labelled with a number. Choose the most logical order of sentences from among the four

given choices to construct a coherent paragraph. A. I find that the one common factor that

connects them all is their tendency to complain, B. Only remarkably few people are grateful

or feel they are lucky or ecstatic about what is happening in their lives C. Having travelled

widely and met a wide cross section of people from all over the world, D. nonstop! If people

are asked to say a few words on their life, it will be lumps, bumps, what is missing, who and
how got hurt and the unfairness of life.

1) CBDA

2) ACDB

3) BDAC

4) CADB

Correct Option is: 4


Your Option is: 3
Result: Wrong
Timetaken: 0.0 secs
Explanation: CA link is missing in all the other options other than CADB.
36.
If Suresh _________, he would have never let her go

1) If Suresh were present here, he would have never let her go.

2) If Suresh was present here, he would have never let her go

3) If Suresh is present here, he would have never let her go

4) If Suresh had been present here, he would have never let her go

Correct
4
Option is:
Your Option
2
is:
Result: Wrong
Timetaken: 0.0 secs
Since this is a conditional tense and there are two connected events
that did not take place, the simple past has turned into had + past
Explanation:
participle, ?had bee n?. The correct sentence is ?If Suresh had been
present here, he would have never let her go?.
37.
Read the sentence to find out whether there is any grammatical error in it. The error, if any,

will be in one part of the sentence. The letter of that part is the answer. If there is no error,
the answer is 'D'. (Ignore - the errors of punctuation, if any) Krish realized that(A) /a friend

in need(B) /was a friend indeed.(C)

1) A

2) B

3) C

4) NONE OF THESE

Correct
3
Option is:
Your Option
3
is:
Result: Correct
Timetaken: 0.0 secs
If the subordinate clause states a universal truth, ?a friend in need is
Explanation: a friend in deed?, it assumes a present tense. The correct sentence is,
?Ravi realized that a friend in need is a friend indeed?.

In the question a part of the sentence is italicised. Alternatives to the italicised part are

given which may improve the sentence. Choose the correct alternative. In case no
38.
improvement is needed. Option ?E? is the answer. The politicians were solemnly ordered by

the judge to make a fair decision.

1) Dictated

2) Commanded

3) Notified

4) Adjured

Correct
4
Option is:
Your Option
2
is:
Result: Wrong
Timetaken: 0.0 secs
Dictated - Determined or decided upon as by an authority
Commanded - Be in command of Notified - Inform (somebody) of
Explanation:
something Adjured - Command solemnly (in a grave/serious
manner) Adjured is the right word here.

In the following question, select the word or phrase that is similar in meaning to the given
39.
word. Chutzpah

1) Temerity

2) Effulgent

3) Eclectic

4) Impetuous

Correct
1
Option is:
Your Option
1
is:
Result: Correct
Timetaken: 0.0 secs
Chutzpah -extreme self-confidence or audacity. Temerity - fearless
daring. Effulgent - radiating or as if radiating light Eclectic -
Explanation: selecting what seems best of various styles or ideas. Impetuous
-characterized by undue haste and lack of thought or deliberation.
Temerity is the synonym of chutzpah
40.
Answer the questions based on the given passage I remember being taught in school about

his landmark experiment in which he recreated the atmosphere of the primitive earth in a

laboratory and shot electricity through it to simulate the effects of lightning. Before long, he

found that amino acids-the building blocks of life-had been created. Sagan called it the

single most significant step in convincing many scientists that life is likely to be abundant in

the cosmos. Chemist William Day said "the experiment showed that this first step in the

creation of life was not a chance event, but it was inevitable". Astronomer Harlow Shapley
said Miller had proven that `the appearance of life is essentially an automatic biochemical

development that comes along naturally when physical conditions are right?. Only two

percent of the material he produced was composed of amino acids. But there was a major

problem with the experiment that has invalidated its results. "Miller and Oparin did not have

any real proof that the earth?s early atmosphere was composed of ammonia, methane, and

hydrogen, which Miller used in his experiment. They based their theory on physical

chemistry. They wanted to get a chemical reaction that would be favorable, and so they

proposed that the atmosphere was rich in those gases. Oparin was smart enough to know

that if you start with inert gases like nitrogen and carbon dioxide, they won?t react." From

1980 on, NASA scientists have shown that the primitive earth never had any methane,

ammonia, or hydrogen to amount to anything. Instead, it was composed of water, carbon

dioxide, and nitrogen and you absolutely cannot get the same experimental results with that

mixture. It just won?t work. More recent experiments have confirmed this to be the case.

Now new discoveries have changed everything-and yet there are generations of former

students still living under the impression that the origin of life issue has been resolved. When

textbooks present the Miller experiment, they should be honest enough to say it was

interesting historically but not terribly relevant to how life actually developed. Which of the

following are true? I. the primitive earth never had any methane, ammonia and hydrogen II.

the primitive earth was composed of water, carbon dioxide and nitrogen III. the primitive

earth may not have had traces of methane, ammonia and hydrogen IV. Miller?s experiment

has failed because the primitive atmosphere is not made of the gases which were used to

create primitive earth?s atmosphere in the lab during the experiment

1) I and II

2) II and III

3) I, II and IV

4) II, III and IV


Correct
3
Option is:
Your Option
3
is:
Result: Correct
Timetaken: 0.0 secs
I- from para 3 " the primitive earth never had any methane, ammonia,
or hydro gen to amount to anything." II " Instead, it was composed of
Explanation: water, carbon dioxide, and nitrogen and you absolutely cannot get the
same experimental results with that mixture. It just w on't work.
More recent experiments have confirmed this to be the case."
41.
Answer the questions based on the given passage I remember being taught in school about

his landmark experiment in which he recreated the atmosphere of the primitive earth in a

laboratory and shot electricity through it to simulate the effects of lightning. Before long, he

found that amino acids-the building blocks of life-had been created. Sagan called it the

single most significant step in convincing many scientists that life is likely to be abundant in

the cosmos. Chemist William Day said "the experiment showed that this first step in the

creation of life was not a chance event, but it was inevitable". Astronomer Harlow Shapley

said Miller had proven that `the appearance of life is essentially an automatic biochemical

development that comes along naturally when physical conditions are right?. Only two

percent of the material he produced was composed of amino acids. But there was a major

problem with the experiment that has invalidated its results. "Miller and Oparin did not have

any real proof that the earth?s early atmosphere was composed of ammonia, methane, and

hydrogen, which Miller used in his experiment. They based their theory on physical

chemistry. They wanted to get a chemical reaction that would be favorable, and so they

proposed that the atmosphere was rich in those gases. Oparin was smart enough to know

that if you start with inert gases like nitrogen and carbon dioxide, they won?t react." From

1980 on, NASA scientists have shown that the primitive earth never had any methane,

ammonia, or hydrogen to amount to anything. Instead, it was composed of water, carbon

dioxide, and nitrogen and you absolutely cannot get the same experimental results with that

mixture. It just won?t work. More recent experiments have confirmed this to be the case.
Now new discoveries have changed everything-and yet there are generations of former

students still living under the impression that the origin of life issue has been resolved. When

textbooks present the Miller experiment, they should be honest enough to say it was

interesting historically but not terribly relevant to how life actually developed. Miller?s

experiment was actually a

1) Chemical reaction

2) Theory on physical chemistry

3) Biochemical experiment

4) Method of creating amino acid

Correct
1
Option is:
Your Option
4
is:
Result: Wrong
Timetaken: 0.0 secs
In the passage from the second paragraph, there is a statement "They
wanted to get a chemical reaction that would be favourable, and so
Explanation: they proposed tha t the atmosphere was rich in those gases." From
this statement, we can conc lude that Miller?s experiment was
actually a Chemical Reaction.

Read the sentence to find out whether there is any grammatical error in it. The error , if any,
42.
will b in one part of the sentence.The letter of that part is the answer. If there is no error,

The answer is D He promised his dad that(A) /he will (B)/be a good boy.(C)

1) A

2) B

3) C

4) NONE OF THESE
Correct
2
Option is:
Your Option
1
is:
Result: Wrong
Timetaken: 0.0 secs
would If the main clause is stated in the past tense, the following
subordinate clause is conditional. ?Will be? is future tense which has
Explanation:
to be turned into a conditiona l ?would be?. The correct sentence is, ?
He promised his mother that he would be a good bo y?.

Fill in the following with appropriate words.


43.
If this dictionary is not yours, _______ can it be?

1) what else

2) who else

3) which else's

4) who else's

Correct
4
Option is:
Your Option
2
is:
Result: Wrong
Timetaken: 0.0 secs
sol:?Who else? is a pronoun used in interrogatives. Here the
Explanation: question is about wh om the dictionary belongs to . Hence
possessive pronoun ?who else?s? should be used.
44.
Change the voice of the sentence. Priya wrote a letter to me.

1) A letter was written to me by Priya.

2) I was written a letter by Priya.

3) Both (1) and (2)


4) A letter had been written by Priya to me

Correct
3
Option is:
Your Option
1
is:
Result: Wrong
Timetaken: 0.0 secs
A letter was written to me by Priya. I was written a letter by Priya.
While writing an active sentence with two objects in passive voice
Explanation: means that, on e of the two objects becomes the subject and the other
one remains as object. Th e object to transform into a subject
depends on what you want to put the focus o n.
45.
Choose the word which is farthest in meaning to the given word Doughty

1) Egregious

2) Effete

3) Potentate

4) Craven

Correct
4
Option is:
Your Option
4
is:
Result: Correct
Timetaken: 0.0 secs
Doughty - Resolute and without fear Egregious - Conspicuously and
outrageously bad or reprehensible Effete - Marked by excessive self-
Explanation: indulgence and moral decay Potentate - A ruler who is unconstrained
by law Craven - Lacking even the rudiments of courage; abjectly
fearful Craven is the right word here

In the following questions, select the word or phrase that is similar in meaning to the given
46.
word. Culpable
1) Harangue

2) Censurable

3) irreproachable

4) Castigate

Correct
2
Option is:
Your Option
2
is:
Result: Correct
Timetaken: 0.0 secs
Culpable - deserving blame or censure as being wrong or evil or
injurious. Harangue - a loud bombastic declamation expressed with
Explanation: strong emotion. irreproachable - beyond criticism; faultless.
Castigate - inflict severe punishment on. Censurable is the synonym
of Culpable
47.
The pilot told the passengers

1) The pilot told the passengers that they need to wait for some more time.

2) The pilot told the passengers about their need to wait for some more time.

3) The pilot told the passengers it was needed to wait for some more time.

4) The pilot told the passengers that they needed to wait for some more time.

Correct
4
Option is:
Your Option
1
is:
Result: Wrong
Timetaken: 0.0 secs
Explanation: If the main clause is stated in the past tense, the following
subordinate clause also assumes a past tense. ?Told? is the past tense,
so the subordinate claus e must have a past tense ?needed?. The
correct sentence is, ?The driver told the passengers that they needed
to w ait for some more time?.

The given sentences when properly sequenced form a coherent paragraph. Each sentence is

labelled with a number. Choose the most logical order of sentences from among the four

given choices to construct a coherent paragraph. A. and Harry Seeley in the 19th century,

when little was known about the diversity of the group, B. resulting in the description of
48.
dozens of species, C. However, more recent findings of pterosaur fossils have challenged

views on their diversity. D. Pterosaurs from the Cretaceous of England were first described

by British naturalists Richard Owen E. all based on very fragmentary remains, represented

mostly by the tips of the snouts of these animals.

1) DCABE

2) DABEC

3) CEBAD

4) DCEBA

Correct
2
Option is:
Your Option
3
is:
Result: Wrong
Timetaken: 0.0 secs
In the given question, statement D marks the beginning of the
paragraph as it is the only introductory topic and introduces
Pterosaurs from the Cretaceous of England. Further statement A
serves as the continuation of statement D. A gain if we check the
Explanation:
ending statement, A&D cannot be an ending statement a nd among
C&E statement C will be the proper ending statement. Looking into
the options, DABEC is the only one such option and hence is the
correct ans wer.
49.
Choose the word which is farthest in meaning to the given word Humongous

1) Expiate
2) Illustrious

3) Hoyden

4) Miniscule

Correct
4
Option is:
Your Option
1
is:
Result: Wrong
Timetaken: 0.0 secs
Humongous - very large Expiate - Make up for, compensate or
causing offence Illustrious - Widely known and esteemed Hoyden -
Explanation:
A girl who behaves in a boyish manner Miniscule - Very small
Miniscule is the right word here.
50.
Choose the word which is farthest in meaning to the given word Compendious

1) Inchoate

2) Windy

3) Condign

4) Compilation

Correct
2
Option is:
Your Option
4
is:
Result: Wrong
Timetaken: 0.0 secs
Compendious - Briefly giving the gist of something, brief. Inchoate -
Only partly in existence; imperfectly formed Windy - Using or
Explanation: containing too many words Condign - Fitting or appropriate and
deserved; used especially of punishment Compilation - Something
that is compiled, Collection.windy is the right word here
1.Read the sentence to find out whether there is any grammatical error in it. The error, if any, will
be in one part of the sentence. The letter of that part is the answer. If there is no error, the answer
is ?E?. (Ignore ? the errors of punctuation, if any) With our sextants (A)/ we find when the center
of the sun(B) /is on the celestial meridian corresponding to the terrestrial one; and at that
instance(C)/ it is noon where we are .(D)/

1) A2) B3) C4) D5) E


Correct
3
Option is:
Your Option
2
is:
Result: Wrong
Timetaken: 0.0 secs
Here, ?is on the celestial meridian corresponding to the terrestrial one; and at t
hat instance? should be replaced with ?is on the celestial meridian correspondi
Explanation:
ng to the terrestrial one; and at that instant? because here we are talking abou t
a particular point of time not example.
2.In the following questions, fill the blank space. Geophysicists first began to appreciate the
smoldering origins of the land ______ the sea, known formally as ocean crust, in the early
1960?s.

1) below2) above3) under4) in


Correct
3
Option is:
Your Option
4
is:
Result: Wrong
Timetaken: 0.0 secs
Here, the correct preposition is "UNDER".Compared to "below", "under" is
Explanation: mo re often used to talk about 3-dimensional objects. For example, you'd talk
about something being under a table, under a book, etc.
3.Read the sentence to find out whether there is any grammatical error in it. The error, if any, will
be in one part of the sentence. The letter of that part is the answer. If there is no error, the answer
is ?E?. (Ignore ? the errors of punctuation, if any) Jane was (A)/ sitting with her mother and
sisters, (B) /reflecting about what she had heard, and doubting whether (C)/ she was authorised
to mention it, when Sir George himself appeared, sent by his lovely daughter, (D)/ to announce
her engagement with the family.

1) A2) B3) C4) D5) E


Correct Option
4
is:
Your Option is: 1
Result: Wrong
Timetaken: 0.0 secs
To announce her engagement with the family should be replaced with ? to
Explanation:
ann ounce her engagement to the family.?
4.Fact, logic, and data drive the world of economics. Experts analyze past and current
information in order to draw conclusions on possible future market behavior. In turn, investors
read these interpretations and combine them with their own data collection, forming a personal
belief and acting accordingly. These actions create more data, which the analysts then collect and
examine, creating an interminable loop of information exchange. This unending interaction
seems simple enough; however, the accuracy of economic forecasting is a testament to the true
complexity of this analyst-data-investor matrix. A major source of uncertainty in the field of
forecasting is the fact that, in order to convey pure data to investors in ways they can understand,
a medium of communication must be used. The sheer volume of data being created each second
cannot simply be passed to the investor for comprehension; the analyst must observe it, draw
conclusions, and somehow deliver his or her ?factual prediction? to the investor. It is in this
transference from numbers and fact to visuals and argument that data speaks slyly. A powerful
example of this phenomenon occurs when viewing any visual image attributed to an economic
forecast. Primarily, the transference of data to graphical representation is of great concern.
Economic graphs are unique in the realm of visual images in that they convey an extremely large
amount of concrete information using simple lines and values. Their ability to reduce complex
theory into a simple visual concept certainly renders them as powerful rhetorical devices.
However, this potency is often misused by those who purport to convey truth in forecasting. The
strong logical appeal of a graph can easily overshadow the content of an economic analysis, a
dangerous effect in the hands of an analyst looking for one small piece of data in the
insurmountable pile of information that exists and that grows constantly. Furthermore, many
economic reporters and quasi-forecasters use non-graphical images in their clouded rhetoric,
obscuring the financial analysis with a pathetic appeal. Clearly, visual rhetoric is a subtle yet
potent source of manipulation in the economic realm. This practice of misrepresentation and
misdirection with images abounds in financial literature, detracts from the truth value of
economic prediction, and casts further doubt on the practice?s merit. According to the passage,
which of the following is a demonstration of true complexity of the analyst-data-investor matrix

1) Unending interaction2) Uncertainty in the field of forecasting3) Interminable loop of


information exchange. The accuracy of economic forecastin4) The accuracy of economic
forecasting
Correct
4
Option is:
Your Option
2
is:
Result: Wrong
Timetaken: 0.0 secs
Explanation: Fourth sentence of the first paragraph "This unending interaction seems simpl
e enough; however, the accuracy of economic forecasting is a testament to th e
true complexity of this analyst-data-investor matrix . "clearly shows that ?The
accuracy of economic forecasting? from the given options is a demonstration
of true complexity of the analyst-data-investor matrix.
5.Choose the contextual meaning of the given word. It is your own indolence that served as a
cause for your failure in the exam.

1) Lethargy2) Vivaciousness3) Concern4) diligence


Correct
1
Option is:
Your Option
2
is:
Result: Wrong
Timetaken: 0.0 secs
indolence-Inactivity resulting from a dislike of work Lethargy-Inactivity;
showing an unusual lack of energy Vivaciousness-Vigorous and animated
Explanation: diligence- Conscientiousness in paying proper attention to a task; giving the d
egree of care required in a given situation;Persevering determination to perfor
m a task Lethargy is the suitable word here
6.In the following questions, select the word or phrase that is similar in the meaning to given
word. Sapient

1) Perspicacious2) Effusively emotional3) ludicrous4) defenseless


Correct
1
Option is:
Your Option
4
is:
Result: Wrong
Timetaken: 0.0 secs
The word means acutely insightful and wise and also from the given options
the word means the same. Hence, is the word which is closet in meaning to the
Explanation:
given word. ludicrous-Broadly or extravagantly humorous; resembling
farce;Incongruous; i nviting ridicule
7.Fact, logic, and data drive the world of economics. Experts analyze past and current
information in order to draw conclusions on possible future market behavior. In turn, investors
read these interpretations and combine them with their own data collection, forming a personal
belief and acting accordingly. These actions create more data, which the analysts then collect and
examine, creating an interminable loop of information exchange. This unending interaction
seems simple enough; however, the accuracy of economic forecasting is a testament to the true
complexity of this analyst-data-investor matrix. A major source of uncertainty in the field of
forecasting is the fact that, in order to convey pure data to investors in ways they can understand,
a medium of communication must be used. The sheer volume of data being created each second
cannot simply be passed to the investor for comprehension; the analyst must observe it, draw
conclusions, and somehow deliver his or her ?factual prediction? to the investor. It is in this
transference from numbers and fact to visuals and argument that data speaks slyly. A powerful
example of this phenomenon occurs when viewing any visual image attributed to an economic
forecast. Primarily, the transference of data to graphical representation is of great concern.
Economic graphs are unique in the realm of visual images in that they convey an extremely large
amount of concrete information using simple lines and values. Their ability to reduce complex
theory into a simple visual concept certainly renders them as powerful rhetorical devices.
However, this potency is often misused by those who purport to convey truth in forecasting. The
strong logical appeal of a graph can easily overshadow the content of an economic analysis, a
dangerous effect in the hands of an analyst looking for one small piece of data in the
insurmountable pile of information that exists and that grows constantly. Furthermore, many
economic reporters and quasi-forecasters use non-graphical images in their clouded rhetoric,
obscuring the financial analysis with a pathetic appeal. Clearly, visual rhetoric is a subtle yet
potent source of manipulation in the economic realm. This practice of misrepresentation and
misdirection with images abounds in financial literature, detracts from the truth value of
economic prediction, and casts further doubt on the practice?s merit. Match the words with their
meanings i. sly-A.having the specious appearance of claiming ii.potent-B.Strong evidence for
something iii.testament-C.Having great influence iv.purport-D.marked by skill in deception

1) i - A, ii - B, iii - C, iv - D2) i - D, ii - C, iii - A, iv - B3) i - B, iii - C, iv - D, ii - A4) i - D, ii - C,


iii - B, iv - A
Correct
4
Option is:
Your Option
2
is:
Result: Wrong
Timetaken: 0.0 secs
Here, means appear to be or doing something especially falsely. Als o means
having great power, influence, or effect. Hence, from the given options we
Explanation:
have only one such option where the above words matches with their
meanings. Therefore, the correct matching option is i - D, ii - C, iii - B, iv - A.
8.The given sentences when properly sequenced form a coherent paragraph. Each sentence is
labelled with a number. Choose the most logical order of sentences from among the four given
choices to construct a coherent paragraph. A. Many citizens argue that serious criminals should
be executed. B. Other citizens say that no one has the right to take a life and that capital
punishment is not a deterrent to crime C. They believe that killing criminals will set an example
for others and also rid society of a cumbersome burden. D. The punishment of criminals has
always been a problem for society. E.Citizens have had to decide whether offenders such as first-
degree murderers should be killed in a gas chamber, imprisoned for life, or rehabilitated and
given a second chance in society.
1) EACBD2) DACBE3) ABCDE4) DEACB
Correct
4
Option is:
Your Option
1
is:
Result: Wrong
Timetaken: 0.0 secs
In the given question, the fragment D presents the beginning of the paragraph
as it discusses an issue about punishment being given to criminals.The AC link
is obvious as "they" in C refers to the many citizens who argue that seriou s
criminals should be executed which is in A.This link is present in DACBE a
Explanation:
nd DEACB. Further in fragment E, the problems with the society regarding cri
minals has been discussed and hence this fragment comes second in the par
agraph.Only one such option meeting the above criteria. Hence, DEACB form
s a coherent paragraph.
9.Read the sentence to find out whether there is any grammatical error in it. The error, if any, will
be in one part of the sentence. The letter of that part is the answer. If there is no error, the answer
is ?E?. (Ignore ? the errors of punctuation, if any) Being a short(A)/ vacation, we(B) /had to
return without(C)/ visiting many of the places(D).

1) A2) B3) C4) D5) E


Correct Option is: 1
Your Option is: 4
Result: Wrong
Timetaken: 0.0 secs
Explanation: Being a short vacation should be replaced with.It being a short vacation.
10.Choose the word which is farthest in meaning to the given word Celerity

1) Celibacy2) Alacrity3) Hurry4) Inertia


Correct
4
Option is:
Your Option
1
is:
Result: Wrong
Timetaken: 0.0 secs
Explanation: Here,means movement and from the given options means to remain
unchanged. Hence, inertia is farthest in meaning to the given word. Celibacy-
An unmarried status Alacrity-Liveliness and eagerness
11.Read the sentence to find out whether there is any grammatical error in it. The error, if any,
will be in one part of the sentence. The letter of that part is the answer. If there is no error, the
answer is . (Ignore ? the errors of punctuation, if any) A course of lectures (A) /were arranged
(B)/ by the students of tenth class.(C)

1) A2) B3) C4) D5) E


Correct Option is: 2
Your Option is: 4
Result: Wrong
Timetaken: 0.0 secs
Explanation: Here, should be replaced with .
12.Choose the word which is farthest in meaning to the given word Foment

1) Deter2) Chicanery3) Expel4) Foster


Correct
1
Option is:
Your Option
1
is:
Result: Correct
Timetaken: 0.0 secs
Foment-Try to stir up public opinion Deter-Try to prevent; show opposition to
Chicanery-The use of tricks to deceive someone (usually to extract money fro
Explanation:
m them) Foster-Promote the growth of expel-Force to leave or move out Deter
is farthest in meaning from Foment
13.The given sentences when properly sequenced form a coherent paragraph. Each sentence is
labelled with a number. Choose the most logical order of sentences from among the four given
choices to construct a coherent paragraph. A. Moreover, private sector competitors claim to be
moving from aluminium manufacture to specialized uses of the metal B. The new concern could
probably supply the metal to established companies for use as input C. As we all know, there is
still shortage of the metal D. All in all, though, the new plant will not threaten existing
manufacturers in a big way

1) ADCB2) BCDA3) CDBA4) DBCA


Correct
4
Option is:
Your Option
3
is:
Result: Wrong
Timetaken: 0.0 secs
D talks about the new plant not being a threat and B talks about the new conc
ern supplying the metal.Thus,B follows D.C talks about the shortage of metal
Explanation:
and we can infer that C follows B.The use of word "moreover" in A helps to
de duce that it follows C.Hence the correct option is DBCA
14.In the following questions, select the word or phrase that is similar in meaning to the given
word. Grovel

1) bemuse2) chafe3) anile4) fawn


Correct Option
4
is:
Your Option is: 2
Result: Wrong
Timetaken: 0.0 secs
means to show submission of fear and from the given options al so means to
Explanation:
show submission of fear. Hence,<="" td="">
15.In the following questions, fill the blank space. Rock found at the bottom of the Grand
Canyon is ______ 2 billion years old

1) below2) around3) under4) about


Correct Option is: 2
Your Option is: 4
Result: Wrong
Timetaken: 0.0 secs
Explanation: "Around" is the most suitable preposition.
16.Choose the contextual meaning of the given word. She has also stirred up controversy among
some feminists, who find her too flippant.

1) disparaging2) Mature3) Frivolous4) Pejorative


Correct
3
Option is:
Your Option
2
is:
Result: Wrong
Timetaken: 0.0 secs
Explanation: means frivolously disrespectful, shallow, or lacking in seriousness a nd
also means "Not serious in content, attitude or behaviour';. Henc e, is the most
appropriate word. Pejorative-Expressing disapproval disparaging-Expressive
of low opinion
17.The given sentences when properly sequenced form a coherent paragraph. Each sentence is
labelled with a number. Choose the most logical order of sentences from among the four given
choices to construct a coherent paragraph. A. The wind at his back also helped him on his course
B. Dark as the night was he found it easy to follow the track C. Not only that he was confident
that there would be few, if any, traces of his passage D. Thanks to the drought, it was hard and
firm and he made good speed

1) CABD2) DACB3) BDCA4) DBAC


Correct
3
Option is:
Your Option
1
is:
Result: Wrong
Timetaken: 0.0 secs
B is the opening sentence as it introduces the night and the track.The 'passa
ge' is on the track;hence, C follows.'Hard and firm' describes the track and foll
Explanation:
ows C.The word 'also' in A signifies that it should follow D.Hence the correct
a nswer is BDCA
18.Read the sentence to find out whether there is any grammatical error in it. The error, if any,
will be in one part of the sentence. The letter of that part is the answer. If there is no error, the
answer is ?E?. (Ignore ? the errors of punctuation, if any) A train journey in Kerala come (A) /
with a series of excellent sightseeing, (B)/ which is something you will (C) /surely take delight
in(D).

1) A2) B3) C4) D5) E


Correct Option
1
is:
Your Option is: 3
Result: Wrong
Timetaken: 0.0 secs
A train journey in Kerala come (A) / with a series of excellent sightseeing,
Explanation:
(B)/ which is something you will (C) /surely take delight in(D).
19.In the question a part of the sentence is italicised. Alternatives to the italicised part are given
which may improve the sentence. Choose the correct alternative. In case no improvement is
needed. Option ?E? is the answer. The candidate has spent much time getting financial support
for his upcoming campaign.

1) Collecting2) Garnering3) reaping4) scattering


Correct Option
2
is:
Your Option is: 4
Result: Wrong
Timetaken: 0.0 secs
Here from the given options ?Garner? means to gather or earn. Hence,
Explanation:
Garnerin g is the most suitable word to replace the one italicized.
20.Choose the word which is farthest in meaning to the given word Hypothesize

1) Surmise2) postulate3) repudiate4) Deduce


Correct
3
Option is:
Your Option
2
is:
Result: Wrong
Timetaken: 0.0 secs
Hypothesize-To believe especially on uncertain or tentative grounds repudiate-
Refuse to acknowledge, ratify, or recognize as valid Deduce-Conclude by
Explanation: reasoning; in logic postulate-a proposition that is accepted as true in order to
provide a basis for l ogical reasoning surmise-A message expressing an
opinion based on incomplete evidence Surmise is the synonym of Hypothesize
21.Fact, logic, and data drive the world of economics. Experts analyze past and current
information in order to draw conclusions on possible future market behavior. In turn, investors
read these interpretations and combine them with their own data collection, forming a personal
belief and acting accordingly. These actions create more data, which the analysts then collect and
examine, creating an interminable loop of information exchange. This unending interaction
seems simple enough; however, the accuracy of economic forecasting is a testament to the true
complexity of this analyst-data-investor matrix. A major source of uncertainty in the field of
forecasting is the fact that, in order to convey pure data to investors in ways they can understand,
a medium of communication must be used. The sheer volume of data being created each second
cannot simply be passed to the investor for comprehension; the analyst must observe it, draw
conclusions, and somehow deliver his or her ?factual prediction? to the investor. It is in this
transference from numbers and fact to visuals and argument that data speaks slyly. A powerful
example of this phenomenon occurs when viewing any visual image attributed to an economic
forecast. Primarily, the transference of data to graphical representation is of great concern.
Economic graphs are unique in the realm of visual images in that they convey an extremely large
amount of concrete information using simple lines and values. Their ability to reduce complex
theory into a simple visual concept certainly renders them as powerful rhetorical devices.
However, this potency is often misused by those who purport to convey truth in forecasting. The
strong logical appeal of a graph can easily overshadow the content of an economic analysis, a
dangerous effect in the hands of an analyst looking for one small piece of data in the
insurmountable pile of information that exists and that grows constantly. Furthermore, many
economic reporters and quasi-forecasters use non-graphical images in their clouded rhetoric,
obscuring the financial analysis with a pathetic appeal. Clearly, visual rhetoric is a subtle yet
potent source of manipulation in the economic realm. This practice of misrepresentation and
misdirection with images abounds in financial literature, detracts from the truth value of
economic prediction, and casts further doubt on the practice's merit. According to the passage,
what is market behavior based upon?

1) The conclusions draw-ed by the experts based on the past and current information.2) The
factual predictions made by the analysts.3) The ability of the investor to understand the content
of the interpretations which in turn are affected by medium of communication through which the
crucial data is represented4) The opinions formed by the investors based on the data they have
collected and on the interpretations by experts.
Correct
4
Option is:
Your Option
3
is:
Result: Wrong
Timetaken: 0.0 secs
The third line of the first paragraph clearly answers the q uestion.."In turn,
Explanation: investors read these interpretations and combine them with their own data
collection, forming a p ersonal belief and acting accordingly."
22.Fact, logic, and data drive the world of economics. Experts analyze past and current
information in order to draw conclusions on possible future market behavior. In turn, investors
read these interpretations and combine them with their own data collection, forming a personal
belief and acting accordingly. These actions create more data, which the analysts then collect and
examine, creating an interminable loop of information exchange. This unending interaction
seems simple enough; however, the accuracy of economic forecasting is a testament to the true
complexity of this analyst-data-investor matrix. A major source of uncertainty in the field of
forecasting is the fact that, in order to convey pure data to investors in ways they can understand,
a medium of communication must be used. The sheer volume of data being created each second
cannot simply be passed to the investor for comprehension; the analyst must observe it, draw
conclusions, and somehow deliver his or her ?factual prediction? to the investor. It is in this
transference from numbers and fact to visuals and argument that data speaks slyly. A powerful
example of this phenomenon occurs when viewing any visual image attributed to an economic
forecast. Primarily, the transference of data to graphical representation is of great concern.
Economic graphs are unique in the realm of visual images in that they convey an extremely large
amount of concrete information using simple lines and values. Their ability to reduce complex
theory into a simple visual concept certainly renders them as powerful rhetorical devices.
However, this potency is often misused by those who purport to convey truth in forecasting. The
strong logical appeal of a graph can easily overshadow the content of an economic analysis, a
dangerous effect in the hands of an analyst looking for one small piece of data in the
insurmountable pile of information that exists and that grows constantly. Furthermore, many
economic reporters and quasi-forecasters use non-graphical images in their clouded rhetoric,
obscuring the financial analysis with a pathetic appeal. Clearly, visual rhetoric is a subtle yet
potent source of manipulation in the economic realm. This practice of misrepresentation and
misdirection with images abounds in financial literature, detracts from the truth value of
economic prediction, and casts further doubt on the practice?s merit. What is the meaning of the
word ?Insurmountable? in the second paragraph according to the passage?

1) Undefeatable2) Overwhelming3) Insoluble4) superable


Correct
2
Option is:
Your Option
4
is:
Result: Wrong
Timetaken: 0.0 secs
In the paragraph, the word ?Insurmountable? means ?too great to be overcom
e?. And from the given options, superable-capable of being overcome;
Explanation: surmountable. ?Overwhelming? means very great in amount and fits the
contextual meanin g. Hence, from the given options ?overwhelming? means ?
insurmountable?.
23.In the following questions, fill the blank space. _____________ before I investigated the
strange noise in the neighbourhood.

1) I called the police2) I had called the police3) I was calling the police4) I should call the police
Correct
2
Option is:
Your Option
2
is:
Result: Correct
Timetaken: 0.0 secs
The past perfect is used when two events happened in the past, with one past
action having occurred even before the other past action. To form the past perf
Explanation:
ect, use had and the past participle of a verb in one part of the sentence. Often,
the regular past tense is used in the other part of the sentence
24.DIRECTIONS: Fill in the following with appropriate words. Question: The aria was ______
with the tempo charging continuously in a unpredictable fashion

1) abstemious2) cacophonous3) impecunious4) capricious


Correct
4
Option is:
Your Option
4
is:
Result: Correct
Timetaken: 0.0 secs
Aria is a song for solo voice. abstemious means indulging only very
moderately in something. cacophonous means involving or producing a harsh
Explanation: mixture of sounds. impecunious means having little or no money. capricious
means given to sudden and unaccountable changes of mood. So,the correct
answer will be option D.
Directions :
Fill in the blanks
1.
There were no fire crackers in the market ____ the end of Christmas that year.

1) By
2) In
3) to
4) at

Correct
4
Option is:
Your Option
4
is:
Result: Correct
Timetaken: 0.0 secs
Here, the most suitable option to fill the blanks is "at". If
you use ?in? you are thinking of the end of a process, or an
Explanation:
argument or a ne gotiation, rather than a point in time or a
particular place. Synonym - at the la st moment

Directions :
Fill in the blanks
2.
There was a picture which looked like a lizard ___ page 10.

1) By
2) In
3) At
4) On
Correct Option
4
is:
Your Option is: 2
Result: Wrong
Timetaken: 0.0 secs
Here, the most suitable word is "on". ie, eg:No movement
Explanation: or action is involve d, the picture is and always will be on
page 10

Every human being, after the first few days of his life, is a product of two
3. factors: on the one hand, there is his inborn endowment; and on the other hand,
there is an effect of the environment.
1) congenital
2) imminent
3) Inbuilt
4) inherent

Correct
1
Option is:
Your Option
2
is:
Result: Wrong
Timetaken: 0.0 secs
Inherent - existing as an essential constituent or
characteristic. Inherent and inbuilt are synonyms which
means existing as an essential cons tituent or characteristic.
Explanation:
'Inborn' means existing from birth and 'Congenital' also
means the same. Hence congenital is the most suitable word
to replace 'inborn'.

Directions for questions: Read the sentence to find out whether there is any
grammatical error in it. The error, if any, will be in one part of the sentence.
4. The letter of that part is the answer. If there is no error, the answer is ?E?.
I have(A) /learnt this(B) /song word (C) / for word(D).

1) A
2) B
3) C
4) D
5) E

Correct Option
4
is:
Your Option is: 2
Result: Wrong
Timetaken: 0.0 secs
Here, in this sentence 'for word' should be replaced with
Explanation:
'by word'.

In the question a part of the sentence is italicized. Alternatives to the italicized


part are given which may improve the sentence. Choose the correct alternative.
5. There were many weeks and months when Bletchley Park could not decode the
intercepted messages.

1) interpret
2) construe
3) comprehend
4) decipher

Correct
4
Option is:
Your Option
3
is:
Result: Wrong
Timetaken: 0.0 secs
'Decode' means to convert (a coded message) into
intelligible language. From the given options 'decipher'
Explanation: means to convert (a text written in code, or a cod ed signal)
into normal language. Hence, 'decipher' is the most suitable
alterna tive.

6. Directions for questions :


In the following questions, select the word or phrase that is similar in meaning
to the given word.
Billingsgate
1) Exaltation
2) Encomium
3) Praise
4) Obloquy

Correct
4
Option is:
Your Option
4
is:
Result: Correct
Timetaken: 0.0 secs
?Billingsgate? means foul-mouthed or obscene abuse and
Explanation: from the given option s ?Obloquy? means abuse. Hence,
Obloquy is the most suitable word.

Directions for questions :


Choose the word which is farthest in meaning to the given word
7.
Expostulate

1) Remonstrate
2) Defalcate
3) Assert
4) Acquiesce

Correct
4
Option is:
Your Option
2
is:
Result: Wrong
Timetaken: 0.0 secs
Explanation: Expostulate - Reason with (somebody) for the purpose of
dissuasionRemonstrate-Argue in protest or opposition ?
Acquiesce? means to accept something reluctantly but
without protest. Henc e, ?Acquiesce? is the word which is
farthest in meaning to the given word.

Directions for questions: Read the sentence to find out whether there is any
grammatical error in it. The error, if any, will be in one part of the sentence.
8. The letter of that part is the answer. If there is no error, the answer is ?E?.
If you wish(A)/, I shall help(B) / find you a more suitable bride (C).

1) A
2) B
3) C
4) E

Correct Option
3
is:
Your Option is: 2
Result: Wrong
Timetaken: 0.0 secs
The phrase 'find you a more suitable bride' should be
Explanation: replaced with 'you find a more suitable bride' to make it
meaningful.

Fill in the blanks with the most suitable word from the given options.
9. He succeeded ___________ perseverance and sheer hard work.

1) by dint of
2) by virtue of
3) in consequence of
4) None of these

Correct Option
1
is:
Your Option is: 2
Result: Wrong
Timetaken: 0.0 secs
The most suitable option from the given options is ?by
Explanation:
dint of? meaning 'by means of'.

The genocides in Bosnia, apart from being misdescribed in the most sinister
10.
and false manner as 'ethnic cleansing' were criticised vehemently.
1) Artless
2) Disingenuous
3) Beguile
4) Ingenuous

Correct
2
Option is:
Your Option
2
is:
Result: Correct
Timetaken: 0.0 secs
Ingenuous - characterized by an inability to mask your
feelings; not devious Disingenuous - not straightforward or
Explanation: candid; giving a false appearance of fran kness From the
given options, 'Disingenuous' also means false. Hence,
'Disingenuou s' is the most suitable word.

Directions :
Choose the contextual meaning of the highlighted word.
11. For his unsavory remarks on Martin L. King in the media, the minister faced
libel charges from his descendants living in the USA.

1) Calumny
2) Perfidy
3) Treason
4) Knavery

Correct
1
Option is:
Your Option
3
is:
Result: Wrong
Timetaken: 0.0 secs
?Libel? means a false and malicious publication printed for
the purpose of defa ming a living person ?Perfidy? is an act
of deliberate betrayal ?Treason? is a crime that undermines
the offender?s government or Disloyalty b y virtue of
subversive behaviour ?Knavery? means lack of honesty; acts
Explanation: of lying, cheating or stealing ?calumny?means ?A false
accusation of an offence or a malicious misrepresent ation of
someone?s words or actions? or ?An abusive attack on a
person?s char acter or good name? ?Calumny? gives the
same meaning with reference to the context. Hence, ?Cal
umny? is the most suitable word.

The given sentences when properly sequenced form a coherent paragraph. Each
sentence is labelled with a number. Choose the most logical order of sentences
from among the four given choices to construct a coherentparagraph.
A. He owned one acre of land, leased two more and grew cotton on all three.
B.?Cotton has given us shattered dreams,? said one old farmer in Nagara
village.
12.
C. Confronted with falling prices, mountingdebts and pest attacks, he
committed harakiri.
D. After making a loss in the first year, he leased yet more land in an attempt to
recover.
E. The suicide of Samala Mallaiah in Nagara village grabbed media headlines.

1) EACDB
2) BAEDC
3) EADCB
4) BEADC

Correct
3
Option is:
Your Option
4
is:
Result: Wrong
Timetaken: 0.0 secs
Explanation: In the given question, statement E discusses an issue and
hence serves as s tarting statement for the paragraph. Further
statement A uses a personal pron oun which is being used for
Samala Mallaiah, who has been introduced in stat ement E
and this statement A further talks more about Samala
Mallaiah. Hen ce, this statement comes second in the
paragraph. Statement D discusses a bout his problem and
hence this statement can be put at number three in the
paragraph to form logical coherent paragraph. Looking into
the options, EADC B and BEADC.In BEADC, cotton in B is
linked with A so A should precede B.So EADCB is the
answer.

Directions :
Choose the contextual meaning of the highlighted word
13.
She is almost incredulous at how her circumstances have changed.

1) Incredible
2) Ingenuous
3) Naive
4) Unbelievable

Correct
4
Option is:
Your Option
2
is:
Result: Wrong
Timetaken: 0.0 secs
?Incredulous? means unbelieving. In the given options we
Explanation: have one such option as unbelievable. Hence, ?
unbelievable? gives the contextual meaning.

14. Directions for questions : The given sentences when properly sequenced form a
coherent paragraph. Each sentence is labelled with a number. Choose the most
logical order of sentences from among the four given choices to construct a
coherentparagraph.
A. Because of their unusual six-sided shape, these satellites create brief flashes
of reflected sunlight, called Iridium flares
B. The far less famous Iridium satellites, a fleet of 66 telecommunication relays
in low-Earth orbit, can shine brighter still.
C. Satellite watching is also ideal for the impatient, since good appearances
happen all the time
D. That can reach a brightness of magnitude -8, about 30 times brighter than
Venus at its peak.

1) ACDB
2) CDAB
3) ABDC
4) BDAC

Correct
4
Option is:
Your Option
3
is:
Result: Wrong
Timetaken: 0.0 secs
Let us go by the process of elimination ACDB and ABDC
starts with A.The us e of pronouns 'their' and 'these',refers to
some kind of satellite which is mentio ned in B.So B should
Explanation:
precede A and Moreover D is the continuation of B.This link
is present in ABDC and BDAC. But the passage cannot start
with A .So t he only option left is BDAC.

Directions for questions :


Choose the word which is farthest in meaning to the given word
15.
Recreant

1) Irksome
2) Saint
3) Courageous
4) Renegade

Correct
3
Option is:
Your Option
2
is:
Result: Wrong
Timetaken: 0.0 secs
?Recreant? is an "abject coward" or "A disloyal person who
betrays or deserts his cause or religion or political party or
Explanation: friend etc.". Renegade is a synonym of Recreant And from
the given options ?Courageous? is the word which is farthest
in meani ng to the given word.

Directions for questions: Answer the questions based on the given passage
The power of music to control the spirit has always been understood, but within
the last decade, new technologies have made visible the interaction between
music and the physical brain. The making and processing of music involves
structures, networks, and pathways throughout the brain, from the highest order
of conscious reaction to the lowest unconscious levels of response.
Music has been shown to stimulate the brain?s primary engines of human
capacity. Musical engagement exercises attentional networks and executive
function, evokes emotional response and stimulates the central nervous system,
and appears to activate the human mirror-neuron system, supporting the
coupling between perceptual events (visual or auditory) and motor actions (leg,
arm/hand, or vocal/articulatory actions).
At one time, theories of human brain development argued that there was little
16. or no growth of brain cells after age 30.
But recent studies of music and the brain have shown this view to be erroneous.
The brain is a plastic organ and music itself has the power to shape the brain?s
development into later life. The implications of this finding are huge. Providing
opportunities for people to experience music in many settings can have a
profound impact on their healthy development. Exposure to music alters the
physical structure of the brain. Engaging in musical activities not only shapes
the organization of the developing brain but also produces long-lasting changes
even after brain maturation is complete. For example, those who frequently
play a musical instrument are less likely to develop dementia compared to
those who do not, revealing that music works not only to train the brain, but
also to protect cognitive functioning.
What is the contextual meaning of the word ?plastic? as used in the passage?

1) Capable of being influenced or formed


2) Able to adjust readily to different conditions
3) Able to acquire new information and skills
4) Ability to improve its utility with use

Correct Option
1
is:
Your Option is: 4
Result: Wrong
Timetaken: 0.0 secs
Third paragraph second line. Here, the word plastic
Explanation:
implies capable of being in fluenced or formed.

Directions :
Choose the contextual meaning of the highlighted word.
17. If you still find yourself completely flummoxed, pay someone else to do the
work for you.

1) Nonplussed
2) Bustle
3) Excogitate
4) Concoct

Correct
1
Option is:
Your Option
2
is:
Result: Wrong
Timetaken: 0.0 secs
?Excogitate? means ?Think deeply about a subject or
question over a period of time? concoct is to devise or invent
Explanation: or cook up ?nonplussed? means confused. Hence,
nonplussed is the most suitable conte xtual meaning for the
given word.

18. Directions for questions: Answer the questions based on the given passage
The power of music to control the spirit has always been understood, but within
the last decade, new technologies have made visible the interaction between
music and the physical brain. The making and processing of music involves
structures, networks, and pathways throughout the brain, from the highest order
of conscious reaction to the lowest unconscious levels of response.
Music has been shown to stimulate the brain?s primary engines of human
capacity. Musical engagement exercises attentional networks and executive
function, evokes emotional response and stimulates the central nervous system,
and appears to activate the human mirror-neuron system, supporting the
coupling between perceptual events (visual or auditory) and motor actions (leg,
arm/hand, or vocal/articulatory actions).
At one time, theories of human brain development arguedthat there was little or
no growth of brain cells after age 30. But recent studies of music and the brain
have shown this view to be erroneous. The brain is a plastic organ and music
itself has the power to shape the brain?s development into later life. The
implications of this finding are huge. Providing opportunities for people to
experience music in many settings can have a profound impact on their healthy
development. Exposure to music alters the physical structure of the brain.
Engaging in musical activities not only shapes the organization of the
developing brain but also produces long-lasting changes even after brain
maturation is complete. For example, those who frequently play a musical
instrument are less likely to develop dementia compared to those who do not,
revealing that music works not only to train the brain, but also to protect
cognitive functioning.
Infer from the passage, what is ?dementia??

1) Loss of memory causing forgetfulness


2) Progressive decline in cognitive function due to damage or disease in the
brain beyond what might be expected from normal aging
3) An abnormality marked by premature aging grey hair and wrinkled skin and
stooped posture in a child
4) Impaired ability to learn to read, learn and develop new skills

Correct
2
Option is:
Your Option
4
is:
Result: Wrong
Timetaken: 0.0 secs
Here, according to the passage the word dementia means
progressive decline in cognitive function due to damage or
Explanation:
disease in the brain beyond what might be expected from
normal aging.

Directions for questions :


Choose the word which is farthest in meaning to the given word
19.
Riveting

1) Boring
2) Gripping
3) Fatuous
4) Acquiesce

Correct Option
1
is:
Your Option is: 2
Result: Wrong
Timetaken: 0.0 secs
?Riveting? means completely engrossing or compelling
Explanation: Gripping is the synonym of Riveting It?s antonym will be
boring

20. Directions for questions: Answer the questions based on the given passage
The power of music to control the spirit has always been understood, but within
the last decade, new technologies have made visible the interaction between
music and the physical brain. The making and processing of music involves
structures, networks, and pathways throughout the brain, from the highest order
of conscious reaction to the lowest unconscious levels of response.
Music has been shown to stimulate the brain?s primary engines of human
capacity. Musical engagement exercises attentional networks and executive
function, evokes emotional response and stimulates the central nervous system,
and appears to activate the human mirror-neuron system, supporting the
coupling between perceptual events (visual or auditory) and motor actions (leg,
arm/hand, or vocal/articulatory actions).
At one time, theories of human brain development arguedthat there was little or
no growth of brain cells after age 30. But recent studies of music and the brain
have shown this view to be erroneous. The brain is a plastic organ and music
itself has the power to shape the brain?s development into later life. The
implications of this finding are huge.
Providing opportunities for people to experience music in many settings can
have a profound impact on their healthy development. Exposure to music alters
the physical structure of the brain. Engaging in musical activities not only
shapes the organization of the developing brain but also produces long-lasting
changes even after brain maturation is complete. For example, those who
frequently play a musical instrument are less likely to develop dementia
compared to those who do not, revealing that music works not only to train the
brain, but also to protect cognitive functioning.
?cognitive functions? refers to a person?s ability to
I. retain memory
II. process thoughts
III. learn new information, skills
IV. read, speak
V. prevent dementia

1) only I
2) all of the above
3) I, II, III, IV
4) II, III, IV

Correct Option
3
is:
Your Option is: 3
Result: Correct
Timetaken: 0.0 secs
According to the passage, it can be inferred that
Explanation:
statements I, II, III, IV are onl y correct.

Directions for questions: Read the sentence to find out whether there is any
grammatical error in it. The error, if any, will be in one part of the sentence.
21. The letter of that part is the answer. If there is no error, the answer is ?E?.
Due to renaissance (A)/ a lot of developments (B)/have happened since(C)

1) A
2) B
3) C
4) E

Correct Option
3
is:
Your Option is: 4
Result: Wrong
Timetaken: 0.0 secs
'have happened since? should be replaced with ?have
Explanation:
happened since then?.
Directions for questions: Read the sentence to find out whether there is any
grammatical error in it. The error, if any, will be in one part of the sentence.
The letter of that part is the answer. If there is no error, the answer is ?E?.
22.
In order to keep fit (A)/, Joyce exercised vigorously(B)/ in the mornings, thrice
a week, at the gym.(C)

1) A
2) B
3) C
4) E

Correct Option
3
is:
Your Option is: 2
Result: Wrong
Timetaken: 0.0 secs
The correct order should be In order to keep fit, Joyce
exercised vigorously at the gym, thrice a week in the
morning.
#1 = Manner (how?) Examples: quickly, with enthusiasm
#2 = Place (where?) Examples: downhole, around the
Explanation:
corner
#3 = Frequency (how often?) Examples: daily, every week
#4 = Time (when?) Examples: before cementing, at noon
#5 = Purpose (why?) Examples: to prevent blowouts, to
keep everyone up todate

DIRECTIONS: Fill in the following with appropriate words.


23. Question: The aria was ______ with the tempo charging continuously in a
unpredictable fashion
1) abstemious
2) cacophonous
3) impecunious
4) capricious

Correct
4
Option is:
Your Option
3
is:
Result: Wrong
Timetaken: 0.0 secs
Aria is a song for solo voice. abstemious means indulging
only very moderately in something cacophonous means
involving or producing a harsh mixture of sounds
Explanation:
impecunious means having little or no money capricious
means given to sudden and unaccountable changes of mood.
So,the correct answer will be Option D

Directions for questions : The given sentences when properly sequenced form a
coherent paragraph. Each sentence is labelled with a number. Choose the most
logical order of sentences from among the four given choices to construct a
coherent paragraph.
A. The role confirmed Smith as a major player
in Hollywood and the go-to guy for summer blockbusters.
B. Heroes need to be able to drop while dispatching their enemies.
24.
C. Smith played an air force pilot leading the counter-attack against the
invading alien forces,
D. The 1996, epic sci-fi disaster movie Independence Day was his next
assignment.
E. And his comedic talents effortlesslytransformed into the pithy one-liners all
action

1) CEBAD
2) DACEB
3) DCAEB
4) AEBCD

Correct
3
Option is:
Your Option
2
is:
Result: Wrong
Timetaken: 0.0 secs
Explanation: In the given question, statement D talks about a movie "The
Independence Da y" ."The role" mentioned in A refers to his
playing an airforce pilot mentioned in C.So C should
precede A.CA link is present only in DCAEB.

Directions for questions: Answer the questions based on the given passage
The power of music to control the spirit has always been understood, but within
the last decade, new technologies have made visible the interaction between
music and the physical brain. The making and processing of music involves
structures, networks, and pathways throughout the brain, from the highest order
of conscious reaction to the lowest unconscious levels of response.
Music has been shown to stimulate the brain?s primary engines of human
capacity. Musical engagement exercises attentional networks and executive
function, evokes emotional response and stimulates the central nervous system,
and appears to activate the human mirror-neuron system, supporting the
coupling between perceptual events (visual or auditory) and motor actions (leg,
arm/hand, or vocal/articulatory actions).
At one time, theories of human brain development arguedthat there was little or
no growth of brain cells after age 30.
But recent studies of music and the brain have shown this view to be erroneous.
25.
The brain is a plastic organ and music itself has the power to shape the brain?s
development into later life. The implications of this finding are huge.
Providing opportunities for people to experience music in many settings can
have a profound impact on their healthy development. Exposure to music alters
the physical structure of the brain. Engaging in musical activities not only
shapes the organization of the developing brain but also produces long-lasting
changes even after brain maturation is complete. For example, those who
frequently play a musical instrument are less likely to develop dementia
compared to those who do not, revealing that music works not only to train the
brain, but also to protect cognitive functioning.
We can conclude from the passage that playing musical instruments benefits a
person.
I. Physiologically.
II. By protecting him from dementia.

1) Both I and II
2) Only II
3) Only I
4) None of these

Correct
3
Option is:
Your Option
2
is:
Result: Wrong
Timetaken: 0.0 secs
From the passage, it is very clear that music helps a person
both physiologic ally and psychologically.In the last line of
the passage, the author states that "For example, those who
Explanation: frequently play a musical instrument are less likely to
develop dementia compared to those who do not."It does?nt
state that pl aying musical instruments guarantees protection
from dementia. Hence, only I is correct.

1.
Directions for questions : Answer the questions based on the given passage

Quantitative easing is an unconventional monetary policy used by central banks to

stimulate the economy when standard monetary policy has become ineffective. A

central bank implements quantitative easing by buying specified amounts of long

term financial assets from commercial banks and other private institutions, thus

increasing the monetary base and lowering the yield on those financial assets. This

is distinguished from the more usual policy of buying or selling government bonds

in order to keep interbank interest rates at a specified target value.Expansionary

monetary policy typically involves the central bank buying short-term government

bonds in order to lower short-term market interest rates. However, when short-term

interest rates are at or close to zero, normal monetary policy can no longer lower

interest rates. Quantitative easing may then be used by monetary authorities to

further stimulate the economy by purchasing assets of longer maturity than short-

term government bonds, and thereby lowering longer-term interest rates further

out on the yield curve. Quantitative easing raises the prices of the financial assets

bought, which lowers their yield. There are two potential benefits. The first is that

the banks get cash in exchange for the gilts they sell back to the government and

the increase in the money supply leads to an increased volume of lending. The

second is that decreasing the supply of gilts pushes up their price. When gilt prices

go up, gilt yields go down and it is gilt yields that determine long-term interest

rates for overdrafts, some fixed-rate mortgage products and most business lending.
Quantitative easing can be used to help ensure that inflation does not fall below

target. Risks include the policy being more effective than intended in acting against

deflation (leading to higher inflation in the longer term, due to increased money

supply), or not being effective enough if banks do not lend out the additional

reserves. According to the IMF and various other economists, quantitative easing

undertaken since the global financial crisis of 2007?08 has mitigated some of the

adverse effects of the crisis.

Infer what the author could be referring to,while talking about the risks involved in

the ?policy being more effective than intended against deflation??

1) the increase in the money supply may lead to an increased volume of borrowing

2) the increase in the money supply may lead to increased volume of lending

3) the already low interest rates could immediately drop leading to excess money in
hands of public

4) It could result in higher inflation due to increased money supply

Correct
4
Option is:
Your Option
2
is:
Result: Wrong
Timetaken: 0.0 secs
Explanation: It is mentioned in the last paragraph,last before line of the
passage that "Risk s include the policy being more effective
than intended in acting against deflati on (leading to higher
inflation in the longer term, due to increased money supp
ly),or not being effective enough if banks do not lend out the
additional reserve s."
The intended effect against deflation is that the volume of
lending should incre ase.If the bank increases volume the
lending, the people will also borrow more increasing the
volume of borrowing.So these two actions cannot be more
than effective than intended.
"the already low interest rates could immediately drop
leading to excess mon ey in hands of public"This is not
mentioned anywhere in the passage.

Directions for questions :

The given sentences when properly sequenced form a coherent paragraph. Each

sentence is labelled with a number. Choose the most logical order of sentences from

among the four given choices to construct a coherent paragraph.

A. And found he had been through life an early riser and a singularly temperate

2. man.

B. An old man named Elm was once a witness in a case tried by Lord Mansfield.

C. The judge was filled with admiration and examined him as to his habitual mode

of living.

D. He gave his evidence with remarkable clearness,

E. Although he was more than eighty year of age.

1) BCEDA

2) ADECB

3) BDECA

4) ABCDE

Correct
3
Option is:
Your Option
2
is:
Result: Wrong
Timetaken: 0.0 secs
Explanation: In the given question, statement B marks the beginning of
the statement as it starts the discussion by addressing an old
man named Elm and the incident associated with him.
Statement D starts with a personal pronoun and further
discusses what Elm did. Hence, statement D comes second in
the paragraph. From the given options, there is only one such
option i.e, BDECA. Hence, BDECA forms the correct
coherent paragraph.

Directions :

Read the sentence to find out whether there is any grammatical error in it. The

error, if any, will be in one part of the sentence. The letter of that part is the

3. answer. If there is no error, the answer is ?E?. (Ignore ? the errors of punctuation, if

any)

Men?s interest in developing (A) /a cure for brain tumour have promoted the (B)

/rapid advances in the abstruse field now known as Genetic Engineering.(C)

1) A

2) B,C

3) B

4) A,B

Correct Option
1
is:
Your Option is: 4
Result: Wrong
Timetaken: 0.0 secs
It should be Man's interest as the subject Here man
Explanation:
represents humankind.

Fill in the blanks:


4.
And so are her __________________________________

1) And so are her big, delicious, annual, decorated, homemade, gingerbread Easter
cookies.
2) And so are her big, home-made, delicious, annual, decorated, gingerbread Easter
cookies.

3) And so are her delicious, big, annual, decorated, home-made, gingerbread Easter
cookies.

4) And so are her big, home-made, annual, delicious, decorated, gingerbread Easter
cookies.

Correct Option
3
is:
Your Option is: 4
Result: Wrong
Timetaken: 0.0 secs
#1 = Determiner ? definite or indefinite articles and
possessives
Examples: the office, your boss
#2 = Observation or opinion
Examples: interesting comment, expensive valve
#3 = Size and shape
Examples: round, three-inch
#4 = Age
Examples: new, five-year-old
Explanation: #5 = Color
Examples: mottled, red
#6 = Origin
Examples: British, home-made
#7 = Material
Examples: stainless steel, limestone
#8 = purpose/Qualifier, often an integral part of the noun
Examples: ball valve, drilling rig
And so are her delicious, big, annual, decorated, home-
made, gingerbread Easter cookies.

Directions for questions:

5. Choose the word which is farthest in meaning to the given word

deciduous

1) temperate
2) tropical

3) boreal

4) evergreen

Correct
4
Option is:
Your Option
1
is:
Result: Wrong
Timetaken: 0.0 secs
?Deciduous? means to shed leaves annually and from the
Explanation: given options it is obvious that ?evergreen? is the word
farthest in meaning to the given word.

Directions for questions : In the question a part of the sentence is bolded.

Alternatives to the bolded part are given which may improve the sentence. Choose

6. the correct alternative. In case no improvement is needed. Option ?E? is the

answer.

He tried hard to persuade his father into buying him a bike.

1) wheedle

2) stimulate

3) induce

4) influence

Correct
1
Option is:
Your Option
4
is:
Result: Wrong
Timetaken: 0.0 secs
From the given options ?wheedle? means to use
endearments or flattery to per suade someone to do
Explanation:
something or give one something. Hence, ?wheedle? is t he
most suitable word for the given italicized word.

Directions : Read the sentence to find out whether there is any grammatical error in

it. The error, if any, will be in one part of the sentence. The letter of that part is the

answer. If there is no error, the answer is . (Ignore ? the errors of punctuation, if

7. any)

The Las Ketchup girls are(A) / a Spanish rock and (B) / roll band.(C)

1) A

2) B

3) C

4) D

Correct
1
Option is:
Your Option
4
is:
Result: Wrong
Timetaken: 0.0 secs
Here "Las Ketchup girls" refers to a band. So the subject is
Explanation: singular. The subject should agree with the verb. The Las
Ketchup girls is a Spanish rock and roll band.

8.
Directions :
Change the speech of the sentence.

"Why don?t you start again?"

1) She suggested me to start again.

2) She suggested why didn?t I start again.

3) She suggested that I started again

4) She suggested that I start again

Correct
3
Option is:
Your Option
2
is:
Result: Wrong
Timetaken: 0.0 secs
The subjunctive is typically used after two structures: the
Explanation: verbs: ask, command, demand, insist, propose, recommend,
request, sug gest + that She suggested that I started again

Directions :

Read the sentence to find out whether there is any grammatical error in it. The

error, if any, will be in one part of the sentence. The letter of that part is the

answer. If there is no error, the answer is ?E?. (Ignore ? the errors of punctuation, if
9.
any)

A week ago newspapers reported that (A) /a handful of the big Western oil

companies were close to unveiling contracts for work in Iraq, (B) / whose

underexploited oil reserves are probably (C) /second only to Saudi Arabia. (D)

1) A

2) B
3) C

4) D

Correct
4
Option is:
Your Option
3
is:
Result: Wrong
Timetaken: 0.0 secs
A week ago newspapers reported that a handful of the big
Western oil companies were close to unveiling contracts for
work in Iraq, whose under exploited oil reserves are probably
Explanation: second only to those in Saudi Arabia Here oil reserves of
Saudi Arabia and oil reserves of Iraq are being
compared.However in the given sentence oil reserves of Iraq
and Saudi Arabia are compared.

Directions:Change the voice of the sentence The professor teaches the students in
10.
the class.

1) The students are being taught by the professor in the class.

2) The students are taught by the professor in the class.

3) The students in the class are being taught by the professor

4) The students in the class are taught by the professor.

Correct Option
4
is:
Your Option is: 4
Result: Correct
Timetaken: 0.0 secs
Explanation: The given sentence in active voice is in simple present
tense.Similarly in passive voice also should be in simple
past tense.

Directions for questions : The given sentences when properly sequenced form a

coherent paragraph. Each sentence is labelled with a number. Choose the most

logical order of sentences from among the four given choices to construct a

coherent paragraph.

A. At this the innkeeper was delighted and hurried off to fetch what he now

perceived the guest to want.

B. And as he could not speak German, he had some difficulty in making known his

11. wants.

C. A French gentleman happened to be in Switzerland at a roadside inn where

German was the only language spoken;

D. Among other things he wanted some mushrooms but could not make his landlord

understand this.

E. At last he seized a piece of charcoal and drew on the wall a rough picture of a

mushroom.

1) CABDE

2) CBDAE

3) CBAED

4) CBDEA

Correct
4
Option is:
Your Option
2
is:
Result: Wrong
Timetaken: 0.0 secs
In the given question, the fragment C presents a general
beginning of the paragraph as it introduces a French
gentleman who happens to be in Switzerland at a roadside
inn where German was the only language spoken. Further in
statement B, it has been state that as he could not speak
German, he had difficulty in making his wants. Hence, this
statement comes second in the paragraph. Further in
Explanation:
statement D, what difficulty he faced has been stated. Hence,
statement D comes third in the paragraph. In statement E, the
French finds a solution to his problem and draws a picture of
what he wanted. Hence, this statement comes fourth
statement in the paragraph. And finally statement A is the last
statement. Therefore, from the given options CBDEA forms
a coherent paragraph.

Directions for questions :

The given sentences when properly sequenced form a coherent paragraph. Each

sentence is labelled with a number. Choose the most logical order of sentences from

among the four given choices to construct a coherent paragraph.

A. who was by trade a stone mason,


12.
B. in his leisure hours, construct

C. proposed once to the adjutant that

D. a sun-dial for the use of the station.

E. if a suitable stone were provided he would,

F. A soldier in one of the Bombay regiments,

1) FBEDAC

2) FBCECD

3) FACEBD

4) FACEDB
Correct
3
Option is:
Your Option
4
is:
Result: Wrong
Timetaken: 0.0 secs
In the given question, statement F marks the beginning of the
paragraph as it introduces a soldier from Bombay regiments.
Further statement A speaks about the soldier that he was
trade a stone mason. Hence, statement A comes just after
Explanation: statement F. And in statement C, what he did has been stated
and hence comes third in the paragraph. Statement E is just
the continuation of statement C and hence is fourth in the
paragraph. Thus, from the given options, the option which
meets our criteria is FACEBD.

Directions for questions :

13. Choose the contextual meaning of the given word.

My tether of playing cricket is hitting sixes.

1) rope used to keep a boat from drifting or an animal from wandering

2) the utmost length to which one can go in action

3) An asset of special worth or utility

4) ability that has been acquired by training

Correct Option is: 2


Your Option is: 2
Result: Correct
Timetaken: 0.0 secs
Explanation: tether - the utmost length to which one can go in action

14.
Directions for questions :

Answer the questions based on the given passage Quantitative easing is an


unconventional monetary policy used by central banks to stimulate the economy

when standard monetary policy has become ineffective. A central bank implements

quantitative easing by buying specified amounts of long term financial assets from

commercial banks and other private institutions, thus increasing the monetary base

and lowering the yield on those financial assets. This is distinguished from the more

usual policy of buying or selling government bonds in order to keep interbank

interest rates at a specified target value. Expansionary monetary policy typically

involves the central bank buying short-term government bonds in order to lower

short-term market interest rates. However, when short-term interest rates are at or

close to zero, normal monetary policy can no longer lower interest rates.

Quantitative easing may then be used by monetary authorities to further stimulate

the economy by purchasing assets of longer maturity than short-term government

bonds, and thereby lowering longer-term interest rates further out on the yield

curve. Quantitative easing raises the prices of the financial assets bought, which

lowers their yield. There are two potential benefits. The first is that the banks get

cash in exchange for the gilts they sell back to the government and the increase in

the money supply leads to an increased volume of lending. The second is that

decreasing the supply of gilts pushes up their price. When gilt prices go up, gilt

yields go down and it is gilt yields that determine long-term interest rates for

overdrafts, some fixed-rate mortgage products and most business lending.

Quantitative easing can be used to help ensure that inflation does not fall below

target. Risks include the policy being more effective than intended in acting against

deflation (leading to higher inflation in the longer term, due to increased money

supply), or not being effective enough if banks do not lend out the additional

reserves. According to the IMF and various other economists, quantitative easing

undertaken since the global financial crisis of 2007?08 has mitigated some of the

adverse effects of the crisis.


The author cites the example global financialcrisis of 2007?08 in order to

1) Convey that fact that QE undertaken then had mitigated some adverse effects of
the crisis

2) Show how QE had been instrumental in keeping the inflation in check then

3) Show how QE played a pivotal role in maintaining the interest rates then.

4) Show how the economy was stimulated by increasing the money supply.

Correct
1
Option is:
Your Option
1
is:
Result: Correct
Timetaken: 0.0 secs
Clearly last paragraph of the passage gives this example of ?
global financial cri sis of 2007?08? to convey the fact that
Explanation:
QE undertaken then had mitigated som e adverse effects of
the crisis.

Directions for questions : In the question a part of the sentence is bolded.

Alternatives to the bolded part are given which may improve the sentence. Choose

15. the correct alternative. In case no improvement is needed. Option ?E? is the

answer.

The commonplace meal was not exciting to the world class cook.

1) bland

2) banal

3) work a day

4) normal
Correct
3
Option is:
Your Option
1
is:
Result: Wrong
Timetaken: 0.0 secs
Here, the contextual meaning of ?Commonplace? is usual.
Explanation: From the given opti ons, ?workaday? also means common.
Hence, ?workaday? is the most suitable word.

Directions for questions : In the following questions, select the word or phrase that

16. is similar in meaning to the given word.

Preternatural

1) Runic

2) Preponderance

3) Uncanny

4) Presage

Correct
3
Option is:
Your Option
4
is:
Result: Wrong
Timetaken: 0.0 secs
Explanation: Preternatural - existing outside of or not in accordance with
nature. Runic - relating to or consisting of rune. Runic
writing consists of ancient characters that are tough to
decipher. These days the word might better fit a doctor's
signature or a confusing instruction manual. Preponderance -
exceeding in heaviness; having greater weight. Uncanny -
surpassing the ordinary or normal. Presage - a foreboding
about what is about to happen.

Directions for questions : In the question a part of the sentence is bolded.

Alternatives to the bolded part are given which may improve the sentence. Choose

17. the correct alternative. In case no improvement is needed. Option ?E? is the

answer.

The candidate took offended at the remark of his opponent.

1) Rancour

2) gall

3) resentment

4) umbrage

Correct
4
Option is:
Your Option
4
is:
Result: Correct
Timetaken: 0.0 secs
?Offended? means resentful or annoyed, typically as a result
of a perceived ins ult and from the given options ?umbrage?
Explanation:
also means offence or annoyance. H ence, ?umbrage? is the
most suitable word here.

18.
Directions:

Read the sentence to find out whether there is any grammatical error in it. The

error, if any, will be in one part of the sentence. The letter of that part is the

answer. If there is no error, the answer is ?E?. (Ignore ? the errors of punctuation, if

any)

Neither the assistants (A) / nor the examiner was (B) /informed about the
cancellation of the examination. (C)

1) B,C

2) B

3) A

4) A,B

5) No error

Correct Option
5
is:
Your Option is: 2
Result: Wrong
Timetaken: 0.0 secs
When one of the subjects are joined by neither/nor is
Explanation:
plural, the verb should a gree with the noun closest to it.

Directions for questions : Choose the contextual meaning of the given word.
19.
His actions traduced his reputation.

1) derogated

2) aspersed

3) belittled

4) maligned

Correct
4
Option is:
Your Option
4
is:
Result: Correct
Timetaken: 0.0 secs
?Traduce? means to speak badly of or tell lies about
(someone) so as to damage their reputation and also from the
Explanation: given options ?malign? means to speak about (someone) in a
spitefully critical manner. Hence, ?malign? gives correct
contextual meaning.

Directions : Read the sentence to find out whether there is any grammatical error in

it. The error, if any, will be in one part of the sentence. The letter of that part is the

answer. If there is no error, the answer is ?E?. (Ignore ? the errors of punctuation, if

20. any)

The rich countries should commit themselves (A) /to finance a massive (B)

/program of technology and (C) / to a program of technology transfer to developing

countries (D).

1) A,D

2) D

3) C,D

4) B

Correct
4
Option is:
Your Option
1
is:
Result: Wrong
Timetaken: 0.0 secs
"commit themselves to" will be followed by a gerund.
Therefore, statement B should read "to financing a
Explanation:
massive..."Also note that commit themselves to a program is
the next idea which is parallel with the first-this is the clue.

21.
Directions for questions : Answer the questions based on the given passage
Quantitative easing is an unconventional monetary policy used by central banks to

stimulate the economy when standard monetary policy has become ineffective. A

central bank implements quantitative easing by buying specified amounts of long

term financial assets from commercial banks and other private institutions, thus

increasing the monetary base and lowering the yield on those financial assets. This

is distinguished from the more usual policy of buying or selling government bonds

in order to keep interbank interest rates at a specified target value. Expansionary

monetary policy typically involves the central bank buying short-term government

bonds in order to lower short-term market interest rates. However, when short-term

interest rates are at or close to zero, normal monetary policy can no longer lower

interest rates. Quantitative easing may then be used by monetary authorities to

further stimulate the economy by purchasing assets of longer maturity than short-

term government bonds, and thereby lowering longer-term interest rates further

out on the yield curve. Quantitative easing raises the prices of the financial assets

bought, which lowers their yield.There are two potential benefits. The first is that

the banks get cash in exchange for the gilts they sell back to the government and

the increase in the money supply leads to an increased volume of lending. The

second is that decreasing the supply of gilts pushes up their price. When gilt prices

go up, gilt yields go down and it is gilt yields that determine long-term interest

rates for overdrafts, some fixed-rate mortgage products and most business lending.

Quantitative easing can be used to help ensure that inflation does not fall below

target. Risks include the policy being more effective than intended in acting against

deflation (leading to higher inflation in the longer term, due to increased money

supply), or not being effective enough if banks do not lend out the additional

reserves. According to the IMF and various other economists, quantitative easing

undertaken since the global financial crisis of 2007?08 has mitigated some of the

adverse effects of the crisis.

Which of the following are true? I. Due to less supply of gilt, there is high demand
for it, it becomes costlier and thus reducing gilt yields.

II. It is essential that the commercial banks have to lend out the extra money

obtained by selling the bonds in order to effectively implement QE.

III. Expansionary monetary policy involves purchasing short term financial assets in

order to reduce short term market interest rates.

IV. QE is used mainly to to keep interbank interest rates at a specified target value.

1) I,II,III

2) III,IV

3) I,IV

4) All the above

Correct
1
Option is:
Your Option
3
is:
Result: Wrong
Timetaken: 0.0 secs
From the given passage we can only conclude that
statements ?Due to less supply of gilt, there is high demand
for it, it becomes costlier and thus reducing gilt yields.?,
statement ?It is essential that the commercial banks have to
Explanation: lend out the extra money obtained by selling the bonds in
order to effectively implement QE.? and statement ?
Expansionary monetary policy involves purchasing short
term financial assets in order to reduce short term market
interest rates.? are only true.

22.
Directions for questions : In the question a part of the sentence is in bold.

Alternatives to the bolded part are given which may improve the sentence. Choose

the correct alternative. In case no improvement is needed. Option ?E? is the

answer.
The witness must confirm the validity of the prisoner?s story if she is to be set

free.

1) essay

2) support

3) corroborate

4) approve

Correct Option
3
is:
Your Option is: 3
Result: Correct
Timetaken: 0.0 secs
Here, from the given options means to confirm or give
Explanation: support to (a statement, theory, or finding). Hence, is the
most suitable word here.

23.
Directions for questions :

Answer the questions based on the given passage Quantitative easing is an

unconventional monetary policy used by central banks to stimulate the economy

when standard monetary policy has become ineffective. A central bank implements

quantitative easing by buying specified amounts of long term financial assets from

commercial banks and other private institutions, thus increasing the monetary base

and lowering the yield on those financial assets. This is distinguished from the more

usual policy of buying or selling government bonds in order to keep interbank

interest rates at a specified target value. Expansionary monetary policy typically

involves the central bank buying short-term government bonds in order to lower

short-term market interest rates. However, when short-term interest rates are at or

close to zero, normal monetary policy can no longer lower interest rates.
Quantitative easing may then be used by monetary authorities to further stimulate

the economy by purchasing assets of longer maturity than short-term government

bonds, and thereby lowering longer-term interest rates further out on the yield

curve. Quantitative easing raises the prices of the financial assets bought, which

lowers their yield. There are two potential benefits. The first is that the banks get

cash in exchange for the gilts they sell back to the government and the increase in

the money supply leads to an increased volume of lending. The second is that

decreasing the supply of gilts pushes up their price. When gilt prices go up, gilt

yields go down and it is gilt yields that determine long-term interest rates for

overdrafts, some fixed-rate mortgage products and most business lending.

Quantitative easing can be used to help ensure that inflation does not fall below

target. Risks include the policy being more effective than intended in acting against

deflation (leading to higher inflation in the longer term, due to increased money

supply), or not being effective enough if banks do not lend out the additional

reserves. According to the IMF and various other economists, quantitative easing

undertaken since the global financial crisis of 2007?08 has mitigated some of the

adverse effects of the crisis.

According to the author, QE is?

1) an unconventional monetary policy used to Stimulate the economy by increasing


money supply without immediately reducing the interest rates.

2) an unconventional monetary policy used to Stimulate the economy by pumping


money into the economy without reducing the interest rates.

3) an unconventional monetary policy used to stimulate the economy when


standard monetary policy has become effective.

4) an unconventional monetary policy used to Stimulate the economy by pumping


money into the economy without reducing the interest rates and keeping the
inflation in check.
Correct
1
Option is:
Your Option
1
is:
Result: Correct
Timetaken: 0.0 secs
The first line of the paragraph itself tells about the QE.
Hence, QE is an uncon ventional monetary policy used to
Explanation:
stimulate the economy by increasing mone y supply without
immediately reducing the interest rates.

Directions for questions:

24. Choose the word which is farthest in meaning to the given word.

Venal

1) Skilled

2) Sardonable

3) Incorruptible

4) Infallible

Correct
3
Option is:
Your Option
4
is:
Result: Wrong
Timetaken: 0.0 secs
?Venal? means showing or motivated by susceptibility to
bribery or corruption. From the given options, ?
Explanation:
incorruptible? is the word which is farthest in meaning to
the given word.

25.
Directions for questions : In the following questions, select the word or phrase that

is similar in meaning to the given word.


Inure

1) Indurate

2) Inundate

3) Inveigle

4) Innuendo

Correct Option
1
is:
Your Option is: 2
Result: Wrong
Timetaken: 0.0 secs
Inure - cause to accept or become hardened to; habituate.
Indurate - cause to accept or become hardened to;
habituate.
Inundate - fill or cover completely, usually with water.
Explanation: Inveigle - influence or urge by gentle urging, caressing,
or flattering.
Innuendo - an indirect (and usually malicious)
implication.
Indurate is the synonym of inure

Directions for questions :In the following questions, select the word or phrase that

1. is similar in meaning to the given word.

Recondite

1) esoteric

2) expatiate

3) prolix

4) puissant
Correct Option
1
is:
Your Option is: 4
Result: Wrong
Timetaken: 0.0 secs
Recondite - Difficult to penetrate; incomprehensible to one
of ordinary understanding or knowledge
esoteric - Confined to and understandable by only an
enlightened inner circle Esoteric is the synonym of
Recondite
Explanation: expatiate - Add details, as to an account or idea; clarify the
meaning of and discourse in a learned way, usually in
writing
Prolix - Tediously prolonged or tending to speak or write at
great length
Puissant ? powerful

Directions for questions : Choose the contextual meaning of the given word.
2.
It will behove the students to buy their textbooks early.

1) be appropriate or necessary for

2) force

3) be advantageous for

4) be benefecial

Correct Option is: 1


Your Option is: 2
Result: Wrong
Timetaken: 0.0 secs
Explanation: Behove - Be appropriate or necessary

3.
Directions for questions : Read the sentence to find out whether there is any

grammatical error in it. The error, if any, will be in one part of the sentence. The
letter of that part is the answer. If there is no error, the answer is ?E?. (Ignore ? the

errors of punctuation, if any)

She would rather (A) /you paid her (B) /by cash.(C)

1) A

2) B

3) C

4) No Error

Correct Option
2
is:
Your Option is: 3
Result: Wrong
Timetaken: 0.0 secs
She prefers that you pay her by cash
This is an example of subjunctive verb which shows a
Explanation:
wish or doubt. For example:
I suggest that Lee play the guitar.

Directions for questions : Fill in the blanks


4.
As ___ your behaviour, I think you were not right

1) of

2) is

3) for

4) to

Correct Option is: 4


Your Option is: 2
Result: Wrong
Timetaken: 0.0 secs
Explanation: as to means "With regard to; According to"

Change the speech of the sentence. Won?t you help me to carry this box?? said
5.
I to my friend.

1) I asked my friend if he would not help me to carry that box.

2) I asked my friend whether he would help me to carry that box.

3) I said to my friend if he would not help me to carry that box

4) I asked my friend whether he would not help me to carry that

Correct
1
Option is:
Your Option
3
is:
Result: Wrong
Timetaken: 0.0 secs
Use ?if? or ?whether? if the sentence inside the quotation
marks begins with a helping verb (Auxiliary verb). To
change questions (which can be answered in yes or no) into
Explanation:
indirect speech, word ?if? or ?whether? is used before the
question in indirect speech. I asked my friend if he would not
help me to carry that box.

6.
Directions for questions :

The given sentences when properly sequenced form a coherent paragraph. Each

sentence is labelled with a number. Choose the most logical order of sentences from

among the four given choices to construct a coherent paragraph.

A. -its history spanning nearly two millennia


B. By far Britain?s largest metropolis, it is also the country?s economic,

transportation, and cultural centre.

C. It is among the oldest of the world?s great cities

D. London, city is the capital of the United Kingdom.

E. and one of the most cosmopolitan.

1) DBCAE

2) DACEB

3) DAEBC

4) DCAEB

Correct
4
Option is:
Your Option
4
is:
Result: Correct
Timetaken: 0.0 secs
In the given question, statement D comes first in the
arrangement as it is starting a topic. Further statement C
Explanation: starts with a pronoun which is used for London and hence,
comes second in the graph. There is only one such option in
given options and hence DCAEB is the correct option.

7.
Directions for questions :

The given sentences when properly sequenced form a coherent paragraph. Each

sentence is labelled with a number. Choose the most logical order of sentences from

among the four given choices to construct a coherent paragraph.

A. At various times they were a friendly tributary state to the Shang, alternatively

warring with them.


B. One of the Zhou ruling houses devised a plan to conquer the Shang,

C. The Zhou coexisted with the Shang for many years, living just west of the Shang

territory in what is now Shaanxi province.

D. Before the whole Shang territory could be consolidated by the Zhou, a rebellion

broke out.

E. And a decisive battle was fought, probably in the mid-11th century.

1) CDABE

2) BEDCA

3) CABED

4) BEDAC

Correct
3
Option is:
Your Option
1
is:
Result: Wrong
Timetaken: 0.0 secs
In the given question, statement C marks the beginning of
the statement as it brings in the topic about the Zhou and the
Shang. Further statement A discusses the topic and talks
Explanation: about Shang. Hence, statements C and A comes together.
Looking into the options, there is only one such option which
satisfies the above criteria. Hence, CABED forms a coherent
paragraph.

Directions: Choose the word which is farthest in meaning to the given word
8.
defection

1) effrontery
2) joining

3) cozen

4) abjure

Correct Option
2
is:
Your Option is: 4
Result: Wrong
Timetaken: 0.0 secs
Defection - Withdrawing support or help despite
allegiance or responsibility
Antonym will be joining
Effrontery - Audacious (even arrogant) behaviour that you
Explanation:
have no right to
Cozen - Be false to; be dishonest with
Abjure - Formally reject or disavow a formerly held
belief, usually under pressure

Directions for questions : Read the sentence to find out whether there is any

grammatical error in it. The error, if any, will be in one part of the sentence. The

9. letter of that part is the answer. If there is no error, the answer is ?E?. (Ignore ? the

errors of punctuation, if any) Both KamalHassan as well as (A) / Shah Rukh Khan

are contenders for the (B) / best actor award in the National film category.(C)

1) A

2) B

3) C

4) No Error
Correct
1
Option is:
Your Option
3
is:
Result: Wrong
Timetaken: 0.0 secs
We need the correlative conjunction pair ?both...and? in this
sentence. Therefore, the sentence should be: Both
Explanation:
KamalHassan and Shah Rukh Khan are contenders for the
best actor award in the National film category.

Directions for questions : Fill in the blanks


10.
I don?t have any dollars. Can I pay rupees _____ dollars?

1) For exchange of

2) In exchange of

3) In face of

4) In lieu of

Correct Option
4
is:
Your Option is: 1
Result: Wrong
Timetaken: 0.0 secs
Lieu - The post or function properly or customarily
Explanation: occupied or served by another in lieu of will be the correct
fit here

11.
Directions for questions :

In the question a part of the sentence is bold. Alternatives to the italicised part are

given which may improve the sentence. Choose the correct alternative. In case no

improvement is needed. Option 'E' is the answer.


The protective glove saved the man?s hand from being burned in the heat.

1) mitt

2) scabbard

3) apron

4) sheath

Correct
1
Option is:
Your Option
4
is:
Result: Wrong
Timetaken: 0.0 secs
Apron - A garment of cloth, leather or plastic that is tied
about the waist and worn to protect one?s clothing Mitt - The
hand wear used by fielders in playing baseball; It is also the
Explanation: gloves used when taking vessels from oven. Scabbard - A
sheath for a sword, dagger or bayonet Sheath - A protective
covering (as for a knife or sword) Mitt is the correct word
here.

12.
Directions for questions:Answer the questions based on the given passage

The Order of the Illuminati was an Enlightenment-age secret society founded by

university professor Adam Weishaupt on 1 May 1776, in Upper Bavaria, Germany.

The movement consisted of advocates of freethought,secularism, liberalism,

republicanism and gender equality, recruited in the German Masonic Lodges, who

sought to teach rationalism through mystery schools. In 1785, the order was

infiltrated, broken up and suppressed by the government agents of Charles

Theodore, Elector of Bavaria, in his pre-emptive campaign to neutralize the threat

of secret societies ever becoming hotbeds of conspiracies to overthrow the Bavarian

monarchy and its state religion, Roman Catholicism. In the late 18th century,
reactionary conspiracy theorists, such as Scottish physicist John Robison and French

Jesuit priest Augustin Barruel, began speculating that the Illuminati survived their

suppression and became the masterminds behind the French Revolution and the

Reign of Terror. During the 19th century, fear of an Illuminati conspiracy was a real

concern of European ruling classes, and their oppressive reactions to this unfounded

fear provoked in 1848 the very revolutions they sought to prevent. During the

interwar period of the 20th century, fascist propagandists, such as British revisionist

historian Nesta Helen Webster and American socialite Edith Starr Miller, not only

popularized the myth of an Illuminati conspiracy but claimed that it was a

subversive secret society which serves the Jewish elites that supposedly propped up

both finance capitalism and Soviet communism in order to divide and rule the

world. Central to some of the most widely known and elaborate conspiracy theories,

the Illuminati have been depicted as lurking in the shadows and pulling the strings

and levers of power in dozens of novels, movies, television shows, comics, video

games and music videos. The Illuminati play a central role in the plots of novels,

such as The Illuminatus! Trilogy by Robert Shea and Robert Anton Wilson; in

Foucaults Pendulum by Umberto Eco; and Angels and Demons by Dan Brown. A

mixture of historical fact, established conspiracy theory, or pure fiction, is used to

portray them.

There is no evidence that the original Bavarian Illuminati survived its suppression in

1785.However, writers such as Mark Dice, David Icke, Texe Marrs, Juri Lina and

Morgan Gricar have argued that the Bavarian Illuminati survived, possibly to this

day. Many of these theories propose that world events are being controlled and

manipulated by a secret society calling itself the Illuminati.

According to Webster and Mille

1) Illuminati conspiracy was a myth and it meant to serve the Jewish elites

2) Illuminati was a secret society meant to spread conspiracy in order to serve the
Jewish elites

3) Illuminati play a pivotal role in the plots of novels like Angels and Demons and
several others by Dan Brown.

4) Illuminati helped the Jewish elites who are purported to have supported finance
capitalism and Soviet communism

Correct
4
Option is:
Your Option
2
is:
Result: Wrong
Timetaken: 0.0 secs
In the paragraph it is mentioned "During the interwar period
of the 20th century, fascist propagandists, such as British
revisionist historian Nesta Helen Webster and American
Explanation:
socialite Edith Starr Miller, not only popularized the myth of
an Illuminati conspiracy but claimed that it was a subversive
secret society which serves the Jewish elites "

Directions for questions : Read the sentence to find out whether there is any

grammatical error in it. The error, if any, will be in one part of the sentence. The

letter of that part is the answer. If there is no error, the answer is ?E?. (Ignore ? the
13.
errors of punctuation, if any)

Many customers in the restaurant found the (A)/tea too bitter to drink, but (B)/it

was still ordered frequently.(C)

1) A

2) B

3) C

4) No Error
Correct
3
Option is:
Your Option
1
is:
Result: Wrong
Timetaken: 0.0 secs
Many customers in the restaurant found the tea too bitter to
drink, but they ordered it frequently.
The given sentence starts in active voice and in C, it
Explanation: switches to passive voice.
The entire sentence should be either in active or passive
voice Many customers in the restaurant found the tea too
bitter to drink, but they ordered it frequently.

Directions for questions :

In the question a part of the sentence is italicised. Alternatives to the italicised part

are given which may improve the sentence. Choose the correct alternative. In case
14.
no improvement is needed. Option 'E' is the answer.

The Solomon's once rich temple was empty after centuries of plundering from

temple robbers during the exile.

1) pilfering

2) ruining

3) purloining

4) depredation

Correct Option
4
is:
Your Option is: 4
Result: Correct
Timetaken: 0.0 secs
Explanation: pilfer - Make off with belongings of others
"purloin" and pilfer means the same.
depredation - An act of plundering and pillaging and
marauding
Depredation is the correct word

Directions for questions : Choose the contextual meaning of the given word.

15. The liberal school of thought trusts in education reform and the sporadic use of

force to remedy the depravity of certain isolated individuals or groups.

1) Infrequent, irregular

2) Persistent, constant

3) Continuous

4) Sparing

5) Corrective and preventive both

Correct Option
1
is:
Your Option is: 2
Result: Wrong
Timetaken: 0.0 secs
sporadic - Recurring in scattered and irregular or
Explanation:
unpredictable instances

Spread over a decade, (A) / the political career of Arvind was more dynamic (B) /
16.
than his opponent Rahul. (C)

1) A

2) B

3) C

4) No Error
Correct
3
Option is:
Your Option
3
is:
Result: Correct
Timetaken: 0.0 secs
When there is a comparison between two things in a
sentence, the two things being compared has to be
Explanation: comparable.Therefore, the sentence should be: Spread over a
decade, the political career of Arvind was more dynamic
than that of his opponent Rahul.

Directions: Choose the word which is farthest in meaning to the given word
17.
extirpate

1) Desuetude

2) exterminate

3) acarpous

4) implant

Correct Option
4
is:
Your Option is: 2
Result: Wrong
Timetaken: 0.0 secs
Extirpate - Pull up by or as if by the roots Antonym will
be implant
Exterminate - kill on a large scale; kill many
Explanation: Acarpous - Producing no fruit
Desuetude - A state of inactivity or disuse
exterminate - Destroy completely, as if down to the
roots;kill on a large scale
Directions for questions : Read the sentence to find out whether there is any

grammatical error in it. The error, if any, will be in one part of the sentence. The

18. letter of that part is the answer. If there is no error, the answer is ?E?. (Ignore ? the

errors of punctuation, if any)

Krish would like (A) /to visit the African jungles(B) /and the deserts of Egypt.(C)

1) A

2) B

3) C

4) No Error

Correct Option
3
is:
Your Option is: 3
Result: Correct
Timetaken: 0.0 secs
Parallelisms when conjunctions are used.Conjunctions join
words, phrases or clauses, which are parallel and
comparable.
Explanation: Krish would like to visit the African jungles and the
Egyptian deserts .
Krish would like to visit the jungles of Africa and the
deserts of Egypt.

Directions for questions :

In the question a part of the sentence is bold. Alternatives to the italicised part are

given which may improve the sentence. Choose the correct alternative. In case no
19.
improvement is needed. Option 'E' is the answer.

The man will talk very long on the subject if you give him too much speaking

time.
1) cover

2) communicate

3) descant

4) homily

Correct Option
3
is:
Your Option is: 2
Result: Wrong
Timetaken: 0.0 secs
Descant - Talk at great length about something of one?s
Explanation: interest Homily - A sermon on a moral or religious topic
Descant is the right word here

Directions for questions :

The given sentences when properly sequenced form a coherent paragraph. Each

sentence is labelled with a number. Choose the most logical order of sentences from

among the four given choices to construct a coherent paragraph.

A. She attended the coeducational Amherst Academy

20. B. and others and was often kept home from school.

C. She also excelled in other subjects emphasized by the school, most notably Latin

and the sciences

D. where she was recognized by teachers and students alike for her prodigious

abilities in composition.

E. As a girl, Emily was seen as frail by her parents

1) EBADC

2) EBDAC

3) EDABC
4) EBDCA

Correct
1
Option is:
Your Option
4
is:
Result: Wrong
Timetaken: 0.0 secs
In the given question, statement E marks the beginning of the
statement as it introduces Emily and also tells about her
parents thinking for her. Further statement B is the
continuation of statement E and hence comes just after
statement E. Now in statement A, it has been given that
Explanation:
where she got her education and also statement D is the
continuation of statement A. Hence, it comes just after
statement A. From the given options there is only one such
option which serves the above criteria. Hence, EBADC
forms a coherent paragraph.

21.
Directions for questions:Answer the questions based on the given passage

The Order of the Illuminati was an Enlightenment-age secret society founded by

university professor Adam Weishaupt on 1 May 1776, in Upper Bavaria, Germany.

The movement consisted of advocates of freethought,secularism, liberalism,

republicanism and gender equality, recruited in the German Masonic Lodges, who

sought to teach rationalism through mystery schools. In 1785, the order was

infiltrated, broken up and suppressed by the government agents of Charles

Theodore, Elector of Bavaria, in his pre-emptive campaign to neutralize the threat

of secret societies ever becoming hotbeds of conspiracies to overthrow the Bavarian

monarchy and its state religion, Roman Catholicism. In the late 18th century,

reactionary conspiracy theorists, such as Scottish physicist John Robison and French

Jesuit priest Augustin Barruel, began speculating that the Illuminati survived their
suppression and became the masterminds behind the French Revolution and the

Reign of Terror. During the 19th century, fear of an Illuminati conspiracy was a real

concern of European ruling classes, and their oppressive reactions to this unfounded

fear provoked in 1848 the very revolutions they sought to prevent. During the

interwar period of the 20th century, fascist propagandists, such as British revisionist

historian Nesta Helen Webster and American socialite Edith Starr Miller, not only

popularized the myth of an Illuminati conspiracy but claimed that it was a

subversive secret society which serves the Jewish elites that supposedly propped up

both finance capitalism and Soviet communism in order to divide and rule the

world. Central to some of the most widely known and elaborate conspiracy theories,

the Illuminati have been depicted as lurking in the shadows and pulling the strings

and levers of power in dozens of novels, movies, television shows, comics, video

games and music videos. The Illuminati play a central role in the plots of novels,

such as The Illuminatus! Trilogy by Robert Shea and Robert Anton Wilson; in

Foucault?s Pendulum by Umberto Eco; and Angels and Demons by Dan Brown. A

mixture of historical fact, established conspiracy theory, or pure fiction, is used to

portray them.

There is no evidence that the original Bavarian Illuminati survived its suppression in

1785.However, writers such as Mark Dice, David Icke, Texe Marrs, Juri Lina and

Morgan Gricar have argued that the Bavarian Illuminati survived, possibly to this

day. Many of these theories propose that world events are being controlled and

manipulated by a secret society calling itself the Illuminati.

According to the most widely known conspiracy theories, the illuminati

1) is influencing people through dozens of novels, movies, television shows, comics,


video games and music videos.

2) s a secret society controlling and manipulating world events

3) Illuminati survived their suppression and became the masterminds behind the
French Revolution and the Reign of Terror.
4) a subversive secret society which serves the Jewish elites

Correct
1
Option is:
Your Option
1
is:
Result: Correct
Timetaken: 0.0 secs
In fourth paragraph it is mentioned "Central to some of the
most widely known and elaborate conspiracy theories, the
Illuminati have been depicted as lurking in the shadows and
Explanation:
pulling the strings and levers of power in dozens of novels,
movies, television shows, comics, video games and music
videos."

Directions for questions :In the following questions, select the word or phrase that

22. is similar in meaning to the given word.

turpitude

1) tenuous

2) quiescence

3) fractious

4) putrefaction

Correct Option
4
is:
Your Option is: 4
Result: Correct
Timetaken: 0.0 secs
Explanation: turpitude - A corrupt or depraved or degenerate act or
practice
tenuous - Lacking substance or significance
Quiescence -quiet and inactive restfulness
fractious - Stubbornly resistant to authority or control
putrefaction - Moral perversion; impairment of virtue and
moral principles
"putrefaction" is the synonym of "turpitude"

Directions for questions : Fill in the blanks


23.
Once he has signed the agreement, he won't be able to ______

1) back up

2) back in

3) back at

4) back out

Correct Option is: 4


Your Option is: 1
Result: Wrong
Timetaken: 0.0 secs
Explanation: back out - withdraw from a commitme

24.
Directions for questions:Answer the questions based on the given passage

The Order of the Illuminati was an Enlightenment-age secret society founded by

university professor Adam Weishaupt on 1 May 1776, in Upper Bavaria, Germany.

The movement consisted of advocates of freethought,secularism, liberalism,

republicanism and gender equality, recruited in the German Masonic Lodges, who

sought to teach rationalism through mystery schools. In 1785, the order was

infiltrated, broken up and suppressed by the government agents of Charles

Theodore, Elector of Bavaria, in his pre-emptive campaign to neutralize the threat

of secret societies ever becoming hotbeds of conspiracies to overthrow the Bavarian

monarchy and its state religion, Roman Catholicism. In the late 18th century,
reactionary conspiracy theorists, such as Scottish physicist John Robison and French

Jesuit priest Augustin Barruel, began speculating that the Illuminati survived their

suppression and became the masterminds behind the French Revolution and the

Reign of Terror. During the 19th century, fear of an Illuminati conspiracy was a real

concern of European ruling classes, and their oppressive reactions to this unfounded

fear provoked in 1848 the very revolutions they sought to prevent. During the

interwar period of the 20th century, fascist propagandists, such as British revisionist

historian Nesta Helen Webster and American socialite Edith Starr Miller, not only

popularized the myth of an Illuminati conspiracy but claimed that it was a

subversive secret society which serves the Jewish elites that supposedly propped up

both finance capitalism and Soviet communism in order to divide and rule the

world. Central to some of the most widely known and elaborate conspiracy theories,

the Illuminati have been depicted as lurking in the shadows and pulling the strings

and levers of power in dozens of novels, movies, television shows, comics, video

games and music videos. The Illuminati play a central role in the plots of novels,

such as The Illuminatus! Trilogy by Robert Shea and Robert Anton Wilson; in

Foucault?s Pendulum by Umberto Eco; and Angels and Demons by Dan Brown. A

mixture of historical fact, established conspiracy theory, or pure fiction, is used to

portray them.

There is no evidence that the original Bavarian Illuminati survived its suppression in

1785.However, writers such as Mark Dice, David Icke, Texe Marrs, Juri Lina and

Morgan Gricar have argued that the Bavarian Illuminati survived, possibly to this

day. Many of these theories propose that world events are being controlled and

manipulated by a secret society calling itself the Illuminati.

Illuminati portrayed in the novels by

I. mixture of historical fact

II. an established conspiracy theory

III. pure fiction


IV. symbolism

V. free thought mystery schools

1) All the above

2) I, II, III

3) I,II,III,IV

4) I & II

Correct
2
Option is:
Your Option
4
is:
Result: Wrong
Timetaken: 0.0 secs
The Illuminati play a central role in the plots of novels, such
as The Illuminatus! Trilogy by Robert Shea and Robert
Anton Wilson; in Foucault?s Pendulum by Umberto Eco;
Explanation:
and Angels and Demons by Dan Brown. A mixture of
historical fact, established conspiracy theory, or pure fiction,
is used to portray them.

25.
Directions for questions:Answer the questions based on the given passage

The Order of the Illuminati was an Enlightenment-age secret society founded by

university professor Adam Weishaupt on 1 May 1776, in Upper Bavaria, Germany.

The movement consisted of advocates of freethought,secularism, liberalism,

republicanism and gender equality, recruited in the German Masonic Lodges, who

sought to teach rationalism through mystery schools. In 1785, the order was

infiltrated, broken up and suppressed by the government agents of Charles

Theodore, Elector of Bavaria, in his pre-emptive campaign to neutralize the threat

of secret societies ever becoming hotbeds of conspiracies to overthrow the Bavarian


monarchy and its state religion, Roman Catholicism. In the late 18th century,

reactionary conspiracy theorists, such as Scottish physicist John Robison and French

Jesuit priest Augustin Barruel, began speculating that the Illuminati survived their

suppression and became the masterminds behind the French Revolution and the

Reign of Terror. During the 19th century, fear of an Illuminati conspiracy was a real

concern of European ruling classes, and their oppressive reactions to this unfounded

fear provoked in 1848 the very revolutions they sought to prevent. During the

interwar period of the 20th century, fascist propagandists, such as British revisionist

historian Nesta Helen Webster and American socialite Edith Starr Miller, not only

popularized the myth of an Illuminati conspiracy but claimed that it was a

subversive secret society which serves the Jewish elites that supposedly propped up

both finance capitalism and Soviet communism in order to divide and rule the

world. Central to some of the most widely known and elaborate conspiracy theories,

the Illuminati have been depicted as lurking in the shadows and pulling the strings

and levers of power in dozens of novels, movies, television shows, comics, video

games and music videos. The Illuminati play a central role in the plots of novels,

such as The Illuminatus! Trilogy by Robert Shea and Robert Anton Wilson; in

Foucault?s Pendulum by Umberto Eco; and Angels and Demons by Dan Brown. A

mixture of historical fact, established conspiracy theory, or pure fiction, is used to

portray them.

There is no evidence that the original Bavarian Illuminati survived its suppression in

1785.However, writers such as Mark Dice, David Icke, Texe Marrs, Juri Lina and

Morgan Gricar have argued that the Bavarian Illuminati survived, possibly to this

day. Many of these theories propose that world events are being controlled and

manipulated by a secret society calling itself the Illuminati.

What is the meaning of the word ?pre-emptive??

1) designed to deter or prevent an anticipated situation


2) imperative

3) subversive

4) to uproot and destroy completely

Correct Option
1
is:
Your Option is: 2
Result: Wrong
Timetaken: 0.0 secs
Pre- emptive - Designed or having the power to deter or
prevent an anticipated situation or occurrence
Explanation: Imperative - Some duty that is essential and urgent
subversive - In opposition to a civil authority or
government

Use appropriate prepositions to fill in the blanks.


1.
We can do this work _______ any extra heap.

1) without

2) withdraw

3) with

4) within

Correct Option
1
is:
Your Option is: 2
Result: Wrong
Timetaken: 0.0 secs
Within - shows time limit. Here we do not have time limit
Explanation:
With - Shows togetherness Without - not in need of
Fill in the following blanks with appropriate articles.
2.
One of the students said, "____ professor is late today."

1) An

2) the

3) a

4) None of these

Correct Option
2
is:
Your Option is: 1
Result: Wrong
Timetaken: 0.0 secs
?Professor? should be preceded by definite article ?the? as
Explanation:
the reference is to a specific professor.

Identify appropriate antonyms for the italicized word, from the following options.
3.
Whenever he won we were exposed to his gasconade.

1) Bluster

2) Modesty

3) Jolty

4) Infrequently

Correct Option
2
is:
Your Option is: 4
Result: Wrong
Timetaken: 0.0 secs
Gasconade ? boasting Modesty ? humbleness, humility
Explanation: Bluster ? noisy bragging Jolty ? characterised by sudden
jerky motion

4.
Read the passage and answer the questions asked below.

Historians have only recently begun to note the increase in demand for luxury

goods and services that took place in Eighteenth century England. McKendrick has

explored the Wedgwood firm?s remarkable success in marketing luxury pottery;

Plumb has written about the proliferation of provincial theatres, musical festivals,

and children?s toys and books. While the fact of this consumer revolution is hardly

in doubt, three key questions remain: Who were the consumers? What were their

motives? And what were the effects of the new demand for luxuries? An answer to

the first of these has been difficult to obtain. Although it has been possible to infer

from the goods and services actually produced what manufacturers and servicing

traders thought their customers wanted, only a study of relevant personal

documents written by actual consumers will provide a precise picture of who wanted

what. We still need to know how large this consumer market was and how far down

the social scale the consumer demand for luxury goods penetrated. With regard to

this last question, we might note in passing that Thompson, while rightly restoring

labouring people to the stage of eighteenth-century English history, has probably

exaggerated the opposition of these people to the inroads of capitalist consumerism

in general; for example, labouring people in eighteenth-century England readily

shifted from home-brewed beer to standardized beer produced by huge, heavily

capitalized urban breweries.

To answer the question of why consumers became so eager to buy, some historians

have pointed to the ability of manufacturers to advertise in a relatively uncensored

press. This, however, hardly seems a sufficient answer. McKendrick favours a Veblen

model of conspicuous consumption stimulated by competition for status. The

middling sort bought goods and services because they wanted to follow fashions set
by the rich. Again, we may wonder whether this explanation is sufficient. Do not

people enjoy buying things as a form of self-gratification? If so, consumerism could

be seen as a product of the rise of new concepts of individualism and materialism,

but not necessarily of the frenzy for conspicuous competition. Finally, what were the

consequences of this consumer demand for luxuries? McKendrick claims that it goes

a long way towards explaining the coming of the Industrial Revolution. But does it?

What, for example, does the production of high-quality pottery and toys have to do

with the development of iron manufacture or textile mills? It is perfectly possible to

have the psychology and reality of a consumer society without a heavy industrial

sector. That future exploration of these key questions is undoubtedly necessary

should not, however, diminish the force of the conclusion of recent studies: the

insatiable demand in eighteenth-century England for frivolous as well as useful

goods and services foreshadows our own world.

According to the passage, a Veblen model of conspicuous consumption has been

used to:

1) Investigate the extent of the demand for luxury goods among social classes in
eighteenth-century England

2) Classify the kinds of luxury goods desired by eighteenth- century consumers

3) explain the motivation of eighteenth-century consumers to buy luxury goods

4) Establish the extent to which the tastes of rich consumers were shaped by the
middle classes in eighteenth-century England

5) Compare luxury consumerism in eighteenth-century England with such


consumerism in the twentieth century

Correct
3
Option is:
Your Option
2
is:
Result: Wrong
Timetaken: 0.0 secs
(Referential question) Both theories put forth in the third
paragraph have been used to answer the qu estion,?what
Explanation:
were the motives of the eighteenth century consumers to
buy lux ury goods??

Use appropriate prepositions to fill in the blanks.


5.
I tried to argue him out _______ his complacency

1) of

2) since

3) by

4) for

Correct
1
Option is:
Your Option
1
is:
Result: Correct
Timetaken: 0.0 secs
Complacency - uncritical satisfaction with oneself or one's
achievements Of - is used when you want to indicate or
Explanation:
implicate something. here his comp lacency is indiacated
hence option 1 is correct

Identify appropriate synonyms for the italicized word, from the following options

6. The meeting went on for hours, accommodating loquacious bores who were each

allowed their say.

1) Silent

2) Loquat

3) Repel
4) Gabby

Correct Option
4
is:
Your Option is: 4
Result: Correct
Timetaken: 0.0 secs
Loquacious - talkative Loquat ? an Asian evergreen tree
Explanation:
Repel ?to turn away, resist Gabby ? talkative, garrulous

Identify appropriate synonyms for the italicized word, from the following options
7.
He seemed vague about the source of the quotation.

1) Noted

2) Conspicuous

3) Imprecise

4) Spick

Correct
3
Option is:
Your Option
1
is:
Result: Wrong
Timetaken: 0.0 secs
Vague- not clearly expressed, not having a precise meaning
Explanation: Conspicuous ? obvious to the eye or mind, noticeable
Imprecise ?not exact, inaccurate

8.
Read the passage and answer the questions asked below
The fight against corruption has intensified with the setting up of Transparency

International (TI) and its chapters in many countries across the world. A non-

governmental organization based in Berlin, transparency International was founded

in 1993 by a group of individuals who had become increasingly aware of the

devastating effects of corruption on human development and its distorting effect on

trade and investment. The group aims to stamp out corruption.

Corruption deepens poverty by distorting social and economic development and

disrupting the provision of essential public services. It also hurts democracy by

undermining principles of fair play and justice. Instead of? contracts being awarded

on the basis of fair competition relying on price, quality and innovation, they are

awarded as a result of competitive bribery. The consequences are dire. Investors

keep away and trade suffers. TI defines corruption as the use of public office for

private gain. Decisions are made not for public benefit but for private interests.

Costs incurred are high and prestigious projects are favoured over cost-efficient

development projects. Access to basic social and economic rights such as

education, medical care, adequate shelter and clean water are jeopardized. The

environment is threatened and human rights abuse flourishes. When corruption

increases, regimes become more secretive, less tolerant of dissent and more fearful

of the loss of power.

TI believes that the stamping out of corruption is not the responsibility of any one

agency but the responsibility of all the parties concerned. Every section of the

society must pitch in because corruption affects everyone, especially the poor. The

government, non-governmental organizations, members of society as well as the

local and international business communities must work together if corruption is to

be combated at all levels. As a first step, TI ropes in governments to set up

chapters in countries. The function of each national chapter is to seek consensus

and bring about systematic reform at both national and international levels. The

media is also brought into the picture as one of the primary aims of each chapter is
to raise public awareness. TI does not believe in broadcasting names or attacking

individuals but on building systems that combat corruption. TI publishes a quarterly

newsletter and an annual Corruption Perception Index. The index ranks countries

from the cleanest to the most corrupt, based on the perception of the international

business community, risk analysts and the general public. In 1998, some 85

countries participated and Denmark topped the list as the country seen to have the

least level of corruption. In 1999, 99 countries participated and again Denmark took

the top spot. Malaysia was ranked 29th and 32nd respectively over the 2 years. In

response to criticisms that the Corruption Perception Index had tended to put unfair

emphasis on developing countries, another ranking system was devised. This is the

Bribe Payers Index (BPI) which was introduced in 1999. This survey attempts to

gauge the tendency to bribe senior public officials by major corporations. The

survey ranks Sweden as the country least likely to offer bribes while China occupies

the bottom spot at number 19. Malaysia is ranked 15th. The Malaysian chapter of

Transparency International is known as the Kuala Lumpur Society for Transparency

and Integrity and at present is headed by Tunku Abdul Aziz. The society believes

that any attempt to redress the deteriorating scenario in Malaysia must be based on

a long- term plan. First, the public must be aware of their constitutional rights and

ensure that these rights are not taken away from them. Then they should assert

their right to good governance. To ensure that there is greater accountability in

both the public and private sectors, there is a need to institute more checks and

balances. Information should also not be withheld but made available to the public

so that they will be better informed to make decisions. To stamp out graft, there are

some who believe that the penalties and consequences that will befall the culprits if

found guilty of corruption should be made known to all. At the same time, a sense

of outrage must be developed in people so that they get angry at corruption

because it is about injustice, dishonesty and the impoverishment of many for the

benefit of a few. Nurturing such an attitude can only begin at home. If what make a
person incorruptible are the values he holds on to -- personal, ethical and religious

-- then the home exerts a mighty influence on inculcating these values in

individuals.

Graft must be stamped out. It is insidious and evil and in time to come, will affect

every level of society. It is time for people to stand up and act in a concerted

manner to rid society of this menace. Otherwise there is every possibility that

corruption becomes a way of life undermining justice and fair play and all that is

good in society.

How does corruption deepen poverty?

1) Cost-efficient development projects are not carried out

2) Money is channelled away to fund more prestigious projects.

3) The poor are totally ignored.

4) Basic amenities are not provided to the poor.

Correct Option
2
is:
Your Option is: 1
Result: Wrong
Timetaken: 0.0 secs
Lines 1-3 of paragraph 2: ?Corruption deepens poverty
Explanation:
by.......public services?

9.
Read the passage and answer the questions asked below.

Historians have only recently begun to note the increase in demand for luxury

goods and services that took place in Eighteenth century England. McKendrick has

explored the Wedgwood firm?s remarkable success in marketing luxury pottery;

Plumb has written about the proliferation of provincial theatres, musical festivals,

and children?s toys and books. While the fact of this consumer revolution is hardly
in doubt, three key questions remain: Who were the consumers? What were their

motives? And what were the effects of the new demand for luxuries? An answer to

the first of these has been difficult to obtain. Although it has been possible to infer

from the goods and services actually produced what manufacturers and servicing

traders thought their customers wanted, only a study of relevant personal

documents written by actual consumers will provide a precise picture of who wanted

what. We still need to know how large this consumer market was and how far down

the social scale the consumer demand for luxury goods penetrated. With regard to

this last question, we might note in passing that Thompson, while rightly restoring

labouring people to the stage of eighteenth-century English history, has probably

exaggerated the opposition of these people to the inroads of capitalist consumerism

in general; for example, labouring people in eighteenth-century England readily

shifted from home-brewed beer to standardized beer produced by huge, heavily

capitalized urban breweries.

To answer the question of why consumers became so eager to buy, some historians

have pointed to the ability of manufacturers to advertise in a relatively uncensored

press. This, however, hardly seems a sufficient answer. McKendrick favours a Veblen

model of conspicuous consumption stimulated by competition for status. The

middling sort bought goods and services because they wanted to follow fashions set

by the rich. Again, we may wonder whether this explanation is sufficient. Do not

people enjoy buying things as a form of self-gratification? If so, consumerism could

be seen as a product of the rise of new concepts of individualism and materialism,

but not necessarily of the frenzy for conspicuous competition. Finally, what were the

consequences of this consumer demand for luxuries? McKendrick claims that it goes

a long way towards explaining the coming of the Industrial Revolution. But does it?

What, for example, does the production of high-quality pottery and toys have to do

with the development of iron manufacture or textile mills? It is perfectly possible to

have the psychology and reality of a consumer society without a heavy industrial
sector.

That future exploration of these key questions is undoubtedly necessary should not,

however, diminish the force of the conclusion of recent studies: the insatiable

demand in eighteenth-century England for frivolous as well as useful goods and

services foreshadows our own world.

In the first paragraph, the author mentions McKendrick and Plumb most probably in

order to:

1) Contrast their views on the subject of luxury consumerism in eighteenth-century


England

2) Indicate the inadequacy of historiographical approaches to eighteenth-century


English history

3) Give examples of historians who have helped to establish the fact of growing
consumerisms in eighteenth century England

4) Support the contention that key questions about eighteenth- century


consumerism remain to be answered

5) Compare one historian interest in luxury goods such as pottery to another


historian?s interest in luxury services such as musical festivals

Correct
3
Option is:
Your Option
1
is:
Result: Wrong
Timetaken: 0.0 secs
Referential question) The first line of the passage talks about
how historians have recently begun to note the increase in
luxury goods in the eighteenth century England. To subst
Explanation:
antiate this, the author follows it up by giving examples of
historians like McK endrick and Plumb who have recently
done researches pertaining to this.

10.
Read the passage and answer the questions asked below

The fight against corruption has intensified with the setting up of Transparency
International (TI) and its chapters in many countries across the world. A non-

governmental organization based in Berlin, transparency International was founded

in 1993 by a group of individuals who had become increasingly aware of the

devastating effects of corruption on human development and its distorting effect on

trade and investment. The group aims to stamp out corruption.

Corruption deepens poverty by distorting social and economic development and

disrupting the provision of essential public services. It also hurts democracy by

undermining principles of fair play and justice. Instead of? contracts being awarded

on the basis of fair competition relying on price, quality and innovation, they are

awarded as a result of competitive bribery. The consequences are dire. Investors

keep away and trade suffers. TI defines corruption as the use of public office for

private gain. Decisions are made not for public benefit but for private interests.

Costs incurred are high and prestigious projects are favoured over cost-efficient

development projects. Access to basic social and economic rights such as

education, medical care, adequate shelter and clean water are jeopardized. The

environment is threatened and human rights abuse flourishes. When corruption

increases, regimes become more secretive, less tolerant of dissent and more fearful

of the loss of power.

TI believes that the stamping out of corruption is not the responsibility of any one

agency but the responsibility of all the parties concerned. Every section of the

society must pitch in because corruption affects everyone, especially the poor. The

government, non-governmental organizations, members of society as well as the

local and international business communities must work together if corruption is to

be combated at all levels. As a first step, TI ropes in governments to set up

chapters in countries. The function of each national chapter is to seek consensus

and bring about systematic reform at both national and international levels. The

media is also brought into the picture as one of the primary aims of each chapter is

to raise public awareness. TI does not believe in broadcasting names or attacking


individuals but on building systems that combat corruption. TI publishes a quarterly

newsletter and an annual Corruption Perception Index. The index ranks countries

from the cleanest to the most corrupt, based on the perception of the international

business community, risk analysts and the general public. In 1998, some 85

countries participated and Denmark topped the list as the country seen to have the

least level of corruption. In 1999, 99 countries participated and again Denmark took

the top spot. Malaysia was ranked 29th and 32nd respectively over the 2 years. In

response to criticisms that the Corruption Perception Index had tended to put unfair

emphasis on developing countries, another ranking system was devised. This is the

Bribe Payers Index (BPI) which was introduced in 1999. This survey attempts to

gauge the tendency to bribe senior public officials by major corporations. The

survey ranks Sweden as the country least likely to offer bribes while China occupies

the bottom spot at number 19. Malaysia is ranked 15th. The Malaysian chapter of

Transparency International is known as the Kuala Lumpur Society for Transparency

and Integrity and at present is headed by Tunku Abdul Aziz. The society believes

that any attempt to redress the deteriorating scenario in Malaysia must be based on

a long- term plan. First, the public must be aware of their constitutional rights and

ensure that these rights are not taken away from them. Then they should assert

their right to good governance. To ensure that there is greater accountability in

both the public and private sectors, there is a need to institute more checks and

balances. Information should also not be withheld but made available to the public

so that they will be better informed to make decisions. To stamp out graft, there are

some who believe that the penalties and consequences that will befall the culprits if

found guilty of corruption should be made known to all. At the same time, a sense

of outrage must be developed in people so that they get angry at corruption

because it is about injustice, dishonesty and the impoverishment of many for the

benefit of a few. Nurturing such an attitude can only begin at home. If what make a

person incorruptible are the values he holds on to -- personal, ethical and religious
-- then the home exerts a mighty influence on inculcating these values in

individuals.

Graft must be stamped out. It is insidious and evil and in time to come, will affect

every level of society. It is time for people to stand up and act in a concerted

manner to rid society of this menace. Otherwise there is every possibility that

corruption becomes a way of life undermining justice and fair play and all that is

good in society.

How is the role of Transparency International crucial in the battle against

corruption?

1) It gets people in different countries to join its organization.

2) It gets governments? agreement to set up a branch in their country.

3) It has global support

4) It declares the finds of a survey showing levels of corruption.

Correct
2
Option is:
Your Option
2
is:
Result: Correct
Timetaken: 0.0 secs
Explanation: Option "It gets people in different countries to join its
organization.": Though TI has chapters in many countries
across the world, it is not explicitly stated that it gets people
of different countries to join its organisation. This opt ion is
eliminated. Option "It gets governments? agreement to set up
a branch in their country.": Lines 8 and 9 of paragraph 4: ?as
the first step TI ropes in governments to set up chapters in
countries?. This role is crucial in the battle against
corruption. T his is the answer. Option "It has global
support" and option "It declares the finds of a survey sho
wing levels of corruption.": Though it has global support and
it conducts a survey (given in the first senten ce of paragraph
5), we do not know how crucial these roles are in the battle a
gainst corruption.

Fill in the following blanks with appropriate articles.


11.
People elected Lincoln ______ President of the United States.

1) the

2) an

3) a

4) None of these

Correct Option
1
is:
Your Option is: 1
Result: Correct
Timetaken: 0.0 secs
?President? should be preceded by definite article ?the? as
Explanation: the reference is to a specific president (president of the
United States)

Fill in the following with appropriate words.


12.
If this dictionary is not yours, _______ can it be?

1) what else

2) who else

3) which else?s

4) who else?s
Correct
4
Option is:
Your Option
4
is:
Result: Correct
Timetaken: 0.0 secs
?Who else? is a pronoun used in interrogatives. Here the
Explanation: question is about wh om the dictionary belongs to . Hence
possessive pronoun ?who else?s? should be used.

Identify appropriate antonyms for the italicized word, from the following options.
13.
Just a tentative schedule.

1) Contingent

2) Certain

3) Tacit

4) Raceme

Correct
2
Option is:
Your Option
2
is:
Result: Correct
Timetaken: 0.0 secs
Tentative ? not fully worked out, uncertain Contingent ?
likely, but not certain to happen Tacit ?implied, unexpressed
Explanation:
Raceme ?an inflorescence having stalked flowers arranged
singly along an un branched axis

14.
Read the passage and answer the questions asked below

The fight against corruption has intensified with the setting up of Transparency

International (TI) and its chapters in many countries across the world. A non-
governmental organization based in Berlin, transparency International was founded

in 1993 by a group of individuals who had become increasingly aware of the

devastating effects of corruption on human development and its distorting effect on

trade and investment. The group aims to stamp out corruption.

Corruption deepens poverty by distorting social and economic development and

disrupting the provision of essential public services. It also hurts democracy by

undermining principles of fair play and justice. Instead of? contracts being awarded

on the basis of fair competition relying on price, quality and innovation, they are

awarded as a result of competitive bribery. The consequences are dire. Investors

keep away and trade suffers. TI defines corruption as the use of public office for

private gain. Decisions are made not for public benefit but for private interests.

Costs incurred are high and prestigious projects are favoured over cost-efficient

development projects. Access to basic social and economic rights such as

education, medical care, adequate shelter and clean water are jeopardized. The

environment is threatened and human rights abuse flourishes. When corruption

increases, regimes become more secretive, less tolerant of dissent and more fearful

of the loss of power.

TI believes that the stamping out of corruption is not the responsibility of any one

agency but the responsibility of all the parties concerned. Every section of the

society must pitch in because corruption affects everyone, especially the poor. The

government, non-governmental organizations, members of society as well as the

local and international business communities must work together if corruption is to

be combated at all levels. As a first step, TI ropes in governments to set up

chapters in countries. The function of each national chapter is to seek consensus

and bring about systematic reform at both national and international levels. The

media is also brought into the picture as one of the primary aims of each chapter is

to raise public awareness. TI does not believe in broadcasting names or attacking

individuals but on building systems that combat corruption. TI publishes a quarterly


newsletter and an annual Corruption Perception Index. The index ranks countries

from the cleanest to the most corrupt, based on the perception of the international

business community, risk analysts and the general public. In 1998, some 85

countries participated and Denmark topped the list as the country seen to have the

least level of corruption. In 1999, 99 countries participated and again Denmark took

the top spot. Malaysia was ranked 29th and 32nd respectively over the 2 years. In

response to criticisms that the Corruption Perception Index had tended to put unfair

emphasis on developing countries, another ranking system was devised. This is the

Bribe Payers Index (BPI) which was introduced in 1999. This survey attempts to

gauge the tendency to bribe senior public officials by major corporations. The

survey ranks Sweden as the country least likely to offer bribes while China occupies

the bottom spot at number 19. Malaysia is ranked 15th. The Malaysian chapter of

Transparency International is known as the Kuala Lumpur Society for Transparency

and Integrity and at present is headed by Tunku Abdul Aziz. The society believes

that any attempt to redress the deteriorating scenario in Malaysia must be based on

a long- term plan. First, the public must be aware of their constitutional rights and

ensure that these rights are not taken away from them. Then they should assert

their right to good governance. To ensure that there is greater accountability in

both the public and private sectors, there is a need to institute more checks and

balances. Information should also not be withheld but made available to the public

so that they will be better informed to make decisions. To stamp out graft, there are

some who believe that the penalties and consequences that will befall the culprits if

found guilty of corruption should be made known to all. At the same time, a sense

of outrage must be developed in people so that they get angry at corruption

because it is about injustice, dishonesty and the impoverishment of many for the

benefit of a few. Nurturing such an attitude can only begin at home. If what make a

person incorruptible are the values he holds on to -- personal, ethical and religious

-- then the home exerts a mighty influence on inculcating these values in


individuals. Graft must be stamped out. It is insidious and evil and in time to come,

will affect every level of society. It is time for people to stand up and act in a

concerted manner to rid society of this menace. Otherwise there is every possibility

that corruption becomes a way of life undermining justice and fair play and all that

is good in society.

What is a good title for this passage?

1) Helping the Poor

2) Fighting Corruption

3) Corruption

4) Ensuring Basic Human Rights

Correct Option
3
is:
Your Option is: 2
Result: Wrong
Timetaken: 0.0 secs
Slightly inferential question) The entire passage talks about
Explanation:
the different measures taken to fight corruptio n.

15.
Rearrange the sentences in a logical sequence.

A. The word failure, it seems, is not good for building self-esteemed school children.

B. Liz Beattie, a 37-year old veteran primary-level instructor, proposed that the

word failure should be banned from classrooms and replaced with the more

appealing phrase deferred success so as not to discourage students from continuing

efforts to achieve.

C. Although the motion ultimately experienced its own ?deferred success,? it was

not without supporters among the 35,000-member teachers association.

D. One Wesley Paxton, a member of the PAT Council, expressed his enthusiastic
agreement, saying: ?It?s time we made the word ?fail? redundant and replaced it

with ?please do a bit more?.?

E. In the summer of 2005, a British school teacher proposed a rather controversial

motion to her union, the Professional Association of Teachers (PAT).

1) ADBCE

2) BACDE

3) EBCDA

4) CEDAB

5) ABCDE

Correct
3
Option is:
Your Option
4
is:
Result: Wrong
Timetaken: 0.0 secs
Sentence E talks about a British school teacher and sentence
B mentions th e name of the teacher (Liz Beattie). Thus EB
Explanation:
link is established. The only opti on with EB link is option
EBCDA .

Use appropriate prepositions to fill in the blanks.


16.
We ran five laps _______the gym and then practiced our jump shots

1) against

2) Across

3) around

4) among
Correct Option
3
is:
Your Option is: 1
Result: Wrong
Timetaken: 0.0 secs
We cannot run across the gym, among the gym and against
Explanation:
the gym. Hence the answer is around

Rearrange the sentences in a logical sequence.

A. The word failure, it seems, is not good for building self-esteemed school children.

B. Liz Beattie, a 37-year old veteran primary-level instructor, proposed that the

word failure should be banned from classrooms and replaced with the more

appealing phrase deferred success so as not to discourage students from continuing

efforts to achieve.

17. C. Although the motion ultimately experienced its own ?deferred success,? it was

not without supporters among the 35,000-member teachers association.

D. One Wesley Paxton, a member of the PAT Council, expressed his enthusiastic

agreement, saying: ?It?s time we made the word ?fail? redundant and replaced it

with ?please do a bit more?.?

E. In the summer of 2005, a British school teacher proposed a rather controversial

motion to her union, the Professional Association of Teachers (PAT).

1) ADBCE

2) BACDE

3) EBCDA

4) CEDAB

5) ABCDE

Correct
3
Option is:
Your Option
4
is:
Result: Wrong
Timetaken: 0.0 secs
Sentence E talks about a British school teacher and sentence
B mentions th e name of the teacher (Liz Beattie). Thus EB
Explanation:
link is established. The only opti on with EB link is option
EBCDA.

18.
Read the passage and answer the questions asked below.

Historians have only recently begun to note the increase in demand for luxury

goods and services that took place in Eighteenth century England. McKendrick has

explored the Wedgwood firm?s remarkable success in marketing luxury pottery;

Plumb has written about the proliferation of provincial theatres, musical festivals,

and children?s toys and books. While the fact of this consumer revolution is hardly

in doubt, three key questions remain: Who were the consumers? What were their

motives? And what were the effects of the new demand for luxuries? An answer to

the first of these has been difficult to obtain. Although it has been possible to infer

from the goods and services actually produced what manufacturers and servicing

traders thought their customers wanted, only a study of relevant personal

documents written by actual consumers will provide a precise picture of who wanted

what. We still need to know how large this consumer market was and how far down

the social scale the consumer demand for luxury goods penetrated. With regard to

this last question, we might note in passing that Thompson, while rightly restoring

labouring people to the stage of eighteenth-century English history, has probably

exaggerated the opposition of these people to the inroads of capitalist consumerism

in general; for example, labouring people in eighteenth-century England readily

shifted from home-brewed beer to standardized beer produced by huge, heavily

capitalized urban breweries.

To answer the question of why consumers became so eager to buy, some historians
have pointed to the ability of manufacturers to advertise in a relatively uncensored

press. This, however, hardly seems a sufficient answer. McKendrick favours a Veblen

model of conspicuous consumption stimulated by competition for status. The

middling sort bought goods and services because they wanted to follow fashions set

by the rich. Again, we may wonder whether this explanation is sufficient. Do not

people enjoy buying things as a form of self-gratification? If so, consumerism could

be seen as a product of the rise of new concepts of individualism and materialism,

but not necessarily of the frenzy for conspicuous competition. Finally, what were the

consequences of this consumer demand for luxuries? McKendrick claims that it goes

a long way towards explaining the coming of the Industrial Revolution. But does it?

What, for example, does the production of high-quality pottery and toys have to do

with the development of iron manufacture or textile mills? It is perfectly possible to

have the psychology and reality of a consumer society without a heavy industrial

sector. That future exploration of these key questions is undoubtedly necessary

should not, however, diminish the force of the conclusion of recent studies: the

insatiable demand in eighteenth-century England for frivolous as well as useful

goods and services foreshadows our own world.

In the third paragraph, the author is primarily concerned with:

1) Contrasting two theses and offering a compromise

2) Questioning two explanations and proposing a possible alternative to them

3) Paraphrasing the work of two historians and questioning their assumptions

4) Examining two theories and endorsing one over the other

5) Raising several questions but implying that they cannot be answered

Correct
2
Option is:
Your Option 3
is:
Result: Wrong
Timetaken: 0.0 secs
(Inferential question) The third paragraph discusses about
two explanations put forth by historians as to why consumers
became so eager to buy. The author however disputes both
Explanation: by saying that they are not sufficient. He then suggests an
alternative th eory that consumerism could be seen as a
product of the rise of new concept s of individualism and
materialism.

19.
Read the passage and answer the questions asked below

The fight against corruption has intensified with the setting up of Transparency

International (TI) and its chapters in many countries across the world. A non-

governmental organization based in Berlin, transparency International was founded

in 1993 by a group of individuals who had become increasingly aware of the

devastating effects of corruption on human development and its distorting effect on

trade and investment. The group aims to stamp out corruption.

Corruption deepens poverty by distorting social and economic development and

disrupting the provision of essential public services. It also hurts democracy by

undermining principles of fair play and justice. Instead of? contracts being awarded

on the basis of fair competition relying on price, quality and innovation, they are

awarded as a result of competitive bribery. The consequences are dire. Investors

keep away and trade suffers. TI defines corruption as the use of public office for

private gain. Decisions are made not for public benefit but for private interests.

Costs incurred are high and prestigious projects are favoured over cost-efficient

development projects. Access to basic social and economic rights such as

education, medical care, adequate shelter and clean water are jeopardized. The

environment is threatened and human rights abuse flourishes. When corruption

increases, regimes become more secretive, less tolerant of dissent and more fearful

of the loss of power.


TI believes that the stamping out of corruption is not the responsibility of any one

agency but the responsibility of all the parties concerned. Every section of the

society must pitch in because corruption affects everyone, especially the poor. The

government, non-governmental organizations, members of society as well as the

local and international business communities must work together if corruption is to

be combated at all levels. As a first step, TI ropes in governments to set up

chapters in countries. The function of each national chapter is to seek consensus

and bring about systematic reform at both national and international levels. The

media is also brought into the picture as one of the primary aims of each chapter is

to raise public awareness. TI does not believe in broadcasting names or attacking

individuals but on building systems that combat corruption. TI publishes a quarterly

newsletter and an annual Corruption Perception Index. The index ranks countries

from the cleanest to the most corrupt, based on the perception of the international

business community, risk analysts and the general public. In 1998, some 85

countries participated and Denmark topped the list as the country seen to have the

least level of corruption. In 1999, 99 countries participated and again Denmark took

the top spot. Malaysia was ranked 29th and 32nd respectively over the 2 years. In

response to criticisms that the Corruption Perception Index had tended to put unfair

emphasis on developing countries, another ranking system was devised. This is the

Bribe Payers Index (BPI) which was introduced in 1999. This survey attempts to

gauge the tendency to bribe senior public officials by major corporations. The

survey ranks Sweden as the country least likely to offer bribes while China occupies

the bottom spot at number 19. Malaysia is ranked 15th. The Malaysian chapter of

Transparency International is known as the Kuala Lumpur Society for Transparency

and Integrity and at present is headed by Tunku Abdul Aziz. The society believes

that any attempt to redress the deteriorating scenario in Malaysia must be based on

a long- term plan. First, the public must be aware of their constitutional rights and

ensure that these rights are not taken away from them. Then they should assert
their right to good governance. To ensure that there is greater accountability in

both the public and private sectors, there is a need to institute more checks and

balances. Information should also not be withheld but made available to the public

so that they will be better informed to make decisions. To stamp out graft, there are

some who believe that the penalties and consequences that will befall the culprits if

found guilty of corruption should be made known to all. At the same time, a sense

of outrage must be developed in people so that they get angry at corruption

because it is about injustice, dishonesty and the impoverishment of many for the

benefit of a few. Nurturing such an attitude can only begin at home. If what make a

person incorruptible are the values he holds on to -- personal, ethical and religious

-- then the home exerts a mighty influence on inculcating these values in

individuals.

Graft must be stamped out. It is insidious and evil and in time to come, will affect

every level of society. It is time for people to stand up and act in a concerted

manner to rid society of this menace. Otherwise there is every possibility that

corruption becomes a way of life undermining justice and fair play and all that is

good in society.

What is the effect of letting the public be aware of the penalties and consequences

if found guilty of corruption? It instills in everyone:

1) the fear of punishment if caught

2) anger towards the guilty

3) a sense of shame if found guilty

4) the complacency to do nothing

Correct
1
Option is:
Your Option
2
is:
Result: Wrong
Timetaken: 0.0 secs
(Slightly inferential question) Lines 1-3 of paragraph 7: ?To
stamp out graft, there ....known to all.? There are some
Explanation: people who believe that if penalties are made known to all,
pe ople will have a fear of the punishment and hence will
refrain from practising c orruption.

Fill in the following with appropriate words. Would you slow down a bit? I can?t

____ you.
20.

1) keep up with

2) make up to

3) put up with

4) hold on to

Correct
1
Option is:
Your Option
4
is:
Result: Wrong
Timetaken: 0.0 secs
Keep up with- to continue at an equal level or pace with
somebody or somethi ng Make up to- to behave in a friendly
way towards someone in order to get som ething for yourself
Explanation:
Put up with - tolerate Hold on to- to continue feeling or
believing something, to not lose something, t o stop someone
from leaving you
21.
Read the passage and answer the questions asked below.

Historians have only recently begun to note the increase in demand for luxury

goods and services that took place in Eighteenth century England. McKendrick has

explored the Wedgwood firm?s remarkable success in marketing luxury pottery;

Plumb has written about the proliferation of provincial theatres, musical festivals,

and children?s toys and books. While the fact of this consumer revolution is hardly

in doubt, three key questions remain: Who were the consumers? What were their

motives? And what were the effects of the new demand for luxuries? An answer to

the first of these has been difficult to obtain. Although it has been possible to infer

from the goods and services actually produced what manufacturers and servicing

traders thought their customers wanted, only a study of relevant personal

documents written by actual consumers will provide a precise picture of who wanted

what. We still need to know how large this consumer market was and how far down

the social scale the consumer demand for luxury goods penetrated. With regard to

this last question, we might note in passing that Thompson, while rightly restoring

labouring people to the stage of eighteenth-century English history, has probably

exaggerated the opposition of these people to the inroads of capitalist consumerism

in general; for example, labouring people in eighteenth-century England readily

shifted from home-brewed beer to standardized beer produced by huge, heavily

capitalized urban breweries.

To answer the question of why consumers became so eager to buy, some historians

have pointed to the ability of manufacturers to advertise in a relatively uncensored

press. This, however, hardly seems a sufficient answer. McKendrick favours a Veblen

model of conspicuous consumption stimulated by competition for status. The

middling sort bought goods and services because they wanted to follow fashions set

by the rich. Again, we may wonder whether this explanation is sufficient. Do not

people enjoy buying things as a form of self-gratification? If so, consumerism could

be seen as a product of the rise of new concepts of individualism and materialism,


but not necessarily of the frenzy for conspicuous competition. Finally, what were the

consequences of this consumer demand for luxuries? McKendrick claims that it goes

a long way towards explaining the coming of the Industrial Revolution. But does it?

What, for example, does the production of high-quality pottery and toys have to do

with the development of iron manufacture or textile mills? It is perfectly possible to

have the psychology and reality of a consumer society without a heavy industrial

sector. That future exploration of these key questions is undoubtedly necessary

should not, however, diminish the force of the conclusion of recent studies: the

insatiable demand in eighteenth-century England for frivolous as well as useful

goods and services foreshadows our own world.

According to the passage, Thompson attributes to labouring people in eighteenth-

century England which of the following attitude towards capitalist consumerism?

1) Enthusiasm

2) Curiosity

3) Ambivalence

4) Stubbornness

5) Hostility

Correct
5
Option is:
Your Option
1
is:
Result: Wrong
Timetaken: 0.0 secs
(Referential question) Lines 10-13 of paragraph 2: ?
Thompson, while rightly......has probably exagger ated the
Explanation:
opposition of these people to the inroads of capitalist
consumeris m...?
In each question below is given a statement followed by two assumptions

numbered I and II. Consider the statement and decide which of the given

assumptions is implicit. Give Answer:

(A) If only assumption I is implicit

(B) If only assumption II is implicit

(C) If either I or II is implicit

22. (D) If neither I nor II is implicit

(E) If both I and II are implicit.

Statement : The civic authority appealed to the people for reduction in usage of

water as there may be an acute shortage during the coming weeks.

Assumptions : I. There will be no rain in recent future.

II. The people are ready to follow the advice of the civic authority.

1) A

2) B

3) C

4) D

5) E

Correct
2
Option is:
Your Option
4
is:
Result: Wrong
Timetaken: 0.0 secs
Explanation: In assumption I,they have mentioned about rain but in
statement they didn't even tell about rain.So,Assumption I is
wrong.Whereas in assumption II people are ready to follow
the instructions.Due to scarcity of water,we have to reduce
the usage of water.So,this is correct answer.

Identify appropriate antonyms for the italicized word, from the following options.
23.
This pill will alleviate your headache.

1) Recover

2) Improve

3) Lesson

4) Aggravate

Correct Option
4
is:
Your Option is: 1
Result: Wrong
Timetaken: 0.0 secs
Alleviate -relieve, lessen Aggravate - to make worse,
Explanation:
increase

Identify appropriate synonyms for the italicized word, from the following options
24.
I was so angry; I had to vent my feelings

1) Drain

2) Quaint

3) Viability

4) Crabbed

Correct
1
Option is:
Your Option 2
is:
Result: Wrong
Timetaken: 0.0 secs
Vent ? release, unleash, take out Drain ? to discharge, to
exhaust physically or emotionally Quaint ? unusual,
Explanation:
strikingly old-fashioned Viability ? workability, feasibility
Crabbed ? gloomy, morose, difficult to read or understand

25.
Read the passage and answer the questions asked below

The fight against corruption has intensified with the setting up of Transparency

International (TI) and its chapters in many countries across the world. A non-

governmental organization based in Berlin, transparency International was founded

in 1993 by a group of individuals who had become increasingly aware of the

devastating effects of corruption on human development and its distorting effect on

trade and investment. The group aims to stamp out corruption.

Corruption deepens poverty by distorting social and economic development and

disrupting the provision of essential public services. It also hurts democracy by

undermining principles of fair play and justice. Instead of? contracts being awarded

on the basis of fair competition relying on price, quality and innovation, they are

awarded as a result of competitive bribery. The consequences are dire. Investors

keep away and trade suffers.

TI defines corruption as the use of public office for private gain. Decisions are made

not for public benefit but for private interests. Costs incurred are high and

prestigious projects are favoured over cost-efficient development projects. Access

to basic social and economic rights such as education, medical care, adequate

shelter and clean water are jeopardized. The environment is threatened and human

rights abuse flourishes. When corruption increases, regimes become more

secretive, less tolerant of dissent and more fearful of the loss of power.

TI believes that the stamping out of corruption is not the responsibility of any one

agency but the responsibility of all the parties concerned. Every section of the
society must pitch in because corruption affects everyone, especially the poor. The

government, non-governmental organizations, members of society as well as the

local and international business communities must work together if corruption is to

be combated at all levels. As a first step, TI ropes in governments to set up

chapters in countries. The function of each national chapter is to seek consensus

and bring about systematic reform at both national and international levels. The

media is also brought into the picture as one of the primary aims of each chapter is

to raise public awareness. TI does not believe in broadcasting names or attacking

individuals but on building systems that combat corruption. TI publishes a quarterly

newsletter and an annual Corruption Perception Index. The index ranks countries

from the cleanest to the most corrupt, based on the perception of the international

business community, risk analysts and the general public. In 1998, some 85

countries participated and Denmark topped the list as the country seen to have the

least level of corruption. In 1999, 99 countries participated and again Denmark took

the top spot. Malaysia was ranked 29th and 32nd respectively over the 2 years. In

response to criticisms that the Corruption Perception Index had tended to put unfair

emphasis on developing countries, another ranking system was devised. This is the

Bribe Payers Index (BPI) which was introduced in 1999. This survey attempts to

gauge the tendency to bribe senior public officials by major corporations. The

survey ranks Sweden as the country least likely to offer bribes while China occupies

the bottom spot at number 19. Malaysia is ranked 15th. The Malaysian chapter of

Transparency International is known as the Kuala Lumpur Society for Transparency

and Integrity and at present is headed by Tunku Abdul Aziz. The society believes

that any attempt to redress the deteriorating scenario in Malaysia must be based on

a long- term plan. First, the public must be aware of their constitutional rights and

ensure that these rights are not taken away from them. Then they should assert

their right to good governance. To ensure that there is greater accountability in

both the public and private sectors, there is a need to institute more checks and
balances. Information should also not be withheld but made available to the public

so that they will be better informed to make decisions. To stamp out graft, there are

some who believe that the penalties and consequences that will befall the culprits if

found guilty of corruption should be made known to all. At the same time, a sense

of outrage must be developed in people so that they get angry at corruption

because it is about injustice, dishonesty and the impoverishment of many for the

benefit of a few. Nurturing such an attitude can only begin at home. If what make a

person incorruptible are the values he holds on to -- personal, ethical and religious

-- then the home exerts a mighty influence on inculcating these values in

individuals.

Graft must be stamped out. It is insidious and evil and in time to come, will affect

every level of society. It is time for people to stand up and act in a concerted

manner to rid society of this menace. Otherwise there is every possibility that

corruption becomes a way of life undermining justice and fair play and all that is

good in society.

What is a good title for this passage?

1) Helping the Poor

2) Fighting Corruption

3) Corruption

4) Ensuring Basic Human Rights

Correct Option
3
is:
Your Option is: 4
Result: Wrong
Timetaken: 0.0 secs
Explanation: (Slightly inferential question) The entire passage talks
about the different measures taken to fight corruptio n.

Identify appropriate synonyms for the italicized word, from the following options
26.
Her emotional intensity was almost tangible in the close air of the holy chamber.

1) Impalpable

2) Real

3) Tandem

4) Abacination

Correct
2
Option is:
Your Option
1
is:
Result: Wrong
Timetaken: 0.0 secs
Tangible ? substantially real, capable of being perceived
especially by the sen se of touch Impalpable ? incapable of
being felt by touch, not readily discerned by the min d
Explanation: Tandem ? working or occurring in conjunction with each
other Abacination ? a form of torture or punishment in which
the victim is blinded by having a red-hot metal plate held
before their eyes

What were the servants dogs when the robbery _______________ places
1.

1) Take

2) Took

3) Was in

4) Were in
Correct Option is: 2
Your Option is: 4
Result: Wrong
Timetaken: 0.0 secs
Explanation: As the sentence is in past tense,'took' must be used.

Select word or phrase which best expresses the meaning of the given word

2. ONRUSH

1) Emergence

2) Surge

3) Flight

Correct Option
2
is:
Your Option is: 2
Result: Correct
Timetaken: 0.0 secs
Direct meaning of ONRUSH is
Explanation:
surge,hurry,flood,storm,etc.

Select the correct option that fills the blank(s) to make the sentence

3. meaningfully complete

The meeting in the office was held behind _____________ doors

1) Close up

2) Close
3) Closing

4) Closed

Correct Option is: 4


Your Option is: 2
Result: Wrong
Timetaken: 0.0 secs
Explanation: meeting held after the door closed.So answer is option 4

Select the correct option that fills the blank(s) to make the sentence

4. meaningfully complete

The roads to hills ____________ closed because of land sides

1) was

2) is

3) were

4) be

Correct Option
3
is:
Your Option is: 1
Result: Wrong
Timetaken: 0.0 secs
As roads is plural and the sentence is in past tense,were
Explanation:
sholud be used

Select the option that is most nearly opposite to the given word
5.
INTENT (Opposite)
1) Distant

2) Target

3) Reluctant

4) Content

5) Soft

Correct Option is: 5


Your Option is: 3
Result: Wrong
Timetaken: 0.0 secs
Explanation: Direct opposite of INTENT is unstable,soft,flexible,etc.

Select the correct option that fills the blank(s) to make the sentence

6. meaningfully complete

In India,women ________ only three percent of senior management

1) Contain

2) Involve

3) Comprise

4) Contains

5) Comprises

Correct Option is: 5


Your Option is: 4
Result: Wrong
Timetaken: 0.0 secs
comprises means 'consists of'.So correct answer is
Explanation:
option 5
In each of the following questions, find out which part of the sentence has

an error. if there is no mistake the answer is 'no error'

7. 3. While proceeding on leave / he had orally committed that / he will resume after

two days/ No error

1) While proceeding on leave

2) he had orally committed that

3) he will resume after two days

4) No error

Correct Option
3
is:
Your Option is: 2
Result: Wrong
Timetaken: 0.0 secs
As 'had' is used in the second part of the sentence,'would'
Explanation:
must be used instead of will in the next part.

Read the sentence to find out whether there is any grammatical error in it.

The error, if any,will be one part of the answer. ignore the error of

8. punctuation, if any

(A) when the capital called the crew(B) Each of the crew members run

together(D)The deck of the ship

1) A

2) B

3) C
4) No error

Correct Option
2
is:
Your Option is: 1
Result: Wrong
Timetaken: 0.0 secs
As 'each' is used before, crew members cannot be used in
Explanation:
plural form

Select the correct option that fills the blank(s) to make the sentence

meaningfully complete

9. ------ being poor,kaveri still dresses more appropriately than most of her group

mate

1) a) Despite

2) b)Although

3) c)Since

4) d)However

Correct Option
1
is:
Your Option is: 4
Result: Wrong
Timetaken: 0.0 secs
Even kaveri is poor,she dresses well.Only by using the
Explanation:
word 'despite',sentence gives proper meaning.

10.
In each of the following questions, find out which part of the sentence has
an error. if there is no mistake the answer is 'no error'

The Cabinet Ministers and the Prime Minister / was at the airport / to receive the

foreign dignitary / No error

1) a)The Cabinet Ministers and the Prime Minister

2) b)was at the airport

3) to receive the foreign dignitary

4) d)No error

Correct Option
2
is:
Your Option is: 1
Result: Wrong
Timetaken: 0.0 secs
The error is 'was at the airport'.As the subject is in plural
Explanation:
form it should be as 'were at the airport '.

In the question, a part of the sentence is italicized. Alternatives to the

italicized part are given which may improve the construction of correct

11. alternative.

People working in high position in a companies tend to shifting their

work burden by delegating tasks to their subordinate

1) Tend for shifting their work

2) Tend to shift their work

3) tend as to shifting their work

4) no improvement needed
Correct Option
2
is:
Your Option is: 4
Result: Wrong
Timetaken: 0.0 secs
As compared with the sentence and the options,option 2 is
Explanation:
the correct answer.

12.
Passage The Unique Iron Age Experimental Centre at Lajire,about 40km west of

Copenhagen serves as a museum, a classroom and a place to get away from it

all.How did people live during thr Iron Age?How did they support themselves/What

did they eat and how did they cultivate the land?These amd myriad of other

question prodded the pioneers of the Lejre experiment Living in the open and

working 10 hours a day,volunteers from all over Scandinavia led by 30 experts,

built the first in the anticent encampment in a matter of months.The house walls

were of clay,the roofs of hay -all based on original design.Then came the secound

stage-getting back to the basics of living.Families were invited to stay in the

'prehistoric village' for a week or two at a tome and rough it Iron Age-style

Initisally, this experiment proved none too easy for modern Danes accustomed to

central heating,but it convinced the central that there was something to the Lejre

project.Little by little,the modern iron Agers learnt that their huts were,after

all,habitable.The problems were numeouus-smoke belching out from the rough-

and-ready fireplaces into the rooms and so on.These problems,however,have led to

some discoveries:domed smoke ovens made of clay,for examples give out more

heat and consume less fuel than an open fire,and when correctly stoked,they are

practically smokless By contacting other musuems, the Lejre team has been able to

reconstruct ancient weaving looms and pottery kilns. Iron Age dyeing

techniques,using local natural vegetatio,have also been revived,as have ancient

banking and cookling methods.


What can be the title of the passage? a) b) c) d)

1) Modern techniques find their way into pre-historic villages

2) Co-existence of ancient and modern times

3) Glad to be living in the 21st Century

4) Turning back time

Correct Option
4
is:
Your Option is: 2
Result: Wrong
Timetaken: 0.0 secs
As the modern people tried to live in irion age,it can be
Explanation:
reffered as turning back time.

The leaves ----------- yellow and dry


13.

1) were

2) had

3) being

4) was

Correct Option
1
is:
Your Option is: 1
Result: Correct
Timetaken: 0.0 secs
Subject 'leaves' is in plural form,so the correct answer is
Explanation:
option 1

Select word or phrase which best expresses the meaning of the given word

14. CONCEITED

1) Arrogant

2) False

3) Deceive

4) Misconspetion

Correct Option
1
is:
Your Option is: 3
Result: Wrong
Timetaken: 0.0 secs
Direct meaning of ARROGANT is arrogant,big-
Explanation:
talking,self-important,rtc.

Select the correct option that fills the blank(s) to make the sentence

meaningfully complete
15.
today --------- the inagural day of the pub,the drinks were served free of cost

1) was

2) been

3) is

4) being
Correct Option
4
is:
Your Option is: 2
Result: Wrong
Timetaken: 0.0 secs
second half of the sentence is the effect of first half of the
Explanation:
sentence.So only being can be used.

Give the antonym for the bold word ,in the given blank
16.
Her hands are too Rough now. I remember last year they were very_______

1) Nice

2) Firm

3) Smooth

4) Fair

Correct Option is: 3


Your Option is: 4
Result: Wrong
Timetaken: 0.0 secs
Direct opposite of ROUGH is
Explanation:
smooth,soft,polite,calm,etc.

17.
Passage

The Unique Iron Age Experimental Centre at Lajire,about 40km west of Copenhagen

serves as a museum, a classroom and a place to get away from it all.How did

people live during thr Iron Age?How did they support themselves/What did they eat

and how did they cultivate the land?These and myriad of other question prodded

the pioneers of the Lejre experiment


Living in the open and working 10 hours a day,volunteers from all over Scandinavia

led by 30 experts, built the first in the anticent encampment in a matter of

months.The house walls were of clay,the roofs of hay -all based on original

design.Then came the second stage-getting back to the basics of living.Families

were invited to stay in the 'prehistoric village' for a week or two at a tome and

rough it Iron Age-style

Initially, this experiment proved none too easy for modern Danes accustomed to

central heating,but it convinced the central that there was something to the Lejre

project.Little by little,the modern iron Agers learnt that their huts were,after

all,habitable.The problems were numeouus-smoke belching out from the rough-

and-ready fireplaces into the rooms and so on.These problems,however,have led to

some discoveries:domed smoke ovens made of clay,for examples give out more

heat and consume less fuel than an open fire,and when correctly stoked,they are

practically smokless

By contacting other musuems, the Lejre team has been able to reconstruct ancient

weaving looms and pottery kilns. Iron Age dyeing techniques,using local natural

vegetation,have also been revived,as have ancient banking and cooking methods

From the passage what can be inferred to be the centre's initial outlook towards the

Lejre project

1) a) It initiated the project

2) b)b. It eagerly supported was very unqiue

3) c) It felt the project was very unique

4) d) It was apprehensive about it


Correct Option
3
is:
Your Option is: 3
Result: Correct
Timetaken: 0.0 secs
From the first paragraph,it is clear that center's outlook is
Explanation:
the initiation of the project.

In each of the following questions, find out which part of the sentence has

an error. if there is no mistake the answer is 'no error'

18. 1. Neither the plans / suits him and therefore / he decided not to / go out yesterday

/ No error

1) Neither the plans

2) suits him and therefore

3) he decided not to

4) go out yesterday

5) No error

Correct Option
2
is:
Your Option is: 2
Result: Correct
Timetaken: 0.0 secs
The sentence is in past tense,so the verb also must be in
Explanation:
past tense.So 'suited him' is the correct answer.

19.
Select the correct option that fills the blank(s) to make the sentence

meaningfully complete

We as human being get easily _________ by materialistic pleasures of modern age


1) Distracted

2) Attentive

3) Devoted

4) Diligent

Correct Option is: 1


Your Option is: 1
Result: Correct
Timetaken: 0.0 secs
Explanation: distraction the result of materialistic pleasures

Select word or phrase which best expresses the meaning of the given word

20. Basis

1) Foundation

2) Words

3) Explanation

4) Correlate

Correct Option
1
is:
Your Option is: 3
Result: Wrong
Timetaken: 0.0 secs
Direct meaning of 'Basis' is
Explanation:
foundation,bottom,support,ground,etc.

Select word or phrase which best expresses the meaning of the given word

21. ADVENTURER

1) Explorer

2) Homely

3) Native

4) Aimless

Correct Option is: 1


Your Option is: 4
Result: Wrong
Timetaken: 0.0 secs
Explanation: Direct meaning of ADVENTURER is explorer,hero,etc.

22.
Passage

The Unique Iron Age Experimental Centre at Lajire,about 40km west of Copenhagen

serves as a museum, a classroom and a place to get away from it all.How did

people live during thr Iron Age?How did they support themselves/What did they eat

and how did they cultivate the land?These amd myriad of other question prodded

the pioneers of the Lejre experiment

Living in the open and working 10 hours a day,volunteers from all over Scandinavia

led by 30 experts, built the first in the anticent encampment in a matter of

months.The house walls were of clay,the roofs of hay -all based on original

design.Then came the secound stage-getting back to the basics of living.Families

were invited to stay in the 'prehistoric village' for a week or two at a tome and
rough it Iron Age-style

Initisally, this experiment proved none too easy for modern Danes accustomed to

central heating,but it convinced the central that there was something to the Lejre

project.Little by little,the modern iron Agers learnt that their huts were,after

all,habitable.The problems were numeouus-smoke belching out from the rough-

and-ready fireplaces into the rooms and so on.These problems,however,have led to

some discoveries:domed smoke ovens made of clay,for examples give out more

heat and consume less fuel than an open fire,and when correctly stoked,they are

practically smokless

By contacting other musuems, the Lejre team has been able to reconstruct ancient

weaving looms and pottery kilns. Iron Age dyeing techniques,using local natural

vegetatio,have also been revived,as have ancient banking and cookling methods

What is the purpose of bulding the Iron Age experimental center?

1) Prehistoric Village where people can stay for a week or two to get away from
modern living

2) Replicate the Iron Age to get a better understanding of the time and people of
the ers

3) To discover the difference between a doomed smoke oven and an open fire to
identify the more efficient of the two

4) Revive activities of ancient women such as weaving,pottery,dyeing,cooking and


baking

Correct Option
1
is:
Your Option is: 2
Result: Wrong
Timetaken: 0.0 secs
Explanation: Option 2 is the correct answer.Last paragraph in the
passage tells that iron age techniques and natural vegetation
is revived.

Select the correct option that fills the blank(s)

23. ---------- the shirt was washed twice,still he refuced refuce to wear it

1) a) Though

2) b)Because

3) c)However

4) d)Since

5) d)While

Correct Option
1
is:
Your Option is: 1
Result: Correct
Timetaken: 0.0 secs
He refused to wear the shirt,even it is washed
Explanation: twice.sentence gives the proper meaning only when using
'though'.

Select the option that is most nearly opposite to the given word
24.
CLARIFY(opposite)

1) Analyze

2) Simplify

3) Confuse

4) Resolve
Correct Option
2
is:
Your Option is: Not Attempted
Result: Not Attempted
Timetaken: 0.0 secs
Direct opposite of CLARIFY is confuse,complicate,mix
Explanation:
up,stc.

25.
Passage

The Unique Iron Age Experimental Centre at Lajire,about 40km west of Copenhagen

serves as a museum, a classroom and a place to get away from it all.How did

people live during thr Iron Age?How did they support themselves/What did they eat

and how did they cultivate the land?These amd myriad of other question prodded

the pioneers of the Lejre experiment

Living in the open and working 10 hours a day,volunteers from all over Scandinavia

led by 30 experts, built the first in the anticent encampment in a matter of

months.The house walls were of clay,the roofs of hay -all based on original

design.Then came the secound stage-getting back to the basics of living.Families

were invited to stay in the 'prehistoric village' for a week or two at a tome and

rough it Iron Age-style

Initially, this experiment proved none too easy for modern Danes accustomed to

central heating,but it convinced the central that there was something to the Lejre

project.Little by little,the modern iron Agers learnt that their huts were,after

all,habitable.The problems were numeouus-smoke belching out from the rough-

and-ready fireplaces into the rooms and so on.These problems,however,have led to

some discoveries:domed smoke ovens made of clay,for examples give out more

heat and consume less fuel than an open fire,and when correctly stoked,they are

practically smokless

By contacting other musuems, the Lejre team has been able to reconstruct ancient

weaving looms and pottery kilns. Iron Age dyeing techniques,using local natural
vegetation,have also been revived,as have ancient banking and cooking methods

What is the meaning of the sentence 'initially, this experiment proved none too easy

for modern Danes assuctomed to central heating,but it conviced the centre that

there was something to the Lejre project'?

1) Even though staying in thr hunts wasn't easy for the modern people the centre
saw merit in the simple living within huts compared to expensive apartments

2) Staying in the hunts was quite easy for the modern people and the centre also
saw merits in the simple living within hunts compared to to expensive apartments

3) The way of living of the Iron Age proved difficult for the people of the modern
age who are used to living in luxury

4) The way of living of the Iron Age proved very easy for the people of the modern
age since it was not inside the hunts,and they were anyway used to heated rooms

Correct
2
Option is:
Your Option
4
is:
Result: Wrong
Timetaken: 0.0 secs
From the given sentence,it is clear that the way of living of
Explanation: the iron age proved difficult for the modern age people,as
they used to live in luxury appartments.

1.
Sixty years ago,on the evening of august 14,1947 a few hours before britain?s

indian empire was formally divided into the nation states of india and pakistan,lord

lousie mountbatten and his wife,edwina sat down in the vicargal mansion in new

g\delhi to watch the katest bob hpe movie.my favourite brunette.?Large parts of

the subconstitnent were descending into chaos.as the implications of partioning the

indian empire along religious lines became clear to the millions of hindus,muslims
twleve million people would be uprooted and as many as a million murdered but on

that night In mid aguest the bloodbath and the fuller consequence of hasty imperial

retreat ?still lay in the future,and the mountbattens probably felt they had earned

their evening?s entertainment. Posterity has enriched this speech,as nehru clearly

intended but today his quaint phrase ?tryst with density?resonate ominously,so

enduring have been the political and psychological scarst of partition.The souls of

the two new nation states immediately found utterance in brutal enmity.In punjab

armed vigligants groups.organised along religious lines and incited by local

politicians ,murdered countless people,abducing nad raping thousands of women

soon.india and pakistan were displacement was shot dead in januray 1946.by a

hindu extrement who believed that the father of the indian nation was soft on

muslims jinnah racked with apperantly burned with him.

What does the author imply about the future of pakistan?

A)Ot becomes a secular country.

B)It becomes unsecular.

C)It is unprosperous

D)It becomes a rogue state.

1) A

2) B

3) C

4) D

Correct Option
2
is:
Your Option is: 2
Result: Correct
Timetaken: 0.0 secs
from this passage author imply about feture of pakistan is
Explanation:
unsecular(connected with religion)

Select the word of phrase which best expresses the MEANING of the given word

2. STERILIZE

1) Freshman

2) potent

3) Mitigate

4) Disinfect

Correct Option is: 4


Your Option is: 4
Result: Correct
Timetaken: 0.0 secs
make free from bacteria option 4 didinfect same
Explanation:
meaning

Select the option that is most nearly OPPOSITE in meaning to the given word.
3.
POMPOUS(OPPOSITE)

1) benevolent

2) boastful

3) modest

4) oppulent
Correct Option is: 3
Your Option is: 1
Result: Wrong
Timetaken: 0.0 secs
meaning of pompous is self important so opposite is
Explanation:
modest

Sixty years ago,on the evening of august 14,1947 a few hours before britain?s

indian empire was formally divided into the nation states of india and pakistan,lord

lousie mountbatten and his wife,edwina sat down in the vicargal mansion in new

g\delhi to watch the latest bob hope movie.my favourite brunette.?Large parts of

the subcontitnent were descending into chaos.as the implications of partioning the

indian empire along religious lines became clear to the millions of hindus,muslims

twleve million people would be uprooted and as many as a million murdered but on

that night In mid aguest the bloodbath and the fuller consequence of hasty imperial

retreat ?still lay in the future,and the mountbattens probably felt they had earned

their evening?s entertainment. Posterity has enriched this speech,as nehru clearly

intended but today his quaint phrase ?tryst with density?resonate ominously,so
4.
enduring have been the political and psychological scarst of partition.The souls of

the two new nation states immediately found utterance in brutal enmity.In punjab

armed vigligants groups.organised along religious lines and incited by local

politicians ,murdered countless people,abducing nad raping thousands of women

soon.india and pakistan were displacement was shot dead in januray 1946.by a

hindu extrement who believed that the father of the indian nation was soft on

muslims jinnah racked with apperantly burned with him.

In the view of the author persists on talking the ?Bob Hope movie? in the

article,why? A)Because the movie was a classic of 1947. B)He thinks it caused the

partition of the sub contitnent. C)He uses it to show the apathy of the britishers

towards constituent. D)It was mountbatten?s favourite movie.


1) A

2) B

3) C

4) D

Correct Option is: 2


Your Option is: 3
Result: Wrong
Timetaken: 0.0 secs
Explanation: from this passage option 2 is suit for this

our bright and beautiful environment is being systematically destroyed under the
5.
pressure of a _______ population

1) proliferating

2) expanding

3) widening

4) enlarging

Correct Option is: 1


Your Option is: 4
Result: Wrong
Timetaken: 0.0 secs
Explanation: proliferating means increase rapidly

6. the leaves ________ yellow and dry

1) were
2) had

3) being

4) was

Correct Option
1
is:
Your Option is: 1
Result: Wrong
Timetaken: 0.0 secs
Subject 'leaves' is in plural form,so the correct answer is
Explanation:
option 1

the efforts put in by the top management to retain him went in ______ as he

7. decided to shift to competitor company

1) failure

2) futility

3) delight

4) vain

Correct Option is: 2


Your Option is: 3
Result: Wrong
Timetaken: 0.0 secs
Explanation: for this sentence option 2 is suit grammatically

8.
the appropriate atmostpheric conditions made it feasible for the astronomers to see
the stars and they could even distinguish the size.

1) and even distinguish the sizes.

2) and they were even distinguising the sizes.

3) and could even distinguish the sizes.

4) and even distinguising the sizes.

Correct Option
1
is:
Your Option is: 2
Result: Wrong
Timetaken: 0.0 secs
in the given sentence we speak about stars ,no need to say
Explanation:
"they" again,so option 1 is grammatically correct

Select the correct option that files the blank(s) to make the sentence meaningfully

complete

9. There are many textile producing mills in the market that compete with _______ to

gain the largest share of the market

1) Person

2) other mills

3) Contestants

4) Individual

Correct Option is: 2


Your Option is: 3
Result: Wrong
Timetaken: 0.0 secs
Explanation: option 2 is apt well because here we speak about mills

people working in high position in companies tend to shifting their work burden by
10.
delegating task to their subordinates.

1) tend for shifting their work

2) tend to shift their work

3) tend as to shifting their work

4) no improvement needed

Correct Option
2
is:
Your Option is: 4
Result: Wrong
Timetaken: 0.0 secs
shifting continuous tense should not come so option 2 is
Explanation:
corrrect

The note books used by_______ 'Evergreen' sociaty are made of recycled paper
11.

1) A

2) An

3) The

4) All
Correct Option is: 2
Your Option is: 1
Result: Wrong
Timetaken: 0.0 secs
Explanation: evergreen start with vowel so AN

Read the sentence to find out whether is any grammatical error in it.The

error, of any will be one part of the answer. Ignore the error of
12.
punctuation if any

A) Guilt and self Pleasure are (B) two most strong drives (c) of any human act

1) (A)

2) (B)

3) (C)

4) No error

Correct Option is: 2


Your Option is: 3
Result: Wrong
Timetaken: 0.0 secs
Explanation: most strong is grammatically wrong

13. tonight i am going to check that will do his homework correctly

1) raju must be doing his homework correctly

2) raju shall do his homework correctly

3) raju does his homework correctly

4) no change
Correct Option is: 3
Your Option is: 1
Result: Wrong
Timetaken: 0.0 secs
Explanation: nill

14. This hostel has a good____service.They park the cars safely.

1) bellboy

2) callboy

3) valet

4) doorman

Correct Option
3
is:
Your Option is: 4
Result: Wrong
Timetaken: 0.0 secs
valet is worker of parking area so option 3 is correct for
Explanation:
this one

Select the option that is most nearly OPPOSITE in meaning to the given word.
15.
PERENNIAL(OPPOSITE)

1) frequent

2) regular

3) lasting

4) rare
Correct Option
4
is:
Your Option is: 1
Result: Wrong
Timetaken: 0.0 secs
endless ,unending,never endind these are the meaning of
Explanation:
perennial so rare is give opposite meaning for this

16. ______ the shirt was washed twice, still he refused to wear it.

1) though

2) because

3) however

4) since

5) while

Correct Option is: 1


Your Option is: 5
Result: Wrong
Timetaken: 0.0 secs
Explanation: nil

Read the sentence to find out whether there is any grammatical error in it.

The error, of any will be one part of the answer. ignore the error of

punctuation, if any
17.
(A)in the film fraternity. There are many people. (B)Who likes to be in the time

light, but many just want (C) to stay away from any kind of complaint

1) (A)
2) (B)

3) (C)

4) No error

Correct Option is: 4


Your Option is: 4
Result: Correct
Timetaken: 0.0 secs
Explanation: this sentence is grammatically correct

18.
Sixty years ago,on the evening of august 14,1947 a few hours before britain?s

indian empire was formally divided into the nation states of india and pakistan,lord

lousie mountbatten and his wife,edwina sat down in the vicargal mansion in new

delhi to watch the latest bob hope movie.my favourite brunette.?Large parts of the

subcontitnent were descending into chaos.as the implications of partioning the

indian empire along religious lines became clear to the millions of hindus,muslims

twleve million people would be uprooted and as many as a million murdered but on

that night In mid aguest the bloodbath and the fuller consequence of hasty imperial

retreat ?still lay in the future,and the mountbattens probably felt they had earned

their evening?s entertainment. Posterity has enriched this speech,as nehru clearly

intended but today his quaint phrase ?tryst with density?resonate ominously,so

enduring have been the political and psychological scarst of partition.The souls of

the two new nation states immediately found utterance in brutal enmity.In punjab

armed vigligants groups.organised along religious lines and incited by local

politicians ,murdered countless people,abducing nad raping thousands of women

soon.india and pakistan were displacement was shot dead in januray 1946.by a

hindu extrement who believed that the father of the indian nation was soft on
muslims jinnah racked with apperantly burned with him.

Why was Gandhi assassinated?

A)Because he was favouring the muslims.

B)he was partial to the muslims.

C)He got killed in the violance after the partition.

D)None of these.

1) A

2) B

3) C

4) D

Correct Option
4
is:
Your Option is: 3
Result: Wrong
Timetaken: 0.0 secs
all the 3 options are not correct for this, so option 4 will
Explanation:
be correct one

Read the sentance to find out whether there is any grammatical error in

it.the error will be one part of the answer. Ignore the error of punctuation,

if any
19.
(A) Juthu beach in Mumbai was filled with (B) innumerable people who had

gathered there (C) to see discovered newly ancient temple

1) (A)

2) (B)
3) (c)

4) No error

Correct Option is: 3


Your Option is: 2
Result: Wrong
Timetaken: 0.0 secs
Explanation:

20.
sixty years ago on the evening of august 14,1947, a few hours before britain,s

indian empire was formally divided into the nation ststes of india and pakisthan.Lot

of lousis mountbatten and his wife .Edwife sat down in the vicarege\al mansion in

new delhi to watch the latest bob hope movie.My favourite Brunette.,Large parts of

the subcontinent were decending into chaos.as the implication of partitioning the

indian empire along religious lines became clear to the million of hindus.musilums

twelve million people would be upoaded and as many a million murdered but on

that night in mid-augest the bloodbath-and the fuller consequences of hasty

imperical retreat-still lay in the future.and the mountbatten probably felt they had

earned their evening;s entertainment.

while the mountbatten were sitting down to their bob hope movie india,s constitent

asembly was convening in new delhi.The moment demanded grandiloquence and

jawaharala nehru .Gandhi's closed displine and soon to be india,s first prime

minister provided it.'long years ago'we made a tryst with distroy.he said ;at the

stoke of the midnight hour.while the world sleeps.india will awakes to life and

freedom.Amoment comes,which but rarely in histroy when we stop out them old to

the new.when an age and when we step out friom the old to the new.when an age

ends and when the soul of a nation long suppressed finds utterence.
posterity has enshrinched this speech as nehru clearly intended but today his

quiants phrase tryst with density resonantes ominoulsy so enduring have been the

political and psychological scars of parttition.the souls of the two new nation states

immediately found utterenate in birtual enmity.In punjab,armed vigilnate groups

oraganized along religious lines and incited by local policians murdered countless

people,abducting and traping thousands of women soon.India and pakistan

countries people fighting war the first of three over the disputed territory of kashmir

.Gandhi reduced to despire by the seemingly endless cycle of retailatory of

kashmir .Gandhi reduced to displacement was shot dead in january 1948 by a hindu

extremenst who belived hat the father of the indian nation was too soft on

muslims.Jinnah racked with tuberculous and overwork died a few months later his

dream of a secular pakishan apparently burried with him.

in the view of author ,what dose nehru's phrase'tryst with density"symbolise

today?

A) celebration of indian independance.

B)An inspirational quote.

C)A reminder of Gandhi's assassination.

D)A symbol of the ills of the partion.

1) A

2) B

3) C

4) D

Correct Option
4
is:
Your Option is: 1
Result: Wrong
Timetaken: 0.0 secs
from this para author means symbolise is symbol of the
Explanation:
ills of a partion

21. JAUNTY(OPPOSITE)

1) youthful

2) ruddy

3) strong

4) unravelled

5) sedate

Correct Option is: 5


Your Option is: 3
Result: Wrong
Timetaken: 0.0 secs
Explanation: jaunty means cheerful so sedate is opposite for this one

as per the recent ethical rules laid down by national institute of health, diseases

22. which can not be treated _______ would qualify for treatment involving human

gene manipulation

1) dangerous

2) similar

3) alternatively

4) uncommon
Correct Option
3
is:
Your Option is: 1
Result: Wrong
Timetaken: 0.0 secs
option 3 is finish the sentence grammatically correct
Explanation:
meaning (treatment have to change)

________ being poor, kaveri still dress more appropriately than most of her group
23.
mets.

1) despite

2) although

3) since

4) however

Correct Option
1
is:
Your Option is: 4
Result: Wrong
Timetaken: 0.0 secs
compare given statement and options "despite being
Explanation:
poor"is grammatically correct

The election verdict was quite surprising as the ruling party was re-elected for the
24.
first time in fifty years.

1) judgement

2) decision

3) order
4) chaos

Correct Option is: 2


Your Option is: 1
Result: Wrong
Timetaken: 0.0 secs
Explanation: statement is as like decision so option 2 is correct

Instigate(opposite)
1.

1) Stimulate

2) Prompt

3) Ferment

4) Prevent

Correct Option
4
is:
Your Option is: 2
Result: Wrong
Timetaken: 0.0 secs
Direct meaning of 'instigate' is initiate,urge,fire up,etc. and
Explanation:
its antonym is 'prevent'

In following questions, a sentence is broken up into parts. One of the parts may

have an error. You are required to identify the part containing the error. If there?s
2.
no error ? choose ?None of the above? as your answer.

1) Meet Arindam Das


2) a resident of Raipur

3) his joy knew no bounds

4) when he got a call for a job opportunity in Mumbai.

5) None of the above

Correct Option is: 3


Your Option is: 4
Result: Wrong
Timetaken: 0.0 secs
Explanation: 'had' must be used instead of 'knew'

3. (A)guilt and self pleasure are (B) two more strong drivers (C) of any human acts.

1) (A)

2) (B)

3) (C)

4) no error

Correct Option
2
is:
Your Option is: 1
Result: Wrong
Timetaken: 0.0 secs
cannot use 'more' with 'strong'.instead of that, 'two
Explanation:
strongest drivers' must be used.

4.
Choose the word which is most nearly the SAME in meaning as the word given
DEGRADING

1) Demeaning

2) Lowering

3) Corrupting

4) Minimizing

Correct Option
1
is:
Your Option is: 2
Result: Wrong
Timetaken: 0.0 secs
Direct meaning of 'degrading' is
Explanation:
humiliating,demeaning,inglorious,etc.

5.
Read the passage below and answer the questions following it.

The Mumbai that we know about is a land of riches and dazzles. It is also the land

of teeming millions who struggle to make a living.

Mumbai is also the tinsel town, where people come with dreams and aspirations to

make it big. That is a familiar Mumbai. It is celebrated in speeches and advertised

on television and in magazines. It has the highest mass standard of living India has

ever known.

In the 1950?s this Mumbai worried about itself yet even its anxieties were products

of abundance. The title of a brilliant book was widely misinterpreted, and the

familiar Mumbai began to call itself the affluent society. There was introspection

about the Maine Lines and tail fins; there was discussion of the emotional suffering

taking place in the suburbs.

In all this, there was an implicit assumption that the basic grinding economic

problems had been solved in the metropolitan Mumbai. In this theory the city's
problems were no longer a matter of basic human needs, of food, shelter and

clothing. Now, they were seen as qualitative, the questions of learning to live

decently amid luxury.

While this discussion was carried on, there existed another Mumbai. In it dwelt

somewhere between 40,000,000 and 50,000,000 citizens of this land. They were

poor. They still are. To be sure, the other Mumbai is not impoverished in the same

sense as those poor nations where millions cling to hunger as a defence against

starvation. The city has escaped such extremes.

That does not change the fact the tens of millions of Mumbaites, at this very

moment, maimed in body and spirit, exist at levels beneath those necessary for

human decency. If these people are not starving, they are hungry and sometimes

fat with hunger, for that is what cheap foods do. They are without adequate housing

and education and medical care.

The clause, 'where millions cling to hunger as a defence against starvation' means

1) That the people of the poor nations foolishly accept hunger as a barrier against
starvation

2) That the people of the poor nations endure both hunger as a barrier against
starvation

3) That the people of the poor nations cannot escape either hunger or starvation

4) That the people of the poor nations can escape either hunger or starvation

Correct Option
3
is:
Your Option is: 4
Result: Wrong
Timetaken: 0.0 secs
From the fifth paragraph,it is clear that correct answer is
Explanation:
option 3
select the word phrase which best expresses the meaning of the given word

6. CONTAGIOUS

1) Communicable

2) Harmfull

3) Preventive

4) Survival

Correct Option is: 1


Your Option is: 1
Result: Correct
Timetaken: 0.0 secs
Explanation: Direct meaning of 'contagious' is 'communicable'.

7. "We need to call this ____________" Monika demanded.

1) out

2) at

3) off

4) away

Correct Option is: 1


Your Option is: 3
Result: Wrong
Timetaken: 0.0 secs
Explanation: 'this' specify one out of others.So correct answer is
option 1

8.
Read the passage below and answer the questions following it.

The Mumbai that we know about is a land of riches and dazzles. It is also the land

of teeming millions who struggle to make a living.

Mumbai is also the tinsel town, where people come with dreams and aspirations to

make it big. That is a familiar Mumbai. It is celebrated in speeches and advertised

on television and in magazines. It has the highest mass standard of living India has

ever known.

In the 1950?s this Mumbai worried about itself yet even its anxieties were products

of abundance. The title of a brilliant book was widely misinterpreted, and the

familiar Mumbai began to call itself the affluent society. There was introspection

about the Maine Lines and tail fins; there was discussion of the emotional suffering

taking place in the suburbs.

In all this, there was an implicit assumption that the basic grinding economic

problems had been solved in the metropolitan Mumbai. In this theory the city's

problems were no longer a matter of basic human needs, of food, shelter and

clothing. Now, they were seen as qualitative, the questions of learning to live

decently amid luxury.

While this discussion was carried on, there existed another Mumbai. In it dwelt

somewhere between 40,000,000 and 50,000,000 citizens of this land. They were

poor. They still are. To be sure, the other Mumbai is not impoverished in the same

sense as those poor nations where millions cling to hunger as a defence against

starvation. The city has escaped such extremes.

That does not change the fact the tens of millions of Mumbaites, at this very

moment, maimed in body and spirit, exist at levels beneath those necessary for

human decency. If these people are not starving, they are hungry and sometimes

fat with hunger, for that is what cheap foods do. They are without adequate housing

and education and medical care.


The phrase,"basic grinding economic problem" means

1) That the basic needs of the people have been solved

2) That the people suffer terriblely due to the absence of adequate housing,
education and medical care.

3) That the peoples are without good housing, education and shelter facilites.

4) That the people have become deseperate due to widespread poverty.

Correct
2
Option is:
Your Option
1
is:
Result: Wrong
Timetaken: 0.0 secs
From the fourth paragraph of this passage,it is clear that the
phrase 'basic grinding economic problem' means that the
Explanation:
people suffer terribly due to the absence of adequate
housing,education and medical care.

in this question a part of the sentence is italicised alternatives to the italicized part

are given bellow which may Improve the construction of the select correct one.
9.
What have we got for dinner?

1) We got for the dinner

2) We got for a dinner

3) We gotten for dinner

4) No change
Correct Option is: 1
Your Option is: 2
Result: Wrong
Timetaken: 0.0 secs
as dinner is used specifically, it must be used as 'the
Explanation:
dinner'

10.
Read the passage below and answer the questions following it.

The Mumbai that we know about is a land of riches and dazzles. It is also the land

of teeming millions who struggle to make a living.

Mumbai is also the tinsel town, where people come with dreams and aspirations to

make it big. That is a familiar Mumbai. It is celebrated in speeches and advertised

on television and in magazines. It has the highest mass standard of living India has

ever known.

In the 1950?s this Mumbai worried about itself yet even its anxieties were products

of abundance. The title of a brilliant book was widely misinterpreted, and the

familiar Mumbai began to call itself the affluent society. There was introspection

about the Maine Lines and tail fins; there was discussion of the emotional suffering

taking place in the suburbs.

In all this, there was an implicit assumption that the basic grinding economic

problems had been solved in the metropolitan Mumbai. In this theory the city?s

problems were no longer a matter of basic human needs, of food, shelter and

clothing. Now, they were seen as qualitative, the questions of learning to live

decently amid luxury.

While this discussion was carried on, there existed another Mumbai. In it dwelt

somewhere between 40,000,000 and 50,000,000 citizens of this land. They were

poor. They still are. To be sure, the other Mumbai is not impoverished in the same

sense as those poor nations where millions cling to hunger as a defence against

starvation. The city has escaped such extremes.


That does not change the fact the tens of millions of Mumbaites, at this very

moment, maimed in body and spirit, exist at levels beneath those necessary for

human decency. If these people are not starving, they are hungry and sometimes

fat with hunger, for that is what cheap foods do. They are without adequate housing

and education and medical care.

The citizens of the other Mumbai

1) Are hungry because they go without any food

2) Are hungry because they do not get a rich diet

3) Are hungry because they get only cheap food

4) Are fat yet hungry because the food that they eat is poor in nutritive value.

Correct Option
4
is:
Your Option is: 3
Result: Wrong
Timetaken: 0.0 secs
From the last two sentences of this passage,it is clear that
Explanation:
option 4 is the correct answer.

The effects put in by the top management to retain him went in____as he decided

11. to shift to a competitor company

1) Failure

2) Futility

3) delight

4) Vain
Correct Option
3
is:
Your Option is: 2
Result: Wrong
Timetaken: 0.0 secs
the words failure,futility and vain means same
Explanation:
meaning,which means failure,useless,etc.

The room is more expensive because it has spectacular view __beach.


12.

1) OF

2) FROM

3) FOR

4) TO

Correct Option
1
is:
Your Option is: 1
Result: Correct
Timetaken: 0.0 secs
The given sentence is about beach's view,So the answer
Explanation:
is option 1

Select the correct option that fills the blank(s)to make the sentence meaningfully

complete.
13.
The note books used by __'evergreen' society are made of recycle paper.

1) A
2) AN

3) THE

4) ALL

Correct Option
3
is:
Your Option is: 4
Result: Wrong
Timetaken: 0.0 secs
As it is definite about a particular garden,we use definite
Explanation:
article.

Choose the word which is most nearly the SAME in meaning as the word given
14.
ARDUOUS

1) Hazardous

2) Difficult

3) Different

4) Plesurable

Correct Option is: 2


Your Option is: 1
Result: Wrong
Timetaken: 0.0 secs
Direct meaning of 'ARDUOUS' is
Explanation:
difficult,hard,heavy,etc.

15.
Read the sentence to find whether there is any grammatical error in it. The error, if
any will be in one part of the sentence.The letter of that part is the answer.Ignore

the error of punctuation, if any.

(A)Fishermen is (B)spotted catching fish on (C)the bank of the river

1) (A)

2) (B)

3) (C)

4) No error

Correct Option
1
is:
Your Option is: 1
Result: Correct
Timetaken: 0.0 secs
cannot use 'fishermen'.Either it should be 'fisherman is' or
Explanation:
'fishermen are'

Read the sentence to find out whether there is any grammatical error in it.The

error, if any, will be in one part of the sentence.The letter of that part is the
16.
answer.Ignore the error of punctuation, if any.

(A)India is a vast country (B)that offers immeasurable experiences (C)to all visitors.

1) (A)

2) (B)

3) (C)

4) No error

Correct Option is: 2


Your Option is: 2
Result: Correct
Timetaken: 0.0 secs
Explanation: cannot use 'experiences',we can use only 'experience'

read the sentence find out whether there grammatical error.

17. (a)haris likes to play cricket (b)and riding bicycle besides (c)playing video games

1) a

2) b

3) c

4) no error

Correct
2
Option is:
Your Option
3
is:
Result: Wrong
Timetaken: 0.0 secs
'besides' must be used infront of riding and 'and' must be
used at the end..So the correct sentence is,'Haris likes to
Explanation:
play cricket besides riding bicycle and playing video
games'.

18.
each passage consists of 6 sentences.the first and 6th are given in beginning. The

middle four have been removed and jumbled up .these are labeled P,Q,R,S ,select

the proper order for the four sentence .

S1:sameer has not been coming to office since last week

S6:since his plaster will be removed after 30 days

P:doctor have advised him complete bed rest for 15 days<


Q:he had six fractures and was rushed to the hospital

R:he met with serious accident a few days ago

S:I think he would be on leave for this whole month

1) PQRS

2) PSQR

3) SPRQ

4) RQPS

Correct Option
4
is:
Your Option is: 2
Result: Wrong
Timetaken: 0.0 secs
Reason for the first sentence is given in R which is
Explanation: followed by Q.Effect of Q is discussed in P.S and S6 is a
pair of sentences.

read the sentence find out whether there grammatical error.

(a) As maria returned to home(b)she found the hall to be empty as(c)everyone


19.
were hiding in the kitchen (d)no error

1) a

2) b

3) c

4) d
Correct Option is: 1
Your Option is: 3
Result: Wrong
Timetaken: 0.0 secs
Explanation: instead of 'returned','return' must be used.

read the sentence find out whether there grammatical error.

(a)We've been saying it separately up till (b)now.but we thought it would be (c)


20.
better if we spoke in one voice.

1) A

2) b

3) c

4) no error

Correct Option is: 3


Your Option is: 2
Result: Wrong
Timetaken: 0.0 secs
Explanation: 'spoke'must be replaced with 'speak'

in this question a part of the sentence is italicised alternatives to the italicized part

are given bellow which may Improve the construction of the select correct one.

21. The election verdict was quite supporting as the ruling party was re-elected for the

first time in fifty years.,


1) Judgement

2) Decision

3) Order

4) Chaos

Correct Option
1
is:
Your Option is: 1
Result: Correct
Timetaken: 0.0 secs
Direct meaning of 'verdict' is
Explanation:
'judgement','decision','result',etc.

futile (opposite)
22.

1) Barren

2) Useless

3) Alive

4) Productive

Correct Option
4
is:
Your Option is: 1
Result: Wrong
Timetaken: 0.0 secs
Meaning of 'futile' is 'unproductive' and its antonym is
Explanation:
'productive'
.read the sentence find out whether there grammatical error.

23. (a) A tie is a very important (b)part of formal dressing (c)for every man

1) a

2) b

3) c

4) no error

Correct Option is: 4


Your Option is: 2
Result: Wrong
Timetaken: 0.0 secs
Explanation: No error in the given sentence

24.
Read the passage below and answer the questions following it.

The Mumbai that we know about is a land of riches and dazzles. It is also the land

of teeming millions who struggle to make a living.

Mumbai is also the tinsel town, where people come with dreams and aspirations to

make it big. That is a familiar Mumbai. It is celebrated in speeches and advertised

on television and in magazines. It has the highest mass standard of living India has

ever known.

In the 1950's this Mumbai worried about itself yet even its anxieties were products

of abundance. The title of a brilliant book was widely misinterpreted, and the

familiar Mumbai began to call itself the affluent society. There was introspection

about the Maine Lines and tail fins; there was discussion of the emotional suffering

taking place in the suburbs.

In all this, there was an implicit assumption that the basic grinding economic
problems had been solved in the metropolitan Mumbai. In this theory the city's

problems were no longer a matter of basic human needs, of food, shelter and

clothing. Now, they were seen as qualitative, the questions of learning to live

decently amid luxury.

While this discussion was carried on, there existed another Mumbai. In it dwelt

somewhere between 40,000,000 and 50,000,000 citizens of this land. They were

poor. They still are. To be sure, the other Mumbai is not impoverished in the same

sense as those poor nations where millions cling to hunger as a defence against

starvation. The city has escaped such extremes.

That does not change the fact the tens of millions of Mumbaites, at this very

moment, maimed in body and spirit, exist at levels beneath those necessary for

human decency. If these people are not starving, they are hungry and sometimes

fat with hunger, for that is what cheap foods do. They are without adequate housing

and education and medical care.

The worries of this Mumbai were:

1) Were a product of the second world war

2) Were a product of world- wide economic crisis.

3) Were the result of its wealth

4) Were products of cut ? throat competition

Correct Option
2
is:
Your Option is: 1
Result: Wrong
Timetaken: 0.0 secs
Explanation: From the third paragraph of this passage,it is very clear that
the worries of mumbai were a product of world-wide
economic crisis.

in this question a part of the sentence is italicised alternatives to the italicized part

are given bellow which may Improve the construction of the select correct one.

25. Prime minister manmohan sihgh received a warm welcome from the political

leaders as he reached Pakistan.

1) Rude

2) Indifferent

3) Angry

4) Friendly

Correct Option is: 4


Your Option is: 2
Result: Wrong
Timetaken: 0.0 secs
Explanation: Direct meaning of 'warm' is 'friendly''kind','pleasant',etc.

Select the option that is most nearly OPPOSITE in meaning to the word or phrase

1. given in bold.

There is a plethora of flora and fauna in the indian subcontinent.

1) destruction

2) dearth

3) uniformity

4) abundance
Correct Option is: 2
Your Option is: 4
Result: Wrong
Timetaken: 0.0 secs
Explanation: antonym of plethora is dearth

Directions:

The following questions, consist of two words each that have a certain relationship

2. to each other, followed by four lettered pairs of words. Select the lettered pair that

has the same relationship as the original pair of words.

Expend : save

1) Exhort : Encourage

2) Formant : Rebellion

3) Defect :good

4) Encroachment : Occupy

Correct Option is: 3


Your Option is: 2
Result: Wrong
Timetaken: 0.0 secs
Explanation: antonym for expend is save as like option 3

3. Which of the following is an appropriate synonym for the word allude?

1) Argued

2) Refer Indirectly

3) Fought
4) Shouted

Correct Option is: 2


Your Option is: 3
Result: Wrong
Timetaken: 0.0 secs
Explanation: suggest is meaning near by option is 2

Select the option that is most nearly OPPOSITE in meaning to the word.
4.
INVINCIBLE (OPPOSITE)

1) Voluble

2) Victorious

3) Visible

4) Vulnerable

Correct Option is: 4


Your Option is: 2
Result: Wrong
Timetaken: 0.0 secs
Explanation: antonym for invincible is vulnerable and defenceless

Read the sentence to find out whether there is any grammatical error in it. The

error, if any, will be in one part of the sentence.The letter of that part is the

5. answer.Ignore the error of punctuation,if any.

(A)We have been saying it separately up till (B)now,but we thought it would be

(C)better if we spoke in one voice.

1) A
2) B

3) C

4) No error

Correct Option is: 3


Your Option is: 3
Result: Correct
Timetaken: 0.0 secs
Explanation: if we "speak" in one voice

Which of the phrases (A), (B), (C) and (D) given below each sentence should

replace the phrase printed in bold in the sentence to make it grammatically

correct ? If the sentence is correct as it is given and no correction is required, mark


6.
(E) as the answer.

We have hired an advertising agency to prepare a campaign to encourage people

votes.

1) Votes by people

2) People to vote

3) Voting for people

4) People from voting

5) No correction required

Correct Option
2
is:
Your Option is: 4
Result: Wrong
Timetaken: 0.0 secs
we are not hired ad for votes we do for make people
Explanation:
should vote so people to vote only correct

Passage

For its new fridge,Whirlpool Corp. spent months investing a shelf with microscope

etching so it can hold a can of spilled soda. The technology is just one weapon

against a dirty kitchen secret.Most Americans clean their fridges only once or twice

a year.

Now appliances makers like Whirlpool,Viking Range Corp. and Sub-Zero Inc, are

tackling the messy fridge problem with a host of new features including souped-up

shelves,bacteria-killing devices and better lighting.General Electric Co., for

example,says it is rolling out new refrigerators in May with 10 lighting sources

inside instead of ots usual three-so proof that might be forgotten in a corner and

spoil will be easier to spot.The new GE models sell for $1,599 or $1,799 for

stainless steel.
7.
In Whirlpool?s 2005 refrigerator habits survey of 2,571 consumers,33% said they

don?t spend any time cleaning the refrigerator before grocery shoping.In order to

make room for items juxt purchased,27% reported shoving everything in and not

worrying about organization.

Whirlpool hopes that increasing the amount of storage space might help.The

company?s new shelves,to be released later this year,are 25% roomier than

previous models.But having more room won?t necessarily limit clutter.People often

don?t store things properly anyway.While the door shelves seem to be a perfect fit

for a carton of milk,suz-Zero says the area is the worst place to store dairy products

because it?s the warmest part of the fridge.

What percentage of consumers said they don?t spend time to clean the

refrigerator?

1) 33%
2) 27%

3) 35%

4) 60%

Correct Option is: 1


Your Option is: 1
Result: Correct
Timetaken: 0.0 secs
Explanation: from 3rd para we can say option 1 is correct

Read the sentence to find out whether there is any grammatical error in it. The

error, if any, will be in one part of the sentence.The letter of that part is the
8.
answer.Ignore the error of punctuation,if any.

(A)Guilt and self pleasure are (B)two most strong drivers (C)of any human act.

1) A

2) B

3) C

4) No error

Correct Option is: 2


Your Option is: 4
Result: Wrong
Timetaken: 0.0 secs
Explanation: "most strong" never come

9. "We need to call this ___________." Monika demanded.


1) out

2) at

3) off

4) away

Correct Option
3
is:
Your Option is: 1
Result: Wrong
Timetaken: 0.0 secs
when we compare given statement and preposition option
Explanation:
3 is correct

Select the option that is most nearly OPPOSITE in meaning to the word.
10.
GHOULISH(OPPOSITE)

1) Gruesome

2) Pleasant

3) Peevish

4) Garrulous

Correct Option
2
is:
Your Option is: 4
Result: Wrong
Timetaken: 0.0 secs
opposite word for ghoulish is good,normal so option 2 is
Explanation:
nearly mean that
Read the sentence to find out whether there is any grammatical error in it. The

error, if any, will be in one part of the sentence.The letter of that part is the

11. answer.Ignore the error of punctuation,if any.

(A)Western culture have (B)influenced many people in (C)India in a very powerful

way.

1) A

2) B

3) C

4) No error

Correct Option is: 1


Your Option is: 2
Result: Wrong
Timetaken: 0.0 secs
Explanation: has incident of have

12. Which of the following is an appropriate synonym for the word Debauch ?

1) Demoralize

2) Encourage

3) Cultivate

Correct Option
1
is:
Your Option is: 1
Result: Correct
Timetaken: 0.0 secs
destroy is exact meaning of debauch here near by
Explanation:
meaning is demoralize

Read the sentence to find out whether there is any grammatical error in it. The

error, if any, will be in one part of the sentence.The letter of that part is the

13. answer.Ignore the error of punctuation,if any.

(A)Big brands like Sony,Samsung and (B)Nokia have been launched many phones

having latest (C)features like Facebook application,Orkut tool and much more.

1) A

2) B

3) C

4) No error

Correct Option is: 2


Your Option is: 2
Result: Correct
Timetaken: 0.0 secs
Explanation: have launched

Directions:

Find which of the phrases (A), (B), (C) and (D) given below should replace the

phrase given in bold in the following sentence to make the sentence grammatically
14.
correct. If the sentence is correct as it is and ?No correction is required?, mark (E)

as the answer.

We must treat any statement as a rumour until they are confirmed with proof.

1) till they are confirmed


2) until they are confirming

3) until it is confirmed

4) until it is confirming

5) No correction required

Correct Option is: 3


Your Option is: 3
Result: Correct
Timetaken: 0.0 secs
Explanation: here we speak about statement so we use "it is"

Directions:

The following questions, consist of two words each that have a certain relationship

15. to each other, followed by four lettered pairs of words. Select the lettered pair that

has the same relationship as the original pair of words.

Plants : Coal

1) Crops : Manure

2) Animals : Oil

3) Cow : Milk

4) Fire : Smoke

Correct Option is: 2


Your Option is: 3
Result: Wrong
Timetaken: 0.0 secs
Explanation: after degradation animal gives oil
Read the sentence to find out whether there is any grammatical error in it. The

error, if any, will be in one part of the sentence.The letter of that part is the
16.
answer.Ignore the error of punctuation,if any.

(A)India is a vast country (B)that offers immeasurable experience (C)to all visitors.

1) A

2) B

3) C

4) No error

Correct Option is: 4


Your Option is: 2
Result: Wrong
Timetaken: 0.0 secs
Explanation: sentence formation is correct so no error in this one

Select the word or phrase which best expresses the meaning of the given word.
17.
CONCEITED

1) Arrogant

2) False

3) Deceive

4) Misconception

Correct Option is: 1


Your Option is: 1
Result: Correct
Timetaken: 0.0 secs
Explanation: conceited means egotistic its near to arrogant

Select the correct option that fills the blank(s) to make the sentence meaningfully

complete.
18.
The labour union decided to go on strike since the management was adamant and

did not agree to their terms,The discussion had reached a/an ________.

1) climax

2) obstacle

3) impetus

4) impasse

Correct Option is: 3


Your Option is: 3
Result: Correct
Timetaken: 0.0 secs
Explanation: impetus means encouragement so option 3is apt for this

Read the sentence to find out whether there is any grammatical error in it. The

error, if any, will be in one part of the sentence.The letter of that part is the

19. answer.Ignore the error of punctuation,if any.

(A)Jeet is a very nice boy (B)and he always listen carefully (C)to what his parents

have to say.(D)No error

1) A

2) B

3) C
4) D

Correct Option is: 3


Your Option is: 4
Result: Wrong
Timetaken: 0.0 secs
Explanation: "to what his parents say" its correct form

Select the word or phrase which best expresses the meaning of the given word.
20.
ARID

1) Dry

2) Separated

3) Arrogant

4) Superfluous

Correct Option is: 1


Your Option is: 1
Result: Correct
Timetaken: 0.0 secs
Explanation: arid means dry

Read the sentence to find out whether there is any grammatical error in it. The

error, if any, will be in one part of the sentence.The letter of that part is the
21.
answer.Ignore the error of punctuation,if any.

(A)Fishermen is (B)spotted catching fish on (C)the bank of the river.

1) A

2) B
3) C

4) No error

Correct Option is: 1


Your Option is: 2
Result: Wrong
Timetaken: 0.0 secs
Explanation: men is plural so is should not come

In the question each passage consists of six sentences.The first and the sixth

sentences are given in the beginning.The middle four sentences have been removed

and jumbled up.These are labeled P,Q,R and S.Select the proper order for the four

sentences.

S1: Rahul has been trying to lose weight.

22. S6:I think it is just a lame excuse for his laziness.

P :As regular morning walk keeps our body fit and healthy.

Q :The trainer has suggested him to start with regular morning walk.

R :He has not yet started his daily walk.

S :He says that because of late night work,it is hard for him to get up early.

1) PRSQ

2) QPRS

3) PQRS

4) SQRP
Correct Option is: 2
Your Option is: 3
Result: Wrong
Timetaken: 0.0 secs
Explanation: QPRS is exact logical order

Which of the phrases (A), (B), (C) and (D) given below each sentence should

replace the phrase printed in bold in the sentence to make it grammatically

correct ? If the sentence is correct as it is given and no correction is required, mark


23.
(E) as the answer.

Since the deadline has been changed from next week to this Thursday you

should give this work priority.

1) Priority this work

2) Prioritised this work

3) Not give priority this work

4) Be given this work priority

5) No correction required

Correct Option is: 5


Your Option is: 2
Result: Wrong
Timetaken: 0.0 secs
statement is correct grammatically so no correction
Explanation:
needed

24. Which of the folloeing sentence is grammatically correct?

1) Youns has played twenty matches last year.

2) Youns have played twenty matches last year.


3) Youns has been played twenty matches last year.

4) Youns have been played twenty matches last year.

5) Youns had been played twenty matches last year.

Correct Option
1
is:
Your Option is: 3
Result: Wrong
Timetaken: 0.0 secs
option one is grammatically(has played-plural (matches))
Explanation:
correct

Select the correct option that fills the blank(s) to make the sentence meaningfully

complete.
25.
The monk wanders here and there in search of silence and peace.His lives a

____________ life.

1) nomadic

2) boring

3) religious

4) busy

Correct Option is: 1


Your Option is: 3
Result: Wrong
Timetaken: 0.0 secs
Explanation: living a life of nomad so option 1 is suit well

1.
Arrange the fragment ts A,B,C,D,E and F in order to form a meaningful sentence

A- disappointed if
B- Not fulfilled

C- do not be

D- or

E- promises are

F- friends let you down

1) CFDAEB

2) CABDEF

3) CAFDBE

4) CAFDEB

5) CBAEDF

Correct Option
4
is:
Your Option is: 2
Result: Wrong
Timetaken: 0.0 secs
C and A is a pair of sentences and A is followed by F,which
Explanation: is followed by D and E and B is a pair of sentences.So
answer option is 4.

In the question a part of the sentence is CAPITALIZED . Alternatives to the

CAPITALIZED are given which may improve the sentence.Select the correct

2. alternative.

What have we got for dinner?

1) we got for the dinner

2) we got for a dinner


3) we gotten for dinner

4) No change

Correct Option
1
is:
Your Option is: 4
Result: Wrong
Timetaken: 0.0 secs
When compared with the sentennce and the options,it is
Explanation:
clear that the answer is option 1

Read the sentence to find out whether there is any grammatical error in it.The

error, if any,will be in one part of the sentence. The letter of that part is the

3. answer.Ignore the error of puntuation,if any.

(A)Yauhan do not understand (B)the importance of money as (C)he never had to

earn himself.

1) A

2) B

3) C

4) No error

Correct Option is: 1


Your Option is: 1
Result: Correct
Timetaken: 0.0 secs
Explanation: 'Do not' must be replaced with 'did not'
In the question a part of the sentence is CAPITALIZED.Alternatives to the

CAPITALIZED parts are given which may improve the construction of the
4.
sentence.Select the correct alternative

The quality and texture of jeans produced in CALIFORNIA would be very good.

1) produced in california are very good

2) produced into california is very good

3) produced in california is very good

4) No improvement needed

Correct Option
3
is:
Your Option is: 2
Result: Wrong
Timetaken: 0.0 secs
when compared with the sentence and the options,it is
Explanation:
clear that the answer is option 3

5.
Passage:

Fasting is an act of homage to the majesty of appetite. So i think we should arrange

to give up our pleasures regularly-our food, our friends, our lovers-in order to

preserve their intensity, and the moment of coming back to them. For this the

moment that renews and refreshers both oneself and the thing one loves.

Sailors and travelers enjoyed this once, and so did hunters, I suppose.

Part of weariness of modern life may be that we live too much on top of each other,

and are entertained and fed too regularly.

Once we were separated by hunger both from our food and families, and then we

learned to value both.

The men went off hunting, and the dogs went with them, the women and children
waved goodbye. The caves was empty of men for days on end; nobody ate, or knew

what to do.

The women crouched by the fire, the wet smoke in their eyes; the children wailed;

everybody was hungry. The one night there were shouts and the barking of dogs

from the hills, and the men came back loaded with meat.

This was the great reunion, and everybody gorged themselves silly, and appetite

came into its own; the long-awaited meal became a feast to remember and an

almost sacred celebration of life.

Now we were we go off to the office and come home in the evenings to cheap

chicken and frozen peas. Very nice, but too much of it, too easy and regular, served

up without effort or wanting.

We eat, we are lucky, our faces are shining with fat, but we don't know the pleasure

of being hungry any more.

Too much of anything-too much music, entertainment, happy snacks, or time spent

with one's friends-creates a kind of importance of living by which one can no longer

hear, or taste, or see, or love, or remember.

Life is short and precious, and appetite is short life we should respect the divinity of

appetite, and keep it eager and not too much blunted.

Select the correct answer option based on the passage.

What are the benefits of fasting?

1) It is an act against the drawbacks of appetite.

2) It brings joy in eating, and one learns to appreciate food.

3) It is the method to understand how civilization envolved.

4) It is a punishment for the greedy unkind.


Correct Option
2
is:
Your Option is: 4
Result: Wrong
Timetaken: 0.0 secs
From the passage,it gives that it brings joy in eating and
Explanation:
one learns to appreciate food.

Improve the sentence by selecting the correct alternatives to the CAPITALIZED part

of the sentence.
6.
The election VERDICT was quit surprising as the ruling part was re-elected for the

first time in fifty years.

1) Judgment

2) Decision

3) Order

4) Chaos

Correct Option is: 2


Your Option is: 2
Result: Correct
Timetaken: 0.0 secs
Explanation: Direct meaning of 'VERDICT' is Decision

Select the option that is most nearly OPPOSITE in meaning to the given word.

7. TORTURE (Opposite)

1) relief

2) Generous
3) Polite

4) Aid

Correct Option is: 1


Your Option is: 3
Result: Wrong
Timetaken: 0.0 secs
Explanation: Direct opposite of 'torture' is 'relief'.

8.
Passage:

Fasting is an act of homage to the majesty of appetite. So i think we should arrange

to give up our pleasures regularly-our food, our friends, our lovers-in order to

preserve their intensity, and the moment of coming back to them. For this the

moment that renews and refreshers both oneself and the thing one love3s.

Sailors and travelers enjoyed this once, and so did hunters, I suppose.

Part of weariness of modern life may be that we live too much on top of each other,

and are entertained and fed too regularly.

Once we were separated by hunger both from our food and families, and then we

learned to value both.

The men went off hunting, and the dogs went with them, the women and children

waved goodbye. The caves was empty of men for days on end; nobody ate, or knew

what to do.

The women crouched by the fire, the wet smoke in their eyes; the children wailed;

everybody was hungry. The one night there were shouts and the barking of dogs

from the hills, and the men came back loaded with meat.

This was the great reunion, and everybody gorged themselves silly, and appetite

came into its own; the long-awaited meal became a feast to remember and an
almost sacred celebration of life.

Now we were we go off to the office and come home in the evenings to cheap

chicken and frozen peas. Very nice, but too much of it, too easy and regular, served

up without effort or wanting.

We eat, we are lucky, our faces are shining with fat, but we don't know the pleasure

of being hungry any more.

Too much of anything-too much music, entertainment, happy snacks, or time spent

with one's friends-creates a kind of importance of living by which one can no longer

hear, or taste, or see, or love, or remember.

Life is short and precious, and appetite is short life we should respect the divinity of

appetite, and keep it eager and not too much blunted.

Select the correct answer option based on the passage.

'The longest-awaited meal became a feast to remember and an almost sacred

celebration of life', what does this line imply?

1) After so many days of being hungry, the cave men and womens felt alive once
again after eating the food.

2) People respected and were thankful for getting food after days of being hungry
and also or being united with their loved ones.

3) Cave men and women ate and celebrated together with the entire community
making the feast really enjoyable.

4) Cave men and women enjoyed themselves in the feast and performed a
ceremony to thank the gods for their safe return back home.

Correct Option
4
is:
Your Option is: 2
Result: Wrong
Timetaken: 0.0 secs
Explanation: according to third para this line meaning is suit with
option 4 perfectly

9.
Passage:

Fasting is an act of homage to the majesty of appetite. So i think we should arrange

to give up our pleasures regularly-our food, our friends, our lovers-in order to

preserve their intensity, and the moment of coming back to them. For this the

moment that renews and refreshers both oneself and the thing one loves.

Sailors and travelers enjoyed this once, and so did hunters, I suppose.

Part of weariness of modern life may be that we live too much on top of each other,

and are entertained and fed too regularly.

Once we were separated by hunger both from our food and families, and then we

learned to value both.

The men went off hunting, and the dogs went with them, the women and children

waved goodbye. The caves was empty of men for days on end; nobody ate, or knew

what to do.

The women crouched by the fire, the wet smoke in their eyes; the children wailed;

everybody was hungry. The one night there were shouts and the barking of dogs

from the hills, and the men came back loaded with meat.

This was the great reunion, and everybody gorged themselves silly, and appetite

came into its own; the long-awaited meal became a feast to remember and an

almost sacred celebration of life.

Now we were we go off to the office and come home in the evenings to cheap

chicken and frozen peas. Very nice, but too much of it, too easy and regular, served

up without effort or wanting.

We eat, we are lucky, our faces are shining with fat, but we don't know the pleasure

of being hungry any more.

Too much of anything-too much music, entertainment, happy snacks, or time spent

with one's friends-creates a kind of importance of living by which one can no longer

hear, or taste, or see, or love, or remember.


Life is short and precious, and appetite is short life we should respect the divinity of

appetite, and keep it eager and not too much blunted.

Select the correct answer option based on the passage.

What is the author's main argument in the passage?

1) The oldest times, when the roles of men and women were clearly divided,
werefar enjoyable than the present time.

2) There is not enough effort required anymore to obtain food and hence the
pleasure derived is not the same.

3) People who don't have enough to eat enjoy life much more than those who have
painful.

4) We should deny ourselves pleasure once in a while in order to whet our desires
and feel more alive.

Correct Option
4
is:
Your Option is: 4
Result: Correct
Timetaken: 0.0 secs
this passage based on ourselves pleasure once in whilein
Explanation:
order about live so option 4 is correct

Improve the sentence by selecting the correct alternative to the CAPITALIZED part

of the sentence.
10.
As per the weather PREDICTION, it will rain heavily for the next one week.

1) Observation

2) Report

3) news
4) forecast

Correct Option is: 4


Your Option is: 2
Result: Wrong
Timetaken: 0.0 secs
Explanation: Direct meaning of the word prediction is 'forecast'.

Select the correct option that fills the blanks to make the sentence meaningfully

complete.

11. Months have passed and no action _________ been taken for a dowry harassment

case filled against the in - laws

1) has

2) have

3) had

4) are

Correct Option
3
is:
Your Option is: 1
Result: Wrong
Timetaken: 0.0 secs
when compared to first half and second half of the given
Explanation:
sentence,correct answer is option 3

12.
Fill in the blanks to make the sentence meaningfully complete
Aspirations of minorities cannot be kept in check ___________ the gun.

1) with

2) from

3) by

4) through

5) under

Correct Option
1
is:
Your Option is: 5
Result: Wrong
Timetaken: 0.0 secs
when we compared with the statements and prepositions,
Explanation:
option 1 is the correct answer.

13. ENFORCE(OPPOSITE)

1) Administer

2) Accompolish

3) Abandon

4) Exert

Correct Option is: 3


Your Option is: 4
Result: Wrong
Timetaken: 0.0 secs
Explanation: nil

14.
Passage:

Fasting is an act of homage to the majesty of appetite. So i think we should arrange

to give up our pleasures regularly-our food, our friends, our lovers-in order to

preserve their intensity, and the moment of coming back to them. For this the

moment that renews and refreshers both oneself and the thing one love3s.

Sailors and travelers enjoyed this once, and so did hunters, I suppose.

Part of weariness of modern life may be that we live too much on top of each other,

and are entertained and fed too regularly.

Once we were separated by hunger both from our food and families, and then we

learned to value both.

The men went off hunting, and the dogs went with them, the women and children

waved goodbye. The caves was empty of men for days on end; nobody ate, or knew

what to do.

The women crouched by the fire, the wet smoke in their eyes; the children wailed;

everybody was hungry. The one night there were shouts and the barking of dogs

from the hills, and the men came back loaded with meat.

This was the great reunion, and everybody gorged themselves silly, and appetite

came into its own; the long-awaited meal became a feast to remember and an

almost sacred celebration of life.

Now we were we go off to the office and come home in the evenings to cheap

chicken and frozen peas. Very nice, but too much of it, too easy and regular, served

up without effort or wanting.

We eat, we are lucky, our faces are shining with fat, but we don't know the pleasure

of being hungry any more.

Too much of anything-too much music, entertainment, happy snacks, or time spent

with one's friends-creates a kind of importance of living by which one can no longer

hear, or taste, or see, or love, or remember.


Life is short and precious, and appetite is short life we should respect the divinity of

appetite, and keep it eager and not too much blunted.

Select the correct answer option based on the passage.

What commonality has been highlighted between the sailors and hunters?

1) Neither were find nor entertained regularly.

2) They renew and frefresh themselves regularly

3) They were regularly seperated from their loved ones and things they liked.

4) The roles of men and women were clearly divided for both professions.

Correct Option
3
is:
Your Option is: 3
Result: Correct
Timetaken: 0.0 secs
From the passage,they were regularly separated from their
Explanation: loved ones and the things they liked.So the answer is option
3

Select the word or phrase which best expresses the MEANING of the given word.

15. PRIMAL

1) Approved

2) Cadrinal

3) Precise

4) permanent
Correct Option is: 2
Your Option is: 1
Result: Wrong
Timetaken: 0.0 secs
Explanation: Direct meaning of 'PRIMAL' is 'Cardinal'

16.
Passage:

At the end of the 19th century. India's maharajahs discovered a Parisian designer

called Louis Vuitton and flooded his small factory with orders for custom-made

Rolls-Royce interiors, leather picnic hampers and modish polo-club bags.

But after independence, when India's princes lost much of their wealth, the orders

dried up. Then in 2002 LVMH, the world's largest luxury-goods group, made a

triumphant return to India, opening a boutique in Delhi and another in Mumbai in

2004.

Its target was the new breed of maharajah produced by India's liberalized

economy:flush, flash and growing in number.

Other purveyors of opulence followed, from Chanel to Bulgari. In recent months a

multitude of swanky brands have announced plans to set up shop in india, including

Dolce & Gabbana, Herms, Jimmy Choo and Gucci.

Though only a tiny fraction of the total population will spend on these brands but it

is India's future prospects that have excited the luxury behemoths.

India has fewer than 100,000 dollar millionaires among its one billion-plus

population, according to American Express, a financial-services firm. It predicts that

this number will grow by 12.8% a year for the next three years.

The longer-term asceridance of India's middle class, meanwhile has been charted

by the MCKinsery Global Institute, which predicts that average incomes will have

tripled by 2025, lifting nearly 300m Indians out of poverty and causing the middle

class to grow more than tenfold to 583m.


Demand for all kinds of consumer products is about to surge, in short. And although

restrictions on foreign investment prevent retail giants such as Wal-Mart and Tesco

from entering India directly, different rules apply to companies that sell their own

products under a singled brand, as luxury-goods firms tend to.

Since January 2006 they have been allowed to take up to 51% in Indian joint

ventures. India is also an attractive market for luxury goods because, unlike China,

it does not have a flourishing counterfeit industry.Credit is becoming more easily

available.

Barriers to growth remain, however. High import duties make luxury goods

expensive.Rich Indians tend to travel widely and may simply buy elsewhere.

Finding suitable retail space is also proving a headache. So far most designer

boutiques are situated in five star hotels.

Select the correct answer option based on the passage.

What is Author most likely to agree to?

1) The current number of dollar millionaries in india is very high.

2) The current number of dollar millionaries in India is low.

3) The current number of dollar millionaries in India match world average.

4) None of these.

Correct Option
1
is:
Your Option is: 2
Result: Wrong
Timetaken: 0.0 secs
From the passage,it is clear that author agrees to the current
Explanation:
number of dollar millionaries in india is very high.

17.
Passage:
At the end of the 19th century. India's maharajahs discovered a Parisian designer

called Louis Vuitton and flooded his small factory with orders for custom-made

Rolls-Royce interiors, leather picnic hampers and modish polo-club bags.

But after independence, when India's princes lost much of their wealth, the orders

dried up. Then in 2002 LVMH, the world's largest luxury-goods group, made a

triumphant return to India, opening a boutique in Delhi and another in Mumbai in

2004.

Its target was the new breed of maharajah produced by India's liberalized

economy:flush, flash and growing in number.

Other purveyors of opulence followed, from Chanel to Bulgari. In recent months a

multitude of swanky brands have announced plans to set up shop in india, including

Dolce & Gabbana, Herms, Jimmy Choo and Gucci.

Though only a tiny fraction of the total population will spend on these brands but it

is India's future prospects that have excited the luxury behemoths.

India has fewer than 100,000 dollar millionaires among its one billion-plus

population, according to American Express, a financial-services firm. It predicts that

this number will grow by 12.8% a year for the next three years.

The longer-term asceridance of India's middle class, meanwhile has been charted

by the MCKinsery Global Institute, which predicts that average incomes will have

tripled by 2025, lifting nearly 300m Indians out of poverty and causing the middle

class to grow more than tenfold to 583m.

Demand for all kinds of consumer products is about to surge, in short. And although

restrictions on foreign investment prevent retail giants such as Wal-Mart and Tesco

from entering India directly, different rules apply to companies that sell their own

products under a singled brand, as luxury-goods firms tend to.

Since January 2006 they have been allowed to take up to 51% in Indian joint

ventures. India is also an attractive market for luxury goods because, unlike China,

it does not have a flourishing counterfeit industry.Credit is becoming more easily


available.

Barriers to growth remain, however. High import duties make luxury goods

expensive.Rich Indians tend to travel widely and may simply buy elsewhere.

Finding suitable retail space is also proving a headache. So far most designer

boutiques are situated in five star hotels.

Select the correct answer option based on the passage.

According to the author, which of these is not a problem for the luxury good firms in

the Indian market

1) High import duty.

2) Difficulty in finding retail space.

3) Restrictions on firms to enter Indian markets.

4) All of these.

Correct Option
3
is:
Your Option is: 2
Result: Wrong
Timetaken: 0.0 secs
According to the passage,restrictions on firms to enter
Explanation:
indian markets is not a problem.

Select the correct option that fills the blanks to make the sentence complete

18. meaningful

The teacher announced in the exam hall. " All you have ____________ hour".

1) is an

2) are an
3) is a

4) is the

Correct Option
1
is:
Your Option is: 4
Result: Wrong
Timetaken: 0.0 secs
As the sentence is in simple present form, then the correct
Explanation:
answer is option 1

Read the sentence to find out whether there is any grammatical error in it. The

error , if any, will be in one part of the sentence.The letter of the part is the

answer.Ignore the error of punctuation, If any.


19.
(A) A tie is a very important

(B) Part of the formal dressing

(C) for every men

1) A

2) B

3) C

4) No error

Correct Option is: 4


Your Option is: 1
Result: Wrong
Timetaken: 0.0 secs
Explanation: No error in the given snetence
20.
Passage:

At the end of the 19th century. India's maharajahs discovered a Parisian designer

called Louis Vuitton and flooded his small factory with orders for custom-made

Rolls-Royce interiors, leather picnic hampers and modish polo-club bags.

But after independence, when India's princes lost much of their wealth, the orders

dried up. Then in 2002 LVMH, the world's largest luxury-goods group, made a

triumphant return to India, opening a boutique in Delhi and another in Mumbai in

2004.

Its target was the new breed of maharajah produced by India's liberalized

economy:flush, flash and growing in number.

Other purveyors of opulence followed, from Chanel to Bulgari. In recent months a

multitude of swanky brands have announced plans to set up shop in india, including

Dolce & Gabbana, Herms, Jimmy Choo and Gucci.

Though only a tiny fraction of the total population will spend on these brands but it

is India's future prospects that have excited the luxury behemoths.

India has fewer than 100,000 dollar millionaires among its one billion-plus

population, according to American Express, a financial-services firm. It predicts that

this number will grow by 12.8% a year for the next three years.

The longer-term asceridance of India's middle class, meanwhile has been charted

by the MCKinsery Global Institute, which predicts that average incomes will have

tripled by 2025, lifting nearly 300m Indians out of poverty and causing the middle

class to grow more than tenfold to 583m.

Demand for all kinds of consumer products is about to surge, in short. And although

restrictions on foreign investment prevent retail giants such as Wal-Mart and Tesco

from entering India directly, different rules apply to companies that sell their own

products under a singled brand, as luxury-goods firms tend to.

Since January 2006 they have been allowed to take up to 51% in Indian joint

ventures. India is also an attractive market for luxury goods because, unlike China,
it does not have a flourishing counterfeit industry.Credit is becoming more easily

available.

Barriers to growth remain, however. High import duties make luxury goods

expensive.Rich Indians tend to travel widely and may simply buy elsewhere.

Finding suitable retail space is also proving a headache. So far most designer

boutiques are situated in five star hotels.

Select the correct answer option based on the passage.

What could be the meaning of the word 'modish', as can be inferred from the

context it is used in first line of the passage?

1) Unattractive

2) Stylish

3) New

4) Beautiful

Correct Option is: 2


Your Option is: 4
Result: Wrong
Timetaken: 0.0 secs
Explanation: Direct meaning of 'modish' is 'stylish'

Select the option that is most nearly OPPOSITE in meaning to the word or phrase

given in CAPITALIZED
21.
Birds are QUARANTINED to prevent the spread of bird flu.

1) immunized
2) butchered

3) secluded

4) Mingled

Correct Option
4
is:
Your Option is: 2
Result: Wrong
Timetaken: 0.0 secs
Meaning of 'QUARANTINE' is isolation and the opposite
Explanation:
word is 'Mingled'

Select the option that is most nearly OPPOSITE in meaning to the given word.
22.
INSTIGATE (OPPOSITE)

1) Stimulate

2) Prompt

3) Ferment

4) Deceive

5) Prevent

Correct Option
5
is:
Your Option is: 4
Result: Wrong
Timetaken: 0.0 secs
Meaning of INSTIGATE is 'stimulate' and opposite of
Explanation:
'INSTIGATE' is 'Prevent'.

23.
In the question each passage consist of a six sentence.The first and the sixth
sentence are given in the beginning.The middle four sentence have been removed

and jumbled up.These are labeled P,Q,R and S.Select the proper order for the four

sentence.

S1: Sameer has not been coming to office since last week.

S6:Since his plaster will be removed after 30 days.

P: Doctors have advised him completed bed rest for 15 days.

Q: He had six fractures and was rushed to the hospital.

R: he met with a serious accident a few days ago. S: I think he would be on leave

for this whole month.

1) RQPS

2) PSQR

3) SPRQ

4) QPRS

Correct Option
1
is:
Your Option is: 2
Result: Wrong
Timetaken: 0.0 secs
reason for the sentence S1 is given in thw sentence R.R is
Explanation: followed by Q which is followed by P and P and S is a pair
of sentences.

24.
Read the sentence to find out whether there is any grammatical error in it.The

error, if any,will be in one part of the sentence. The letter of that part is the

answer.Ignore the error of punctuation,if any.

(A)The phrase 'Be the change you want (B)to see in the world'was (C)said through
Mahatma Gandhi.(D)No error

1) A

2) B

3) C

4) No error

Correct Option is: 1


Your Option is: 1
Result: Correct
Timetaken: 0.0 secs
Explanation: instead of 'want', 'wish' must be used

25.
Passage:

At the end of the 19th century. India's maharajahs discovered a Parisian designer

called Louis Vuitton and flooded his small factory with orders for custom-made

Rolls-Royce interiors, leather picnic hampers and modish polo-club bags.

But after independence, when India's princes lost much of their wealth, the orders

dried up. Then in 2002 LVMH, the world's largest luxury-goods group, made a

triumphant return to India, opening a boutique in Delhi and another in Mumbai in

2004.

Its target was the new breed of maharajah produced by India's liberalized

economy:flush, flash and growing in number.

Other purveyors of opulence followed, from Chanel to Bulgari. In recent months a

multitude of swanky brands have announced plans to set up shop in india, including

Dolce & Gabbana, Herms, Jimmy Choo and Gucci.

Though only a tiny fraction of the total population will spend on these brands but it

is India's future prospects that have excited the luxury behemoths.


India has fewer than 100,000 dollar millionaires among its one billion-plus

population, according to American Express, a financial-services firm. It predicts that

this number will grow by 12.8% a year for the next three years.

The longer-term asceridance of India's middle class, meanwhile has been charted

by the MCKinsery Global Institute, which predicts that average incomes will have

tripled by 2025, lifting nearly 300m Indians out of poverty and causing the middle

class to grow more than tenfold to 583m.

Demand for all kinds of consumer products is about to surge, in short. And although

restrictions on foreign investment prevent retail giants such as Wal-Mart and Tesco

from entering India directly, different rules apply to companies that sell their own

products under a singled brand, as luxury-goods firms tend to.

Since January 2006 they have been allowed to take up to 51% in Indian joint

ventures. India is also an attractive market for luxury goods because, unlike China,

it does not have a flourishing counterfeit industry.Credit is becoming more easily

available.

Barriers to growth remain, however. High import duties make luxury goods

expensive.Rich Indians tend to travel widely and may simply buy elsewhere.

Finding suitable retail space is also proving a headache. So far most designer

boutiques are situated in five star hotels.

Select the correct answer option based on the passage.

What is a good estimate of the middle class population in India today as inferred

from the passage?

1) 583m

2) 100,000

3) 58m

4) 300m
Correct Option
1
is:
Your Option is: 2
Result: Wrong
Timetaken: 0.0 secs
From the passage,it is clear that middle class population
Explanation:
is 583m.

26. 24.SHABBY

1) A. Pure

2) B. Dirty

3) C. Interesting

4) D. Curious

Correct Option is: 2


Your Option is: 2
Result: Correct
Timetaken: 0.0 secs
Explanation: -

27. VENT(Meaning)

1) Opening

2) Stodgy

3) End

4) Past tense of Go
Correct Option is: 1
Your Option is: 2
Result: Wrong
Timetaken: 0.0 secs
Explanation: Nil

Select the word or phrase which best expresses the meaning of the given word.
28.
VANISH

1) Evacuate

2) Decrease

3) Disappear

4) Harm

Correct Option is: 3


Your Option is: 4
Result: Wrong
Timetaken: 0.0 secs
Explanation: Direct meaning of 'Vanish' is 'disappear'

In each of the following questions, a sentence has been given in Active (or passive)

voice. Out of the four alternatives suggested select the one which best express the
1.
same sentence in Passive (or Active) Voice.

I saw him conducting the rehearsal.

1) He was seen conducting the rehearsal

2) I saw the rehearsal to be conducted by him

3) He was seen by me to conduct the rehearsal


4) I saw the rehearsal being conducted by him

Correct Option
4
is:
Your Option is: 2
Result: Wrong
Timetaken: 0.0 secs
In the options,only option 4 gives yhe same meaning of
Explanation:
given statemenet.

In the following the questions choose the word which best expresses the meaning

2. of the given word.

IMPROMPTU

1) Offhand

2) Effective

3) Unreal

4) Unimportant

Correct Option is: 1


Your Option is: 3
Result: Wrong
Timetaken: 0.0 secs
Explanation: 'Offhand is the direct meaning of 'impromptu'

3. Please distribute these sweets........................the children

1) in

2) between
3) amid

4) among

Correct
4
Option is:
Your Option
2
is:
Result: Wrong
Timetaken: 0.0 secs
Among : Surrounded by; in the company of. eg. Being a
member or members of (a larger set): "he was among the
first 29 students enrolled". Between :(intermediate to, in
Explanation: time, quantity, or degree) when it is categorized as boys or
girls. Amid : during; in or throughout the course of, in the
middle of; (eg. to stand weeping amid the ruins.) So, Option
(D) is the right answer.

In each of the following questions,find out which part of the sentence has an error.If

there is no mistake the answer is "no error"

as soon as I will reach Bombay(A)

I will send (B)


4.
you the books (C)

you have asked for(D)

No error(E)

1) A

2) B

3) C

4) D

5) E
Correct Option is: 1
Your Option is: 2
Result: Wrong
Timetaken: 0.0 secs
Explanation: 'will' should not be there infront of reach.

In the each of the following question find out which part of the sentence has an

error.if there is no mis take the answer is "no error"

Our conception of (A)

5. what should a science of mental life be (B)

has changed considerably since James' time (C)

No error(D)

1) A

2) B

3) C

4) D

Correct Option is: 2


Your Option is: 1
Result: Wrong
Timetaken: 0.0 secs
Explanation: 'be' should not come with 'has'.

6.
The following questions, consist of two words each that have a certain relationship

to each other, followed by four lettered pairs of words. Select the lettered pair that
has the same relationship as the original pair of words.

AVER : AFFIRMATION : :

1) proclaim: objection

2) denounce : defiance

3) reduse : stress

4) reduce : distress

Correct Option
4
is:
Your Option is: 2
Result: Wrong
Timetaken: 0.0 secs
Both aver and affirmation gives same meaning as reduce
Explanation:
and distress are same.So answer is option 4

In the following questions choose the word which is the exact OPPOSITE of the
7.
given words. amused

1) jolted

2) frightened

3) saddened

4) astonished

Correct Option is: 3


Your Option is: 4
Result: Wrong
Timetaken: 0.0 secs
opposite of amused is 'bore','depress'.So correct option is
Explanation:
3

The following questions, consist of two words each that have a certain relationship

to each other, followed by four lettered pairs of words. Select the lettered pair that
8.
has the same relationship as the original pair of words.

Duralumin : Aircraft

1) Brass : Alloy

2) Stone : Sculptor

3) Iron : Steel

4) Bronze : Statue

Correct Option
4
is:
Your Option is: 1
Result: Wrong
Timetaken: 0.0 secs
Aircrafts are mate up of duralumin, similarly in the
Explanation:
options,Statue will be made of bronze.

In the following questions choose the word which is the exact OPPOSITE of the

9. given words.

disparage

1) please

2) denigrate

3) praise

4) downgrade
Correct Option is: 3
Your Option is: 1
Result: Wrong
Timetaken: 0.0 secs
Explanation: Direct antonym of 'disparage' is 'praise'

In the following questions choose the word which is the exact OPPOSITE of the

10. given words.

abet

1) Prevent

2) Aid

3) Pacify

4) Risk

Correct Option is: 1


Your Option is: 4
Result: Wrong
Timetaken: 0.0 secs
Explanation: Nil

In the following questions choose the word which is the exact OPPOSITE of the

11. given words.

ornate

1) flamboyant

2) unpretending

3) baroque
4) florid

Correct Option
2
is:
Your Option is: 2
Result: Correct
Timetaken: 0.0 secs
Meaning of 'ornate' is pretentious.So opposite is
Explanation:
unpretending.

In each of the following questions, a sentence has been given in Active (or passive)

voice. Out of the four alternatives suggested select the one which best express the
12.
same sentence in Passive (or Active) Voice.

Do you imitate others ?

1) Are others imitated by you ?

2) Are others being imitated by you ?

3) Are others being imitated by you ?

4) Were others being imitated by you ?

5) Have others been imitated by you ?

Correct Option
1
is:
Your Option is: 3
Result: Wrong
Timetaken: 0.0 secs
In the options,Only option 1 gives the same meaning of
Explanation:
given statement.
In the following questions choose the word which is the exact OPPOSITE of the

13. given words.

adventure

1) explore

2) abstention

3) emprise

4) speculation

Correct Option is: 2


Your Option is: 4
Result: Wrong
Timetaken: 0.0 secs
Explanation: 'Abstention' is the direct opposite of 'adventure'

The following questions, consist of two words each that have a certain relationship

to each other, followed by four lettered pairs of words. Select the lettered pair that
14.
has the same relationship as the original pair of words. OFFHAND : PREMEDITATION

::

1) upright : integrity

2) aboveboard : guile

3) cutthroat : competition

4) backward : direction

Correct
2
Option is:
Your Option
2
is:
Result: Correct
Timetaken: 0.0 secs
OFFHAND : 1. Ungraciously or offensively nonchalant or
cool in manner. 2. Without previous thought or consideration.
PREMEDITATION : he act of speculating, arranging, or
Explanation:
plotting in advance. So. Offhand remark is made without
forethought or premeditation. In the same way Aboveboard
(open) is done without trickery or guile.

In the following the questions choose the word which best expresses the meaning

15. of the given word.

INSOLVENT

1) ability

2) Broke

3) Bankrupt

4) Poor

Correct Option is: 3


Your Option is: 4
Result: Wrong
Timetaken: 0.0 secs
Explanation: Meaning of insolvent is collapsed,ruined,bankrupt.

In the following the questions choose the word which best expresses the meaning

16. of the given word.

REPERCUSSION

1) Reaction
2) Remuneration

3) Recollection

4) Clever reply

Correct Option
1
is:
Your Option is: 2
Result: Wrong
Timetaken: 0.0 secs
Meaning of 'repercussion' is
Explanation:
effect,result,outcome,reaction,etc.

In the following questions choose the word which is the exact OPPOSITE of the

17. given words.

culpable

1) censurable

2) innocent

3) guilty

4) responsible

Correct Option is: 2


Your Option is: 1
Result: Wrong
Timetaken: 0.0 secs
Explanation: Direct opposite of 'culpable' is innocent.

18.
In the following the questions choose the word which best expresses the meaning

of the given word.


LAUD

1) Extolled

2) Praise

3) Eulogy

4) Lord

Correct Option is: 2


Your Option is: 2
Result: Correct
Timetaken: 0.0 secs
Explanation: 'Praise' is the direct meaning of 'LAUD'

While facts are ________ and data hard to come by, even scientists occasionally

throw side the professional pretense of ______ and tear into each other with
19.
shameless appeals to authority and arguments that shameless appeals to authority

and arguments that are unabashedly ad hominid.

1) elusive...objectivity

2) establish...courtesy

3) demonstrate .. neutrality

4) ineluctable...cooperation

Correct
1
Option is:
Your Option
4
is:
Result: Wrong
Timetaken: 0.0 secs
From the last sentence it is clear that scientists are not able to
do rectify the matter. establish : should not come in first
black as if facts are establish then data is not hard to come as
same for option c. So option B and C are false.
Ineluctable :Unable to be resisted or avoided; inescapable:
Explanation: Elusive : Difficult to find or catch. is the right word which
comes in the first black.
So, Elusive is the right word come in place. The scientist
would not able to see the matter with great objectivity and
argument in Ad hominid (appealing to feelings or prejudices
rather than intellect) manner to blame each other.

The following questions, consist of two words each that have a certain relationship

to each other, followed by four lettered pairs of words. Select the lettered pair that
20.
has the same relationship as the original pair of words.

Cool : Frigid

1) Cool : Frigid

2) Pool : Placid

3) Tepid : Torried

4) Lack : Abundant

Correct Option
3
is:
Your Option is: 2
Result: Wrong
Timetaken: 0.0 secs
Frigid is the extreme of cooling to make things to
Explanation: freeze.Similarly tepid is only lukewarm and torried is to dry
by heating.

21.
While the disease is in ______ state it is almost impossible to determine its
existence by_____ .

1) a dormant ..postulate

2) a critical...examination

3) an acute ...analysis

4) a latent...observation

Correct
4
Option is:
Your Option
4
is:
Result: Correct
Timetaken: 0.0 secs
B is incorrect as if disease is in critical state, then it is
possible to determine by its examination. C is incorrect as
disease is able to determine by analysis. A is incorrect
because if disease is in dormant state(asleep/inactive) it can
be determined by postulate(Suggest or assume the existence,
Explanation:
fact, or truth of (something)). Patent means : Present or
potential but not evident or active or 2. Pathology In a
dormant or hidden stage: a latent infection. Thus cannot be
determine by just Observation (an act or instance of
regarding attentively or watching.) Hence D is the answer.

In each of the following questions, find out which part of the sentence has an error.

if there is no mistake the answer is 'no error'

If I was you (A)

22. I would have (B)

terminated his services then and there (C)

No error(D)

1) A
2) B

3) C

4) D

Correct Option is: 1


Your Option is: 2
Result: Wrong
Timetaken: 0.0 secs
Explanation: 'were' must be used instead of 'was'

The following questions, consist of two words each that have a certain relationship

to each other, followed by four lettered pairs of words. Select the lettered pair that
23.
has the same relationship as the original pair of words.

POLTERGEIST : APPARATION::

1) dwarf : stature

2) witch : familiar

3) ogre : monster

4) sorcerer : spell

Correct Option
3
is:
Your Option is: 4
Result: Wrong
Timetaken: 0.0 secs
Here, meaning of potergeist and apparation is same.In
Explanation:
options, ogre means monster.So answer is option 3
24. The storehouse was infested.........................rats.

1) by

2) of

3) with

4) in

Correct
3
Option is:
Your Option
1
is:
Result: Wrong
Timetaken: 0.0 secs
infested: (of insects or other animals) Be present (in a place
or site) in large numbers, typically so as to cause damage or
Explanation: disease. 2). To live as a parasite in or on. So, answer should
be with as rat should be present in a place or site. Hence
Answer is (C).

25. This custom seems to have originated .....................East European country.

1) in

2) from

3) by

4) with

Correct
1
Option is:
Your Option
2
is:
Result: Wrong
Timetaken: 0.0 secs
Originated : 1. Have a specified beginning: "the word
originated as a marketing term". 2. Create or initiate
Explanation: (something): "he originated this particular clich". Initiate :
So, custom seem to be originated in East European country.
Option A is correct answer.

In the following the questions choose the word which best expresses the meaning

1. of the given word.

CONCEITED

1) Arrogant

2) False

3) Deceive

4) Misconception

Correct Option is: 1


Your Option is: 2
Result: Wrong
Timetaken: 0.0 secs
Explanation: self admiring, vain, egoistic

2.
Sixty years ago, on the evening of August 14, 1947, a few hours before Britain?s

Indian Empire was formally divided into the nation-states of India and Pakistan,

Lord Louis Mountbatten and his wife, Edwina, sat down in the vice regal mansion in

New Delhi to watch the latest Bob Hope movie, ?My Favorite Brunette.? Large parts

of the subcontinent were descending into chaos, as the implications of partitioning

the Indian Empire along religious lines became clear to the millions of Hindus,

Muslims, and Sikhs caught on the wrong side of the border. In the next few months,
some twelve million people would be uprooted and as many as a million murdered.

But on that night in mid-August the bloodbath?and the fuller consequences of hasty

imperial retreat?still lay in the future, and the Mountbatten probably felt they had

earned their evening?s entertainment.

Mountbatten, the last viceroy of India, had arrived in New Delhi in March, 1947,

charged with an almost impossible task. Irrevocably enfeebled by the Second World

War, the British belatedly realized that they had to leave the subcontinent, which

had spiraled out of their control through the nineteen-forties. But plans for brisk

disengagement ignored messy realities on the ground. Mountbatten had a clear

remit to transfer power to the Indians within fifteen months. Leaving India to God,

or anarchy, as Mohandas Gandhi, the foremost Indian leader, exhorted, wasn?t a

political option, however tempting. Mountbatten had to work hard to figure out how

and to whom power was to be transferred.

The dominant political party, the Congress Party, took inspiration from Gandhi in

claiming to be a secular organization, representing all four hundred million Indians.

But many Muslim politicians saw it as a party of upper-caste Hindus and demanded

a separate homeland for their hundred million co-religionists, who were

intermingled with non-Muslim populations across the subcontinent?s villages,

towns, and cities. Eventually, as in Palestine, the British saw partition along

religious lines as the quickest way to the exit. But sectarian riots in Punjab and

Bengal dimmed hopes for a quick and dignified British withdrawal, and boded ill for

India?s assumption of power. Not surprisingly, there were some notable absences at

the Independence Day celebrations in New Delhi on August 15th. Gandhi,

denouncing freedom from imperial rule as a ?wooden loaf,? had remained in

Calcutta, trying, with the force of his moral authority, to stop Hindus and Muslims

from killing each other. His great rival Mohammed Ali Jinnah, who had fought

bitterly for a separate homeland for Indian Muslims, was in Karachi, trying to hold

together the precarious nation-state of Pakistan.


Nevertheless, the significance of the occasion was not lost on many. While the

Mountbattens were sitting down to their Bob Hope movie, India?s constituent

assembly was convening in New Delhi. The moment demanded grandiloquence, and

Jawaharlal Nehru, Gandhi?s closest disciple and soon to be India?s first Prime

Minister, provided it. ?Long years ago, we made a tryst with destiny,? he said. ?At

the stroke of the midnight hour, while the world sleeps, India will awaken to life and

freedom. A moment comes, which comes but rarely in history, when we step out

from the old to the new, when an age ends, and when the soul of a nation, long

suppressed, finds utterance.?

Posterity has enshrined this speech, as Nehru clearly intended. But today his quaint

phrase ?tryst with destiny? resonates ominously, so enduring have been the political

and psychological scars of partition. The souls of the two new nation-states

immediately found utterance in brutal enmity. In Punjab, armed vigilante groups,

organized along religious lines and incited by local politicians, murdered countless

people, abducting and raping thousands of women. Soon, India and Pakistan were

fighting a war?the first of three?over the disputed territory of Kashmir. Gandhi,

reduced to despair by the seemingly endless cycle of retaliatory mass murders and

displacement, was shot dead in January, 1948, by a Hindu extremist who believed

that the father of the Indian nation was too soft on Muslims. Jinnah, racked with

tuberculosis and overwork, died a few months later, his dream of a secular Pakistan

apparently buried with him.

What does the author imply about the future of Pakistan?

1) It becomes a secular country

2) It becomes unsecular

3) It is unprosperous

4) It becomes a rogue state


Correct Option is: 3
Your Option is: 2
Result: Wrong
Timetaken: 0.0 secs
Explanation: Nil

Our bright and beautiful environment is being systematically destroyed under the
3.
pressure of a _________ population.

1) proliferating

2) Expanding

3) Widening

4) Enlarging

Correct Option
1
is:
Your Option is: 1
Result: Correct
Timetaken: 0.0 secs
1: increasing rapidly in number 2,3,4: increasing in size.
Explanation:
hence 1

Select the correct option to fill in the blank.


4.
The leaves __________ yellow and dry.

1) were

2) being

3) was
4) had

Correct Option
1
is:
Your Option is: 2
Result: Wrong
Timetaken: 0.0 secs
Subject 'leaves' is in plural form,so the correct answer is
Explanation:
option 1

Select the correct option to fill in the blank.


5.
___________the shirt was washed twice, still he refuse to wear it.

1) Though

2) Because

3) However

4) Since

5) While

Correct Option
1
is:
Your Option is: 4
Result: Wrong
Timetaken: 0.0 secs
Because, since- reason. Opposing word for the sentence
Explanation:
still he refuse is 'though'

6.
Sixty years ago on the evening of August 14,1947 a few hours before Britain Indian

empire was formally divided into the nation- states of India and Pakistan , Lord

Louis Mountbatten and his wife,Edwin sat down in the viceregal mansion in New

Delhi to watch the last bob hope movie.?My favorite Brunette.? Large parts of the
subcontinent were descending into chaos,as the implications of partitioning the

Indian empire along religious lines became clear to the million of Hindus,Muslims,

and Sikhs caught on the wrong side of the border. In the next few months,some

twelve million people would be uprooted and as many as a million murdered.But on

that night in mid-August the blood bath-and the fuller consequence of hasty

imperial retreat ? still lay in the future,and the Mountbatten's probably felt they had

earned .

While the Mount battens were sitting down to their Bob Hope movie,India's

constitute assembly was convening in New Delhi.The moment demanded

grandiloquence,and Jawaharlal Nehru,Gandhi's closest discipline and soon to be

India's first prime minister ,provided along it.?Long years ago while the world

sleeps ,India will awaken to life and to freedom .A moment comes which rarely in

history when we step out from the old to the new,when an age ends,and when the

soul of a nation ,long suppressed,finds utterance.

Posterity has enshrined this speech ,as Nehru clearly intended. But today his quaint

phrase ?tryst with destiny? ominously,so enduring have been political and

psychological scars of partitions.Soon,India and Pakistan were fighting a war-the

first of three ? over the disputed territorial of Kashmir. Gandhi reduces to despair to

the seemingly endless cycle of retaliatory mass murders and displacement was shot

dead in January 1948,by Hindu extremist who believed that the father of the Indian

nation was too soft on Muslim,Jinnah,racked with tuberculous and overwork died a

few months later ,his dreams of secular Pakistan apparently buried with him.

Why was Gandhiji assassinated?

1) Because he was favouring the muslim

2) His assassin thought he was partial to the muslim

3) He got killed in the violence after partition

4) None of these
Correct Option is: 2
Your Option is: 1
Result: Wrong
Timetaken: 0.0 secs
Explanation: Nil

In the following questions choose the word which is the exact OPPOSITE of the

7. given words.

.JAUNTY

1) Youthful

2) Rude

3) Strong

4) Unravelled

5) Sedate

Correct Option is: 2


Your Option is: 4
Result: Wrong
Timetaken: 0.0 secs
Explanation: cheerful, glad, merry. hence 2

In the following questions choose the word which is the exact OPPOSITE of the

8. given words.

RUDE

1) Detest
2) Beastly

3) Respectful

4) Hideous

Correct Option is: 3


Your Option is: 1
Result: Wrong
Timetaken: 0.0 secs
Explanation: 1:dislike 2:very unpleasant 4:ugly hence 3

9.
Give people power and discretion, and whether they are grand viziers or border

guards, some will use their position to enrich themselves. The problem can be big

enough to hold back a country's development. One study has shown that bribes

account for 8% of the total cost of running a business in Uganda. Another found

that corruption boosted the price of hospital supplies in Buenos Aires by 15%. Paul

Wolfowitz, the head of the World Bank, is devoting special efforts during his

presidency there to a drive against corruption. GIVE people power and discretion,

and whether they are grand viziers or border guards, some will use their position to

enrich themselves. The problem can be big enough to hold back a country's

development. One study has shown that bribes account for 8% of the total cost of

running a business in Uganda. Another found that corruption boosted the price of

hospital supplies in Buenos Aires by 15%. Paul Wolfowitz, the head of the World

Bank, is devoting special efforts during his presidency there to a drive against

corruption.

For most people in the world, though, the worry is not that corruption may slow

down their country's GDP growth. It is that their daily lives are pervaded by endless

hassles, big and small. And for all the evidence that some cultures suffer endemic

corruption while others are relatively clean, attitudes towards corruption, and even
the language describing bribery, is remarkably similar around the world.

In a testament to most people's basic decency, bribe-takers and bribe-payers have

developed an elaborate theatre of dissimulation. This is not just to avoid detection.

Even in countries where corruption is so common as to be unremarkable and

unprosecutable?and even when the transaction happens far from snooping eyes?a

bribe is almost always dressed up as some other kind of exchange. Though most of

the world is plagued by corruption, even serial offenders try to conceal it.

One manifestation of this is linguistic. Surprisingly few people say: ?You are going

to have to pay me if you want to get that done.? Instead, they use a wide variety of

euphemisms. One type is quasi-official terminology. The first bribe paid by your

correspondent, in Ukraine in 1998, went to two policemen so they would let him

board a train leaving the country. On the train into Ukraine, the customs officer had

absconded with a form that is needed again later to leave the country. The

policemen at the station kindly explained that there was a shtraf, a ?fine? that could

be paid instead of producing the document. The policemen let him off with the

minimum shtraf of 50 hryvnia ($25).

A second type of euphemism dresses up a dodgy payment as a friendly favour done

by the bribe-payer. There is plenty of creative scope. Nigerian policemen are known

to ask for ?a little something for the weekend?. A North African term is ?un petit

cadeau?, a little gift. Mexican traffic police will suggest that you buy them a

refresco, a soft drink, as will Angolan and Mozambican petty officials, who call it a

gazoso in Portuguese. A businessman in Iraq told Reuters that although corruption

there is quite overt, officials still insist on being given a ?good coffee?

Double meaning can help soothe the awkwardness of bribe-paying. Baksheesh,

originally a Persian word now found in many countries of the Middle East, can mean

?tip?, ?alms? and ?bribe?. Swahili-speakers can take advantage of another

ambiguous term. In Kenya a machine-gun-wielding guard suggested to a terrified

Canadian aid worker: ?Perhaps you would like to discuss this over tea?? The young
Canadian was relieved: the difficulty could be resolved with some chai, which

means both ?tea? and ?bribe?.

Along with the obscurantist language, bribe-taking culture around the world often

involves the avoidance of physically handing the money from one person to another.

One obvious reason is to avoid detection, which is why bribes are known as ?

envelopes? in countries from China to Greece. But avoidance of a direct hand-over

is common even where there is no chance of detection. There will always be some

officials who will take money right from a bribe-payer's hands, but most seem to

prefer to find some way to hide the money from view. A bribe to a border guard

may be folded into a passport. A sweetener to a traffic cop is often placed in the

ticket-book that is handed to the driver. Parag Khanna, who is writing a book about

countries on the edge of the rich world that are trying to get rich themselves,

describes a bribe-taker he spotted in Georgia who he was sure was a rookie. Why?

The scrawny young soldier, forgoing any subtleties, merely rubbed his fingers

together in an age-old gesture. Rich Westerners may not think of their societies as

plagued by corruption. But the definition of bribery clearly differs from person to

person. A New Yorker might pity the third-world businessman who must pay bribes

just to keep his shop open. But the same New Yorker would not think twice about

slipping the maitre d' $50 to sneak into a nice restaurant without a reservation.

Poor people the world over are most infuriated by the casual corruption of the elites

rather than by the underpaid, tip seeking soldier or functionary. What could be the

meaning of the word dissimulation as inferred from the passage?

1) Hypocrispy

2) Clarity

3) Frankness

4) Pretense
Correct Option is: 4
Your Option is: 2
Result: Wrong
Timetaken: 0.0 secs
Explanation: Nil

10.
Sixty years ago, on the evening of August 14, 1947, a few hours before Britain?s

Indian Empire was formally divided into the nation-states of India and Pakistan,

Lord Louis Mountbatten and his wife, Edwina, sat down in the vice regal mansion in

New Delhi to watch the latest Bob Hope movie, ?My Favorite Brunette.? Large parts

of the subcontinent were descending into chaos, as the implications of partitioning

the Indian Empire along religious lines became clear to the millions of Hindus,

Muslims, and Sikhs caught on the wrong side of the border. In the next few months,

some twelve million people would be uprooted and as many as a million murdered.

But on that night in mid-August the bloodbath?and the fuller consequences of hasty

imperial retreat?still lay in the future, and the Mountbatten probably felt they had

earned their evening?s entertainment. Mountbatten, the last viceroy of India, had

arrived in New Delhi in March, 1947, charged with an almost impossible task.

Irrevocably enfeebled by the Second World War, the British belatedly realized that

they had to leave the subcontinent, which had spiraled out of their control through

the nineteen-forties. But plans for brisk disengagement ignored messy realities on

the ground. Mountbatten had a clear remit to transfer power to the Indians within

fifteen months. Leaving India to God, or anarchy, as Mohandas Gandhi, the

foremost Indian leader, exhorted, wasn?t a political option, however tempting.

Mountbatten had to work hard to figure out how and to whom power was to be

transferred.

The dominant political party, the Congress Party, took inspiration from Gandhi in

claiming to be a secular organization, representing all four hundred million Indians.


But many Muslim politicians saw it as a party of upper-caste Hindus and demanded

a separate homeland for their hundred million co-religionists, who were

intermingled with non-Muslim populations across the subcontinent?s villages,

towns, and cities. Eventually, as in Palestine, the British saw partition along

religious lines as the quickest way to the exit.

But sectarian riots in Punjab and Bengal dimmed hopes for a quick and dignified

British withdrawal, and boded ill for India?s assumption of power. Not surprisingly,

there were some notable absences at the Independence Day celebrations in New

Delhi on August 15th. Gandhi, denouncing freedom from imperial rule as a ?wooden

loaf,? had remained in Calcutta, trying, with the force of his moral authority, to stop

Hindus and Muslims from killing each other. His great rival Mohammed Ali Jinnah,

who had fought bitterly for a separate homeland for Indian Muslims, was in Karachi,

trying to hold together the precarious nation-state of Pakistan.

Nevertheless, the significance of the occasion was not lost on many. While the

Mountbattens were sitting down to their Bob Hope movie, India?s constituent

assembly was convening in New Delhi. The moment demanded grandiloquence, and

Jawaharlal Nehru, Gandhi?s closest disciple and soon to be India?s first Prime

Minister, provided it. ?Long years ago, we made a tryst with destiny,? he said. ?At

the stroke of the midnight hour, while the world sleeps, India will awaken to life and

freedom. A moment comes, which comes but rarely in history, when we step out

from the old to the new, when an age ends, and when the soul of a nation, long

suppressed, finds utterance.?

Posterity has enshrined this speech, as Nehru clearly intended. But today his quaint

phrase ?tryst with destiny? resonates ominously, so enduring have been the political

and psychological scars of partition. The souls of the two new nation-states

immediately found utterance in brutal enmity. In Punjab, armed vigilante groups,

organized along religious lines and incited by local politicians, murdered countless

people, abducting and raping thousands of women. Soon, India and Pakistan were
fighting a war?the first of three?over the disputed territory of Kashmir. Gandhi,

reduced to despair by the seemingly endless cycle of retaliatory mass murders and

displacement, was shot dead in January, 1948, by a Hindu extremist who believed

that the father of the Indian nation was too soft on Muslims. Jinnah, racked with

tuberculosis and overwork, died a few months later, his dream of a secular Pakistan

apparently buried with him.

What does the author imply about the future of Pakistan?

1) It becomes a secular country

2) It becomes unsecular

3) It is unprosperous

4) It becomes arough state

Correct Option is: 3


Your Option is: 1
Result: Wrong
Timetaken: 0.0 secs
Explanation: Nil

11.
Sixty years ago, on the evening of August 14, 1947, a few hours before Britain?s

Indian Empire was formally divided into the nation-states of India and Pakistan,

Lord Louis Mountbatten and his wife, Edwina, sat down in the vice regal mansion in

New Delhi to watch the latest Bob Hope movie, ?My Favorite Brunette.? Large parts

of the subcontinent were descending into chaos, as the implications of partitioning

the Indian Empire along religious lines became clear to the millions of Hindus,

Muslims, and Sikhs caught on the wrong side of the border. In the next few months,

some twelve million people would be uprooted and as many as a million murdered.

But on that night in mid-August the bloodbath?and the fuller consequences of hasty
imperial retreat?still lay in the future, and the Mountbatten probably felt they had

earned their evening?s entertainment.

Mountbatten, the last viceroy of India, had arrived in New Delhi in March, 1947,

charged with an almost impossible task. Irrevocably enfeebled by the Second World

War, the British belatedly realized that they had to leave the subcontinent, which

had spiraled out of their control through the nineteen-forties. But plans for brisk

disengagement ignored messy realities on the ground. Mountbatten had a clear

remit to transfer power to the Indians within fifteen months. Leaving India to God,

or anarchy, as Mohandas Gandhi, the foremost Indian leader, exhorted, wasn?t a

political option, however tempting. Mountbatten had to work hard to figure out how

and to whom power was to be transferred.

The dominant political party, the Congress Party, took inspiration from Gandhi in

claiming to be a secular organization, representing all four hundred million Indians.

But many Muslim politicians saw it as a party of upper-caste Hindus and demanded

a separate homeland for their hundred million co-religionists, who were

intermingled with non-Muslim populations across the subcontinent?s villages,

towns, and cities. Eventually, as in Palestine, the British saw partition along

religious lines as the quickest way to the exit. But sectarian riots in Punjab and

Bengal dimmed hopes for a quick and dignified British withdrawal, and boded ill for

India?s assumption of power. Not surprisingly, there were some notable absences at

the Independence Day celebrations in New Delhi on August 15th. Gandhi,

denouncing freedom from imperial rule as a ?wooden loaf,? had remained in

Calcutta, trying, with the force of his moral authority, to stop Hindus and Muslims

from killing each other. His great rival Mohammed Ali Jinnah, who had fought

bitterly for a separate homeland for Indian Muslims, was in Karachi, trying to hold

together the precarious nation-state of Pakistan.

Nevertheless, the significance of the occasion was not lost on many. While the

Mountbattens were sitting down to their Bob Hope movie, India?s constituent
assembly was convening in New Delhi. The moment demanded grandiloquence, and

Jawaharlal Nehru, Gandhi?s closest disciple and soon to be India?s first Prime

Minister, provided it. ?Long years ago, we made a tryst with destiny,? he said. ?At

the stroke of the midnight hour, while the world sleeps, India will awaken to life and

freedom. A moment comes, which comes but rarely in history, when we step out

from the old to the new, when an age ends, and when the soul of a nation, long

suppressed, finds utterance.?

Posterity has enshrined this speech, as Nehru clearly intended. But today his quaint

phrase ?tryst with destiny? resonates ominously, so enduring have been the political

and psychological scars of partition. The souls of the two new nation-states

immediately found utterance in brutal enmity. In Punjab, armed vigilante groups,

organized along religious lines and incited by local politicians, murdered countless

people, abducting and raping thousands of women. Soon, India and Pakistan were

fighting a war?the first of three?over the disputed territory of Kashmir. Gandhi,

reduced to despair by the seemingly endless cycle of retaliatory mass murders and

displacement, was shot dead in January, 1948, by a Hindu extremist who believed

that the father of the Indian nation was too soft on Muslims. Jinnah, racked with

tuberculosis and overwork, died a few months later, his dream of a secular Pakistan

apparently buried with him.

Give people power and discretion, and whether they are grand viziers or border

guards, some will use their position to enrich themselves. The problem can be big

enough to hold back a country's development. One study has shown that bribes

account for 8% of the total cost of running a business in Uganda. Another found

that corruption boosted the price of hospital supplies in Buenos Aires by 15%. Paul

Wolfowitz, the head of the World Bank, is devoting special efforts during his

presidency there to a drive against corruption. GIVE people power and discretion,

and whether they are grand viziers or border guards, some will use their position to

enrich themselves. The problem can be big enough to hold back a country's
development. One study has shown that bribes account for 8% of the total cost of

running a business in Uganda. Another found that corruption boosted the price of

hospital supplies in Buenos Aires by 15%. Paul Wolfowitz, the head of the World

Bank, is devoting special efforts during his presidency there to a drive against

corruption.

What could be the meaning of the word ?obscurantist? as inferred from the

passage?

1) clear

2) Unclear

3) Nasty

4) Polite

Correct Option is: Select


Your Option is: 2
Result: Wrong
Timetaken: 0.0 secs
Explanation: Nil

12.
Sixty years ago, on the evening of August 14, 1947, a few hours before Britain?s

Indian Empire was formally divided into the nation-states of India and Pakistan,

Lord Louis Mountbatten and his wife, Edwina, sat down in the vice regal mansion in

New Delhi to watch the latest Bob Hope movie, ?My Favorite Brunette.? Large parts

of the subcontinent were descending into chaos, as the implications of partitioning

the Indian Empire along religious lines became clear to the millions of Hindus,

Muslims, and Sikhs caught on the wrong side of the border. In the next few months,

some twelve million people would be uprooted and as many as a million murdered.

But on that night in mid-August the bloodbath?and the fuller consequences of hasty
imperial retreat?still lay in the future, and the Mountbatten probably felt they had

earned their evening?s entertainment. Mountbatten, the last viceroy of India, had

arrived in New Delhi in March, 1947, charged with an almost impossible task.

Irrevocably enfeebled by the Second World War, the British belatedly realized that

they had to leave the subcontinent, which had spiraled out of their control through

the nineteen-forties. But plans for brisk disengagement ignored messy realities on

the ground. Mountbatten had a clear remit to transfer power to the Indians within

fifteen months. Leaving India to God, or anarchy, as Mohandas Gandhi, the

foremost Indian leader, exhorted, wasn?t a political option, however tempting.

Mountbatten had to work hard to figure out how and to whom power was to be

transferred. The dominant political party, the Congress Party, took inspiration from

Gandhi in claiming to be a secular organization, representing all four hundred

million Indians. But many Muslim politicians saw it as a party of upper-caste Hindus

and demanded a separate homeland for their hundred million co-religionists, who

were intermingled with non-Muslim populations across the subcontinent?s villages,

towns, and cities. Eventually, as in Palestine, the British saw partition along

religious lines as the quickest way to the exit. But sectarian riots in Punjab and

Bengal dimmed hopes for a quick and dignified British withdrawal, and boded ill for

India?s assumption of power. Not surprisingly, there were some notable absences at

the Independence Day celebrations in New Delhi on August 15th. Gandhi,

denouncing freedom from imperial rule as a ?wooden loaf,? had remained in

Calcutta, trying, with the force of his moral authority, to stop Hindus and Muslims

from killing each other. His great rival Mohammed Ali Jinnah, who had fought

bitterly for a separate homeland for Indian Muslims, was in Karachi, trying to hold

together the precarious nation-state of Pakistan. Nevertheless, the significance of

the occasion was not lost on many. While the Mountbattens were sitting down to

their Bob Hope movie, India?s constituent assembly was convening in New Delhi.

The moment demanded grandiloquence, and Jawaharlal Nehru, Gandhi?s closest


disciple and soon to be India?s first Prime Minister, provided it. ?Long years ago, we

made a tryst with destiny,? he said. ?At the stroke of the midnight hour, while the

world sleeps, India will awaken to life and freedom. A moment comes, which comes

but rarely in history, when we step out from the old to the new, when an age ends,

and when the soul of a nation, long suppressed, finds utterance.? Posterity has

enshrined this speech, as Nehru clearly intended. But today his quaint phrase ?tryst

with destiny? resonates ominously, so enduring have been the political and

psychological scars of partition. The souls of the two new nation-states immediately

found utterance in brutal enmity. In Punjab, armed vigilante groups, organized

along religious lines and incited by local politicians, murdered countless people,

abducting and raping thousands of women. Soon, India and Pakistan were fighting

a war?the first of three?over the disputed territory of Kashmir. Gandhi, reduced to

despair by the seemingly endless cycle of retaliatory mass murders and

displacement, was shot dead in January, 1948, by a Hindu extremist who believed

that the father of the Indian nation was too soft on Muslims. Jinnah, racked with

tuberculosis and overwork, died a few months later, his dream of a secular Pakistan

apparently buried with him.

In the view of author what is the phrase 'tryst with destiny' symbolize today?

1) a celebration of Indian independence

2) An inspirational quote

3) A reminder of Gandhi's assassination

4) A symbol of ills of the partition

Correct Option is: 4


Your Option is: 4
Result: Correct
Timetaken: 0.0 secs
Explanation: Nil

The appropriate atmospheric conditions made it feasible for the astronomers to see
13.
the stars "AND THEY COULD EVEN DISTINGUISH THE SIZES"

1) And even distinguish the sizes

2) And they were even distinguish the sizes

3) And he could even distinguish the sizes

4) And even distinguishing the sizes

Correct
4
Option is:
Your Option
2
is:
Result: Wrong
Timetaken: 0.0 secs
The subject is Astronomers, he is incorrect. The sentence
Explanation: makes use of 'to see the stars' so 'even to distinguish' but not
given. Hence more appropriate is 4

In the following the questions choose the word which best expresses the meaning

14. of the given word.

AGITATED

1) Soothe

2) Refresh

3) Disturb

4) Suppress
Correct Option is: 3
Your Option is: 2
Result: Wrong
Timetaken: 0.0 secs
Explanation: restless, nervous hence, 3

In the following the questions choose the word which best expresses the meaning

15. of the given word.

ABSURD

1) Absent

2) Equitable

3) Present

4) Level

5) Inane

Correct Option
5
is:
Your Option is: 2
Result: Wrong
Timetaken: 0.0 secs
Absurd- illogical, unreasonable. Negative word is
Explanation:
Inane(silly)

The appropriate atmospheric conditions made it feasible for the astronomers to see
16.
the stars "AND THEY COULD EVEN DISTINGUISH THE SIZES"

1) And even distinguish the sizes

2) And they were even distinguish the sizes


3) And he could even distinguish the sizes

4) And even distinguishing the sizes

Correct Option is: 4


Your Option is: 1
Result: Wrong
Timetaken: 0.0 secs
Explanation: Nil

17.
Sixty years ago, on the evening of August 14, 1947, a few hours before Britain?s

Indian Empire was formally divided into the nation-states of India and Pakistan,

Lord Louis Mountbatten and his wife, Edwina, sat down in the vice regal mansion in

New Delhi to watch the latest Bob Hope movie, ?My Favorite Brunette.? Large parts

of the subcontinent were descending into chaos, as the implications of partitioning

the Indian Empire along religious lines became clear to the millions of Hindus,

Muslims, and Sikhs caught on the wrong side of the border. In the next few months,

some twelve million people would be uprooted and as many as a million murdered.

But on that night in mid-August the bloodbath?and the fuller consequences of hasty

imperial retreat?still lay in the future, and the Mountbatten probably felt they had

earned their evening?s entertainment. Mountbatten, the last viceroy of India, had

arrived in New Delhi in March, 1947, charged with an almost impossible task.

Irrevocably enfeebled by the Second World War, the British belatedly realized that

they had to leave the subcontinent, which had spiraled out of their control through

the nineteen-forties. But plans for brisk disengagement ignored messy realities on

the ground. Mountbatten had a clear remit to transfer power to the Indians within

fifteen months. Leaving India to God, or anarchy, as Mohandas Gandhi, the

foremost Indian leader, exhorted, wasn?t a political option, however tempting.

Mountbatten had to work hard to figure out how and to whom power was to be
transferred.

The dominant political party, the Congress Party, took inspiration from Gandhi in

claiming to be a secular organization, representing all four hundred million Indians.

But many Muslim politicians saw it as a party of upper-caste Hindus and demanded

a separate homeland for their hundred million co-religionists, who were

intermingled with non-Muslim populations across the subcontinent?s villages,

towns, and cities. Eventually, as in Palestine, the British saw partition along

religious lines as the quickest way to the exit.

But sectarian riots in Punjab and Bengal dimmed hopes for a quick and dignified

British withdrawal, and boded ill for India?s assumption of power. Not surprisingly,

there were some notable absences at the Independence Day celebrations in New

Delhi on August 15th. Gandhi, denouncing freedom from imperial rule as a ?wooden

loaf,? had remained in Calcutta, trying, with the force of his moral authority, to stop

Hindus and Muslims from killing each other. His great rival Mohammed Ali Jinnah,

who had fought bitterly for a separate homeland for Indian Muslims, was in Karachi,

trying to hold together the precarious nation-state of Pakistan.

Nevertheless, the significance of the occasion was not lost on many. While the

Mountbattens were sitting down to their Bob Hope movie, India?s constituent

assembly was convening in New Delhi. The moment demanded grandiloquence, and

Jawaharlal Nehru, Gandhi?s closest disciple and soon to be India?s first Prime

Minister, provided it. ?Long years ago, we made a tryst with destiny,? he said. ?At

the stroke of the midnight hour, while the world sleeps, India will awaken to life and

freedom. A moment comes, which comes but rarely in history, when we step out

from the old to the new, when an age ends, and when the soul of a nation, long

suppressed, finds utterance.?

Posterity has enshrined this speech, as Nehru clearly intended. But today his quaint

phrase ?tryst with destiny? resonates ominously, so enduring have been the political

and psychological scars of partition. The souls of the two new nation-states
immediately found utterance in brutal enmity. In Punjab, armed vigilante groups,

organized along religious lines and incited by local politicians, murdered countless

people, abducting and raping thousands of women. Soon, India and Pakistan were

fighting a war?the first of three?over the disputed territory of Kashmir. Gandhi,

reduced to despair by the seemingly endless cycle of retaliatory mass murders and

displacement, was shot dead in January, 1948, by a Hindu extremist who believed

that the father of the Indian nation was too soft on Muslims. Jinnah, racked with

tuberculosis and overwork, died a few months later, his dream of a secular Pakistan

apparently buried with him.

The author persists on talking about the ?bob Hope Movies? in the article. Why?

1) Because the movie was a classic of 1947

2) He thinks it caused the partition of the sub-continent

3) he uses it to show the apathy of the britishers towards the sub-continent

4) It was Mountbatten?s favourite movie

Correct Option is: 3


Your Option is: 1
Result: Wrong
Timetaken: 0.0 secs
Explanation: Nil

18.
In questions given below, a part of the sentence is italicised and underlined. Below

are given alternatives to the italicised part which may improve the sentence.

Choose the correct alternative. In case no improvement is needed, option 'D' is the

answer.

People working in high position in companies tend to shifting their work burden by
delegating tasks to their subordinates.

1) Tend for shifting their work

2) Tend to shift their work

3) Tend as to shifting their work

4) No improvement needed.

Correct Option
2
is:
Your Option is: 4
Result: Wrong
Timetaken: 0.0 secs
'to' follows 'tend', 1 incorrect, 'to' won't take with it 'ing', 3
Explanation:
incorrect. hence correct 2

In the following questions choose the word which is the exact OPPOSITE of the

19. given words.

AFFABLE

1) Rude

2) Ruddy

3) Needy

4) Useless

5) Conscious

Correct Option is: 1


Your Option is: 2
Result: Wrong
Timetaken: 0.0 secs
Explanation: Affable: Good natured Hence Rude. Ruddy: healthy

Select the correct option to fill in the blank.

As per the recent ethical rules laid down by National Institute of Health, diseases
20.
which cannot be treated_______would qualify for treatment involving human gene

manipulation.

1) Dangerous

2) Similar

3) Alternatively

4) Uncommon

Correct Option
3
is:
Your Option is: 1
Result: Wrong
Timetaken: 0.0 secs
Treated- verb can be modified only by an adverb
Explanation:
alternatively

21.
Sixty years ago, on the evening of August 14, 1947, a few hours before Britain?s

Indian Empire was formally divided into the nation-states of India and Pakistan,

Lord Louis Mountbatten and his wife, Edwina, sat down in the vice regal mansion in

New Delhi to watch the latest Bob Hope movie, ?My Favorite Brunette.? Large parts

of the subcontinent were descending into chaos, as the implications of partitioning

the Indian Empire along religious lines became clear to the millions of Hindus,

Muslims, and Sikhs caught on the wrong side of the border. In the next few months,

some twelve million people would be uprooted and as many as a million murdered.
But on that night in mid-August the bloodbath?and the fuller consequences of hasty

imperial retreat?still lay in the future, and the Mountbatten probably felt they had

earned their evening?s entertainment. Mountbatten, the last viceroy of India, had

arrived in New Delhi in March, 1947, charged with an almost impossible task.

Irrevocably enfeebled by the Second World War, the British belatedly realized that

they had to leave the subcontinent, which had spiraled out of their control through

the nineteen-forties. But plans for brisk disengagement ignored messy realities on

the ground. Mountbatten had a clear remit to transfer power to the Indians within

fifteen months. Leaving India to God, or anarchy, as Mohandas Gandhi, the

foremost Indian leader, exhorted, wasn?t a political option, however tempting.

Mountbatten had to work hard to figure out how and to whom power was to be

transferred.

The dominant political party, the Congress Party, took inspiration from Gandhi in

claiming to be a secular organization, representing all four hundred million Indians.

But many Muslim politicians saw it as a party of upper-caste Hindus and demanded

a separate homeland for their hundred million co-religionists, who were

intermingled with non-Muslim populations across the subcontinent?s villages,

towns, and cities. Eventually, as in Palestine, the British saw partition along

religious lines as the quickest way to the exit.

But sectarian riots in Punjab and Bengal dimmed hopes for a quick and dignified

British withdrawal, and boded ill for India?s assumption of power. Not surprisingly,

there were some notable absences at the Independence Day celebrations in New

Delhi on August 15th. Gandhi, denouncing freedom from imperial rule as a ?wooden

loaf,? had remained in Calcutta, trying, with the force of his moral authority, to stop

Hindus and Muslims from killing each other. His great rival Mohammed Ali Jinnah,

who had fought bitterly for a separate homeland for Indian Muslims, was in Karachi,

trying to hold together the precarious nation-state of Pakistan.

Nevertheless, the significance of the occasion was not lost on many. While the
Mountbattens were sitting down to their Bob Hope movie, India?s constituent

assembly was convening in New Delhi. The moment demanded grandiloquence, and

Jawaharlal Nehru, Gandhi?s closest disciple and soon to be India?s first Prime

Minister, provided it. ?Long years ago, we made a tryst with destiny,? he said. ?At

the stroke of the midnight hour, while the world sleeps, India will awaken to life and

freedom. A moment comes, which comes but rarely in history, when we step out

from the old to the new, when an age ends, and when the soul of a nation, long

suppressed, finds utterance.?

Posterity has enshrined this speech, as Nehru clearly intended. But today his quaint

phrase ?tryst with destiny? resonates ominously, so enduring have been the political

and psychological scars of partition. The souls of the two new nation-states

immediately found utterance in brutal enmity. In Punjab, armed vigilante groups,

organized along religious lines and incited by local politicians, murdered countless

people, abducting and raping thousands of women. Soon, India and Pakistan were

fighting a war?the first of three?over the disputed territory of Kashmir. Gandhi,

reduced to despair by the seemingly endless cycle of retaliatory mass murders and

displacement, was shot dead in January, 1948, by a Hindu extremist who believed

that the father of the Indian nation was too soft on Muslims. Jinnah, racked with

tuberculosis and overwork, died a few months later, his dream of a secular Pakistan

apparently buried with him.

What is the author most likely agree to?

1) People generally do not try to hide money taken as bribe

2) People hide money taken as bribe primarily to avoid detection

3) People hide money taken as bribe from view even if detection possibility is low

4) None of these
Correct Option is: 2
Your Option is: 1
Result: Wrong
Timetaken: 0.0 secs
Explanation: Nil

________being poor,Kaveri still dresses more appropriately than most of her group
22.
mates.

1) Despite

2) Although

3) Since

4) However

Correct
1
Option is:
Your Option
4
is:
Result: Wrong
Timetaken: 0.0 secs
Connectivity used to show contrast: Despite, inspite of
Inspite of the fact that + Subject + Verb- Although, Though
Explanation:
Even then + Subject + Verb - However, Nevertheless Hence,
1

In the following the questions choose the word which best expresses the meaning

23. of the given word.

INFER

1) Deadly

2) Deduce
3) Interfere

4) Envious

Correct Option is: 2


Your Option is: 1
Result: Wrong
Timetaken: 0.0 secs
Explanation: conclude or deduce

24.
Give people power and discretion, and whether they are grand viziers or border

guards, some will use their position to enrich themselves. The problem can be big

enough to hold back a country's development. One study has shown that bribes

account for 8% of the total cost of running a business in Uganda. Another found

that corruption boosted the price of hospital supplies in Buenos Aires by 15%. Paul

Wolfowitz, the head of the World Bank, is devoting special efforts during his

presidency there to a drive against corruption. GIVE people power and discretion,

and whether they are grand viziers or border guards, some will use their position to

enrich themselves. The problem can be big enough to hold back a country's

development. One study has shown that bribes account for 8% of the total cost of

running a business in Uganda. Another found that corruption boosted the price of

hospital supplies in Buenos Aires by 15%. Paul Wolfowitz, the head of the World

Bank, is devoting special efforts during his presidency there to a drive against

corruption.

For most people in the world, though, the worry is not that corruption may slow

down their country's GDP growth. It is that their daily lives are pervaded by endless

hassles, big and small. And for all the evidence that some cultures suffer endemic

corruption while others are relatively clean, attitudes towards corruption, and even
the language describing bribery, is remarkably similar around the world.

In a testament to most people's basic decency, bribe-takers and bribe-payers have

developed an elaborate theatre of dissimulation. This is not just to avoid detection.

Even in countries where corruption is so common as to be unremarkable and

unprosecutable?and even when the transaction happens far from snooping eyes?a

bribe is almost always dressed up as some other kind of exchange. Though most of

the world is plagued by corruption, even serial offenders try to conceal it.

One manifestation of this is linguistic. Surprisingly few people say: ?You are going

to have to pay me if you want to get that done.? Instead, they use a wide variety of

euphemisms. One type is quasi-official terminology. The first bribe paid by your

correspondent, in Ukraine in 1998, went to two policemen so they would let him

board a train leaving the country. On the train into Ukraine, the customs officer had

absconded with a form that is needed again later to leave the country. The

policemen at the station kindly explained that there was a shtraf, a ?fine? that could

be paid instead of producing the document. The policemen let him off with the

minimum shtraf of 50 hryvnia ($25).

A second type of euphemism dresses up a dodgy payment as a friendly favour done

by the bribe-payer. There is plenty of creative scope. Nigerian policemen are known

to ask for ?a little something for the weekend?. A North African term is ?un petit

cadeau?, a little gift. Mexican traffic police will suggest that you buy them a

refresco, a soft drink, as will Angolan and Mozambican petty officials, who call it a

gazoso in Portuguese. A businessman in Iraq told Reuters that although corruption

there is quite overt, officials still insist on being given a ?good coffee?

Double meaning can help soothe the awkwardness of bribe-paying. Baksheesh,

originally a Persian word now found in many countries of the Middle East, can mean

?tip?, ?alms? and ?bribe?. Swahili-speakers can take advantage of another

ambiguous term. In Kenya a machine-gun-wielding guard suggested to a terrified

Canadian aid worker: ?Perhaps you would like to discuss this over tea?? The young
Canadian was relieved: the difficulty could be resolved with some chai, which

means both ?tea? and ?bribe?.

Along with the obscurantist language, bribe-taking culture around the world often

involves the avoidance of physically handing the money from one person to another.

One obvious reason is to avoid detection, which is why bribes are known as ?

envelopes? in countries from China to Greece. But avoidance of a direct hand-over

is common even where there is no chance of detection. There will always be some

officials who will take money right from a bribe-payer's hands, but most seem to

prefer to find some way to hide the money from view. A bribe to a border guard

may be folded into a passport. A sweetener to a traffic cop is often placed in the

ticket-book that is handed to the driver. Parag Khanna, who is writing a book about

countries on the edge of the rich world that are trying to get rich themselves,

describes a bribe-taker he spotted in Georgia who he was sure was a rookie. Why?

The scrawny young soldier, forgoing any subtleties, merely rubbed his fingers

together in an age-old gesture. Rich Westerners may not think of their societies as

plagued by corruption. But the definition of bribery clearly differs from person to

person. A New Yorker might pity the third-world businessman who must pay bribes

just to keep his shop open. But the same New Yorker would not think twice about

slipping the maitre d' $50 to sneak into a nice restaurant without a reservation.

Poor people the world over are most infuriated by the casual corruption of the elites

rather than by the underpaid, tip seeking soldier or functionary.

What is the best represent that author toward the rich people in the west?

1) Appreciate

2) Mildly critical

3) Heavily critical

4) Mildly appreciate
Correct Option is: Select
Your Option is: 2
Result: Wrong
Timetaken: 0.0 secs
Explanation: Nil

Read the each sentence to find out whether there is any grammatical error in it.

The error, if any will be in one part of the sentence. The letter of that part is the
25.
answer. If there is no error, the answer is 'D'. (Ignore the errors of punctuation, if

any).

1) Guilt and self pleasure are

2) two most strong drivers

3) of any human act

4) No error

Correct Option is: 2


Your Option is: 1
Result: Wrong
Timetaken: 0.0 secs
Explanation: Strong superlative is strongest, strike most

Find the synonyms for the below word:


1.
Misrepresent

1) Represent

2) cover

3) misinterpret
4) mislead

Correct Option
3
is:
Your Option is: 2
Result: Wrong
Timetaken: 0.0 secs
meaning of misrepresent is "give a false "misinterpret"is
Explanation:
same meaning so option 3 is correct

2.
Since the late 1970s when the technology for sex determination first came into

being, sex selective abortion has unleashed a saga of horror. Experts are calling it ?

Sanitised Barbarism?. Demographic trends indicate the country is fast heading

towards a million foetuses aborted each year.

Although foetal sex determination and sex selection is an offence in India, the

practice is rampant. Private clinics with ultrasound machines are doing brisk

business. Everywhere, people are paying to know the sex of the unborn child, and

paying more to abort the female child. The technology has reached even remote

places through mobile clinics. Dr. Puneet Bedi obstetrician and specialist in foetal

medicine, says these days he hardly sees a family with two daughters. People are

getting the sex determination even for the first child, he says.

In the 1991 census showed that two districts had a child sex ratio (number of girls

per thousand boys) less than 850; by 2001 it was 51 districts. Child rights activist

Dr. Sabu George says foeticide is the most extreme form of violence against

women. ?Today a girl is several times more likely to be eliminated before birth than

die of various causes during the first year. Nature intended the womb to be a safe

space. Today Doctors have made it the most unsafe space for the female child, ?he

says?. He believes that Doctors must be held responsible ? ?They have aggressively
promoted the misuse of technology and legitimised foeticide.?

Akhila Sivadas, Centre for Advocacy and Research, Delhi, feels that the PCPNDT Act

(Pre-Conception and Pre-Natal Diagnostic Techniques ? Regulation and Prevention

of Misuse) is very well conceived and easy to use. The need of the hour is the legal

literacy to ensure the law is implemented. ?The demand and supply debate has

been going on for some time. Doctors say there is a social demand and they are

fulfilling it. They argue that social attitudes must change. However, in this case

supply fuels demand. Technology will have to be regulated. Technology in the hands

of greedy, vested interests cannot be neutral. There is a law to prevent misuse and

we must be able to use it,? she says.

On the Demands side, experts such as Dr. Agnihotri argue that women?s

participation in workforce, having disposable incomes and making a contribution to

the larger society will make a difference to how women are seen. Youth icons and

role models such as Sania Mirza are making an impact, he says.

Others feel there needs to be widespread visible contempt and anger in society

against this ?Genocide?- ?the kind we saw against the Nithari killings,? says Dr

Bedi. ?Today nobody can say that female foeticide is not their problem.? Time we all

did our bit to help save the girl child. Time?s running out.

Select the correct answer option based on the based on the passage.

What does the word sanitised imply in the first paragraph of the passage?

1) Unforgivable

2) Legitimate

3) Free from dirt

4) None of these

Correct Option is: 2


Your Option is: 2
Result: Correct
Timetaken: 0.0 secs
Explanation: from the para 1,sanitised is legitimate

Find the synonyms for the below word:


3.
Absurd

1) Crazy

2) logical

3) moronic

4) silly

Correct Option
1
is:
Your Option is: 1
Result: Correct
Timetaken: 0.0 secs
meaning is "unreasonable,illogical " so option 1 is well
Explanation:
suit for this

4.
Read the passage and answer the given question below: Since the late 1970s when

the technology of sex determination first came into being, sex selective abortion

has unleashed a saga of horror. Experts are calling it ?sanitized barbarism?.

Demographic trends indicate the country is fast heading towards a million female

foetuses aborted each year.

Although foetal sex determination and sex selection is a criminal offence in India,

the practice is rampant. Private clinics with ultrasound machines are doing brisk

business. Everywhere, people are paying to know the sex of an unborn child and
paying more to abort the female child. The technology has even reached remote

areas through mobile clinics. Dr. Puneet Bedi, obstetrician and specialist in foetal

medicine, says these days he hardly sees a family with two daughters. People are

getting sex determination done even for the first child, he says.

If the 1991 Census showed that two districts had a child sex ratio (number of girls

per thousand boys) less that 850; by 2001 it was 51 districts. Child rights activist

Dr. Sabu George says foeticide is the most extreme form of violence against

women. ?Today a girl is several times more likely to be eliminated before a birth

than die of various causes in the first year. Nature intended the womb to be safe

space. Today, doctors have made it the most unsafe space for the female child?, he

says. He believes that doctors must be held responsible- ?they have aggressively

promoted the misuse of technology and legitimized foeticide?.

What is Akhila Sivadas's opinion on the PCPNDT act?

1) The act is inconsistent

2) The act needs reform

3) The act encourages demand for foeticide

4) The act is sound, but needs enforcement

Correct Option
1
is:
Your Option is: 4
Result: Wrong
Timetaken: 0.0 secs
ACCORDING TO THIS PASSAGE OPTION 1 IS
Explanation:
OPINION OF SIVSDAS ABOUT THE PSPNDT

5.
Find the antonyms for the below word:
JAUNTY

1) exuberant

2) vigorous

3) lethargic

4) blithe

Correct Option is: 3


Your Option is: 2
Result: Wrong
Timetaken: 0.0 secs
Explanation: antonym for jaunty is lethargic

6.
Read the passage. Then answer the questions below.

The Battle of Chancellorsville, one of the most famous battles of the Civil War, took

place in Virginia in the spring of 1863. For months, the two armies had been staked

out on opposite banks of a narrow river. The Confederate troops were led by

perhaps the most revered military tactician in American history, General Robert E.

Lee. The Union soldiers were led by "Fighting" Joe Hooker.

In appearance, personality, and lifestyle, these men were nearly perfect opposites.

Lee, an older man in poor health with a gray beard, had a somber, measured

demeanor. Hooker was a blond, strapping young man whose vanity over his

appearance was but one aspect of his egotism. Whereas Lee was devout and

principled, Hooker was known for his rollicking enjoyment of both women and

whiskey.

Despite the fact that the Confederacy had won the last four major battles and the

Union soldiers were famished, exhausted, and demoralized, Hooker proclaimed, "My

plans are perfect. And when I start to carry them out, may God have mercy on
Bobby Lee, for I shall have none." Why, aside from a propensity for narcissism, was

Hooker so confident?

Hooker had used spies, analysts, and even hot air balloons to compile a vast

amount of intelligence about Lee?s army. He had discerned, for example, that Lee

had only 61,000 men to Hooker?s own 134,000. Buoyed by his superior numbers,

Hooker covertly moved 70,000 of his men fifteen miles up and across the river, and

then ordered them to sneak back down to position themselves behind Lee?s army.

In effect, Hooker had cut off the Confederate soldiers in front and behind. They

were trapped. Satisfied with his advantage, Hooker became convinced that Lee?s

only option was to retreat to Richmond, thus assuring a Union victory.

Yet Lee, despite his disadvantages of both numbers and position, did not retreat.

Instead, he moved his troops into position to attack. Union soldiers who tried to

warn Hooker that Lee was on the offensive were dismissed as cowards. Having

become convinced that Lee had no choice but to retreat, Hooker began to ignore

reality. When Lee?s army attacked the Union soldiers at 5:00 p.m., they were

eating supper, completely unprepared for battle. They abandoned their rifles and

fled as Lee?s troops came shrieking out of the brush, bayonets drawn. Against all

odds, Lee won the Battle of Chancellorsville, and Hooker?s forces withdrew in

defeat.

As used in paragraph 4, buoyed most nearly means

1) strengthened

2) anchored

3) floated

4) sharpened

5) heartened

Correct Option is: 5


Your Option is: 4
Result: Wrong
Timetaken: 0.0 secs
IN THAT PARA BUOYED MEAN THAT
Explanation:
HEARTENED

Read the passage and answer the given question below: Since the late 1970s when

the technology of sex determination first came into being, sex selective abortion

has unleashed a saga of horror. Experts are calling it ?sanitized barbarism?.

Demographic trends indicate the country is fast heading towards a million female

foetuses aborted each year.

Although foetal sex determination and sex selection is a criminal offence in India,

the practice is rampant. Private clinics with ultrasound machines are doing brisk

business. Everywhere, people are paying to know the sex of an unborn child and

paying more to abort the female child. The technology has even reached remote

areas through mobile clinics. Dr. Puneet Bedi, obstetrician and specialist in foetal

medicine, says these days he hardly sees a family with two daughters. People are
7.
getting sex determination done even for the first child, he says.

If the 1991 Census showed that two districts had a child sex ratio (number of girls

per thousand boys) less that 850; by 2001 it was 51 districts. Child rights activist

Dr. Sabu George says foeticide is the most extreme form of violence against

women. ?Today a girl is several times more likely to be eliminated before a birth

than die of various causes in the first year. Nature intended the womb to be safe

space. Today, doctors have made it the most unsafe space for the female child?, he

says. He believes that doctors must be held responsible- ?they have aggressively

promoted the misuse of technology and legitimized foeticide?.

What is the solution to the problem of female foeticide as envisioned by Dr. Bedi?

1) Effective use of law


2) Mass public outrage

3) Comparison with Nithari killing

4) Contempt towards doctors

Correct Option is: 2


Your Option is: 2
Result: Correct
Timetaken: 0.0 secs
Explanation: according to second para option 2 is correct

8.
Read the passage. Then answer the questions below.

The Battle of Chancellorsville, one of the most famous battles of the Civil War, took

place in Virginia in the spring of 1863. For months, the two armies had been staked

out on opposite banks of a narrow river. The Confederate troops were led by

perhaps the most revered military tactician in American history, General Robert E.

Lee. The Union soldiers were led by "Fighting" Joe Hooker.

In appearance, personality, and lifestyle, these men were nearly perfect opposites.

Lee, an older man in poor health with a gray beard, had a somber, measured

demeanor. Hooker was a blond, strapping young man whose vanity over his

appearance was but one aspect of his egotism. Whereas Lee was devout and

principled, Hooker was known for his rollicking enjoyment of both women and

whiskey.

Despite the fact that the Confederacy had won the last four major battles and the

Union soldiers were famished, exhausted, and demoralized, Hooker proclaimed, "My

plans are perfect. And when I start to carry them out, may God have mercy on

Bobby Lee, for I shall have none." Why, aside from a propensity for narcissism, was

Hooker so confident?
Hooker had used spies, analysts, and even hot air balloons to compile a vast

amount of intelligence about Lee?s army. He had discerned, for example, that Lee

had only 61,000 men to Hooker?s own 134,000. Buoyed by his superior numbers,

Hooker covertly moved 70,000 of his men fifteen miles up and across the river, and

then ordered them to sneak back down to position themselves behind Lee?s army.

In effect, Hooker had cut off the Confederate soldiers in front and behind. They

were trapped. Satisfied with his advantage, Hooker became convinced that Lee?s

only option was to retreat to Richmond, thus assuring a Union victory.

Yet Lee, despite his disadvantages of both numbers and position, did not retreat.

Instead, he moved his troops into position to attack. Union soldiers who tried to

warn Hooker that Lee was on the offensive were dismissed as cowards. Having

become convinced that Lee had no choice but to retreat, Hooker began to ignore

reality. When Lee?s army attacked the Union soldiers at 5:00 p.m., they were

eating supper, completely unprepared for battle. They abandoned their rifles and

fled as Lee?s troops came shrieking out of the brush, bayonets drawn. Against all

odds, Lee won the Battle of Chancellorsville, and Hooker?s forces withdrew in

defeat.

In paragraph 3, the author quotes Hooker as saying, ?My plans are perfect. And

when I start to carry them out, may God have mercy on Bobby Lee, for I shall have

none. The author most likely includes this quote in order to

1) demonstrate Hooker?s belief in his own infallibility

2) provide an example of the way language has changed since 1863

3) reveal that Hooker was a deeply religious man in spite of his lifestyle

4) foreshadow Hooker?s defeat at the hands of Lee?s army

5) portray Hooker as a merciless general who was compelled by his hated of Lee
Correct
1
Option is:
Your Option
4
is:
Result: Wrong
Timetaken: 0.0 secs
FROM THE ABOVE PASSAGE ,THIS QUOTES ORDER
Explanation: TO BELIEF OF OWN INFALLIBILITY OF HOOKER SO
OPTION 1 IS CORRECT

9.
Read the passage and answer the question:

Democratic institutions are devices for reconciling social order with individual

freedom and initiative, and for making the immediate power of a country's rulers

subject to the ultimate power of the ruled. The fact that, in Western Europe 5 and

America, these devices have worked, all things considered, not too badly is proof

enough that the eighteenth century optimists were not entirely wrong. Given a fair

chance, I repeat; for the fair chance is an indispensable prerequisite No people that

pass abruptly from a state of subservience 10 under the rule of a despot to the

completely unfamiliar state of political independence can be said to have a fair

chance of being able to govern itself democratically. Liberalism flourishes in an

atmosphere of prosperity and declines as declining prosperity makes it necessary

for the government 15 to intervene ever more frequently and drastically in the

affairs of its subjects. Over-population and over-organization are two conditions

which ... deprive a society of a fair chance of making democratic institutions work

effectively. We see, then, that there are certain 20 historical, economic,

demographic and technological conditions which make it very hard for Jefferson's

rational animals, endowed by nature with inalienable rights and an innate sense of

justice, to exercise their reason, claim their rights and act justly within a

democratically organized 25 society. We in the West have been supremely fortunate

in having been given a fair chance of making the great experiment in self-
government. Unfortunately, it now looks as though, owing to recent changes in our

circumstances, this infinitely precious fair chance were being, little by little, taken

30 away from us.

The author's primary purpose is apparently to

1) Explain a requirement and introduce a warning about that requirement

2) Argue for the limitation of a certain form of government

3) Define the conditions for social order

4) Advocate liberalism in government of a certain era

5) Credit certain thinkers with foresight

Correct Option
1
is:
Your Option is: 3
Result: Wrong
Timetaken: 0.0 secs
PRIMARY PURPOSE IS APPARENTLY TO
Explanation:
RECUIREMENT SO OPTION 1 IS CORRECT

10.
Read the passage. Then answer the questions below.

The Battle of Chancellorsville, one of the most famous battles of the Civil War, took

place in Virginia in the spring of 1863. For months, the two armies had been staked

out on opposite banks of a narrow river. The Confederate troops were led by

perhaps the most revered military tactician in American history, General Robert E.

Lee. The Union soldiers were led by "Fighting" Joe Hooker.

In appearance, personality, and lifestyle, these men were nearly perfect opposites.

Lee, an older man in poor health with a gray beard, had a somber, measured

demeanor. Hooker was a blond, strapping young man whose vanity over his

appearance was but one aspect of his egotism. Whereas Lee was devout and
principled, Hooker was known for his rollicking enjoyment of both women and

whiskey.

Despite the fact that the Confederacy had won the last four major battles and the

Union soldiers were famished, exhausted, and demoralized, Hooker proclaimed, "My

plans are perfect. And when I start to carry them out, may God have mercy on

Bobby Lee, for I shall have none." Why, aside from a propensity for narcissism, was

Hooker so confident?

Hooker had used spies, analysts, and even hot air balloons to compile a vast

amount of intelligence about Lee?s army. He had discerned, for example, that Lee

had only 61,000 men to Hooker?s own 134,000. Buoyed by his superior numbers,

Hooker covertly moved 70,000 of his men fifteen miles up and across the river, and

then ordered them to sneak back down to position themselves behind Lee?s army.

In effect, Hooker had cut off the Confederate soldiers in front and behind. They

were trapped. Satisfied with his advantage, Hooker became convinced that Lee?s

only option was to retreat to Richmond, thus assuring a Union victory.

Yet Lee, despite his disadvantages of both numbers and position, did not retreat.

Instead, he moved his troops into position to attack. Union soldiers who tried to

warn Hooker that Lee was on the offensive were dismissed as cowards. Having

become convinced that Lee had no choice but to retreat, Hooker began to ignore

reality. When Lee?s army attacked the Union soldiers at 5:00 p.m., they were

eating supper, completely unprepared for battle. They abandoned their rifles and

fled as Lee?s troops came shrieking out of the brush, bayonets drawn. Against all

odds, Lee won the Battle of Chancellorsville, and Hooker?s forces withdrew in

defeat.

The contrast drawn between Lee and Hooker in paragraph 2 is intended to

1) showcase the different backgrounds and personal histories of these two enemy
soldiers

2) provide support for the idea that Lee was a more virtuous person than Hooker,
and therefore a better military commander

3) prove that two men with very different values could end up in similar positions of
power

4) suggest that if Hooker had been more devout and principled, he might not have
been outwitted by Lee

5) imply that these men fundamentally differed in their approaches to nearly


everything, including battle

Correct
5
Option is:
Your Option
2
is:
Result: Wrong
Timetaken: 0.0 secs
FROM PARA 2,LE AND HOOKER ,IMPLY THAT THESE
Explanation: MEN FUNDAMENTALLY DIFFERED IN THERE
APPROACH SO OPTION 5 IS WELL APT

11.
Since the late 1970s when the technology for sex determination first came into

being, sex selective abortion has unleashed a saga of horror. Experts are calling it ?

Sanitised Barbarism?. Demographic trends indicate the country is fast heading

towards a million foetuses aborted each year.

Although foetal sex determination and sex selection is an offence in India, the

practice is rampant. Private clinics with ultrasound machines are doing brisk

business. Everywhere, people are paying to know the sex of the unborn child, and

paying more to abort the female child. The technology has reached even remote

places through mobile clinics. Dr. Puneet Bedi obstetrician and specialist in foetal

medicine, says these days he hardly sees a family with two daughters. People are

getting the sex determination even for the first child, he says.

In the 1991 census showed that two districts had a child sex ratio (number of girls
per thousand boys) less than 850; by 2001 it was 51 districts. Child rights activist

Dr. Sabu George says foeticide is the most extreme form of violence against

women. ?Today a girl is several times more likely to be eliminated before birth than

die of various causes during the first year. Nature intended the womb to be a safe

space. Today Doctors have made it the most unsafe space for the female child, ?he

says?. He believes that Doctors must be held responsible ? ?They have aggressively

promoted the misuse of technology and legitimised foeticide.?

Akhila Sivadas, Centre for Advocacy and Research, Delhi, feels that the PCPNDT Act

(Pre-Conception and Pre-Natal Diagnostic Techniques ? Regulation and Prevention

of Misuse) is very well conceived and easy to use. The need of the hour is the legal

literacy to ensure the law is implemented. ?The demand and supply debate has

been going on for some time. Doctors say there is a social demand and they are

fulfilling it. They argue that social attitudes must change. However, in this case

supply fuels demand. Technology will have to be regulated. Technology in the hands

of greedy, vested interests cannot be neutral. There is a law to prevent misuse and

we must be able to use it,? she says.

On the ?Demand? side, experts such as Dr. Agnihotri argue that women?s

participation in workforce, having disposable incomes and making a contribution to

the larger society will make a difference to how women are seen. Youth icons and

role models such as Sania Mirza are making an impact, he says.

Others feel there needs to be widespread visible contempt and anger in society

against this ?Genocide?- ?the kind we saw against the Nithari killings,? says Dr

Bedi. ?Today nobody can say that female foeticide is not their problem.? Time we all

did our bit to help save the girl child. Time?s running out.

Select the correct answer option based on the passage

Which demand the author refers to in paragraph 5?

1) Demand for principled doctors


2) Demand for high income jobs for women

3) Demand for youth icons

4) Demand for sex determination and abortion

Correct Option
2
is:
Your Option is: 4
Result: Wrong
Timetaken: 0.0 secs
from the para no 5 auther refers high income jobs for
Explanation:
women.

12.
Since the late 1970s when the technology for sex determination first came into

being, sex selective abortion has unleashed a saga of horror. Experts are calling it ?

Sanitised Barbarism?. Demographic trends indicate the country is fast heading

towards a million foetuses aborted each year.

Although foetal sex determination and sex selection is an offence in India, the

practice is rampant. Private clinics with ultrasound machines are doing brisk

business. Everywhere, people are paying to know the sex of the unborn child, and

paying more to abort the female child. The technology has reached even remote

places through mobile clinics. Dr. Puneet Bedi obstetrician and specialist in foetal

medicine, says these days he hardly sees a family with two daughters. People are

getting the sex determination even for the first child, he says.

In the 1991 census showed that two districts had a child sex ratio (number of girls

per thousand boys) less than 850; by 2001 it was 51 districts. Child rights activist

Dr. Sabu George says foeticide is the most extreme form of violence against

women. ?Today a girl is several times more likely to be eliminated before birth than

die of various causes during the first year. Nature intended the womb to be a safe

space. Today Doctors have made it the most unsafe space for the female child, ?he
says?. He believes that Doctors must be held responsible ? ?They have aggressively

promoted the misuse of technology and legitimised foeticide.?

Akhila Sivadas, Centre for Advocacy and Research, Delhi, feels that the PCPNDT Act

(Pre-Conception and Pre-Natal Diagnostic Techniques ? Regulation and Prevention

of Misuse) is very well conceived and easy to use. The need of the hour is the legal

literacy to ensure the law is implemented. ?The demand and supply debate has

been going on for some time. Doctors say there is a social demand and they are

fulfilling it. They argue that social attitudes must change. However, in this case

supply fuels demand. Technology will have to be regulated. Technology in the hands

of greedy, vested interests cannot be neutral. There is a law to prevent misuse and

we must be able to use it,? she says.

On the ?Demand? side, experts such as Dr. Agnihotri argue that women?s

participation in workforce, having disposable incomes and making a contribution to

the larger society will make a difference to how women are seen. Youth icons and

role models such as Sania Mirza are making an impact, he says.

Others feel there needs to be widespread visible contempt and anger in society

against this ?Genocide?- ?the kind we saw against the Nithari killings,? says Dr

Bedi. ?Today nobody can say that female foeticide is not their problem.? Time we all

did our bit to help save the girl child. Time?s running out.

Select the correct answer option based on the passage.

Which of the two people mentioned in the passage suggest similar solution to the

problem?

1) Dr. Agnihotri and Dr. George

2) Dr. Bedi and Dr. Agnihotri

3) Dr. George and Dr. Bedi

4) Dr. George and Miss. Sivadas


Correct Option
4
is:
Your Option is: 3
Result: Wrong
Timetaken: 0.0 secs
from para 1 ND PARA 2 WE CAN FINALIZE BOTH
Explanation:
GEORGE AND SIVADAS ARE SAME SOLUTION

13.
Read the passage and answer the given question below: Since the late 1970s when

the technology of sex determination first came into being, sex selective abortion

has unleashed a saga of horror. Experts are calling it ?sanitized barbarism?.

Demographic trends indicate the country is fast heading towards a million female

foetuses aborted each year.

Although foetal sex determination and sex selection is a criminal offence in India,

the practice is rampant. Private clinics with ultrasound machines are doing brisk

business. Everywhere, people are paying to know the sex of an unborn child and

paying more to abort the female child. The technology has even reached remote

areas through mobile clinics. Dr. Puneet Bedi, obstetrician and specialist in foetal

medicine, says these days he hardly sees a family with two daughters. People are

getting sex determination done even for the first child, he says.

If the 1991 Census showed that two districts had a child sex ratio (number of girls

per thousand boys) less that 850; by 2001 it was 51 districts. Child rights activist

Dr. Sabu George says foeticide is the most extreme form of violence against

women. ?Today a girl is several times more likely to be eliminated before a birth

than die of various causes in the first year. Nature intended the womb to be safe

space. Today, doctors have made it the most unsafe space for the female child?, he

says. He believes that doctors must be held responsible- ?they have aggressively

promoted the misuse of technology and legitimized foeticide?.

1) Which of the following will Dr. George agree to?


1) The girl child is as safe in the mother's womb as after birth

2) The girl child is safe in the mother's womb in comparison to after birth

3) the act encouraThe girl child is safer after birth as compared to mother's wombes
demand for foeticide

4) None of these

Correct Option
4
is:
Your Option is: 1
Result: Wrong
Timetaken: 0.0 secs
according to compression of this passage and options first
Explanation:
all the 3 are not suit so option 4 is correct one

Find the antonyms for the below word:


14.
Exemplify

1) depict

2) represent

3) instance

4) conceal

Correct Option
4
is:
Your Option is: 4
Result: Correct
Timetaken: 0.0 secs
meaning exemplify is serve as an example opponent
Explanation:
meaning for this is conceal

Read the passage and answer the given question below: Since the late 1970s when

the technology of sex determination first came into being, sex selective abortion

has unleashed a saga of horror. Experts are calling it ?sanitized barbarism?.

Demographic trends indicate the country is fast heading towards a million female

foetuses aborted each year.

Although foetal sex determination and sex selection is a criminal offence in India,

the practice is rampant. Private clinics with ultrasound machines are doing brisk

business. Everywhere, people are paying to know the sex of an unborn child and

paying more to abort the female child. The technology has even reached remote

areas through mobile clinics. Dr. Puneet Bedi, obstetrician and specialist in foetal

medicine, says these days he hardly sees a family with two daughters. People are
15.
getting sex determination done even for the first child, he says.

If the 1991 Census showed that two districts had a child sex ratio (number of girls

per thousand boys) less that 850; by 2001 it was 51 districts. Child rights activist

Dr. Sabu George says foeticide is the most extreme form of violence against

women. ?Today a girl is several times more likely to be eliminated before a birth

than die of various causes in the first year. Nature intended the womb to be safe

space. Today, doctors have made it the most unsafe space for the female child?, he

says. He believes that doctors must be held responsible- ?they have aggressively

promoted the misuse of technology and legitimized foeticide?.

What is the topic of the passage?

1) Factual

2) Biased

3) Aggressive
4) Sad

Correct Option
2
is:
Your Option is: 2
Result: Correct
Timetaken: 0.0 secs
from the above passage we can say factual,aggressive,sad
Explanation:
is not a suitable topic BIASED IS SUIT WELL

Find the synonyms for the below word:


16.
Celestial

1) hellish

2) Spiritual

3) infrenal

4) mortal

Correct Option is: 2


Your Option is: 4
Result: Wrong
Timetaken: 0.0 secs
Explanation: "relating to sky" equal meaning is spiritual

Find the synonyms for the below word:


17.
Conceited

1) vain

2) shy
3) humble

4) meek

Correct Option
1
is:
Your Option is: Not Attempted
Result: Not Attempted
Timetaken: 0.0 secs
excessively proud of one self him so equal meaning is
Explanation:
vain in option1

18.
Read the passage. Then answer the questions below.

The Battle of Chancellorsville, one of the most famous battles of the Civil War, took

place in Virginia in the spring of 1863. For months, the two armies had been staked

out on opposite banks of a narrow river. The Confederate troops were led by

perhaps the most revered military tactician in American history, General Robert E.

Lee. The Union soldiers were led by "Fighting" Joe Hooker.

In appearance, personality, and lifestyle, these men were nearly perfect opposites.

Lee, an older man in poor health with a gray beard, had a somber, measured

demeanor. Hooker was a blond, strapping young man whose vanity over his

appearance was but one aspect of his egotism. Whereas Lee was devout and

principled, Hooker was known for his rollicking enjoyment of both women and

whiskey.

Despite the fact that the Confederacy had won the last four major battles and the

Union soldiers were famished, exhausted, and demoralized, Hooker proclaimed, "My

plans are perfect. And when I start to carry them out, may God have mercy on

Bobby Lee, for I shall have none." Why, aside from a propensity for narcissism, was

Hooker so confident?

Hooker had used spies, analysts, and even hot air balloons to compile a vast
amount of intelligence about Lee?s army. He had discerned, for example, that Lee

had only 61,000 men to Hooker?s own 134,000. Buoyed by his superior numbers,

Hooker covertly moved 70,000 of his men fifteen miles up and across the river, and

then ordered them to sneak back down to position themselves behind Lee?s army.

In effect, Hooker had cut off the Confederate soldiers in front and behind. They

were trapped. Satisfied with his advantage, Hooker became convinced that Lee?s

only option was to retreat to Richmond, thus assuring a Union victory.

Yet Lee, despite his disadvantages of both numbers and position, did not retreat.

Instead, he moved his troops into position to attack. Union soldiers who tried to

warn Hooker that Lee was on the offensive were dismissed as cowards. Having

become convinced that Lee had no choice but to retreat, Hooker began to ignore

reality. When Lee?s army attacked the Union soldiers at 5:00 p.m., they were

eating supper, completely unprepared for battle. They abandoned their rifles and

fled as Lee?s troops came shrieking out of the brush, bayonets drawn. Against all

odds, Lee won the Battle of Chancellorsville, and Hooker?s forces withdrew in

defeat.

Based on its use in paragraph 3, it can be inferred that the word propensity belongs

to which of the following word groups?

1) fondness, partiality, affection

2) flaw, fault, shortcoming

3) distaste, aversion, dissatisfaction

4) tendency, inclination, predisposition

5) confidence, self-assurance, certitude

Correct Option
4
is:
Your Option is: 3
Result: Wrong
Timetaken: 0.0 secs
From the third paragraph in the passage,it is clear that the
Explanation: word propensity belongs to tendency, inclination and
predisposition.

Find the synonyms for the below word:


19.
Forewarn

1) Impel

2) Admonish

3) dare

4) risk

Correct Option
2
is:
Your Option is: 4
Result: Wrong
Timetaken: 0.0 secs
meaning of forewarn is "inform of a possible future danger
Explanation: or problem", so from given option admonish is give same
meaning

Find the antonyms for the below word:


20.
Descry

1) Disclose

2) overlook

3) see

4) behold
Correct Option is: 2
Your Option is: Not Attempted
Result: Not Attempted
Timetaken: 0.0 secs
Explanation: antonym for descry is overlook

21.
Read the passage. Then answer the questions below.

The Battle of Chancellorsville, one of the most famous battles of the Civil War, took

place in Virginia in the spring of 1863. For months, the two armies had been staked

out on opposite banks of a narrow river. The Confederate troops were led by

perhaps the most revered military tactician in American history, General Robert E.

Lee. The Union soldiers were led by "Fighting" Joe Hooker.

In appearance, personality, and lifestyle, these men were nearly perfect opposites.

Lee, an older man in poor health with a gray beard, had a somber, measured

demeanor. Hooker was a blond, strapping young man whose vanity over his

appearance was but one aspect of his egotism. Whereas Lee was devout and

principled, Hooker was known for his rollicking enjoyment of both women and

whiskey.

Despite the fact that the Confederacy had won the last four major battles and the

Union soldiers were famished, exhausted, and demoralized, Hooker proclaimed, "My

plans are perfect. And when I start to carry them out, may God have mercy on

Bobby Lee, for I shall have none." Why, aside from a propensity for narcissism, was

Hooker so confident?

Hooker had used spies, analysts, and even hot air balloons to compile a vast

amount of intelligence about Lee?s army. He had discerned, for example, that Lee

had only 61,000 men to Hooker?s own 134,000. Buoyed by his superior numbers,

Hooker covertly moved 70,000 of his men fifteen miles up and across the river, and

then ordered them to sneak back down to position themselves behind Lee?s army.
In effect, Hooker had cut off the Confederate soldiers in front and behind. They

were trapped. Satisfied with his advantage, Hooker became convinced that Lee?s

only option was to retreat to Richmond, thus assuring a Union victory.

Yet Lee, despite his disadvantages of both numbers and position, did not retreat.

Instead, he moved his troops into position to attack. Union soldiers who tried to

warn Hooker that Lee was on the offensive were dismissed as cowards. Having

become convinced that Lee had no choice but to retreat, Hooker began to ignore

reality. When Lee?s army attacked the Union soldiers at 5:00 p.m., they were

eating supper, completely unprepared for battle. They abandoned their rifles and

fled as Lee?s troops came shrieking out of the brush, bayonets drawn. Against all

odds, Lee won the Battle of Chancellorsville, and Hooker?s forces withdrew in

defeat.

Based on information in the passage, it can be concluded that Hooker lost the Battle

of Chancellorsville mostly because of his

1) Vanity

2) ignorane

3) Overconfidence

4) faculty information

Correct
3
Option is:
Your Option
3
is:
Result: Correct
Timetaken: 0.0 secs
WITH HELP OF THIS PASSAGE WE CAN CONCLUDE
Explanation: HOOKER LOST ,REASON IS OVERCONFIDENCE SO
OPTION 3 WILL BE CORRECT
Find the antonyms for the below word:
22.
photographic

1) Visual

2) vivid

3) accurate

4) outlined

Correct Option is: 4


Your Option is: 4
Result: Correct
Timetaken: 0.0 secs
Explanation: meaning of photographic visual,so opponent is outlined

23.
Since the late 1970s when the technology for sex determination first came into

being, sex selective abortion has unleashed a saga of horror. Experts are calling it ?

Sanitised Barbarism?. Demographic trends indicate the country is fast heading

towards a million foetuses aborted each year.

Although foetal sex determination and sex selection is an offence in India, the

practice is rampant. Private clinics with ultrasound machines are doing brisk

business. Everywhere, people are paying to know the sex of the unborn child, and

paying more to abort the female child. The technology has reached even remote

places through mobile clinics. Dr. Puneet Bedi obstetrician and specialist in foetal

medicine, says these days he hardly sees a family with two daughters. People are

getting the sex determination even for the first child, he says.

In the 1991 census showed that two districts had a child sex ratio (number of girls

per thousand boys) less than 850; by 2001 it was 51 districts. Child rights activist

Dr. Sabu George says foeticide is the most extreme form of violence against
women. ?Today a girl is several times more likely to be eliminated before birth than

die of various causes during the first year. Nature intended the womb to be a safe

space. Today Doctors have made it the most unsafe space for the female child, ?he

says?. He believes that Doctors must be held responsible ? ?They have aggressively

promoted the misuse of technology and legitimised foeticide.?

Akhila Sivadas, Centre for Advocacy and Research, Delhi, feels that the PCPNDT Act

(Pre-Conception and Pre-Natal Diagnostic Techniques ? Regulation and Prevention

of Misuse) is very well conceived and easy to use. The need of the hour is the legal

literacy to ensure the law is implemented. ?The demand and supply debate has

been going on for some time. Doctors say there is a social demand and they are

fulfilling it. They argue that social attitudes must change. However, in this case

supply fuels demand. Technology will have to be regulated. Technology in the hands

of greedy, vested interests cannot be neutral. There is a law to prevent misuse and

we must be able to use it,? she says.

On the ?Demand? side, experts such as Dr. Agnihotri argue that women?s

participation in workforce, having disposable incomes and making a contribution to

the larger society will make a difference to how women are seen. Youth icons and

role models such as Sania Mirza are making an impact, he says.

Others feel there needs to be widespread visible contempt and anger in society

against this ?Genocide?- ?the kind we saw against the Nithari killings,? says Dr

Bedi. ?Today nobody can say that female foeticide is not their problem.? Time we all

did our bit to help save the girl child. Time?s running out.

Select the correct answer option based on the passage.

What is the Doctors's explanation for foeticide?

1) They think it is legitimate

2) They do it because people demand it

3) The technology is available and there is no harm using it


4) None of these

Correct Option is: 2


Your Option is: 3
Result: Wrong
Timetaken: 0.0 secs
Explanation: FROM para no 3,we can say option 2 is apt well

24.
Read the passage and answer the question:

Democratic institutions are devices for reconciling social order with individual

freedom and initiative, and for making the immediate power of a country's rulers

subject to the ultimate power of the ruled. The fact that, in Western Europe 5 and

America, these devices have worked, all things considered, not too badly is proof

enough that the eighteenth century optimists were not entirely wrong. Given a fair

chance, I repeat; for the fair chance is an indispensable prerequisite No people that

pass abruptly from a state of subservience 10 under the rule of a despot to the

completely unfamiliar state of political independence can be said to have a fair

chance of being able to govern itself democratically. Liberalism flourishes in an

atmosphere of prosperity and declines as declining prosperity makes it necessary

for the government 15 to intervene ever more frequently and drastically in the

affairs of its subjects. Over-population and over-organization are two conditions

which ... deprive a society of a fair chance of making democratic institutions work

effectively. We see, then, that there are certain 20 historical, economic,

demographic and technological conditions which make it very hard for Jefferson's

rational animals, endowed by nature with inalienable rights and an innate sense of

justice, to exercise their reason, claim their rights and act justly within a

democratically organized 25 society. We in the West have been supremely fortunate


in having been given a fair chance of making the great experiment in self-

government. Unfortunately, it now looks as though, owing to recent changes in our

circumstances, this infinitely precious fair chance were being, little by little, taken

30 away from us.

The infinitely precious fair chance highlighted in the last sentence, according to the

author is

1) Unlikely to emerge in an atmosphere of liberalism

2) Incompatible with Jefferson?s views

3) Vitiated in an atmosphere of prosperity

4) An essential precondition for the success of democracy

5) Only possible in a large, advanced and highly organised society

Correct
4
Option is:
Your Option
1
is:
Result: Wrong
Timetaken: 0.0 secs
ACCORDING TO THIS PASSAGE LAST SENTENCE
Explanation: MEANS ESSENTIAL PRE CONDITION FOR THE
SUCCESS OF DEMOCRACY

Find the antonyms for the below word:


25.
Musty

1) putrid

2) crumbling

3) smelly

4) rotten
Correct Option
4
is:
Your Option is: 3
Result: Wrong
Timetaken: 0.0 secs
meaning of musty is having a stale , mouldy so equal
Explanation:
antononym is rotten

1.
In regard to propaganda the early advocates of universal literacy and a free press

envisaged only two possibilities:the propaganda might be true, or it might be false.

They did not foresee what in fact has happened, above all in our Western capitalist

democracies - the development of a vast mass communications industry, concerned

in the main neither with the true nor the false, but with the unreal, the more or less

totally irrelevant. In a word, they failed to take into account man's almost infinite

appetite for distractions.In the past most people never got a chance of fully

satisfying this appetite. They might long for distractions,but the distractions were

not provided. Christmas came but once a year, feasts were "solemn and rare," there

were few readers and very little to read, and the nearest approach to a

neighborhood movie theater was the parish church, where the performances,

though infrequent, were somewhat monotonous.For conditions even remotely

comparable to those now prevailing we must return to imperial Rome, where the

populace was kept in good humor by frequent, gratuitous doses of many kinds of

entertainment - from poetical dramas to gladiatorial fights,from recitations of Virgil

to all-out boxing, from concerts to military reviews and public executions.< br> But

even in Rome there was nothing like the non-stop distraction now provided by

newspapers and magazines, by radio, television and the cinema. In Brave New

World non-stop distractions of the most fascinating nature (the feelies, orgy-porgy,
centrifugal bumble puppy) are deliberately used as instruments of policy,for the

purpose of preventing people from paying too much attention to the realities of the

social and political situation. The other world of religion is different from the other

world of entertainment; but they resemble one another in being most decidedly

"not of this world." Both are distractions and, if lived in too continuously, both can

become, in Marx's phrase, "the opium of the people" and so a threat to freedom.

Only the vigilant can maintain their liberties, and only those who are constantly and

intelligently on the spot can hope to govern themselves effectively by democratic

procedures. A society, most of whose members spend a great part of their time, not

on the spot, not here and now and in the calculable future, but somewhere else, in

the irrelevant other worlds of sport and soap opera, of mythology and metaphysical

fantasy, will find it hard to resist the encroachments of those who would manipulate

and control it.

In their propaganda today's dictators rely for the most part on repetition,

suppression and rationalization - the repetition of catchwords which they wish to be

accepted as true, the suppression of facts which they wish to be ignored,the arousal

and rationalization of passions which may be used in the interests of the Party or

the State. As the art and science of manipulation come to be better understood,the

dictators of the future will doubtless learn to combine these techniques with the

non-stop distractions which, in the West, are now threatening to drown in a sea of

irrelevance the rational propaganda essential to the maintenance of individual

liberty and the survival of democratic institutions.

By ?intelligently on the spot? the author apparently means

1) alert to the dangers of propaganda

2) in a particular society at a particular time

3) in a specific time and place

4) conscious of political and social realities


5) deeply aware of current trends

Correct
4
Option is:
Your Option
2
is:
Result: Wrong
Timetaken: 0.0 secs
The author uses the phrase ?on the spot? twice. In the second
instance he amplifies thus: not here and now and in the
calculable future, but somewhere else, in the irrelevant other
Explanation: worlds of sport and soap opera, of mythology and
metaphysical fantasy. It is obvious he means the phrase to
relate to an awareness of the actually and potentially real as
opposed to the unreal.

In the following the questions choose the word which best expresses the meaning

2. of the given word.

ADVENTURER

1) Explorer

2) Homely

3) Native

4) Aimless

Correct Option
1
is:
Your Option is: 2
Result: Wrong
Timetaken: 0.0 secs
Explanation: 'Adventurer' means 'a person who enjoys or seek
adventure,who is also called as Explorer.

Choose the correct alternative from the following options.


3.
Due to these reason we are all in favour of universal compulsory education.

1) out of these reasons

2) For these reasons

3) by these reasons

4) no improvement

Correct Option
2
is:
Your Option is: 3
Result: Wrong
Timetaken: 0.0 secs
In the given sentence,second half of the sentence is not a
Explanation:
result of that 'reasons', but action made for the reason.

Choose the correct alternative for the word or phrase is in Bold. Markets like
4.
Janpath and Connaught Place have a lots of shop selling very trendy clothes.

1) Have lots of shops selling

2) Have lot many number of shops selling

3) Have a lot number of shops that were selling

4) No improvement needed

Correct Option
1
is:
Your Option is: 2
Result: Wrong
Timetaken: 0.0 secs
As 'lots' is in plural form, used in the sentence 'shop'
Explanation:
cannot be used. The word 'shops' must be used.

5. In India women _______only three percent of senior management

1) Contain

2) Involve

3) comprise

4) contains

5) Comprises

Correct Option
3
is:
Your Option is: 3
Result: Correct
Timetaken: 0.0 secs
Three percent of senior management consists of
Explanation:
women.So the correct answer is option 3

6.
The pioneers of the teaching of science imagined that its introduction into education

would remove the conventionality,artificiality, and backward-lookingness which were

characteristic;of classical studies, but they were gravely disappointed. So, too, in

their time had the humanists thought that the study of the classical authors in the

original would banish at once the dull pedantry and superstition of mediaeval

scholasticism. The professional schoolmaster was a match for both of them, and has

almost managed to make the understanding of chemical reactions as dull and as

dogmatic an affair as the reading of Virgil's Aeneid.The chief claim for the use of

science in education is that it teaches a child something about the actual universe in
which he is living, in making him acquainted with the results of scientific discovery,

and at the same time teaches him how to think logically and inductively by studying

scientific method.

A certain limited success has been reached in the first of these aims, but practically

none at all in the second. Those privileged members of the community who have

been through a secondary or public school education may be expected to know

something about the elementary physics and chemistry of a hundred years ago, but

they probably know hardly more than any bright boy can pick up from an interest in

wireless or scientific hobbies out of school hours.As to the learning of scientific

method, the whole thing is palpably a farce. Actually, for the convenience of

teachers and the requirements of the examination system, it is necessary that the

pupils not only do not learn scientific method but learn precisely the reverse, that

is, to believe exactly what they are told and to reproduce it when asked, whether it

seems nonsense to them or not. The way in which educated people respond to such

quackeries as spiritualism or astrology, not to say more dangerous ones such as

racial theories or currency myths, shows that fifty years of education in the method

of science in Britain or Germany has produced no visible effect whatever.

The only way of learning the method of science is the long and bitter way of

personal experience, and, until the educational or social systems are altered to

make this possible, the best we can expect is the production of a minority of people

who are able to acquire some of the techniques of science and a still smaller

minority who are able to use and develop them.

The author's attitude to secondary and public school education in the sciences is

1) ambivalent

2) neutral

3) supportive

4) satirical
5) contemptuous

Correct
5
Option is:
Your Option
4
is:
Result: Wrong
Timetaken: 0.0 secs
From the passage.it is obvious that nothing very valuable is
learned in school about science and scientific method.He is
Explanation: therefore negative.So options 1,2 and 3 is ruled out.He
seems to be expresing contempt rather than mocking.So the
correct answer is option 5.

7.
The pioneers of the teaching of science imagined that its introduction into education

would remove the conventionality,artificiality, and backward-lookingness which were

characteristic;of classical studies, but they were gravely disappointed. So, too, in

their time had the humanists thought that the study of the classical authors in the

original would banish at once the dull pedantry and superstition of mediaeval

scholasticism. The professional schoolmaster was a match for both of them, and has

almost managed to make the understanding of chemical reactions as dull and as

dogmatic an affair as the reading of Virgil's Aeneid.The chief claim for the use of

science in education is that it teaches a child something about the actual universe in

which he is living, in making him acquainted with the results of scientific discovery,

and at the same time teaches him how to think logically and inductively by studying

scientific method.

A certain limited success has been reached in the first of these aims, but practically

none at all in the second. Those privileged members of the community who have

been through a secondary or public school education may be expected to know

something about the elementary physics and chemistry of a hundred years ago, but
they probably know hardly more than any bright boy can pick up from an interest in

wireless or scientific hobbies out of school hours.As to the learning of scientific

method, the whole thing is palpably a farce. Actually, for the convenience of

teachers and the requirements of the examination system, it is necessary that the

pupils not only do not learn scientific method but learn precisely the reverse, that

is, to believe exactly what they are told and to reproduce it when asked, whether it

seems nonsense to them or not. The way in which educated people respond to such

quackeries as spiritualism or astrology, not to say more dangerous ones such as

racial theories or currency myths, shows that fifty years of education in the method

of science in Britain or Germany has produced no visible effect whatever.

The only way of learning the method of science is the long and bitter way of

personal experience, and, until the educational or social systems are altered to

make this possible, the best we can expect is the production of a minority of people

who are able to acquire some of the techniques of science and a still smaller

minority who are able to use and develop them.

1. The author implies that the 'professional schoolmaster' has

1) no interest in teaching science

2) thwarted attempts to enliven education

3) aided true learning

4) supported the humanists

5) been a pioneer in both science and humanities.

Correct
2
Option is:
Your Option
3
is:
Result: Wrong
Timetaken: 0.0 secs
From the First paragraph in the passage,"The professional
school master was a match for both of them,and has almost
managed to make the understanding of chemical reaction as
dull and as dogmatic an affair as the reading of Virgil
Aeneid',This tells that schoolmaster has made learning
Explanation:
dull.So option 3 and 4 is ruled out.To find out the exact
answer,read previous sentence also.Those sentences tells that
other people tried to alter the nature of education.He
therefore prevented(thwarted) these attempts.Therefore
answer is option 2

In the following questions choose the word which is the exact OPPOSITE of the

8. given words.

hamper

1) Hinder

2) Impede

3) Bolster

4) promote

5) pannier

Correct Option
4
is:
Your Option is: 4
Result: Correct
Timetaken: 0.0 secs
Antonym of 'Hamper' is
Explanation:
Host,Encourage,Forward,Promote,Further,etc.

The origin alternative deficit disorder as believed by many medical experts can be
9.
congenital whereas others believe it to be ________

1) Exogenous
2) Deleterious

3) Pathological

4) celestial

Correct Option
1
is:
Your Option is: 3
Result: Wrong
Timetaken: 0.0 secs
Exogenous means originating from the outside body,that
Explanation:
is external factors

10.
In regard to propaganda the early advocates of universal literacy and a free press

envisaged only two possibilities:the propaganda might be true, or it might be false.

They did not foresee what in fact has happened, above all in our Western capitalist

democracies - the development of a vast mass communications industry, concerned

in the main neither with the true nor the false, but with the unreal, the more or less

totally irrelevant. In a word, they failed to take into account man's almost infinite

appetite for distractions.In the past most people never got a chance of fully

satisfying this appetite. They might long for distractions,but the distractions were

not provided. Christmas came but once a year, feasts were "solemn and rare," there

were few readers and very little to read, and the nearest approach to a

neighborhood movie theater was the parish church, where the performances,

though infrequent, were somewhat monotonous.For conditions even remotely

comparable to those now prevailing we must return to imperial Rome, where the

populace was kept in good humor by frequent, gratuitous doses of many kinds of

entertainment - from poetical dramas to gladiatorial fights,from recitations of Virgil

to all-out boxing, from concerts to military reviews and public executions.< br> But

even in Rome there was nothing like the non-stop distraction now provided by
newspapers and magazines, by radio, television and the cinema. In Brave New

World non-stop distractions of the most fascinating nature (the feelies, orgy-porgy,

centrifugal bumble puppy) are deliberately used as instruments of policy,for the

purpose of preventing people from paying too much attention to the realities of the

social and political situation. The other world of religion is different from the other

world of entertainment; but they resemble one another in being most decidedly

"not of this world." Both are distractions and, if lived in too continuously, both can

become, in Marx's phrase, "the opium of the people" and so a threat to freedom.

Only the vigilant can maintain their liberties, and only those who are constantly and

intelligently on the spot can hope to govern themselves effectively by democratic

procedures. A society, most of whose members spend a great part of their time, not

on the spot, not here and now and in the calculable future, but somewhere else, in

the irrelevant other worlds of sport and soap opera, of mythology and metaphysical

fantasy, will find it hard to resist the encroachments of those who would manipulate

and control it.

In their propaganda today's dictators rely for the most part on repetition,

suppression and rationalization - the repetition of catchwords which they wish to be

accepted as true, the suppression of facts which they wish to be ignored,the arousal

and rationalization of passions which may be used in the interests of the Party or

the State. As the art and science of manipulation come to be better understood,the

dictators of the future will doubtless learn to combine these techniques with the

non-stop distractions which, in the West, are now threatening to drown in a sea of

irrelevance the rational propaganda essential to the maintenance of individual

liberty and the survival of democratic institutions.

The best title to above passage is

1) puzzling phenomena

2) propaganda
3) origins of life

4) Empedocles

5) prove that selection is the only possible way of looking at evolutionary biology

Correct Option
2
is:
Your Option is: 2
Result: Correct
Timetaken: 0.0 secs
This entire passage gives the only information about
Explanation:
propaganda.

Choose the correct alternative from the following options.


11.
After Michael typed the letter,he gave it to Jane to sign.

1) He was giving it to Jane to sign

2) He gives to jane to sign

3) He had been giving to jane to sign

4) No change

Correct Option is: 4


Your Option is: 1
Result: Wrong
Timetaken: 0.0 secs
Explanation: The given sentence itself correct.

12.
The pioneers of the teaching of science imagined that its introduction into education

would remove the conventionality,artificiality, and backward-lookingness which were

characteristic;of classical studies, but they were gravely disappointed. So, too, in

their time had the humanists thought that the study of the classical authors in the
original would banish at once the dull pedantry and superstition of mediaeval

scholasticism. The professional schoolmaster was a match for both of them, and has

almost managed to make the understanding of chemical reactions as dull and as

dogmatic an affair as the reading of Virgil's Aeneid.The chief claim for the use of

science in education is that it teaches a child something about the actual universe in

which he is living, in making him acquainted with the results of scientific discovery,

and at the same time teaches him how to think logically and inductively by studying

scientific method.

A certain limited success has been reached in the first of these aims, but practically

none at all in the second. Those privileged members of the community who have

been through a secondary or public school education may be expected to know

something about the elementary physics and chemistry of a hundred years ago, but

they probably know hardly more than any bright boy can pick up from an interest in

wireless or scientific hobbies out of school hours.As to the learning of scientific

method, the whole thing is palpably a farce. Actually, for the convenience of

teachers and the requirements of the examination system, it is necessary that the

pupils not only do not learn scientific method but learn precisely the reverse, that

is, to believe exactly what they are told and to reproduce it when asked, whether it

seems nonsense to them or not. The way in which educated people respond to such

quackeries as spiritualism or astrology, not to say more dangerous ones such as

racial theories or currency myths, shows that fifty years of education in the method

of science in Britain or Germany has produced no visible effect whatever.

The only way of learning the method of science is the long and bitter way of

personal experience, and, until the educational or social systems are altered to

make this possible, the best we can expect is the production of a minority of people

who are able to acquire some of the techniques of science and a still smaller

minority who are able to use and develop them.


The word 'palpably' most nearly means

1) empirically

2) obviously

3) tentatively

4) markedly

5) ridiculously

Correct
2
Option is:
Your Option
4
is:
Result: Wrong
Timetaken: 0.0 secs
In the sentence,'As to the learning of scientific method,the
Explanation: whole thing is palpably a farce'.Here i could substitute
'obviously' or 'clearly'.So option 2 is the correct answer.

Re-arrange the following sentences.

P. Mrs. Parasuram started a petrol pump in madras.

Q.thus she has shown the way for many others.

13. R.a total of twelve girls now work at the pump.

S.she advertised in news paper for women staff.

T.they operate in two shifts

U.the response was good

1) PQRSTU

2) URSPQT

3) PURTSQ

4) RSQPTU
5) PRUTSQ

Correct Option
3
is:
Your Option is: 3
Result: Correct
Timetaken: 0.0 secs
Starting os the passage is sentence P.So options 2 and 4 is
Explanation:
ruled out.Ant the next sentence is U.So correct option is 3

This technology has the potential to provide handsome returns even though it is at
14.
a___stage in India

1) Turbulent

2) Peculiar

3) Nascent

4) unknown

Correct Option is: 3


Your Option is: 4
Result: Wrong
Timetaken: 0.0 secs
Nascent means beginning.So option 1 is the correct
Explanation:
answer.

15.
The pioneers of the teaching of science imagined that its introduction into education

would remove the conventionality,artificiality, and backward-lookingness which were

characteristic;of classical studies, but they were gravely disappointed. So, too, in

their time had the humanists thought that the study of the classical authors in the
original would banish at once the dull pedantry and superstition of medieval

scholasticism. The professional schoolmaster was a match for both of them, and has

almost managed to make the understanding of chemical reactions as dull and as

dogmatic an affair as the reading of Virgil's Aeneid.The chief claim for the use of

science in education is that it teaches a child something about the actual universe in

which he is living, in making him acquainted with the results of scientific discovery,

and at the same time teaches him how to think logically and inductively by studying

scientific method.

A certain limited success has been reached in the first of these aims, but practically

none at all in the second. Those privileged members of the community who have

been through a secondary or public school education may be expected to know

something about the elementary physics and chemistry of a hundred years ago, but

they probably know hardly more than any bright boy can pick up from an interest in

wireless or scientific hobbies out of school hours.As to the learning of scientific

method, the whole thing is palpably a farce. Actually, for the convenience of

teachers and the requirements of the examination system, it is necessary that the

pupils not only do not learn scientific method but learn precisely the reverse, that

is, to believe exactly what they are told and to reproduce it when asked, whether it

seems nonsense to them or not. The way in which educated people respond to such

quackeries as spiritualism or astrology, not to say more dangerous ones such as

racial theories or currency myths, shows that fifty years of education in the method

of science in Britain or Germany has produced no visible effect whatever.

The only way of learning the method of science is the long and bitter way of

personal experience, and, until the educational or social systems are altered to

make this possible, the best we can expect is the production of a minority of people

who are able to acquire some of the techniques of science and a still smaller

minority who are able to use and develop them.

If the author were to study current education in science to see how things have
changed since he wrote the piece, he would probably be most interested in the

answer to which of the following questions?

1) Do students know more about the world about them?

2) Do students spend more time in laboratories?

3) Can students apply their knowledge logically?

4) Have textbooks improved?

5) Do they respect their teachers?

Correct
3
Option is:
Your Option
1
is:
Result: Wrong
Timetaken: 0.0 secs
The concern of the author will tell us what he would like to
see.From the first paragraph,we learnt that he is especially
Explanation:
interested in whether a student can apply his or her
knowledge.So anwer is option 3.

Select the word or phrase which best expression the meaning of the given word.
16.
APPETITE

1) Hunger

2) Revulision

3) Antipathy

4) Fasting

Correct Option
1
is:
Your Option is: 4
Result: Wrong
Timetaken: 0.0 secs
Direct meaning of 'Appetite' is
Explanation:
'Hunger','Hungriness','Ravenousness'.

17.
In regard to propaganda the early advocates of universal literacy and a free press

envisaged only two possibilities:the propaganda might be true, or it might be false.

They did not foresee what in fact has happened, above all in our Western capitalist

democracies - the development of a vast mass communications industry, concerned

in the main neither with the true nor the false, but with the unreal, the more or less

totally irrelevant. In a word, they failed to take into account man's almost infinite

appetite for distractions.In the past most people never got a chance of fully

satisfying this appetite. They might long for distractions,but the distractions were

not provided. Christmas came but once a year, feasts were "solemn and rare," there

were few readers and very little to read, and the nearest approach to a

neighborhood movie theater was the parish church, where the performances,

though infrequent, were somewhat monotonous.For conditions even remotely

comparable to those now prevailing we must return to imperial Rome, where the

populace was kept in good humor by frequent, gratuitous doses of many kinds of

entertainment - from poetical dramas to gladiatorial fights,from recitations of Virgil

to all-out boxing, from concerts to military reviews and public executions.< br> But

even in Rome there was nothing like the non-stop distraction now provided by

newspapers and magazines, by radio, television and the cinema. In Brave New

World non-stop distractions of the most fascinating nature (the feelies, orgy-porgy,

centrifugal bumble puppy) are deliberately used as instruments of policy,for the

purpose of preventing people from paying too much attention to the realities of the

social and political situation. The other world of religion is different from the other

world of entertainment; but they resemble one another in being most decidedly
"not of this world." Both are distractions and, if lived in too continuously, both can

become, in Marx's phrase, "the opium of the people" and so a threat to freedom.

Only the vigilant can maintain their liberties, and only those who are constantly and

intelligently on the spot can hope to govern themselves effectively by democratic

procedures. A society, most of whose members spend a great part of their time, not

on the spot, not here and now and in the calculable future, but somewhere else, in

the irrelevant other worlds of sport and soap opera, of mythology and metaphysical

fantasy, will find it hard to resist the encroachments of those who would manipulate

and control it.

In their propaganda today's dictators rely for the most part on repetition,

suppression and rationalization - the repetition of catchwords which they wish to be

accepted as true, the suppression of facts which they wish to be ignored,the arousal

and rationalization of passions which may be used in the interests of the Party or

the State. As the art and science of manipulation come to be better understood,the

dictators of the future will doubtless learn to combine these techniques with the

non-stop distractions which, in the West, are now threatening to drown in a sea of

irrelevance the rational propaganda essential to the maintenance of individual

liberty and the survival of democratic institutions.

The author refers to ?Brave New World? as a fictional example of a society in which

A. B. C. D. E.

1) non-stop distractions are the main instrument of government policy

2) people are totally unaware of political realities

3) entertainment is used to keep people from full awareness of social realities

4) entertainment resembles religion in its effects on the masses

5) non-stop entertainment is provided as it was in Rome


Correct
3
Option is:
Your Option
1
is:
Result: Wrong
Timetaken: 0.0 secs
The non-stop distractions in Brave New World are used ?for
the purpose of preventing people from paying too much
Explanation:
attention to the realities of the social and political situation?.
This is paraphrased in answer C.

Choose the answer option that arranges the given set of words in the "most"

meaningful order. The words when put in order should make logical sense according

to size,quality, occurrence of events, value,appearance, nature, process etc,.

1.Network
18.
2.Talk

3.Mobile

4.Ring

5.Connection

1) 3,5,1,4,2

2) 3,1,5,4,2

3) 5,3,1,4,2

4) 5,1,3,4,2

Correct Option is: 2


Your Option is: 2
Result: Correct
Timetaken: 0.0 secs
Explanation: First thing is Mobile,Then
Network,Connection,Ring,Talk

Find out the error in the following sentence.

19. A.When the captain called the crew B.each of the crew member ran towards C.the

deck of the ship

1) A

2) B

3) C

4) No error

Correct Option
2
is:
Your Option is: 3
Result: Wrong
Timetaken: 0.0 secs
each of/either of/neither of/one of/anyone of/none of....
Explanation:
+noun(plural). So correct answer is option 2.

20.
In regard to propaganda the early advocates of universal literacy and a free press

envisaged only two possibilities:the propaganda might be true, or it might be false.

They did not foresee what in fact has happened, above all in our Western capitalist

democracies - the development of a vast mass communications industry, concerned

in the main neither with the true nor the false, but with the unreal, the more or less

totally irrelevant. In a word, they failed to take into account man's almost infinite

appetite for distractions.In the past most people never got a chance of fully

satisfying this appetite. They might long for distractions,but the distractions were

not provided. Christmas came but once a year, feasts were "solemn and rare," there

were few readers and very little to read, and the nearest approach to a
neighborhood movie theater was the parish church, where the performances,

though infrequent, were somewhat monotonous.For conditions even remotely

comparable to those now prevailing we must return to imperial Rome, where the

populace was kept in good humor by frequent, gratuitous doses of many kinds of

entertainment - from poetical dramas to gladiatorial fights,from recitations of Virgil

to all-out boxing, from concerts to military reviews and public executions.< br> But

even in Rome there was nothing like the non-stop distraction now provided by

newspapers and magazines, by radio, television and the cinema. In Brave New

World non-stop distractions of the most fascinating nature (the feelies, orgy-porgy,

centrifugal bumble puppy) are deliberately used as instruments of policy,for the

purpose of preventing people from paying too much attention to the realities of the

social and political situation. The other world of religion is different from the other

world of entertainment; but they resemble one another in being most decidedly

"not of this world." Both are distractions and, if lived in too continuously, both can

become, in Marx's phrase, "the opium of the people" and so a threat to freedom.

Only the vigilant can maintain their liberties, and only those who are constantly and

intelligently on the spot can hope to govern themselves effectively by democratic

procedures. A society, most of whose members spend a great part of their time, not

on the spot, not here and now and in the calculable future, but somewhere else, in

the irrelevant other worlds of sport and soap opera, of mythology and metaphysical

fantasy, will find it hard to resist the encroachments of those who would manipulate

and control it.

In their propaganda today's dictators rely for the most part on repetition,

suppression and rationalization - the repetition of catchwords which they wish to be

accepted as true, the suppression of facts which they wish to be ignored,the arousal

and rationalization of passions which may be used in the interests of the Party or

the State. As the art and science of manipulation come to be better understood,the

dictators of the future will doubtless learn to combine these techniques with the
non-stop distractions which, in the West, are now threatening to drown in a sea of

irrelevance the rational propaganda essential to the maintenance of individual

liberty and the survival of democratic institutions.

The 'early advocates of universal literacy' are mentioned as

1) advocates of propaganda

2) opponents of an idea that the author thinks is correct

3) proponents of an idea that the author wishes to counter

4) people who made wrong predictions about freedom of the press

5) social commentators unaware of man?s appetite for distractions

Correct
3
Option is:
Your Option
2
is:
Result: Wrong
Timetaken: 0.0 secs
The ?early advocates? envisaged two possibilities: that
propaganda could be concerned with the true or the false.
The author wishes to show that their ideas were limited and
Explanation: that propaganda today is mainly concerned with the ?unreal?.
In other words that author only mentions these ideas so that
he can knock them down and thus to form an introduction to
the idea that he wishes to discuss.

21.
The pioneers of the teaching of science imagined that its introduction into education

would remove the conventionality,artificiality, and backward-lookingness which were

characteristic;of classical studies, but they were gravely disappointed. So, too, in

their time had the humanists thought that the study of the classical authors in the

original would banish at once the dull pedantry and superstition of mediaeval

scholasticism. The professional schoolmaster was a match for both of them, and has

almost managed to make the understanding of chemical reactions as dull and as


dogmatic an affair as the reading of Virgil's Aeneid.The chief claim for the use of

science in education is that it teaches a child something about the actual universe in

which he is living, in making him acquainted with the results of scientific discovery,

and at the same time teaches him how to think logically and inductively by studying

scientific method.

A certain limited success has been reached in the first of these aims, but practically

none at all in the second. Those privileged members of the community who have

been through a secondary or public school education may be expected to know

something about the elementary physics and chemistry of a hundred years ago, but

they probably know hardly more than any bright boy can pick up from an interest in

wireless or scientific hobbies out of school hours.As to the learning of scientific

method, the whole thing is palpably a farce. Actually, for the convenience of

teachers and the requirements of the examination system, it is necessary that the

pupils not only do not learn scientific method but learn precisely the reverse, that

is, to believe exactly what they are told and to reproduce it when asked, whether it

seems nonsense to them or not. The way in which educated people respond to such

quackeries as spiritualism or astrology, not to say more dangerous ones such as

racial theories or currency myths, shows that fifty years of education in the method

of science in Britain or Germany has produced no visible effect whatever.

The only way of learning the method of science is the long and bitter way of

personal experience, and, until the educational or social systems are altered to

make this possible, the best we can expect is the production of a minority of people

who are able to acquire some of the techniques of science and a still smaller

minority who are able to use and develop them.

The author blames all of the following for the failure to impart scientific method

through the education system except

1) poor teaching
2) examination methods

3) lack of direct experience

4) the social and education systems

5) lack of interest on the part of students

Correct
5
Option is:
Your Option
4
is:
Result: Wrong
Timetaken: 0.0 secs
In the passage,'Author blame poor teaching,exams,social
Explanation: and education system,lack of direct experience.But he never
blames the students.Hence answer is option 5.

22. Performance appraisals that contain ___ criticism are appreciated.

1) constructive

2) Scathing

3) Soft

4) Indirect

Correct Option
1
is:
Your Option is: 3
Result: Wrong
Timetaken: 0.0 secs
Criticism is used for the improvement.So constructive
Explanation:
criticism is appreciated.

23.
Read the sentence find out the grammatical error in it.
(a)Fatimah is a girl of principle and she (b)would never go against her values (c) to

get admission in that institute

1) A

2) B

3) C

4) No error

Correct Option is: 1


Your Option is: 1
Result: Correct
Timetaken: 0.0 secs
Explanation: 'Principles' must be used instead of 'principle'.

24.
In regard to propaganda the early advocates of universal literacy and a free press

envisaged only two possibilities:the propaganda might be true, or it might be false.

They did not foresee what in fact has happened, above all in our Western capitalist

democracies - the development of a vast mass communications industry, concerned

in the main neither with the true nor the false, but with the unreal, the more or less

totally irrelevant. In a word, they failed to take into account man's almost infinite

appetite for distractions.In the past most people never got a chance of fully

satisfying this appetite. They might long for distractions,but the distractions were

not provided. Christmas came but once a year, feasts were "solemn and rare," there

were few readers and very little to read, and the nearest approach to a

neighborhood movie theater was the parish church, where the performances,

though infrequent, were somewhat monotonous.For conditions even remotely

comparable to those now prevailing we must return to imperial Rome, where the

populace was kept in good humor by frequent, gratuitous doses of many kinds of
entertainment - from poetical dramas to gladiatorial fights,from recitations of Virgil

to all-out boxing, from concerts to military reviews and public executions.< br> But

even in Rome there was nothing like the non-stop distraction now provided by

newspapers and magazines, by radio, television and the cinema. In Brave New

World non-stop distractions of the most fascinating nature (the feelies, orgy-porgy,

centrifugal bumble puppy) are deliberately used as instruments of policy,for the

purpose of preventing people from paying too much attention to the realities of the

social and political situation. The other world of religion is different from the other

world of entertainment; but they resemble one another in being most decidedly

"not of this world." Both are distractions and, if lived in too continuously, both can

become, in Marx's phrase, "the opium of the people" and so a threat to freedom.

Only the vigilant can maintain their liberties, and only those who are constantly and

intelligently on the spot can hope to govern themselves effectively by democratic

procedures. A society, most of whose members spend a great part of their time, not

on the spot, not here and now and in the calculable future, but somewhere else, in

the irrelevant other worlds of sport and soap opera, of mythology and metaphysical

fantasy, will find it hard to resist the encroachments of those who would manipulate

and control it.

In their propaganda today's dictators rely for the most part on repetition,

suppression and rationalization - the repetition of catchwords which they wish to be

accepted as true, the suppression of facts which they wish to be ignored,the arousal

and rationalization of passions which may be used in the interests of the Party or

the State. As the art and science of manipulation come to be better understood,the

dictators of the future will doubtless learn to combine these techniques with the

non-stop distractions which, in the West, are now threatening to drown in a sea of

irrelevance the rational propaganda essential to the maintenance of individual

liberty and the survival of democratic institutions.


The author would be most likely to agree that propaganda

1) can serve a vital function in democracy

2) is concerned mainly with the irrelevant

3) is now combined with entertainment

4) is universally recognized as a danger

5) needs constant vigilance to avoid

Correct
1
Option is:
Your Option
2
is:
Result: Wrong
Timetaken: 0.0 secs
The last sentence will tell us that rational propaganda is
'essential to the maitenance of individual liberty and the
Explanation:
survival of democratic institutions'.This clearly suggests that
answer 1 is correct.

In the following questions choose the word which is the exact OPPOSITE of the

25. given words.

CLARIFY

1) Analyze

2) Simplify

3) Confuse

4) Response

Correct Option is: 3


Your Option is: 3
Result: Correct
Timetaken: 0.0 secs
Explanation: Exact opposite of 'Clarify' is "Confuse'

In each of the following questions, a paragraph or a sentence has been broken up

into different parts. The parts have been scrambled and numbered as given below.

Choose the correct order of these parts from the given alternatives.

1) you can take

2) you can't take

1. 3) the boy

4) the village

5) out of the village

6) out of the boy

7) but

1) 1,3,5,7,2,4,6

2) 1,5,2,6,3,7,4

3) 1,6,5,3,7,2,4

4) 1,7,2,4,3,5,6

Correct Option
1
is:
Your Option is: 1
Result: Correct
Timetaken: 0.0 secs
You can take the boy out of the village but you cannot
Explanation:
take the village out of the boy.
Read each sentence to find if there is any grammatical error in it. If there is any

error, it will be only one part of the sentence. The number or alphabet of that part is

2. your answer.( Disregard punctuation errors if any)

His supporters are / not as enthusiastic / and co-operative as / that of his

opponent's/ no error

1) A

2) B

3) C

4) D

5) E

Correct Option
4
is:
Your Option is: 2
Result: Wrong
Timetaken: 0.0 secs
No need of using 'that of' as 'S' is used in the word
Explanation:
'opponent's'.

Read each sentence to find if there is any grammatical error in it. If there is any

error, it will be only one part of the sentence. The number or alphabet of that part is

3. your answer.( Disregard punctuation errors if any)

No sooner the news appeared in the paper/ than / there was a rush / in the

counter/ no error

1) A

2) B

3) C
4) D

5) E

Correct Option
1
is:
Your Option is: 2
Result: Wrong
Timetaken: 0.0 secs
'No sooner had the newspaper appeared' in place of 'No
Explanation: sooner the news appeared'.'No sooner' is used in two forms
(i)'No sooner had' and (ii)'No sooner did'

4.
Read the passage and answer the questions that follow on the basis of the

information provided in the passage.

Outside, the rain continued to run down the screened windows of Mrs. Sennett's

little Cape Cod cottage. The long weeds and grass that composed the front yard

dripped against the blurred background of the bay, where the water was almost the

color of the grass. Mrs. Sennett's five charges were vigorously playing house in the

dining room. (In the wintertime, Mrs. Sennett was housekeeper for a Mr. Curley, in

Boston, and during the summers the Curley children boarded with her on the

Cape.) My expression must have changed. " Are those children making too much

noise?" Mrs. Sennett demanded, a sort of wave going over her that might mark the

beginning of her getting up out of her chair. I shook my head no, and gave her a

little push on the shoulder to keep her seated. Mrs. Sennett was almost stone-deaf

and had been for a long time, but she could read lips. You could talk to her without

making any sound yourself, if you wanted to, and she more than kept up her side of

the conversation in a loud, rusty voice that dropped weirdly every now and then

into a whisper. She adored talking. To look at Mrs. Sennett made me think of

eighteenth-century England and its literary figures. Her hair must have been sadly

thin, because she always wore, indoors and out, either a hat or a sort of turban,
and sometimes she wore both. The rims of her eyes were dark; she looked very ill.

Mrs. Sennett and I continued talking. She said she really didn't think she'd stay with

the children another winter. Their father wanted her to, but it was too much for her.

She wanted to stay right here in the cottage. The afternoon was getting along, and

I finally left because I knew that at four o'clock Mrs. Sennett's "sit down" was over

and she started to get supper. At six o'clock, from my nearby cottage, I saw

Theresa coming through the rain with a shawl over her head. She was bringing me

a six-inchsquare piece of spice cake , still hot from the oven and kept warm

between two soup plates. A few days later I learned from the twins, who brought

over gifts of firewood and blackberries, that their father was coming the next

morning, bringing their aunt and her husband and their cousin. Mrs. Sennett had

promised to take them all on a picnic at the pond some pleasant day. On the fourth

day of their visit, Xavier arrived with a note. It was from Mrs. Sennett, written in

blue ink, in a large, serene, ornamented hand, on linenfinish paper:

Tomorrow is the last day Mr. Curley has and the Children all wanted the Picnic so

much. The Men can walk to the Pond but it is too far for the Children. I see your

Friend has a car and I hate to ask this but could you possibly drive us to the Pond

tomorrow morning? . . .Very sincerely yours, Carmen Sennett After the picnic, Mrs.

Sennett's presents to me were numberless. It was almost time for the children to

go back to school in South Boston. Mrs. Sennett insisted that she was not going;

their father was coming down again to get them and she was just going to stay. He

would have to get another housekeeper. She said this over and over to me, loudly,

and her turbans and kerchiefs grew more and more distrait. One evening, Mary

came to call on me and we sat on an old table in the back yard to watch the sunset.

"Papa came today, " she said, "and we've got to go back day after tomorrow. ""Is

Mrs. Sennett going to stay here? ""She said at supper she was. She said this time

she really was, because she'd said that last year and came back, but now she

means it ."I said, "Oh dear," scarcely knowing which side I was on. "It was awful at
supper. I cried and cried." "Did Theresa cry?" "Oh, we all cried. Papa cried, too. We

always do." "But don't you think Mrs. Sennett needs a rest?" "Yes, but I think she'll

come, though. Papa told her he'd cry every single night at supper if she didn't, and

then we all did." The next day I heard that Mrs. Sennett was going back with them

just to " help settle." She came over the following morning to say goodbye,

supported by all five children. She was wearing her traveling hat of black satin and

black straw, with sequins. High and somber, above her ravaged face, it had quite a

Spanishgrandee air. "This isn't really goodbye," she said. "I'll be backas soon as I

get these bad ,noisy children off my hands." But the children hung on to her skirt

and tugged at her sleeves, shaking their heads frantically, silently saying, "No! No!

No!" to her with their puckered-up mouths

What is the main insight suggested by the conversation in last paragraph?

1) The Curley family cries to manipulate Mrs. Sennett into doing what they want.

2) The narrator regrets that she is not going to Boston and is a little jealous of Mrs.
Sennett.

3) Mrs. Sennett is happy to leave the Curley family because they are always
whining and crying.

4) Mrs. Sennett intends to return to the Cape soon because she has discovered that
they have been manipulating and taking advantage of her.

Correct Option
1
is:
Your Option is: 2
Result: Wrong
Timetaken: 0.0 secs
From the last paragraph,it is clear that The curley family
Explanation:
cried to manipulate Mrs.Sennett into doing what they want.

5.
Read the passage and answer the questions that follow on the basis of the
information provided in the passage.

Outside, the rain continued to run down the screened windows of Mrs. Sennett's

little Cape Cod cottage. The long weeds and grass that composed the front yard

dripped against the blurred background of the bay, where the water was almost the

color of the grass. Mrs. Sennett's five charges were vigorously playing house in the

dining room. (In the wintertime, Mrs. Sennett was housekeeper for a Mr. Curley, in

Boston, and during the summers the Curley children boarded with her on the

Cape.) My expression must have changed. " Are those children making too much

noise?" Mrs. Sennett demanded, a sort of wave going over her that might mark the

beginning of her getting up out of her chair. I shook my head no, and gave her a

little push on the shoulder to keep her seated. Mrs. Sennett was almost stone-deaf

and had been for a long time, but she could read lips. You could talk to her without

making any sound yourself, if you wanted to, and she more than kept up her side of

the conversation in a loud, rusty voice that dropped weirdly every now and then

into a whisper. She adored talking. To look at Mrs. Sennett made me think of

eighteenth-century England and its literary figures. Her hair must have been sadly

thin, because she always wore, indoors and out, either a hat or a sort of turban,

and sometimes she wore both. The rims of her eyes were dark; she looked very ill.

Mrs. Sennett and I continued talking. She said she really didn't think she'd stay with

the children another winter. Their father wanted her to, but it was too much for her.

She wanted to stay right here in the cottage. The afternoon was getting along, and

I finally left because I knew that at four o'clock Mrs. Sennett's "sit down" was over

and she started to get supper. At six o'clock, from my nearby cottage, I saw

Theresa coming through the rain with a shawl over her head. She was bringing me

a six-inchsquare piece of spice cake , still hot from the oven and kept warm

between two soup plates. A few days later I learned from the twins, who brought

over gifts of firewood and blackberries, that their father was coming the next

morning, bringing their aunt and her husband and their cousin. Mrs. Sennett had
promised to take them all on a picnic at the pond some pleasant day. On the fourth

day of their visit, Xavier arrived with a note. It was from Mrs. Sennett, written in

blue ink, in a large, serene, ornamented hand, on linenfinish paper:

Tomorrow is the last day Mr. Curley has and the Children all wanted the Picnic so

much. The Men can walk to the Pond but it is too far for the Children. I see your

Friend has a car and I hate to ask this but could you possibly drive us to the Pond

tomorrow morning? . . .Very sincerely yours, Carmen Sennett After the picnic, Mrs.

Sennett's presents to me were numberless. It was almost time for the children to

go back to school in South Boston. Mrs. Sennett insisted that she was not going;

their father was coming down again to get them and she was just going to stay. He

would have to get another housekeeper. She said this over and over to me, loudly,

and her turbans and kerchiefs grew more and more distrait. One evening, Mary

came to call on me and we sat on an old table in the back yard to watch the sunset.

"Papa came today, " she said, "and we've got to go back day after tomorrow. ""Is

Mrs. Sennett going to stay here? ""She said at supper she was. She said this time

she really was, because she'd said that last year and came back, but now she

means it ."I said, "Oh dear," scarcely knowing which side I was on. "It was awful at

supper. I cried and cried." "Did Theresa cry?" "Oh, we all cried. Papa cried, too. We

always do." "But don't you think Mrs. Sennett needs a rest?" "Yes, but I think she'll

come, though. Papa told her he'd cry every single night at supper if she didn't, and

then we all did." The next day I heard that Mrs. Sennett was going back with them

just to " help settle." She came over the following morning to say goodbye,

supported by all five children. She was wearing her traveling hat of black satin and

black straw, with sequins. High and somber, above her ravaged face, it had quite a

Spanishgrandee air. "This isn't really goodbye," she said. "I'll be back as soon as I

get these bad ,noisy children off my hands." But the children hung on to her skirt

and tugged at her sleeves, shaking their heads frantically, silently saying, "No! No!

No!" to her with their puckered-up mouths


Given the evidence provided throughout the passage, the children probably silently

mouth the word "no" because:

1) Mrs. Sennett has just called them bad, noisy children, and they are defending
themselves.

2) they do not want to leave the Cape before the summer is over and are
protesting.

3) they are letting the narrator know that Mrs. Sennett is thinking about returning
to the Cape.

4) they are continuing their battle against Mrs. Sennett's intention to return to the
Cape

Correct
4
Option is:
Your Option
2
is:
Result: Wrong
Timetaken: 0.0 secs
From the passage,it is very clear that the children did not
like her returning back to cape.They silently mouth the word
Explanation:
'no' because they are continuing their battle against
Mrs.Sennett's intention to return to the cape.

6.
Read the passage and answer the questions that follow on the basis of the

information provided in the passage.

Outside, the rain continued to run down the screened windows of Mrs. Sennett's

little Cape Cod cottage. The long weeds and grass that composed the front yard

dripped against the blurred background of the bay, where the water was almost the

color of the grass. Mrs. Sennett's five charges were vigorously playing house in the

dining room. (In the wintertime, Mrs. Sennett was housekeeper for a Mr. Curley, in

Boston, and during the summers the Curley children boarded with her on the

Cape.) My expression must have changed. " Are those children making too much
noise?" Mrs. Sennett demanded, a sort of wave going over her that might mark the

beginning of her getting up out of her chair. I shook my head no, and gave her a

little push on the shoulder to keep her seated. Mrs. Sennett was almost stone-deaf

and had been for a long time, but she could read lips. You could talk to her without

making any sound yourself, if you wanted to, and she more than kept up her side of

the conversation in a loud, rusty voice that dropped weirdly every now and then

into a whisper. She adored talking. To look at Mrs. Sennett made me think of

eighteenth-century England and its literary figures. Her hair must have been sadly

thin, because she always wore, indoors and out, either a hat or a sort of turban,

and sometimes she wore both. The rims of her eyes were dark; she looked very ill.

Mrs. Sennett and I continued talking. She said she really didn't think she'd stay with

the children another winter. Their father wanted her to, but it was too much for her.

She wanted to stay right here in the cottage. The afternoon was getting along, and

I finally left because I knew that at four o'clock Mrs. Sennett's "sit down" was over

and she started to get supper. At six o'clock, from my nearby cottage, I saw

Theresa coming through the rain with a shawl over her head. She was bringing me

a six-inchsquare piece of spice cake , still hot from the oven and kept warm

between two soup plates. A few days later I learned from the twins, who brought

over gifts of firewood and blackberries, that their father was coming the next

morning, bringing their aunt and her husband and their cousin. Mrs. Sennett had

promised to take them all on a picnic at the pond some pleasant day. On the fourth

day of their visit, Xavier arrived with a note. It was from Mrs. Sennett, written in

blue ink, in a large, serene, ornamented hand, on linenfinish paper:

Tomorrow is the last day Mr. Curley has and the Children all wanted the Picnic so

much. The Men can walk to the Pond but it is too far for the Children. I see your

Friend has a car and I hate to ask this but could you possibly drive us to the Pond

tomorrow morning? . . .Very sincerely yours, Carmen Sennett After the picnic, Mrs.

Sennett's presents to me were numberless. It was almost time for the children to
go back to school in South Boston. Mrs. Sennett insisted that she was not going;

their father was coming down again to get them and she was just going to stay. He

would have to get another housekeeper. She said this over and over to me, loudly,

and her turbans and kerchiefs grew more and more distrait. One evening, Mary

came to call on me and we sat on an old table in the back yard to watch the sunset.

"Papa came today, " she said, "and we've got to go back day after tomorrow. ""Is

Mrs. Sennett going to stay here? ""She said at supper she was. She said this time

she really was, because she'd said that last year and came back, but now she

means it ."I said, "Oh dear," scarcely knowing which side I was on. "It was awful at

supper. I cried and cried." "Did Theresa cry?" "Oh, we all cried. Papa cried, too. We

always do." "But don't you think Mrs. Sennett needs a rest?" "Yes, but I think she'll

come, though. Papa told her he'd cry every single night at supper if she didn't, and

then we all did." The next day I heard that Mrs. Sennett was going back with them

just to " help settle." She came over the following morning to say goodbye,

supported by all five children. She was wearing her traveling hat of black satin and

black straw, with sequins. High and somber, above her ravaged face, it had quite a

Spanishgrandee air. "This isn't really goodbye," she said. "I'll be backas soon as I

get these bad ,noisy children off my hands." But the children hung on to her skirt

and tugged at her sleeves, shaking their heads frantically, silently saying, "No! No!

No!" to her with their puckered-up mouths

According to the narrator, Mrs. Sennett wears a hat because she:

1) is often outside.

2) wants to look like a literary figure

3) has thin hair

4) has unique taste in clothing.


Correct Option
3
is:
Your Option is: 2
Result: Wrong
Timetaken: 0.0 secs
From the second paragraph,it is clear that Mrs. Sennett
Explanation:
wears a hat because she has thin hair.

Read each sentence to find if there is any grammatical error in it. If there is any

error, it will be only one part of the sentence. The number or alphabet of that part is

7. your answer.( Disregard punctuation errors if any)

Unlike the other/ rich men of his community,/ he does not look/ down upon the

poor/ no error

1) A

2) B

3) C

4) D

5) E

Correct Option is: 5


Your Option is: 4
Result: Wrong
Timetaken: 0.0 secs
Explanation: No error

8.
Read the passage and answer the questions that follow on the basis of the

information provided in the passage.

From the 197 million square miles, which make up the surface of the globe, 71 per

cent is covered by the interconnecting bodies of marine water; the Pacific Ocean
alone covers half the Earth and averages near 14,000 feet in depth. The portions

which rise above sea level are the continents-Eurasia, Africa; North America, South

America, Australia, and Antarctica. The submerged borders of the continental

masses are the continental shelves, beyond which lie the deep-sea basins.

The ocean are deepest not in the center but in some elongated furrows, or long

narrow troughs, called deeps. These profound troughs have a peripheral

arrangement, notably around the borders of the pacific and Indian oceans. The

position of the deeps, like the highest mountains, are of recent origin, since

otherwise they would have been filled with waste from the lands. This is further

strengthened by the observation that the deeps are quite often, where world-

shaking earthquakes occur. To cite an example, the "tidal wave" that in April, 1946,

caused widespread destruction along Pacific coasts resulted from a strong

earthquake on the floor of the Aleutian Deep.

The topography of the ocean floors is none too well known, since in great areas the

available soundings are hundreds or even thousands of miles apart. However, the

floor of the Atlantic is becoming fairly well known as a result of special surveys

since 1920. A broad, well-defined ridge-the Mid-Atlantic ridgeruns north and south

between Africa and the two Americas and numerous other major irregularities

diversify the Atlantic floor. Closely spaced soundings show that many parts of the

oceanic floors are as rugged as mountainous regions of the continents. Use of the

recently perfected method of submarine topography. During world war II great

strides were made in mapping submarine surfaces, particularly in many parts of the

vast Pacific basin

Most of the continents stand on an average of 2870 feet above sea level. North

America averages 2300 feet; Europe averages only 1150 feet; and Asia, the highest

of the larger continental subdivisions, averages 3200 feet. Mount Everest, which is

the highest point in the globe, is 29,000 feet above the sea; and as the greatest

known depth in the sea is over 35,000 feet, the maximum relief (that is, the
difference in altitude between the lowest and highest points) exceeds 64,000 feet,

or exceeds 12 miles. The continental masses and the deep-sea basins are relief

features of the first order; the deeps, ridges, and volcanic cones that diversify the

sea floor, as well as the plains, plateaus, and mountains of the continents, are relief

features of the second order. The lands are unendingly subject to a complex of

activities summarized in the term erosion, which first sculptures them in great

detail and then tends to reduce them ultimately to sea level. The modeling of the

landscape by weather, running water, and other agents is apparent to the keenly

observant eye and causes thinking people to speculate on what must be the final

result of the ceaseless wearing down of the lands. Much before there was any

recognizable science as geology, Shakespeare wrote "the revolution of the times

makes mountains level."

The deeps are subject to change caused by

1) erosion

2) soundings

3) earthquakes

4) waste

5) weathering

Correct Option
3
is:
Your Option is: 1
Result: Wrong
Timetaken: 0.0 secs
From the second paragraph in the passage,it is very clear
Explanation:
that the deeps are subject to change caused by earthquakes.

9.
In each of the following questions, a paragraph or a sentence has been broken up
into different parts. The parts have been scrambled and numbered as given below.

Choose the correct order of these parts from the given alternatives.

1) food supply

2) storage, distribution and handling

3) pastoral industry and fishing

4) besides increasing

5) by preventing wastage in

6) the productivity from agriculture

7)can be increased

1) 1,7,5,2,4,3,6

2) 4,1,6,7,5,3,2

3) 4,6,3,1,7,5,2

4) 6,3,5,7,4,1,2

Correct
3
Option is:
Your Option
2
is:
Result: Wrong
Timetaken: 0.0 secs
Correct sequence is,'Besides increasing the productivity
from agriculture pastoral industry and fishing, food supply
Explanation:
can be increased by preventing wastage in
storage,distribution and handling'. So the answer option is 3.

10.
Read the passage and answer the questions that follow on the basis of the

information provided in the passage.

From the 197 million square miles, which make up the surface of the globe, 71 per
cent is covered by the interconnecting bodies of marine water; the Pacific Ocean

alone covers half the Earth and averages near 14,000 feet in depth. The portions

which rise above sea level are the continents-Eurasia, Africa; North America, South

America, Australia, and Antarctica. The submerged borders of the continental

masses are the continental shelves, beyond which lie the deep-sea basins.

The ocean are deepest not in the center but in some elongated furrows, or long

narrow troughs, called deeps. These profound troughs have a peripheral

arrangement, notably around the borders of the pacific and Indian oceans. The

position of the deeps, like the highest mountains, are of recent origin, since

otherwise they would have been filled with waste from the lands. This is further

strengthened by the observation that the deeps are quite often, where world-

shaking earthquakes occur. To cite an example, the "tidal wave" that in April, 1946,

caused widespread destruction along Pacific coasts resulted from a strong

earthquake on the floor of the Aleutian Deep.

The topography of the ocean floors is none too well known, since in great areas the

available soundings are hundreds or even thousands of miles apart. However, the

floor of the Atlantic is becoming fairly well known as a result of special surveys

since 1920. A broad, well-defined ridge-the Mid-Atlantic ridgeruns north and south

between Africa and the two Americas and numerous other major irregularities

diversify the Atlantic floor. Closely spaced soundings show that many parts of the

oceanic floors are as rugged as mountainous regions of the continents. Use of the

recently perfected method of submarine topography. During world war II great

strides were made in mapping submarine surfaces, particularly in many parts of the

vast Pacific basin

Most of the continents stand on an average of 2870 feet above sea level. North

America averages 2300 feet; Europe averages only 1150 feet; and Asia, the highest

of the larger continental subdivisions, averages 3200 feet. Mount Everest, which is

the highest point in the globe, is 29,000 feet above the sea; and as the greatest
known depth in the sea is over 35,000 feet, the maximum relief (that is, the

difference in altitude between the lowest and highest points) exceeds 64,000 feet,

or exceeds 12 miles. The continental masses and the deep-sea basins are relief

features of the first order; the deeps, ridges, and volcanic cones that diversify the

sea floor, as well as the plains, plateaus, and mountains of the continents, are relief

features of the second order. The lands are unendingly subject to a complex of

activities summarized in the term erosion, which first sculptures them in great

detail and then tends to reduce them ultimately to sea level. The modeling of the

landscape by weather, running water, and other agents is apparent to the keenly

observant eye and causes thinking people to speculate on what must be the final

result of the ceaseless wearing down of the lands. Much before there was any

recognizable science as geology, Shakespeare wrote "the revolution of the times

makes mountains level."

The highest point on North America is

1) 2870 feet above sea level

2) not mentioned in the passage

3) higher than the highest point in Europe

4) 2300 feet above sea level

5) in Mexico

Correct Option
2
is:
Your Option is: 2
Result: Correct
Timetaken: 0.0 secs
There is no information about the highest point in North
Explanation:
America.
Read each sentence to find if there is any grammatical error in it. If there is any

error, it will be only one part of the sentence. The number or alphabet of that part is
11.
your answer.( Disregard punctuation errors if any)

The firm show / began / when we arrived / in the hall/no error

1) A

2) B

3) C

4) D

5) E

Correct Option is: 2


Your Option is: 1
Result: Wrong
Timetaken: 0.0 secs
Explanation: 'Had begun' must be used in place of 'began'

12.
Read the passage and answer the questions that follow on the basis of the

information provided in the passage.

Outside, the rain continued to run down the screened windows of Mrs. Sennett's

little Cape Cod cottage. The long weeds and grass that composed the front yard

dripped against the blurred background of the bay, where the water was almost the

color of the grass. Mrs. Sennett's five charges were vigorously playing house in the

dining room. (In the wintertime, Mrs. Sennett was housekeeper for a Mr. Curley, in

Boston, and during the summers the Curley children boarded with her on the

Cape.) My expression must have changed. " Are those children making too much

noise?" Mrs. Sennett demanded, a sort of wave going over her that might mark the

beginning of her getting up out of her chair. I shook my head no, and gave her a
little push on the shoulder to keep her seated. Mrs. Sennett was almost stone-deaf

and had been for a long time, but she could read lips. You could talk to her without

making any sound yourself, if you wanted to, and she more than kept up her side of

the conversation in a loud, rusty voice that dropped weirdly every now and then

into a whisper. She adored talking. To look at Mrs. Sennett made me think of

eighteenth-century England and its literary figures. Her hair must have been sadly

thin, because she always wore, indoors and out, either a hat or a sort of turban,

and sometimes she wore both. The rims of her eyes were dark; she looked very ill.

Mrs. Sennett and I continued talking. She said she really didn't think she'd stay with

the children another winter. Their father wanted her to, but it was too much for her.

She wanted to stay right here in the cottage. The afternoon was getting along, and

I finally left because I knew that at four o'clock Mrs. Sennett's "sit down" was over

and she started to get supper. At six o'clock, from my nearby cottage, I saw

Theresa coming through the rain with a shawl over her head. She was bringing me

a six-inchsquare piece of spice cake , still hot from the oven and kept warm

between two soup plates. A few days later I learned from the twins, who brought

over gifts of firewood and blackberries, that their father was coming the next

morning, bringing their aunt and her husband and their cousin. Mrs. Sennett had

promised to take them all on a picnic at the pond some pleasant day. On the fourth

day of their visit, Xavier arrived with a note. It was from Mrs. Sennett, written in

blue ink, in a large, serene, ornamented hand, on linenfinish paper:

Tomorrow is the last day Mr. Curley has and the Children all wanted the Picnic so

much. The Men can walk to the Pond but it is too far for the Children. I see your

Friend has a car and I hate to ask this but could you possibly drive us to the Pond

tomorrow morning? . . .Very sincerely yours, Carmen Sennett After the picnic, Mrs.

Sennett's presents to me were numberless. It was almost time for the children to

go back to school in South Boston. Mrs. Sennett insisted that she was not going;

their father was coming down again to get them and she was just going to stay. He
would have to get another housekeeper. She said this over and over to me, loudly,

and her turbans and kerchiefs grew more and more distrait. One evening, Mary

came to call on me and we sat on an old table in the back yard to watch the sunset.

"Papa came today, " she said, "and we've got to go back day after tomorrow. ""Is

Mrs. Sennett going to stay here? ""She said at supper she was. She said this time

she really was, because she'd said that last year and came back, but now she

means it ."I said, "Oh dear," scarcely knowing which side I was on. "It was awful at

supper. I cried and cried." "Did Theresa cry?" "Oh, we all cried. Papa cried, too. We

always do." "But don't you think Mrs. Sennett needs a rest?" "Yes, but I think she'll

come, though. Papa told her he'd cry every single night at supper if she didn't, and

then we all did." The next day I heard that Mrs. Sennett was going back with them

just to " help settle." She came over the following morning to say goodbye,

supported by all five children. She was wearing her traveling hat of black satin and

black straw, with sequins. High and somber, above her ravaged face, it had quite a

Spanishgrandee air. "This isn't really goodbye," she said. "I'll be backas soon as I

get these bad ,noisy children off my hands." But the children hung on to her skirt

and tugged at her sleeves, shaking their heads frantically, silently saying, "No! No!

No!" to her with their puckered-up mouths

At what point does Mr. Curley cry at the supper table?

1) Before Mary and the narrator sit and watch the sunset

2) Before Mrs. Sennett tells the narrator she doubts she will stay another winter
with the children

3) Before the children spend a rainy afternoon playing house in the dining room

4) After the narrator learns that Mrs. Sennett will return to Boston

Correct Option
1
is:
Your Option is: 1
Result: Correct
Timetaken: 0.0 secs
From the last paragraph,it is clear that Mr.Curley cry at the
Explanation: supper table before Mary and narrator sit and watch the
sunset.

In each of the following questions, a paragraph or a sentence has been broken up

into different parts. The parts have been scrambled and numbered as given below.

Choose the correct order of these parts from the given alternatives.

1) not only for

2) but also for

3) lumbering

13. 4) construction purposes

5) as an occupation

6) on modern lines

7) the manufacture of wood pulp, paper , resins etc.

8) owing to the great demand for timber

9) has developed

1) 3,9,6,5,8,1,4,2,7

2) 3,8,9,5,6,1,7,2,4

3) 3,5,9,6,8,1,4,2,7

4) 5,3,9,6,8,1,7,2,4

Correct
3
Option is:
Your Option 4
is:
Result: Wrong
Timetaken: 0.0 secs
Correct sequence is,'Lumbering as an occupation has
developed on modern lines owing to the great demand for
Explanation: timber not only for construction purposes but also for the
manufacture of wood pulp, paper,resins, ets.'. Correct option
is 3

In each of the following questions, a paragraph or a sentence has been broken up

into different parts. The parts have been scrambled and numbered as given below.

Choose the correct order of these parts from the given alternatives.

1) The African elephant is usually larger

2) being about three and a half metres in height

3) than the Indian


14.
4) and 6000 kg in weight

5) It has enormous ears

6) which are valued for the ivory

7) and very long tusks

8) that they contain

1) 1,3,2,4,5,7,6,8

2) 1,4,2,3,6,5,8,7

3) 5,3,6,2,7,4,1,8

4) 5,6,2,4,1,3,7,8

Correct
1
Option is:
Your Option 2
is:
Result: Wrong
Timetaken: 0.0 secs
Correct sequence is,'The African elephant is usually larger
than the Indian being about three and a half metres in height
Explanation: and 6000 kg in weight.It has enormous ears and very long
tusks which are valued for the ivory that they contain'.
Answer option is 1.

In each of the following questions, some sentence are given which are on the same

theme. decide which sentence is the most preferable with respect to grammar;

meaning and usage, suitable for formal writing in English. Find the correct

sentence.

15. A) Without your help must try to carry out my task alone.

B) Barring your help I should try to carry out my task alone

C) Besides help from you, I must try to carry out my task alone.

D) Failing your help I must try to carry out my task alone

1) A

2) B

3) C

4) D

Correct Option
4
is:
Your Option is: 4
Result: Correct
Timetaken: 0.0 secs
Explanation: In first three sentences,First and second half of the
sentences are not connected properly.Correct answer is 4

Read each sentence to find if there is any grammatical error in it. If there is any

error, it will be only one part of the sentence. The number or alphabet of that part is

16. your answer.( Disregard punctuation errors if any)

Our company can / no longer afford / over-manned unit as/; out profit has lower./

no error

1) A

2) B

3) C

4) D

5) E

Correct Option is: 4


Your Option is: 4
Result: Correct
Timetaken: 0.0 secs
Explanation: Instead of 'has', the word 'is' must be used.

17.
Read the passage and answer the questions that follow on the basis of the

information provided in the passage.

Outside, the rain continued to run down the screened windows of Mrs.Sennett's

little Cape Cod cottage. The long weeds and grass that composed the front yard

dripped against the blurred background of the bay, where the water was almost the

color of the grass. Mrs. Sennett's five charges were vigorously playing house in the

dining room. (In the wintertime, Mrs. Sennett was housekeeper for a Mr. Curley, in

Boston, and during the summers the Curley children boarded with her on the Cape.

My expression must have changed. " Are those children making too much noise?"
Mrs. Sennett demanded, a sort of wave going over her that might mark the

beginning of her getting up out of her chair. I shook my head no, and gave her a

little push on the shoulder to keep her seated. Mrs. Sennett was almost stone-deaf

and had been for a long time, but she could read lips. You could talk to her without

making any sound yourself, if you wanted to, and she more than kept up her side of

the conversation in a loud, rusty voice that dropped weirdly every nowand then into

a whisper. She adored talking. To look at Mrs. Sennett made me think of

eighteenth-century England and its literary figures. Her hair must have been sadly

thin, because she always wore, indoors and out, either a hat or a sort of turban,

and sometimes she wore both. The rims of her eyes were dark; she looked very ill.

Mrs. Sennett and I continued talking. She said she really didn't think she'd stay with

the children another winter. Their father wanted her to, but it was too much for her.

She wanted to stay right here in the cottage. The afternoon was getting along, and

I finally left because I knew that at four o'clock Mrs. Sennett's "sit down" was over

and she started to get supper. At six o'clock, from my nearby cottage, I saw

Theresa coming through the rain with a shawl over her head. She was bringing me

a six-inchsquare piece of spice cake , still hot from the oven and kept warm

between two soup plates. A few days later I learned from the twins, who brought

over gifts of firewood and blackberries, that their father was coming the next

morning, bringing their aunt and her husband and their cousin. Mrs. Sennett had

promised to take them all on a picnic at the pond some pleasant day. On the fourth

day of their visit, Xavier arrived with a note. It was from Mrs. Sennett, written in

blue ink, in a large, serene, ornamented hand, on linenfinish paper:

Tomorrow is the last day Mr. Curley has and the Children all wanted the Picnic so

much. The Men can walk to the Pond but it is too far for the Children. I see your

Friend has a car and I hate to ask this but could you possibly drive us to the Pond

tomorrow morning? . . .Very sincerely yours, Carmen Sennett After the picnic, Mrs.

Sennett's presents to me were numberless. It was almost time for the children to
go back to school in South Boston. Mrs. Sennett insisted that she was not going;

their father was coming down again to get them and she was just going to stay. He

would have to get another housekeeper. She said this over and over to me, loudly,

and her turbans and kerchiefs grew more and more distrait. One evening, Mary

came to call on me and we sat on an old table in the back yard to watch the sunset.

"Papa came today, " she said, "and we've got to go back day after tomorrow. ""Is

Mrs. Sennett going to stay here? ""She said at supper she was. She said this time

she really was, because she'd said that last year and came back, but now she

means it ."I said, "Oh dear," scarcely knowing which side I was on. "It was awful at

supper. I cried and cried." "Did Theresa cry?" "Oh, we all cried. Papa cried, too. We

always do." "But don't you think Mrs. Sennett needs a rest?" "Yes, but I think she'll

come, though. Papa told her he'd cry every single night at supper if she didn't, and

then we all did." The next day I heard that Mrs. Sennett was going back with them

just to " help settle." She came over the following morning to say goodbye,

supported by all five children. She was wearing her traveling hat of black satin and

black straw, with sequins. High and somber, above her ravaged face, it had quite a

Spanishgrandee air.

"This isn't really goodbye," she said. "I'll be back as soon as I get these bad ,noisy

children off my hands." But the children hung on to her skirt and tugged at her

sleeves, shaking their heads frantically, silently saying, "No! No! No!" to her with

their puckered-up mouths

Considering the events of the entire passage, it is most reasonable to infer that

Mrs. Sennett calls the children bad because she:

1) is bothered by the noise they are making.

2) doesn't like them hanging on her skirt.

3) doesn't want to reveal her affection for them.

4) is angry that they never do what she tells them.


Correct Option
3
is:
Your Option is: 2
Result: Wrong
Timetaken: 0.0 secs
From the passage,it is very clear that Mrs.Sennett calls the
Explanation: children bad because she does not want to reveal her
affection for them.

18.
Read the passage and answer the questions that follow on the basis of the

information provided in the passage.

From the 197 million square miles, which make up the surface of the globe, 71 per

cent is covered by the interconnecting bodies of marine water; the Pacific Ocean

alone covers half the Earth and averages near 14,000 feet in depth. The portions

which rise above sea level are the continents-Eurasia, Africa; North America, South

America, Australia, and Antarctica. The submerged borders of the continental

masses are the continental shelves, beyond which lie the deep-sea basins.

The ocean are deepest not in the center but in some elongated furrows, or long

narrow troughs, called deeps. These profound troughs have a peripheral

arrangement, notably around the borders of the pacific and Indian oceans. The

position of the deeps, like the highest mountains, are of recent origin, since

otherwise they would have been filled with waste from the lands. This is further

strengthened by the observation that the deeps are quite often, where world-

shaking earthquakes occur. To cite an example, the "tidal wave" that in April, 1946,

caused widespread destruction along Pacific coasts resulted from a strong

earthquake on the floor of the Aleutian Deep.

The topography of the ocean floors is none too well known, since in great areas the

available soundings are hundreds or even thousands of miles apart. However, the
floor of the Atlantic is becoming fairly well known as a result of special surveys

since 1920. A broad, well-defined ridge-the Mid-Atlantic ridgeruns north and south

between Africa and the two Americas and numerous other major irregularities

diversify the Atlantic floor. Closely spaced soundings show that many parts of the

oceanic floors are as rugged as mountainous regions of the continents. Use of the

recently perfected method of submarine topography. During world war II great

strides were made in mapping submarine surfaces, particularly in many parts of the

vast Pacific basin

Most of the continents stand on an average of 2870 feet above sea level. North

America averages 2300 feet; Europe averages only 1150 feet; and Asia, the highest

of the larger continental subdivisions, averages 3200 feet. Mount Everest, which is

the highest point in the globe, is 29,000 feet above the sea; and as the greatest

known depth in the sea is over 35,000 feet, the maximum relief (that is, the

difference in altitude between the lowest and highest points) exceeds 64,000 feet,

or exceeds 12 miles. The continental masses and the deep-sea basins are relief

features of the first order; the deeps, ridges, and volcanic cones that diversify the

sea floor, as well as the plains, plateaus, and mountains of the continents, are relief

features of the second order. The lands are unendingly subject to a complex of

activities summarized in the term erosion, which first sculptures them in great

detail and then tends to reduce them ultimately to sea level. The modeling of the

landscape by weather, running water, and other agents is apparent to the keenly

observant eye and causes thinking people to speculate on what must be the final

result of the ceaseless wearing down of the lands. Much before there was any

recognizable science as geology, Shakespeare wrote "the revolution of the times

makes mountains level."

The peripheral furrows or deeps are found

1) only in the pacific and Indian oceans


2) near earthquakes

3) near the shore

4) in the center of the ocean

5) to be 14,000 feet in depth in the pacific.

Correct Option
3
is:
Your Option is: 3
Result: Correct
Timetaken: 0.0 secs
From the second paragraph in the passage,it is clear that
Explanation:
deeps are found in the center of the ocean.

In each of the following questions, a paragraph or a sentence has been broken up

into different parts. The parts have been scrambled and numbered as given below.

Choose the correct order of these parts from the given alternatives.

1) in some of the developed countries

2) in the developing countries

3) mostly in the form of beef, pork etc.

19. 4) about 180 kilograms

5) per capita consumption is

6) of grain per capita annually

7) the people

8) five times that amount

9) consume only

1) 1,7,9,4,6,2,5,8,3

2) 2,5,8,2,7,9,4,6,3
3) 7,2,9,4,6,1,5,8,3

4) 5,8,1,2,7,9,4,6,3

Correct
3
Option is:
Your Option
1
is:
Result: Wrong
Timetaken: 0.0 secs
Correct sequence is,'The people in the developing countries
consume only about 180 kilograms of grain per capita
Explanation: annually .In some of the developed countries per capita
consumption is five times that amount mostly in the form of
beef,pork,etc.' Correct option is 3.

20.
Read the passage and answer the questions that follow on the basis of the

information provided in the passage.

From the 197 million square miles, which make up the surface of the globe, 71 per

cent is covered by the interconnecting bodies of marine water; the Pacific Ocean

alone covers half the Earth and averages near 14,000 feet in depth. The portions

which rise above sea level are the continents-Eurasia, Africa; North America, South

America, Australia, and Antarctica. The submerged borders of the continental

masses are the continental shelves, beyond which lie the deep-sea basins.

The ocean are deepest not in the center but in some elongated furrows, or long

narrow troughs, called deeps. These profound troughs have a peripheral

arrangement, notably around the borders of the pacific and Indian oceans. The

position of the deeps, like the highest mountains, are of recent origin, since

otherwise they would have been filled with waste from the lands. This is further

strengthened by the observation that the deeps are quite often, where world-

shaking earthquakes occur. To cite an example, the "tidal wave" that in April, 1946,

caused widespread destruction along Pacific coasts resulted from a strong


earthquake on the floor of the Aleutian Deep.

The topography of the ocean floors is none too well known, since in great areas the

available soundings are hundreds or even thousands of miles apart. However, the

floor of the Atlantic is becoming fairly well known as a result of special surveys

since 1920. A broad, well-defined ridge-the Mid-Atlantic ridgeruns north and south

between Africa and the two Americas and numerous other major irregularities

diversify the Atlantic floor. Closely spaced soundings show that many parts of the

oceanic floors are as rugged as mountainous regions of the continents. Use of the

recently perfected method of submarine topography. During world war II great

strides were made in mapping submarine surfaces, particularly in many parts of the

vast Pacific basin

Most of the continents stand on an average of 2870 feet above sea level. North

America averages 2300 feet; Europe averages only 1150 feet; and Asia, the highest

of the larger continental subdivisions, averages 3200 feet. Mount Everest, which is

the highest point in the globe, is 29,000 feet above the sea; and as the greatest

known depth in the sea is over 35,000 feet, the maximum relief (that is, the

difference in altitude between the lowest and highest points) exceeds 64,000 feet,

or exceeds 12 miles. The continental masses and the deep-sea basins are relief

features of the first order; the deeps, ridges, and volcanic cones that diversify the

sea floor, as well as the plains, plateaus, and mountains of the continents, are relief

features of the second order. The lands are unendingly subject to a complex of

activities summarized in the term erosion, which first sculptures them in great

detail and then tends to reduce them ultimately to sea level. The modeling of the

landscape by weather, running water, and other agents is apparent to the keenly

observant eye and causes thinking people to speculate on what must be the final

result of the ceaseless wearing down of the lands. Much before there was any

recognizable science as geology, Shakespeare wrote "the revolution of the times

makes mountains level."


The highest mountains are

1) oldest

2) in excess of 12 miles

3) near the deeps

4) relief features of the first order

5) of recent origin

Correct Option
5
is:
Your Option is: 2
Result: Wrong
Timetaken: 0.0 secs
From the third line in second paragraph,it is very clear that
Explanation:
the highest mountains are of recent origin.

In each of the following questions, some sentence are given which are on the same

theme. decide which sentence is the most preferable with respect to grammar;

meaning and usage, suitable for formal writing in English. Find the correct

sentence.

A) Since the dividend being declared than the notices were prepared for mailing.

B) Scarcely had the dividend been declared than the notices were sent out.

21. C) They had no sooner declared the dividend when they sent the notices to the

stockholders.

D) No sooner had the dividend been declared than the notices were prepared for

mailing.

E) The company hardly declared the dividend till the notices were prepared for

mailing
1) A

2) B

3) C

4) D

5) E

Correct Option is: 4


Your Option is: 4
Result: Correct
Timetaken: 0.0 secs
1.First clause is wrong.
2.Instead of than, 'when' should be used.
Explanation: 3.Instead of when,'than' must be used.
4.No error.
5.First clause is wrong.

In each of the following questions, some sentence are given which are on the same

theme. decide which sentence is the most preferable with respect to grammar;

meaning and usage, suitable for formal writing in English. Find the correct

sentence.

A) As one travels from Karjat to Khandala, be finds the line most beautifully laid.
22.
B) Travelling from Karjat to Khandala, one finds the line most beautifully laid

C) While travelling from Karjat to Khandala, one would find the line most beautifully

laid

D) If one travels from Karjat to Khandala, he will find the line most beautifully laid

1) A

2) B

3) C
4) D

Correct
2
Option is:
Your Option
2
is:
Result: Correct
Timetaken: 0.0 secs
In sentences 1,3 and 4,First and second half of the sentences
Explanation: are not connected properly to form a meaningful
sentence.Correct answer is option 2.

23.
Read the passage and answer the questions that follow on the basis of the

information provided in the passage.

From the 197 million square miles, which make up the surface of the globe, 71 per

cent is covered by the interconnecting bodies of marine water; the Pacific Ocean

alone covers half the Earth and averages near 14,000 feet in depth. The portions

which rise above sea level are the continents-Eurasia, Africa; North America, South

America, Australia, and Antarctica. The submerged borders of the continental

masses are the continental shelves, beyond which lie the deep-sea basins.

The ocean are deepest not in the center but in some elongated furrows, or long

narrow troughs, called deeps. These profound troughs have a peripheral

arrangement, notably around the borders of the pacific and Indian oceans. The

position of the deeps, like the highest mountains, are of recent origin, since

otherwise they would have been filled with waste from the lands. This is further

strengthened by the observation that the deeps are quite often, where world-

shaking earthquakes occur. To cite an example, the "tidal wave" that in April, 1946,

caused widespread destruction along Pacific coasts resulted from a strong

earthquake on the floor of the Aleutian Deep.


The topography of the ocean floors is none too well known, since in great areas the

available soundings are hundreds or even thousands of miles apart. However, the

floor of the Atlantic is becoming fairly well known as a result of special surveys

since 1920. A broad, well-defined ridge-the Mid-Atlantic ridgeruns north and south

between Africa and the two Americas and numerous other major irregularities

diversify the Atlantic floor. Closely spaced soundings show that many parts of the

oceanic floors are as rugged as mountainous regions of the continents. Use of the

recently perfected method of submarine topography. During world war II great

strides were made in mapping submarine surfaces, particularly in many parts of the

vast Pacific basin

Most of the continents stand on an average of 2870 feet above sea level. North

America averages 2300 feet; Europe averages only 1150 feet; and Asia, the highest

of the larger continental subdivisions, averages 3200 feet. Mount Everest, which is

the highest point in the globe, is 29,000 feet above the sea; and as the greatest

known depth in the sea is over 35,000 feet, the maximum relief (that is, the

difference in altitude between the lowest and highest points) exceeds 64,000 feet,

or exceeds 12 miles. The continental masses and the deep-sea basins are relief

features of the first order; the deeps, ridges, and volcanic cones that diversify the

sea floor, as well as the plains, plateaus, and mountains of the continents, are relief

features of the second order. The lands are unendingly subject to a complex of

activities summarized in the term erosion, which first sculptures them in great

detail and then tends to reduce them ultimately to sea level. The modeling of the

landscape by weather, running water, and other agents is apparent to the keenly

observant eye and causes thinking people to speculate on what must be the final

result of the ceaseless wearing down of the lands. Much before there was any

recognizable science as geology, Shakespeare wrote "the revolution of the times

makes mountains level."


We may conclude from this passage that earth quakes

1) Occur more frequently in newly formed land or sea formations

2) Are caused by the weight of the water

3) Cause erosion

4) Occur in the deeps

5) Will ultimately "make mountains level".

Correct Option
1
is:
Your Option is: 2
Result: Wrong
Timetaken: 0.0 secs
From the passage,it is very clear that earthquakes occur
Explanation:
more frequently in newly formed land or sea formations

In each of the following questions, some sentence are given which are on the same

theme. decide which sentence is the most preferable with respect to grammar;

meaning and usage, suitable for formal writing in English. Find the correct

sentence.

24. A) Nobody will want to play in his team if he does not treat people kindly

B) If he does not treat people kindly, nobody will play to want his team

C) Nobody will treat people kindly, he does not want to play in this team

D) Nobody will want to treat people, if he does not play in his team kindly

1) A

2) B

3) C
4) D

Correct Option
1
is:
Your Option is: 4
Result: Wrong
Timetaken: 0.0 secs
A)No error.Correct answer.
B)In second half of the sentence,'want to play in' must be
used in place of 'play to want'.
Explanation: C)First and second half of the sentence is not connected
properly.
D)First and second half of the sentence is not connected
properly.

In each of the following questions, a paragraph or a sentence has been broken up

into different parts. The parts have been scrambled and numbered as given below.

Choose the correct order of these parts from the given alternatives.

1. 1) not only for 2) but also for 3) lumbering 4) construction purposes 5) as an

occupation6) on modern lines 7) the manufacture of wood pulp. paper, resins etc.

8) owing to the great demand for timber 9) has developed

A) 3,9,6,5,8,1,4,2,7 B) 3,8,9,5,6,1,7,2,4

C) 3,5,9,6,8,1,4,2,7 D)5,3,9,6,8,1,7,2,4

1) A

2) B

3) C

4) D
Correct Option is: 3
Your Option is: 2
Result: Wrong
Timetaken: 0.0 secs
Explanation: No Explanation

2.

Read the passage and answer the questions that follow on the basis of the

information provided in the passage

One of the most dangerous drugs for pregnant women to consume is alcohol.

Because alcohol is delivered quickly into the blood and passes quickly into the

tissues and membranes, the human fetus is particularly vulnerable to its effects. In

fact, the negative effects on a fetus are so pronounced that babies born after

exposure to alcohol are said to be suffering from fetal alcohol syndrome. As a

pregnant woman drinks alcohol, the alcohol is passed into her her bloodstream

almost simultaneously. Moreover, because the bloodstream of the fetus is

inextricably tied to that of the mother, the alcohol passes directly into the

bloodstream of the fetus as well. And, what is more, the concentration of alcohol in

the fetus is exactly the same as in the mother. For the mother, this concentration is

not a problem because her liver can remove one ounce of alcohol from her system

per hour. However, the fetus's liver is not completely developed (how developed it is

depends on its stage of development). The rate at which it is able to eliminate the

alcohol from the blood of the fetus is much slower. Eventually, the alcohol will be

returned to the mother\'s system by passing across the placenta, but this process is

slow. By the time this takes place, major neurological damage may have already

occurred. Research has shown that as little as one drink of alcohol can produce

significant, irreversible damage to the fetus. Babies born after exposure to alcohol

generally exhibit facial distortion, inability to concentrate, and difficulty in

remembering. Simply speaking, it is imperative that pregnant women avoid


alcohol.

Following are some sample questions on this passage:

It can be inferred that the development of a fetal liver depends on

a how many months pregnant the mother is

b how much alcohol the mother has consumed

c how large the fetus is

d how well the mother has taken care of the fetus

1) a

2) b

3) c

4) d

Correct Option is: 1


Your Option is: 2
Result: Wrong
Timetaken: 0.0 secs
Explanation: No Explanations

Read each sentence to find if there is any grammatical error in it. If there is any

error, it will be only one part of the sentence. The number or alphabet of that part is
3.
your answer.(Disregard punctuation errors if any)

The major / along with / his soldiers / were killed in the field / no error.

ABCDE

1) A

2) B
3) C

4) D

5) E

Correct Option is: 4


Your Option is: 3
Result: Wrong
Timetaken: 0.0 secs
Explanation: No Explanation

In each of the following questions, some sentence are given which are on the same

theme. decide which sentence is the most preferable with respect to grammar;

meaning and usage, suitable for formal writing in English. Find the correct

sentence.
4.
A) The dacoits being stopped to divide the booty, the police overtook them.

B) The dacoits having stopped to divide the booty, the police had overtaken them

C) The dacoits having stopped for dividing the booty, the police overtook them

D) The dacoits having been stopped for dividing the booty, the police overtook

them

1) A

2) B

3) C

4) D

Correct Option is: 3


Your Option is: 2
Result: Wrong
Timetaken: 0.0 secs
Explanation: No Explanations

5.

Read the passage and answer the questions that follow on the basis of the

information provided in the passage.

The establishment of the third Reich influenced events in American history by

starting a chain of events which culminated in war between Germany and the

United States. The complete destruction of democracy, the persecution of laws, the

war on religion, the cruelty and barbarism of the Nazis and especially, the plans of

Germany and her allies, Italy and Japan, for world conquest caused great

indignation in this country and brought on fear of another world war. While speaking

out against Hitler\'s atrocities, the American profile generally favored isolationist

policies, and neutrality. The neutrality acts of 1935 and 1936 prohibited trade with

any belligerents or loans to them. In 1937 the president was empowered to declare

an arms embargo in wars between nations at his discretion American opinion began

to change somewhat after President Roosevelt\'s quarantine the aggressor speech

at Chicago (1937) in which he severely criticized Hitler\'s policies. Germany's

seizure of Austria and Munich pact for the partition of Czechoslovakia (1938) also

around the American people. The conquest of Czechoslovakia in March 1939 was

another rude awakening to the menace of the third Reich. In August, 1939, came

the shock of the Nazi - Soviet pact and in September the attack on Poland and the

outbreak of European war. The United States attempt to maintain neutrality in spite

of sympathy for the democracies arranged against the Third Reich. The Neutrality

act of 1939 repeated the arms embargo and permitted 'cash' and 'carry' exports of

arms to belligerent nations. A strong national defense program was begun. A draft

act was passed (1940) to strengthen the military services. A Lend - Lease Act

(1940) authorized the president to sell, exchange or lend materials to any county
deemed necessary by him for the defence of the United States. Help was given to

Britain territory in the western Hemisphere. In August 1941, President Roosevelt

and prime minister Churchill met and issued the Atlantic Charter which proclaimed

the kind of a world which should be established after the war. In December 1941,

Japan launched the unprovoked attack on the United States at Pearl harbour,

immediately thereafter Germany declared war on the United States.

One item occurring 1937 that the author does not mention in the list of actions that

alienated the American Public was

a The persecution of religious groups

b Nazi barbarism

c The pacts with Italy

d German plans for conquest of the world

e The burning of the Reich tag.

1) a

2) b

3) c

4) d

5) e

Correct Option is: 5


Your Option is: 2
Result: Wrong
Timetaken: 0.0 secs
Explanation: No Explanations

6.

Read the passage and answer the questions that follow on the basis of the
information provided in the passage.

The establishment of the third Reich influenced events in American history by

starting a chain of events which culminated in war between Germany and the

United States. The complete destruction of democracy, the persecution of laws, the

war on religion, the cruelty and barbarism of the Nazis and especially, the plans of

Germany and her allies, Italy and Japan, for world conquest caused great

indignation in this country and brought on fear of another world war. While speaking

out against Hitler\'s atrocities, the American profile generally favored isolationist

policies, and neutrality. The neutrality acts of 1935 and 1936 prohibited trade with

any belligerents or loans to them. In 1937 the president was empowered to declare

an arms embargo in wars between nations at his discretion American opinion began

to change somewhat after President Roosevelt\'s quarantine the aggressor speech

at Chicago (1937) in which he severely criticized Hitler\'s policies. Germany's

seizure of Austria and Munich pact for the partition of Czechoslovakia (1938) also

around the American people. The conquest of Czechoslovakia in March 1939 was

another rude awakening to the menace of the third Reich. In August, 1939, came

the shock of the Nazi - Soviet pact and in September the attack on Poland and the

outbreak of European war. The United States attempt to maintain neutrality in spite

of sympathy for the democracies arranged against the Third Reich. The Neutrality

act of 1939 repeated the arms embargo and permitted 'cash' and 'carry' exports of

arms to belligerent nations. A strong national defense program was begun. A draft

act was passed (1940) to strengthen the military services. A Lend - Lease Act

(1940) authorized the president to sell, exchange or lend materials to any county

deemed necessary by him for the defence of the United States. Help was given to

Britain territory in the western Hemisphere. In August 1941, President Roosevelt

and prime minister Churchill met and issued the Atlantic Charter which proclaimed

the kind of a world which should be established after the war. In December 1941,

Japan launched the unprovoked attack on the United States at Pearl harbour,
immediately thereafter Germany declared war on the United States.

USA entered the war against Germany

a because Pearl Harbor was attacked

b after peaceful efforts had failed

c because Germany declare war against it

d because Japan was an ally of Germany

e after Germany had signed the Nazi - Soviet pact

1) a

2) b

3) c

4) d

5) e

Correct Option is: 3


Your Option is: 4
Result: Wrong
Timetaken: 0.0 secs
Explanation: No Explanations

7.

In each of the following questions, some sentence are given which are on the same

theme. decide which sentence is the most preferable with respect to grammar;

meaning and usage, suitable for formal writing in English. Find the correct

sentence.

A) The harassed wife shot herself after bidding her husband the last good bye with

a gun

B) The harassed wife with a gun shot herself after bidding her husband the last
goodbye

C) The harassed wife shot herself with a gun after bidding her husband the last

goodbye.

D) With a gun the harassed wife shot herself, after bidding her husband the last

goodbye.

1) A

2) B

3) C

4) D

Correct Option is: 3


Your Option is: 2
Result: Wrong
Timetaken: 0.0 secs
Explanation: No Explanation

8.

Read the passage and answer the questions that follow on the basis of the

information provided in the passage.

The establishment of the third Reich influenced events in American history by

starting a chain of events which culminated in war between Germany and the

United States. The complete destruction of democracy, the persecution of laws, the

war on religion, the cruelty and barbarism of the Nazis and especially, the plans of

Germany and her allies, Italy and Japan, for world conquest caused great

indignation in this country and brought on fear of another world war. While speaking

out against Hitler\'s atrocities, the American profile generally favored isolationist

policies, and neutrality. The neutrality acts of 1935 and 1936 prohibited trade with
any belligerents or loans to them. In 1937 the president was empowered to declare

an arms embargo in wars between nations at his discretion American opinion began

to change somewhat after President Roosevelt\'s quarantine the aggressor speech

at Chicago (1937) in which he severely criticized Hitler\'s policies. Germany's

seizure of Austria and Munich pact for the partition of Czechoslovakia (1938) also

around the American people. The conquest of Czechoslovakia in March 1939 was

another rude awakening to the menace of the third Reich. In August, 1939, came

the shock of the Nazi - Soviet pact and in September the attack on Poland and the

outbreak of European war. The United States attempt to maintain neutrality in spite

of sympathy for the democracies arranged against the Third Reich. The Neutrality

act of 1939 repeated the arms embargo and permitted 'cash' and 'carry' exports of

arms to belligerent nations. A strong national defense program was begun. A draft

act was passed (1940) to strengthen the military services. A Lend - Lease Act

(1940) authorized the president to sell, exchange or lend materials to any county

deemed necessary by him for the defence of the United States. Help was given to

Britain territory in the western Hemisphere. In August 1941, President Roosevelt

and prime minister Churchill met and issued the Atlantic Charter which proclaimed

the kind of a world which should be established after the war. In December 1941,

Japan launched the unprovoked attack on the United States at Pearl harbour,

immediately thereafter Germany declared war on the United States.

The Neutrality Act of 1939 favored Great Britain because

a the British had command of the sea

b the law permitted U.S.A. to trade only with the allies.

c it antagonized Japan

d it led to the Land - Lease Act

e it agreed with the British on the principle of the Atlantic Charter

1) a
2) b

3) c

4) d

5) e

Correct Option is: 1


Your Option is: 4
Result: Wrong
Timetaken: 0.0 secs
Explanation: No Explanations

9.

Read the passage and answer the questions that follow on the basis of the

information provided in the passage.

The establishment of the third Reich influenced events in American history by

starting a chain of events which culminated in war between Germany and the

United States. The complete destruction of democracy, the persecution of laws, the

war on religion, the cruelty and barbarism of the Nazis and especially, the plans of

Germany and her allies, Italy and Japan, for world conquest caused great

indignation in this country and brought on fear of another world war. While speaking

out against Hitler\'s atrocities, the American profile generally favored isolationist

policies, and neutrality. The neutrality acts of 1935 and 1936 prohibited trade with

any belligerents or loans to them. In 1937 the president was empowered to declare

an arms embargo in wars between nations at his discretion American opinion began

to change somewhat after President Roosevelt\'s quarantine the aggressor speech

at Chicago (1937) in which he severely criticized Hitler\'s policies. Germany's

seizure of Austria and Munich pact for the partition of Czechoslovakia (1938) also

around the American people. The conquest of Czechoslovakia in March 1939 was

another rude awakening to the menace of the third Reich. In August, 1939, came
the shock of the Nazi - Soviet pact and in September the attack on Poland and the

outbreak of European war. The United States attempt to maintain neutrality in spite

of sympathy for the democracies arranged against the Third Reich. The Neutrality

act of 1939 repeated the arms embargo and permitted 'cash' and 'carry' exports of

arms to belligerent nations. A strong national defense program was begun. A draft

act was passed (1940) to strengthen the military services. A Lend - Lease Act

(1940) authorized the president to sell, exchange or lend materials to any county

deemed necessary by him for the defence of the United States. Help was given to

Britain territory in the western Hemisphere. In August 1941, President Roosevelt

and prime minister Churchill met and issued the Atlantic Charter which proclaimed

the kind of a world which should be established after the war. In December 1941,

Japan launched the unprovoked attack on the United States at Pearl harbour,

immediately thereafter Germany declared war on the United States.

The Land - Lease Act has designed to

a Strengthen USA\'s national defense

bProvide battle shit to the Allies

c Help the British

d the Atlantic Charter

e Avenge Pearl Harbor

1) a

2) b

3) c

4) d

5) e

Correct Option is: 1


Your Option is: 2
Result: Wrong
Timetaken: 0.0 secs
Explanation: No Explanations

Read each sentence to find if there is any grammatical error in it. If there is any

error, it will be only one part of the sentence. The number or alphabet of that part is

10. your answer.(Disregard punctuation errors if any)

The apparently obvious solutions / to most of his problems / were overlook by

/many of his friends / no error.

ABCDE

1) A

2) B

3) C

4) D

5) E

Correct Option is: 3


Your Option is: 3
Result: Correct
Timetaken: 0.0 secs
Explanation: No Explanations

11.

In each of the following questions, a paragraph or a sentence has been broken up

into different parts. The parts have been scrambled and numbered as given below.

Choose the correct order of these parts from the given alternatives.

1) it is of vital importance 2) if this can be prevented 3) since man depends for his
food 4) upon articles 5) that none of this soil should be wasted 6) produced from

the earth\'s layer of fertile soil.

A) 3,1,2,4,6,5 B) 3,1,5,4,6,2

C) 3,4,6,1,5,2 D) 3,5,1,2,6,4

1) A

2) B

3) C

4) D

Correct Option is: 3


Your Option is: 3
Result: Correct
Timetaken: 0.0 secs
Explanation: No Explanation

In each of the following questions, a paragraph or a sentence has been broken up

into different parts. The parts have been scrambled and numbered as given below.

Choose the correct order of these parts from the given alternatives.

1) Along the gutters is lingered, 2) its tired breath a pale of chrysanthemum about

12. the street lamps 3) sliding its warm tongue over silent pavements, 4) The gentle

fox curled softly down 5) and rubbed its back against the huddled houses 6) then it

curled up and slept on corners 7) Slowly it rose and fell, 8) and pressing its blurred

face against shop- windows 9) It coiled its body around the black railings.

A) 1,2,3,4,5,6,7,8,9 B) 2,9,6,8,5,7,4,3,1

C) 4,9,5,1,3,8,7,2,6 D) 9,6,8,5,7,1,4,2,3
1) A

2) B

3) C

4) D

Correct Option is: 3


Your Option is: 1
Result: Wrong
Timetaken: 0.0 secs
Explanation: No Explanations

Read each sentence to find if there is any grammatical error in it. If there is any

error, it will be only one part of the sentence. The number or alphabet of that part is

13. your answer.(Disregard punctuation errors if any)

By arresting the local criminals / and encouraging good people / we can end

/hostilities of that area / no error.

ABCDE

1) A

2) B

3) C

4) D

5) E

Correct Option is: 3


Your Option is: 4
Result: Wrong
Timetaken: 0.0 secs
Explanation: we can bring to end

In each of the following questions, a paragraph or a sentence has been broken up

into different parts. The parts have been scrambled and numbered as given below.

Choose the correct order of these parts from the given alternatives.

14. 1) keeper of the 2) guardian of 3) as well as 4) The U.N.O is supposed to be the 5)

the morally conceived 6) political conscience 7) expressly defined rights and duties

of sovereign states. 8) and

A) 4,1,7,3,2,5,8,6 B) 4,2,6,8,1,3,5,7

C) 4,1,6,8,2,5,3,7 D) 4,2,7,3,1,5,8,6

1) A

2) B

3) C

4) D

Correct Option is: 3


Your Option is: 2
Result: Wrong
Timetaken: 0.0 secs
Explanation: No Explanation

15.

In each of the following questions, some sentence are given which are on the same

theme. decide which sentence is the most preferable with respect to grammar;

meaning and usage, suitable for formal writing in English. Find the correct
sentence.

A) If you had told me that you were in Bombay I had certainly contacted by you

instead of getting bored there

B) If you had told me that you were in Bombay I would have certainly contacted

you despite getting bored there

C) If you had told me that you were in Bombay I would have certainly contacted

you instead of getting bored there.

D) If you would have told me that you were in Bombay I had certainly contacted

you instead of getting bored there

1) A

2) B

3) C

4) D

Correct Option is: 3


Your Option is: 1
Result: Wrong
Timetaken: 0.0 secs
Explanation: No Explanation

16.

Read the passage and answer the questions that follow on the basis of the

information provided in the passage.

The establishment of the third Reich influenced events in American history by

starting a chain of events which culminated in war between Germany and the

United States. The complete destruction of democracy, the persecution of laws, the

war on religion, the cruelty and barbarism of the Nazis and especially, the plans of
Germany and her allies, Italy and Japan, for world conquest caused great

indignation in this country and brought on fear of another world war. While speaking

out against Hitler\'s atrocities, the American profile generally favored isolationist

policies, and neutrality. The neutrality acts of 1935 and 1936 prohibited trade with

any belligerents or loans to them. In 1937 the president was empowered to declare

an arms embargo in wars between nations at his discretion American opinion began

to change somewhat after President Roosevelt\'s quarantine the aggressor speech

at Chicago (1937) in which he severely criticized Hitler\'s policies. Germany's

seizure of Austria and Munich pact for the partition of Czechoslovakia (1938) also

around the American people. The conquest of Czechoslovakia in March 1939 was

another rude awakening to the menace of the third Reich. In August, 1939, came

the shock of the Nazi - Soviet pact and in September the attack on Poland and the

outbreak of European war. The United States attempt to maintain neutrality in spite

of sympathy for the democracies arranged against the Third Reich. The Neutrality

act of 1939 repeated the arms embargo and permitted 'cash' and 'carry' exports of

arms to belligerent nations. A strong national defense program was begun. A draft

act was passed (1940) to strengthen the military services. A Lend - Lease Act

(1940) authorized the president to sell, exchange or lend materials to any county

deemed necessary by him for the defence of the United States. Help was given to

Britain territory in the western Hemisphere. In August 1941, President Roosevelt

and prime minister Churchill met and issued the Atlantic Charter which proclaimed

the kind of a world which should be established after the war. In December 1941,

Japan launched the unprovoked attack on the United States at Pearl harbour,

immediately thereafter Germany declared war on the United States.

An event that did not occur in 1939 was the

a invasion of Poland

b invasion of Czechoslovakia

c passing of the Neutrality Act


d passing of the Land - Lease act

e outbreak of the war in Europe

1) a

2) b

3) c

4) d

5) e

Correct Option is: 4


Your Option is: 4
Result: Correct
Timetaken: 0.0 secs
Explanation: No Explanations

Read each sentence to find if there is any grammatical error in it. If there is any

error, it will be only one part of the sentence. The number or alphabet of that part is

17. your answer.(Disregard punctuation errors if any)

In this way nuclear fission / or the splitting / of the atom / have been achieved / no

error.

ABCDE

1) A

2) B

3) C

4) D

5) E
Correct Option is: 4
Your Option is: 3
Result: Wrong
Timetaken: 0.0 secs
Explanation: No Explanations

18.
Read the passage and answer the questions that follow on the basis of the

information provided in the passage

One of the most dangerous drugs for pregnant women to consume is alcohol.

Because alcohol is delivered quickly into the blood and passes quickly into the

tissues and membranes, the human fetus is particularly vulnerable to its effects. In

fact, the negative effects on a fetus are so pronounced that babies born after

exposure to alcohol are said to be suffering from fetal alcohol syndrome. As a

pregnant woman drinks alcohol, the alcohol is passed into her her bloodstream

almost simultaneously. Moreover, because the bloodstream of the fetus is

inextricably tied to that of the mother, the alcohol passes directly into the

bloodstream of the fetus as well. And, what is more, the concentration of alcohol in

the fetus is exactly the same as in the mother. For the mother, this concentration is

not a problem because her liver can remove one ounce of alcohol from her system

per hour. However, the fetus's liver is not completely developed (how developed it is

depends on its stage of development). The rate at which it is able to eliminate the

alcohol from the blood of the fetus is much slower. Eventually, the alcohol will be

returned to the mother\'s system by passing across the placenta, but this process is

slow. By the time this takes place, major neurological damage may have already

occurred. Research has shown that as little as one drink of alcohol can produce

significant, irreversible damage to the fetus. Babies born after exposure to alcohol

generally exhibit facial distortion, inability to concentrate, and difficulty in


remembering. Simply speaking, it is imperative that pregnant women avoid

alcohol.

Following are some sample questions on this passage:

Which one of the following was NOT mentioned as a sign of fetal alcohol syndrome?

a) disfigurement of the face

b) concentration difficulties

c) increased aggression

d) memory problems

1) a

2) b

3) c

4) d

Correct Option is: 3


Your Option is: 1
Result: Wrong
Timetaken: 0.0 secs
Explanation: No Explanations

19.

Read the passage and answer the questions that follow on the basis of the

information provided in the passage

One of the most dangerous drugs for pregnant women to consume is alcohol.

Because alcohol is delivered quickly into the blood and passes quickly into the

tissues and membranes, the human fetus is particularly vulnerable to its effects. In

fact, the negative effects on a fetus are so pronounced that babies born after

exposure to alcohol are said to be suffering from fetal alcohol syndrome. As a


pregnant woman drinks alcohol, the alcohol is passed into her her bloodstream

almost simultaneously. Moreover, because the bloodstream of the fetus is

inextricably tied to that of the mother, the alcohol passes directly into the

bloodstream of the fetus as well. And, what is more, the concentration of alcohol in

the fetus is exactly the same as in the mother. For the mother, this concentration is

not a problem because her liver can remove one ounce of alcohol from her system

per hour. However, the fetus's liver is not completely developed (how developed it is

depends on its stage of development). The rate at which it is able to eliminate the

alcohol from the blood of the fetus is much slower. Eventually, the alcohol will be

returned to the mother\'s system by passing across the placenta, but this process is

slow. By the time this takes place, major neurological damage may have already

occurred. Research has shown that as little as one drink of alcohol can produce

significant, irreversible damage to the fetus. Babies born after exposure to alcohol

generally exhibit facial distortion, inability to concentrate, and difficulty in

remembering. Simply speaking, it is imperative that pregnant women avoid

alcohol.

Following are some sample questions on this passage:

According to the passage, how is alcohol finally returned to the mother\'s system?

a it is carried through the bloodstream

b it is transferred across the placenta

c it is expelled by the fetus\'s liver

d it is not completely returned

1) a

2) b

3) c

4) d
Correct Option is: 2
Your Option is: 3
Result: Wrong
Timetaken: 0.0 secs
Explanation: No Explanations

In each of the following questions, a paragraph or a sentence has been broken up

into different parts. The parts have been scrambled and numbered as given below.

Choose the correct order of these parts from the given alternatives.
20.
1) are free from 2) grow abundantly 3) low plants 4) Tundra regions 5) during short

summer 6) like mosses and lichens 7) and 8) ice

A) 3,6,2,5,7,4,1,8 B) 4,1,8,5,7,3,6,2

C) 5,3,6,2,7,4,1,8 D) 5,4,1,8,7,2,3,6

1) A

2) B

3) C

4) D

Correct Option is: 2


Your Option is: 4
Result: Wrong
Timetaken: 0.0 secs
Explanation: No Explanation

21.

Read the passage and answer the questions that follow on the basis of the
information provided in the passage

One of the most dangerous drugs for pregnant women to consume is alcohol.

Because alcohol is delivered quickly into the blood and passes quickly into the

tissues and membranes, the human fetus is particularly vulnerable to its effects. In

fact, the negative effects on a fetus are so pronounced that babies born after

exposure to alcohol are said to be suffering from fetal alcohol syndrome. As a

pregnant woman drinks alcohol, the alcohol is passed into her her bloodstream

almost simultaneously. Moreover, because the bloodstream of the fetus is

inextricably tied to that of the mother, the alcohol passes directly into the

bloodstream of the fetus as well. And, what is more, the concentration of alcohol in

the fetus is exactly the same as in the mother. For the mother, this concentration is

not a problem because her liver can remove one ounce of alcohol from her system

per hour. However, the fetus's liver is not completely developed (how developed it is

depends on its stage of development). The rate at which it is able to eliminate the

alcohol from the blood of the fetus is much slower. Eventually, the alcohol will be

returned to the mother\'s system by passing across the placenta, but this process is

slow. By the time this takes place, major neurological damage may have already

occurred. Research has shown that as little as one drink of alcohol can produce

significant, irreversible damage to the fetus. Babies born after exposure to alcohol

generally exhibit facial distortion, inability to concentrate, and difficulty in

remembering. Simply speaking, it is imperative that pregnant women avoid

alcohol.

Following are some sample questions on this passage:

According to the passage, how does the concentration of alcohol in a fetus compare

to that in the mother?

a The concentration is more.

b The concentration is less.

c The concentration is equivalent.


d The concentration cannot be measured.

1) a

2) b

3) c

4) d

Correct Option is: 3


Your Option is: 3
Result: Correct
Timetaken: 0.0 secs
Explanation: No Explanations

In each of the following questions, some sentence are given which are on the same

theme. decide which sentence is the most preferable with respect to grammar;

meaning and usage, suitable for formal writing in English. Find the correct

sentence.

A) The receptionist must answer courteously the questions what are asked by the
22.
callers

B) The receptionist must answer courteously the questions of all the callers

C) The receptionist should answer courteously the questions of all callers

D) The receptionist courteously should answer the questions of all callers

E) There would have been no trouble if the receptionist had have always answered

courteously.

1) A

2) B
3) C

4) D

5) E

Correct Option is: 3


Your Option is: 2
Result: Wrong
Timetaken: 0.0 secs
Explanation: No Explanation

Read each sentence to find if there is any grammatical error in it. If there is any

error, it will be only one part of the sentence. The number or alphabet of that part is

23. your answer.(Disregard punctuation errors if any)

The trust has succeeded / admirably in raising / money for / its future programs /

no error.

ABCDE

1) A

2) B

3) C

4) D

5) E

Correct Option is: 5


Your Option is: 3
Result: Wrong
Timetaken: 0.0 secs
Explanation: No Explanations
24.

Read the passage and answer the questions that follow on the basis of the

information provided in the passage

One of the most dangerous drugs for pregnant women to consume is alcohol.

Because alcohol is delivered quickly into the blood and passes quickly into the

tissues and membranes, the human fetus is particularly vulnerable to its effects. In

fact, the negative effects on a fetus are so pronounced that babies born after

exposure to alcohol are said to be suffering from fetal alcohol syndrome. As a

pregnant woman drinks alcohol, the alcohol is passed into her her bloodstream

almost simultaneously. Moreover, because the bloodstream of the fetus is

inextricably tied to that of the mother, the alcohol passes directly into the

bloodstream of the fetus as well. And, what is more, the concentration of alcohol in

the fetus is exactly the same as in the mother. For the mother, this concentration is

not a problem because her liver can remove one ounce of alcohol from her system

per hour. However, the fetus's liver is not completely developed (how developed it is

depends on its stage of development). The rate at which it is able to eliminate the

alcohol from the blood of the fetus is much slower. Eventually, the alcohol will be

returned to the mother\'s system by passing across the placenta, but this process is

slow. By the time this takes place, major neurological damage may have already

occurred. Research has shown that as little as one drink of alcohol can produce

significant, irreversible damage to the fetus. Babies born after exposure to alcohol

generally exhibit facial distortion, inability to concentrate, and difficulty in

remembering. Simply speaking, it is imperative that pregnant women avoid

alcohol.

Following are some sample questions on this passage:

How much time can it be inferred that it takes alcohol to enter a woman\'s

bloodstream after she takes a drink?

a about one hour


b a few seconds

c several minutes

d at least 24 hours

1) a

2) b

3) c

4) d

Correct Option is: 2


Your Option is: 4
Result: Wrong
Timetaken: 0.0 secs
Explanation: No Explanations

25.
Directions for Questions : Read the passage and answer the questions that follow

on the basis of the information provided in the passage. For years now, critics of

United States cities have pictured city centers as empty places, avoid of life.

Fortunately, a recent trend in urban development is making some cities lively. One

example of this trend is Baltimore's Harbor place, a stunning waterfront complex of

one hundred shops set in glass pavilions. Not long ago, Baltimore's dockland 5 area,

like clock areas in other older cities, was a collection of abandoned warehouses.

Now the area is expected to add thirty million dollars to Baltimore's annual tax base

and to draw home buyers back to the city and away from the nearby Washington,

DC area. Baltimore has long been a pioneer in urban renewal, not only building

from scratch, as at Harbor place, but preserving and restoring buildings throughout

the town center. Although 10 few cities show this much care for their old residential

areas, the officials of many cities are starting to believe that central districts ought
to be efficiently attractive to draw permanent residents to them. Several other cities

built around water are also exploiting their easily accessible waterfront areas. San

Francisco, in its Ghirardelli Square project, converted an old factory at the edge of

the bay into a labyrinth of boutiques. St. Louis 15 Savannah, Louisville, and

Portland all have "riverside malls" in progress, while Boston's Quincy Market also on

the waterfront , is expected to bring in 1.5 million dollars in city taxes annually. This

trend reflects an important change in social attitudes in the United States, brought

on by new economic factors. In the 1960's, the availability of low-cost housing

beyond 20 city limits caused many cities in the east and Midwest to lose residents,

thus leaving city centers relatively empty of life. In the southwest a different

phenomenon ,the relentless impetus of cities like TU Houston, and Phoenix to

extend their orders across the desert, has had much the same effect: centers full of

skyscrapers and streets that are empty after five o'clock ,for years it seemed that

the "ideal" home was tranquil suburban 25 residence. But recently, the rise in gas

prices has made long automobile journeys to work less tolerable. Trips of fifty miles

are not uncommon for the suburban commuter. In addition, suburban property

taxes have risen sharply as utility lines are stretched ever farther from central

sources.30 The rise in house prices has increased the differential between city and

suburban property; now it is generally cheaper to buy an old urban house than a

newer suburban one. Young working people, usually without children, are the most

likely purchasers of urban homes. Following these people are huge projects

including apartments, offices, and shops, all embracing the increasingly attractive

belief that people should live, work, and shop within 35 as little traveling distance

as possible. The author suggests that, in the United States, southwestern cities can

be distinguished from eastern cities by the fact that a)Cities in the southwest have

declined at a different rate than have cities in the east

b)Officials in southwestern cities have shown less interest in revitalizing their city

centers than have officials in eastern cities.


c)Southwestern cities have had more success than have eastern cities in attracting

new home buyers.

d)The centers of southwestern cities have lost residents for different reasons than

have the centers of eastern cities

1) a

2) b

3) c

4) d

5) e

Correct Option is: 4


Your Option is: 2
Result: Wrong
Timetaken: 0.0 secs
Explanation: nil

1.
Read the passage and answer the questions that follow on the basis of the

information provided in the passage.

After his father's death, writer Laurence Yep returned to San Francisco to look for

the apartment house where his family had lived, which also housed their grocery

store. It had been replaced by a two-story parking garage for a nearby college.

There were trees growing where the store door had been. I had to look at the street

signs on the corner to make sure I was in the right spot. Behind the trees was a

door of solid metal painted a battleship gray Stretching to either side were concrete

walls with metal grates bolted over the openings in the sides. The upper story of
the garage was open to the air but through the grates I could look into the lower

level. The gray, oil- stained concrete spread onward endlessly, having replaced the

red cement floor of our store. Lines marked parking places where my parents had

laid wooden planks to ease the ache and chill on their feet. Where the old-fashioned

glass store counter had been was a row of cars. I looked past the steel I-beams

that formed the columns and ceiling of the garage, peering through the dimness in

an attempt to locate where my father's garden had been; but there was only an

endless stretch of cars within the painted stalls. We called it the garden though that

was stretching the definition of the word because it was only a small, narrow

cement courtyard on the north side of our apartment house. There was only a brief

time during the day when the sun could reach the tiny courtyard; but fuchsia

bushes, which loved the shade, grew as tall as trees from the dirt plot there. Next

to it my father had fashioned shelves from old hundred-pound rice cans and planks;

and on these makeshift shelves he had his miniature flower patches growing in old

soda pop crates from which he had removed the wooden dividers. He would go out

periodically to a wholesale nursery by the beach and load the car with boxes full of

little flowers and seedlings which he would lovingly transplant in his shadowy

garden. If you compared our crude little garden to your own backyards, you would

probably laugh; and yet the cats in the neighbourhood loved my father's garden

almost as much as he did--to his great dismay The cats loved to roll among the

flowers, crushing what were just about the only green growing things in the area.

Other times, they ate them-perhaps as a source of greens. Whatever the case, my

father could have done without their destructive displays of appreciation. I don't

know where my father came by his love of growing things. He had come to San

Francisco as a boy and, except for a brief time spent picking fruit, had lived most of

his life among cement, brick, and asphalt. I hadn't thought of my father's garden in

years; and yet it was the surest symbol of my father. Somehow he could persuade

flowers to grow within the old, yellow soda pop crates though the sun seldom
touched them; and he could coax green shoots out of what seemed like lifeless

sticks. His was the gift of renewal. However, though I stared and stared, I could not

quite figure out where it had been. Everything looked the same; more concrete and

more cars. Store, home and garden had all been torn down and replaced by

something as cold, massive and impersonal as a prison. Even if I could have gone

through the gate, there was nothing for me inside there. If I wanted to return to

that lost garden, I would have to go back into my own memories. Award-winning

author Laurence Yep did return to his father's garden in his memories. In 1991 he

published The Lost Garden an autobiography in which he tells of growing up in San

Francisco and of coming to use his writing to celebrate his family and his ethnic

heritage.

The author is searching for something as he looks through the window of a parking

garage. What is he searching for?

A. A particular car

B. The red cement floor of an old store

C. Reminders of the past

D. Evidence of his father's financial success

1) A

2) B

3) C

4) d

5) E

Correct Option is: 3


Your Option is: 2
Result: Wrong
Timetaken: 0.0 secs
Explanation: nil

Directions for Questions : In each of the following questions, some sentences are

given which are on the same theme. Decide which sentence is the most preferable

with respect to grammar; meaning and usage, suitable for formal writing in English.

2. Find the correct sentence.

A) I was asked to stop writing.

B) She denied to go with me.

C) My hairs stood on end.

D) I am reading this novel for four days.

1) a

2) b

3) c

4) d

Correct Option is: 4


Your Option is: 4
Result: Correct
Timetaken: 0.0 secs
Explanation: I am reading this novel for four days.

3.
In each of the following questions, a paragraph or a sentence has been broken up

into different parts. The parts have been scrambled and numbered as given below.

Choose the correct order of these parts from the given alternatives. 1) Pentium 4

2) any

3) conflicts
4) handle

5) It seems

6) can

7) that

8) without

9) it

A) 5, 7, 1, 4, 6, 9, 8, 3, 2

B) 5, 7, 2, 4, 6, 8, 9, 1, 3

C) 5, 7, 1, 4, 6, 9, 8, 2, 3

D) 5, 7, 1, 6, 4, 9, 8, 2, 3

1) A

2) B

3) C

4) D

5) E

Correct Option is: 4


Your Option is: 1
Result: Wrong
Timetaken: 0.0 secs
Explanation: NIL

4.

Directions for Questions : In each of the following questions, some sentences are

given which are on the same theme. Decide which sentence is the most preferable

with respect to grammar; meaning and usage, suitable for formal writing in English.

Find the correct sentence.


A. Since he lacked needed money, he never turned down anyone who needed help.

B. He wasn't rich by any means, although he never turned down anyone who

needed help.

C. Being not rich by any means, but he never turned away anyone who needed

help.

D. He wasn't rich by any means, but he never turned away anyone who needed

help.

E. Since he wasn't rich by any means, he never turned away anyone who needed

help.

1) a

2) b

3) c

4) d

Correct Option
4
is:
Your Option is: 3
Result: Wrong
Timetaken: 0.0 secs
He wasn't rich by any means, but he never turned away
Explanation:
anyone who needed help.

5.
Read the passage and answer the questions that follow on the basis of the

information provided in the passage.

For a period of more than two centuries palaeontologists have been intrigued by the

fossilized remains of pterosaurs, the first flying vertebrates. The issues, which

puzzle them, are how these heavy creatures, having a wingspan of about 8-12

meters managed the various problems associated with powered flight and whether
these creatures were reptiles or birds. Perhaps the least controversial assertion

about the pterosaurs is that they were reptiles. Their skulls, pelvises, and hind feet

are reptilian. The anatomy of their wings suggests that they did not evolve into the

class of birds. In pterosaurs a greatly elongated fourth finger of each forelimb

supported a wing like membrane. The other fingers were short and reptilian, with

sharp claws. In birds the second finger is the principal strut of the wing, which

consists primarily of feathers. If the pterosaurs walked on all fours, the three short

fingers may have been employed for grasping. When a pterosaurs walked or

remained stationary, the fourth finger, and with it the wing, could only urn upward

in an extended inverted V- shape along each side of the animal's body. In

resemblance they were extremely similar to both birds and bats, with regard to

their overall body structure and proportion. This is hardly surprising as the design of

any flying vertebrate is subject to aerodynamic constraints. Both the pterosaurs and

the birds have hollow bones, a feature that represents a savings in weight. There is

a difference, which is that the bones of the birds are more massively reinforced by

internal struts. Although scales typically cover reptiles, the pterosaurs probably had

hairy coats. T.H. Huxley reasoned that flying vertebrates must have been warm-

blooded because flying implies a high rate of metabolism, which in turn implies a

high internal temperature. Huxley speculated that a coat of hair would insulate

against loss of body heat and might streamline the body to reduce drag in flight.

The recent discovery of a pterosaur specimen covered in long, dense, and relatively

thick hair like fossil material was the first clear evidence that his reasoning was

correct. Some palaeontologists are of the opinion that the pterosaurs jumped from

s dropped from trees or perhaps rose into the light winds from the crests of waves

in order to become airborne. Each theory has its associated difficulties. The first

makes a wrong assumption that the pterosaurs hind feet resembled a bat's and

could serve as hooks by which the animal could hang in preparation for flight. The

second hypothesis seems unlikely because large pterosaurs could not have landed
in trees without damaging their wings. The third calls for high aces to channel

updrafts. The pterosaurs would have been unable to control their flight once

airborne as the wind from which such waves arose would have been too strong.

As inferred from the passage, the skeleton of a pterosaur is 5. According to the

passage, some scientists believe that distinguishable from that of a bird by the

pterosaurs

A.length of its wingspan

B.hollow spaces in its bones

C.anatomic origin of its wing strut

D.evidence of the hook like projections on its hind feet

E.location of the shoulder joint joining the wing to its body.

1) A

2) B

3) c

4) D

5) E

Correct Option is: 3


Your Option is: 3
Result: Correct
Timetaken: 0.0 secs
Explanation: nil

6.
Read each sentence to find if there is any grammatical error in it.

If there is any error, it will be only one part of the sentence. The number or

alphabet of that part is your answer.(Disregard punctuation errors if any)

I shall / ring him / tomorrow / in the afternoon.


ABCD

1) A

2) B

3) C

4) D

5) E

Correct Option is: 2


Your Option is: 2
Result: Correct
Timetaken: 0.0 secs
Explanation: NIL

7.
In each of the following questions, a paragraph or a sentence has been broken up

into different parts. The parts have been scrambled and numbered as given below.

Choose the correct order of these parts from the given alternatives. 1) language

2) of

3) two

4) the

5) official

6) countries

7) is

8) English

9) the

A) 8, 7, 4, 5, 1, 2, 9, 3, 6

B) 8, 7, 1, 5, 4, 2, 9, 3, 6

C) 8, 7, 4, 1, 5, 2 9, 3, 6
D) 8, 7, 4, 5, 1, 9, 2, 3, 6

1) A

2) B

3) C

4) D

5) E

Correct Option is: 1


Your Option is: 4
Result: Wrong
Timetaken: 0.0 secs
Explanation: NIL

8.
Read the passage and answer the questions that follow on the basis of the

information provided in the passage.

For a period of more than two centuries palaeontologists have been intrigued by the

fossilized remains of pterosaurs, the first flying vertebrates. The issues, which

puzzle them, are how these heavy creatures, having a wingspan of about 8-12

meters managed the various problems associated with powered flight and whether

these creatures were reptiles or birds. Perhaps the least controversial assertion

about the pterosaurs is that they were reptiles. Their skulls, pelvises, and hind feet

are reptilian. The anatomy of their wings suggests that they did not evolve into the

class of birds. In pterosaurs a greatly elongated fourth finger of each forelimb

supported a wing like membrane. The other fingers were short and reptilian, with

sharp claws. In birds the second finger is the principal strut of the wing, which

consists primarily of feathers. If the pterosaurs walked on all fours, the three short

fingers may have been employed for grasping. When a pterosaurs walked or
remained stationary, the fourth finger, and with it the wing, could only urn upward

in an extended inverted V- shape along each side of the animal's body. In

resemblance they were extremely similar to both birds and bats, with regard to

their overall body structure and proportion. This is hardly surprising as the design of

any flying vertebrate is subject to aerodynamic constraints. Both the pterosaurs and

the birds have hollow bones, a feature that represents a savings in weight. There is

a difference, which is that the bones of the birds are more massively reinforced by

internal struts. Although scales typically cover reptiles, the pterosaurs probably had

hairy coats. T.H. Huxley reasoned that flying vertebrates must have been warm-

blooded because flying implies a high rate of metabolism, which in turn implies a

high internal temperature. Huxley speculated that a coat of hair would insulate

against loss of body heat and might streamline the body to reduce drag in flight.

The recent discovery of a pterosaur specimen covered in long, dense, and relatively

thick hair like fossil material was the first clear evidence that his reasoning was

correct. Some palaeontologists are of the opinion that the pterosaurs jumped from

s dropped from trees or perhaps rose into the light winds from the crests of waves

in order to become airborne. Each theory has its associated difficulties. The first

makes a wrong assumption that the pterosaurs hind feet resembled a bat's and

could serve as hooks by which the animal could hang in preparation for flight. The

second hypothesis seems unlikely because large pterosaurs could not have landed

in trees without damaging their wings. The third calls for high aces to channel

updrafts. The pterosaurs would have been unable to control their flight once

airborne as the wind from which such waves arose would have been too strong.

According to the passage, some scientists believe that pterosaurs

A. Lived near large bodies of water

B. Had sharp teeth for tearing food

C. Were attacked and eaten by larger reptiles


D. Had longer tails than many birds

E. Consumed twice their weight daily to maintain their body temperature.

1) A

2) B

3) C

4) D

5) E

Correct Option is: 1


Your Option is: 3
Result: Wrong
Timetaken: 0.0 secs
Explanation: nil

One of the four sentences given in each question is grammatically wrong. Find the

incorrect sentence.

A) He has no desire for fame.


9.
B) I intend going to Calcutta.

C) He is too miserly to part with his money.

D) He has invited me for dinner.

1) A

2) B

3) C

4) D

5) E
Correct Option is: 3
Your Option is: 4
Result: Wrong
Timetaken: 0.0 secs
Explanation: nil

10.
Read the passage and answer the questions that follow on the basis of the

information provided in the passage.

After his father's death, writer Laurence Yep returned to San Francisco to look for

the apartment house where his family had lived, which also housed their grocery

store. It had been replaced by a two-story parking garage for a nearby college.

There were trees growing where the store door had been. I had to look at the street

signs on the corner to make sure I was in the right spot. Behind the trees was a

door of solid metal painted a battleship gray Stretching to either side were concrete

walls with metal grates bolted over the openings in the sides. The upper story of

the garage was open to the air but through the grates I could look into the lower

level. The gray, oil- stained concrete spread onward endlessly, having replaced the

red cement floor of our store. Lines marked parking places where my parents had

laid wooden planks to ease the ache and chill on their feet. Where the old-fashioned

glass store counter had been was a row of cars. I looked past the steel I-beams

that formed the columns and ceiling of the garage, peering through the dimness in

an attempt to locate where my father's garden had been; but there was only an

endless stretch of cars within the painted stalls. We called it the garden though that

was stretching the definition of the word because it was only a small, narrow

cement courtyard on the north side of our apartment house. There was only a brief

time during the day when the sun could reach the tiny courtyard; but fuchsia

bushes, which loved the shade, grew as tall as trees from the dirt plot there. Next

to it my father had fashioned shelves from old hundred-pound rice cans and planks;

and on these makeshift shelves he had his miniature flower patches growing in old
soda pop crates from which he had removed the wooden dividers. He would go out

periodically to a wholesale nursery by the beach and load the car with boxes full of

little flowers and seedlings which he would lovingly transplant in his shadowy

garden. If you compared our crude little garden to your own backyards, you would

probably laugh; and yet the cats in the neighbourhood loved my father's garden

almost as much as he did--to his great dismay The cats loved to roll among the

flowers, crushing what were just about the only green growing things in the area.

Other times, they ate them-perhaps as a source of greens. Whatever the case, my

father could have done without their destructive displays of appreciation. I don't

know where my father came by his love of growing things. He had come to San

Francisco as a boy and, except for a brief time spent picking fruit, had lived most of

his life among cement, brick, and asphalt. I hadn't thought of my father's garden in

years; and yet it was the surest symbol of my father. Somehow he could persuade

flowers to grow within the old, yellow soda pop crates though the sun seldom

touched them; and he could coax green shoots out of what seemed like lifeless

sticks. His was the gift of renewal. However, though I stared and stared, I could not

quite figure out where it had been. Everything looked the same; more concrete and

more cars. Store, home and garden had all been torn down and replaced by

something as cold, massive and impersonal as a prison. Even if I could have gone

through the gate, there was nothing for me inside there. If I wanted to return to

that lost garden, I would have to go back into my own memories. Award-winning

author Laurence Yep did return to his father's garden in his memories. In 1991 he

published The Lost Garden an autobiography in which he tells of growing up in San

Francisco and of coming to use his writing to celebrate his family and his ethnic

heritage.

Why are details about the neighbourhood cats included in this story?

A. To show how much the garden meant to the family.

B. To show how important this garden was to the author's father.


C. To show how had the author worked at helping his father.

D. To show that the author's father loved animals as well as plants.

1) A

2) B

3) C

4) D

5) E

Correct Option is: 3


Your Option is: 2
Result: Wrong
Timetaken: 0.0 secs
Explanation: NIL

Directions for Questions : In each of the following questions, some sentences are

given which are on the same theme. Decide which sentence is the most preferable

with respect to grammar; meaning and usage, suitable for formal writing in English.

Find the correct sentence.


11.
A. Our school had won the match if only we have concentrated.

B. Our school would have won the match if only we would have concentrated.

C. Our school would win the match if only we had concentrated.

D. Our school had won the match if only we would have concentrated.

E. Our school would have won the match if only we had concentrated.

1) a

2) b
3) c

4) d

5) e

Correct Option
5
is:
Your Option is: 3
Result: Wrong
Timetaken: 0.0 secs
Our school would have won the match if only we had
Explanation:
concentrated

Read each sentence to find if there is any grammatical error in it.

If there is any error, it will be only one part of the sentence. The number or

12. alphabet of that part is your answer.(Disregard punctuation errors if any)

I enjoyed / during my / stay in / England.

ABCD

1) A

2) B

3) C

4) D

5) E

Correct Option is: 1


Your Option is: 4
Result: Wrong
Timetaken: 0.0 secs
Explanation: NIL
13.
Read the passage and answer the questions that follow on the basis of the

information provided in the passage.

For a period of more than two centuries palaeontologists have been intrigued by the

fossilized remains of pterosaurs, the first flying vertebrates. The issues, which

puzzle them, are how these heavy creatures, having a wingspan of about 8-12

meters managed the various problems associated with powered flight and whether

these creatures were reptiles or birds. Perhaps the least controversial assertion

about the pterosaurs is that they were reptiles. Their skulls, pelvises, and hind feet

are reptilian. The anatomy of their wings suggests that they did not evolve into the

class of birds. In pterosaurs a greatly elongated fourth finger of each forelimb

supported a wing like membrane. The other fingers were short and reptilian, with

sharp claws. In birds the second finger is the principal strut of the wing, which

consists primarily of feathers. If the pterosaurs walked on all fours, the three short

fingers may have been employed for grasping. When a pterosaurs walked or

remained stationary, the fourth finger, and with it the wing, could only urn upward

in an extended inverted V- shape along each side of the animal's body. In

resemblance they were extremely similar to both birds and bats, with regard to

their overall body structure and proportion. This is hardly surprising as the design of

any flying vertebrate is subject to aerodynamic constraints. Both the pterosaurs and

the birds have hollow bones, a feature that represents a savings in weight. There is

a difference, which is that the bones of the birds are more massively reinforced by

internal struts. Although scales typically cover reptiles, the pterosaurs probably had

hairy coats. T.H. Huxley reasoned that flying vertebrates must have been warm-

blooded because flying implies a high rate of metabolism, which in turn implies a

high internal temperature. Huxley speculated that a coat of hair would insulate

against loss of body heat and might streamline the body to reduce drag in flight.

The recent discovery of a pterosaur specimen covered in long, dense, and relatively

thick hair like fossil material was the first clear evidence that his reasoning was
correct. Some palaeontologists are of the opinion that the pterosaurs jumped from

s dropped from trees or perhaps rose into the light winds from the crests of waves

in order to become airborne. Each theory has its associated difficulties. The first

makes a wrong assumption that the pterosaurs hind feet resembled a bat's and

could serve as hooks by which the animal could hang in preparation for flight. The

second hypothesis seems unlikely because large pterosaurs could not have landed

in trees without damaging their wings. The third calls for high aces to channel

updrafts. The pterosaurs would have been unable to control their flight once

airborne as the wind from which such waves arose would have been too strong.

The organization of the last paragraph of the passage can best be described as:

A. New data is introduced in order to support a traditional point of view

B. Three explanations are put forth and each of them is disputed by means of

specific information

C. An outline of three hypotheses are given and evidence supporting each of them

is given

D. Description of three recent discoveries is presented, and their implications for

future study are projected

E. The material in the earlier paragraphs is summarized and certain conclusions are

from it.

1) A

2) B

3) C

4) D

5) E

Correct Option is: 2


Your Option is: 2
Result: Correct
Timetaken: 0.0 secs
Explanation: nil

14.
Read the passage and answer the questions that follow on the basis of the

information provided in the passage.

For a period of more than two centuries palaeontologists have been intrigued by the

fossilized remains of pterosaurs, the first flying vertebrates. The issues, which

puzzle them, are how these heavy creatures, having a wingspan of about 8-12

meters managed the various problems associated with powered flight and whether

these creatures were reptiles or birds. Perhaps the least controversial assertion

about the pterosaurs is that they were reptiles. Their skulls, pelvises, and hind feet

are reptilian. The anatomy of their wings suggests that they did not evolve into the

class of birds. In pterosaurs a greatly elongated fourth finger of each forelimb

supported a wing like membrane. The other fingers were short and reptilian, with

sharp claws. In birds the second finger is the principal strut of the wing, which

consists primarily of feathers. If the pterosaurs walked on all fours, the three short

fingers may have been employed for grasping. When a pterosaurs walked or

remained stationary, the fourth finger, and with it the wing, could only urn upward

in an extended inverted V- shape along each side of the animal's body. In

resemblance they were extremely similar to both birds and bats, with regard to

their overall body structure and proportion. This is hardly surprising as the design of

any flying vertebrate is subject to aerodynamic constraints. Both the pterosaurs and

the birds have hollow bones, a feature that represents a savings in weight. There is

a difference, which is that the bones of the birds are more massively reinforced by

internal struts. Although scales typically cover reptiles, the pterosaurs probably had

hairy coats. T.H. Huxley reasoned that flying vertebrates must have been warm-

blooded because flying implies a high rate of metabolism, which in turn implies a

high internal temperature. Huxley speculated that a coat of hair would insulate
against loss of body heat and might streamline the body to reduce drag in flight.

The recent discovery of a pterosaur specimen covered in long, dense, and relatively

thick hair like fossil material was the first clear evidence that his reasoning was

correct. Some palaeontologists are of the opinion that the pterosaurs jumped from

s dropped from trees or perhaps rose into the light winds from the crests of waves

in order to become airborne. Each theory has its associated difficulties. The first

makes a wrong assumption that the pterosaurs hind feet resembled a bat's and

could serve as hooks by which the animal could hang in preparation for flight. The

second hypothesis seems unlikely because large pterosaurs could not have landed

in trees without damaging their wings. The third calls for high aces to channel

updrafts. The pterosaurs would have been unable to control their flight once

airborne as the wind from which such waves arose would have been too strong.

As seen in the above passage scientists generally agree that:

A. the pterosaurs could fly over large distances because of their large wingspan.

B. a close evolutionary relationship can be seen between the pterosaurs and bats,

when the structure of their skeletons is studied.

C. the study of the fossilized remains of the pterosaurs reveals how they solved the

problem associated with powered flight

D. the pterosaurs were reptiles

E. Pterosaurs walked on all fours

1) A

2) B

3) C

4) D

5) E

Correct Option is: 4


Your Option is: 4
Result: Correct
Timetaken: 0.0 secs
Explanation: nil

In each of the following questions, a paragraph or a sentence has been broken up

into different parts. The parts have been scrambled and numbered as given below.

Choose the correct order of these parts from the given alternatives.

1) is decidedly harmful

2) disregarding other equally important aspects,

3) to the total neglect of others

4) in the life of a man or a woman

5) is not wisdom but


15.
6) cultivating only one quality

7) giving all attention and energy to one aspect of national life only,

8) folly

9) similarly in the life of a nation.

A) 4,6,2,5,8,9,7,1,3

B) 4,6,3,1,9,7,2,5,8

C) 6,2,4,5,1,9,7,3,8

D) 6,4,2,1,9,7,3,5,8

1) A

2) B

3) C

4) D

5) E
Correct Option is: 2
Your Option is: 3
Result: Wrong
Timetaken: 0.0 secs
Explanation: nil

In each of the following questions, a paragraph or a sentence has been broken up

into different parts. The parts have been scrambled and numbered as given below.

Choose the correct order of these parts from the given alternatives. 1) Zealand

2) islands

3) Australia

4) of

5) new

16. 6)consist

7) both

8) and

9) two

A) 2,4,3,6,5,7,1,8,9

B) 5,1, 8 3,7,6,9,2,4

C) 5,1,8,3,7,6,4,9,2

D) 5,1,8,2,3,7,6,4,9

1) A

2) B

3) C

4) D

5) E
Correct Option is: 3
Your Option is: 3
Result: Correct
Timetaken: 0.0 secs
Explanation: nil

In each of the following questions, a paragraph or a sentence has been broken up

into different parts. The parts have been scrambled and numbered as given below.

Choose the correct order of these parts from the given alternatives. 1) two

2) there

3) some

4) however

5) countries

17. 6)between

7) are

8) differences

9) the

A) 4, 2, 8, 3, 7, 6, 9, 1, 5

B) 4, 2, 5, 3, 8, 6, 9, 1, 7,

C) 4, 2, 7, 3, 8, 6, 9, 1, 5

D) 4, 2, 7, 3, 8, 6, 9, 1, 5,

1) A

2) B

3) C

4) D

5) E
Correct Option is: 3
Your Option is: 4
Result: Wrong
Timetaken: 0.0 secs
Explanation: NIL

Directions for Questions : In each of the following questions, some sentences are

given which are on the same theme. Decide which sentence is the most preferable

with respect to grammar; meaning and usage, suitable for formal writing in English.

18. Find the correct sentence.

A. He will not pay unless he is not compelled

B. He will not pay unless he will be compelled

C. He will not pay unless he is compelled

D. he will not pay till he is compelled

1) a

2) b

3) c

4) d

Correct Option is: 3


Your Option is: 2
Result: Wrong
Timetaken: 0.0 secs
Explanation: He will not pay unless he is compelled

19.
One of the four sentences given in each question is grammatically wrong. Find the

incorrect sentence.
A) the odds are against him.

B) Let me thread the needle .

C) A nurse is taking care of him.

D) I don't know if snow is falling.

1) A

2) B

3) C

4) D

Correct Option is: 4


Your Option is: 1
Result: Wrong
Timetaken: 0.0 secs
Explanation: NIL

20.
Read the passage and answer the questions that follow on the basis of the

information provided in the passage.

For a period of more than two centuries palaeontologists have been intrigued by the

fossilized remains of pterosaurs, the first flying vertebrates. The issues, which

puzzle them, are how these heavy creatures, having a wingspan of about 8-12

meters managed the various problems associated with powered flight and whether

these creatures were reptiles or birds. Perhaps the least controversial assertion

about the pterosaurs is that they were reptiles. Their skulls, pelvises, and hind feet

are reptilian. The anatomy of their wings suggests that they did not evolve into the

class of birds. In pterosaurs a greatly elongated fourth finger of each forelimb

supported a wing like membrane. The other fingers were short and reptilian, with

sharp claws. In birds the second finger is the principal strut of the wing, which
consists primarily of feathers. If the pterosaurs walked on all fours, the three short

fingers may have been employed for grasping. When a pterosaurs walked or

remained stationary, the fourth finger, and with it the wing, could only urn upward

in an extended inverted V- shape along each side of the animal's body. In

resemblance they were extremely similar to both birds and bats, with regard to

their overall body structure and proportion. This is hardly surprising as the design of

any flying vertebrate is subject to aerodynamic constraints. Both the pterosaurs and

the birds have hollow bones, a feature that represents a savings in weight. There is

a difference, which is that the bones of the birds are more massively reinforced by

internal struts. Although scales typically cover reptiles, the pterosaurs probably had

hairy coats. T.H. Huxley reasoned that flying vertebrates must have been warm-

blooded because flying implies a high rate of metabolism, which in turn implies a

high internal temperature. Huxley speculated that a coat of hair would insulate

against loss of body heat and might streamline the body to reduce drag in flight.

The recent discovery of a pterosaur specimen covered in long, dense, and relatively

thick hair like fossil material was the first clear evidence that his reasoning was

correct. Some palaeontologists are of the opinion that the pterosaurs jumped from

s dropped from trees or perhaps rose into the light winds from the crests of waves

in order to become airborne. Each theory has its associated difficulties. The first

makes a wrong assumption that the pterosaurs hind feet resembled a bat's and

could serve as hooks by which the animal could hang in preparation for flight. The

second hypothesis seems unlikely because large pterosaurs could not have landed

in trees without damaging their wings. The third calls for high aces to channel

updrafts. The pterosaurs would have been unable to control their flight once

airborne as the wind from which such waves arose would have been too strong.

From the viewpoint of T.H.Huxley, as given in the passage, which of the following

statements is he most likely to agree with?

A.An animal can master complex behaviors irrespective of the size of it's brain.
B.Environmental capabilities and physical capabilities often influence the

appearance of an animal.

C.Usually animals in a particular family group do not change their appearance

dramatically over a period of time

D.The origin of flight in vertebrates was an accidental development rather than the

outcome of specialization or adaption

E.The pterosaurs should be classified as birds, not reptiles.

1) A

2) B

3) C

4) D

5) E

Correct Option is: 2


Your Option is: 3
Result: Wrong
Timetaken: 0.0 secs
Explanation: nil

21.
Read the passage and answer the questions that follow on the basis of the

information provided in the passage.

After his father's death, writer Laurence Yep returned to San Francisco to look for

the apartment house where his family had lived, which also housed their grocery

store. It had been replaced by a two-story parking garage for a nearby college.

There were trees growing where the store door had been. I had to look at the street

signs on the corner to make sure I was in the right spot. Behind the trees was a
door of solid metal painted a battleship gray Stretching to either side were concrete

walls with metal grates bolted over the openings in the sides. The upper story of

the garage was open to the air but through the grates I could look into the lower

level. The gray, oil- stained concrete spread onward endlessly, having replaced the

red cement floor of our store. Lines marked parking places where my parents had

laid wooden planks to ease the ache and chill on their feet. Where the old-fashioned

glass store counter had been was a row of cars. I looked past the steel I-beams

that formed the columns and ceiling of the garage, peering through the dimness in

an attempt to locate where my father's garden had been; but there was only an

endless stretch of cars within the painted stalls. We called it the garden though that

was stretching the definition of the word because it was only a small, narrow

cement courtyard on the north side of our apartment house. There was only a brief

time during the day when the sun could reach the tiny courtyard; but fuchsia

bushes, which loved the shade, grew as tall as trees from the dirt plot there. Next

to it my father had fashioned shelves from old hundred-pound rice cans and planks;

and on these makeshift shelves he had his miniature flower patches growing in old

soda pop crates from which he had removed the wooden dividers. He would go out

periodically to a wholesale nursery by the beach and load the car with boxes full of

little flowers and seedlings which he would lovingly transplant in his shadowy

garden. If you compared our crude little garden to your own backyards, you would

probably laugh; and yet the cats in the neighbourhood loved my father's garden

almost as much as he did--to his great dismay The cats loved to roll among the

flowers, crushing what were just about the only green growing things in the area.

Other times, they ate them-perhaps as a source of greens. Whatever the case, my

father could have done without their destructive displays of appreciation. I don't

know where my father came by his love of growing things. He had come to San

Francisco as a boy and, except for a brief time spent picking fruit, had lived most of

his life among cement, brick, and asphalt. I hadn't thought of my father's garden in
years; and yet it was the surest symbol of my father. Somehow he could persuade

flowers to grow within the old, yellow soda pop crates though the sun seldom

touched them; and he could coax green shoots out of what seemed like lifeless

sticks. His was the gift of renewal. However, though I stared and stared, I could not

quite figure out where it had been. Everything looked the same; more concrete and

more cars. Store, home and garden had all been torn down and replaced by

something as cold, massive and impersonal as a prison. Even if I could have gone

through the gate, there was nothing for me inside there. If I wanted to return to

that lost garden, I would have to go back into my own memories. Award-winning

author Laurence Yep did return to his father's garden in his memories. In 1991 he

published The Lost Garden an autobiography in which he tells of growing up in San

Francisco and of coming to use his writing to celebrate his family and his ethnic

heritage.

What kind of work did the author's father do?

A. He was a professional gardener

B. He worked in a parking garage.

C. He owned a restaurant.

D. He owned a store.

1) A

2) B

3) C

4) D

5) E

Correct Option is: 4


Your Option is: 4
Result: Correct
Timetaken: 0.0 secs
Explanation: nil

One of the four sentences given in each question is grammatically wrong. Find the

incorrect sentence.

A) Let me put my sign here.


22.
B) These cattle are mine.

c) He examined the book closely.

D) He has no knowledge of and no interest in music.

1) A

2) B

3) C

4) D

5) E

Correct Option is: 1


Your Option is: 1
Result: Correct
Timetaken: 0.0 secs
Explanation: NIL

23.
Read the passage and answer the questions that follow on the basis of the

information provided in the passage.

After his father's death, writer Laurence Yep returned to San Francisco to look for

the apartment house where his family had lived, which also housed their grocery

store. It had been replaced by a two-story parking garage for a nearby college.

There were trees growing where the store door had been. I had to look at the street
signs on the corner to make sure I was in the right spot. Behind the trees was a

door of solid metal painted a battleship gray Stretching to either side were concrete

walls with metal grates bolted over the openings in the sides. The upper story of

the garage was open to the air but through the grates I could look into the lower

level. The gray, oil- stained concrete spread onward endlessly, having replaced the

red cement floor of our store. Lines marked parking places where my parents had

laid wooden planks to ease the ache and chill on their feet. Where the old-fashioned

glass store counter had been was a row of cars. I looked past the steel I-beams

that formed the columns and ceiling of the garage, peering through the dimness in

an attempt to locate where my father's garden had been; but there was only an

endless stretch of cars within the painted stalls. We called it the garden though that

was stretching the definition of the word because it was only a small, narrow

cement courtyard on the north side of our apartment house. There was only a brief

time during the day when the sun could reach the tiny courtyard; but fuchsia

bushes, which loved the shade, grew as tall as trees from the dirt plot there. Next

to it my father had fashioned shelves from old hundred-pound rice cans and planks;

and on these makeshift shelves he had his miniature flower patches growing in old

soda pop crates from which he had removed the wooden dividers. He would go out

periodically to a wholesale nursery by the beach and load the car with boxes full of

little flowers and seedlings which he would lovingly transplant in his shadowy

garden. If you compared our crude little garden to your own backyards, you would

probably laugh; and yet the cats in the neighbourhood loved my father's garden

almost as much as he did--to his great dismay The cats loved to roll among the

flowers, crushing what were just about the only green growing things in the area.

Other times, they ate them-perhaps as a source of greens. Whatever the case, my

father could have done without their destructive displays of appreciation. I don't

know where my father came by his love of growing things. He had come to San

Francisco as a boy and, except for a brief time spent picking fruit, had lived most of
his life among cement, brick, and asphalt. I hadn't thought of my father's garden in

years; and yet it was the surest symbol of my father. Somehow he could persuade

flowers to grow within the old, yellow soda pop crates though the sun seldom

touched them; and he could coax green shoots out of what seemed like lifeless

sticks. His was the gift of renewal. However, though I stared and stared, I could not

quite figure out where it had been. Everything looked the same; more concrete and

more cars. Store, home and garden had all been torn down and replaced by

something as cold, massive and impersonal as a prison. Even if I could have gone

through the gate, there was nothing for me inside there. If I wanted to return to

that lost garden, I would have to go back into my own memories. Award-winning

author Laurence Yep did return to his father's garden in his memories. In 1991 he

published The Lost Garden an autobiography in which he tells of growing up in San

Francisco and of coming to use his writing to celebrate his family and his ethnic

heritage.

What idea does the story suggest about the author's parents?

A. They both worked hard to support their family

B. They had encouraged their son to become a writer

C. They had not wanted to see a parking garage replace their home.

D. They had been farmers most of their lives.

1) A

2) B

3) C

4) D

5) E

Correct Option is: 3


Your Option is: 4
Result: Wrong
Timetaken: 0.0 secs
Explanation: nil

Directions for Questions : In each of the following questions, some sentences are

given which are on the same theme. Decide which sentence is the most preferable

with respect to grammar; meaning and usage, suitable for formal writing in English.

24. Find the correct sentence.

A. The teacher asked the student with a frown on his face, to leave the room.

B. The teacher asked with a frawn on his face the student to leave the room.

C. With a frawn on his face, the teacher asked the student to leave the room.

D. The teacher asked the student to leave the room with a frawn on his face.

1) a

2) b

3) c

4) d

Correct Option
3
is:
Your Option is: 1
Result: Wrong
Timetaken: 0.0 secs
With a frawn on his face, the teacher asked the student to
Explanation:
leave the room

25.
Read the passage and answer the questions that follow on the basis of the

information provided in the passage.


After his father's death, writer Laurence Yep returned to San Francisco to look for

the apartment house where his family had lived, which also housed their grocery

store. It had been replaced by a two-story parking garage for a nearby college.

There were trees growing where the store door had been. I had to look at the street

signs on the corner to make sure I was in the right spot. Behind the trees was a

door of solid metal painted a battleship gray Stretching to either side were concrete

walls with metal grates bolted over the openings in the sides. The upper story of

the garage was open to the air but through the grates I could look into the lower

level. The gray, oil- stained concrete spread onward endlessly, having replaced the

red cement floor of our store. Lines marked parking places where my parents had

laid wooden planks to ease the ache and chill on their feet. Where the old-fashioned

glass store counter had been was a row of cars. I looked past the steel I-beams

that formed the columns and ceiling of the garage, peering through the dimness in

an attempt to locate where my father's garden had been; but there was only an

endless stretch of cars within the painted stalls. We called it the garden though that

was stretching the definition of the word because it was only a small, narrow

cement courtyard on the north side of our apartment house. There was only a brief

time during the day when the sun could reach the tiny courtyard; but fuchsia

bushes, which loved the shade, grew as tall as trees from the dirt plot there. Next

to it my father had fashioned shelves from old hundred-pound rice cans and planks;

and on these makeshift shelves he had his miniature flower patches growing in old

soda pop crates from which he had removed the wooden dividers. He would go out

periodically to a wholesale nursery by the beach and load the car with boxes full of

little flowers and seedlings which he would lovingly transplant in his shadowy

garden. If you compared our crude little garden to your own backyards, you would

probably laugh; and yet the cats in the neighbourhood loved my father's garden

almost as much as he did--to his great dismay The cats loved to roll among the

flowers, crushing what were just about the only green growing things in the area.
Other times, they ate them-perhaps as a source of greens. Whatever the case, my

father could have done without their destructive displays of appreciation. I don't

know where my father came by his love of growing things. He had come to San

Francisco as a boy and, except for a brief time spent picking fruit, had lived most of

his life among cement, brick, and asphalt. I hadn't thought of my father's garden in

years; and yet it was the surest symbol of my father. Somehow he could persuade

flowers to grow within the old, yellow soda pop crates though the sun seldom

touched them; and he could coax green shoots out of what seemed like lifeless

sticks. His was the gift of renewal. However, though I stared and stared, I could not

quite figure out where it had been. Everything looked the same; more concrete and

more cars. Store, home and garden had all been torn down and replaced by

something as cold, massive and impersonal as a prison. Even if I could have gone

through the gate, there was nothing for me inside there. If I wanted to return to

that lost garden, I would have to go back into my own memories. Award-winning

author Laurence Yep did return to his father's garden in his memories. In 1991 he

published The Lost Garden an autobiography in which he tells of growing up in San

Francisco and of coming to use his writing to celebrate his family and his ethnic

heritage.

What do you know about the father's garden?

A. It grew in spite of being neglected.

B. The cats would eat all the plants before they grew

C. It flourished in an unlikely spot.

D. It didn't grow well because of lack of sun.

1) A

2) B

3) C

4) D
5) E

Correct Option is: 4


Your Option is: 2
Result: Wrong
Timetaken: 0.0 secs
Explanation: nil

1.
Read the passage and answer the questions that follow on the basis of the

information provided in the passage.

MARK HUGHES is a master of the fine art of survival. His Los Angeles-based

Herbalife International Inc. is a pyramid outfit that peddles weight-loss and

nutrition concoctions of dubious value. Bad publicity and regulatory crackdowns

hurt his U.S. business in the late 1980s. But Hughes, 41, continues to enjoy a

luxurious lifestyle in a $20 million Beverly Hills mansion. He has been sharing the

pad and a yacht with his third wife, a former Miss Petite U.S.A. He can finance this

lavish lifestyle just on his salary and bonus, which last year came to $7.3 million.

He survived his troubles in the U.S. by moving overseas, where regulators are less

zealous and consumers even more naive, at least initially. Today 77% of Herbalife

retail sales derive from overseas. Its new prowling grounds: Asia and Russia. Last

year Herbalife's net earnings doubled, to $45 million, on net sales of $632million.

Based on Herbalife's Nasdaq-traded stock, the company has a market capitalization

of $790 million, making Hughes 58% worth $454 million.

There's a worm, though, in Hughes apple. Foreigners aren't stupid. In the end they

know when they've been had. In France, for instance, retail sales rose to $97

million by 1993 and then plunged to $12 million last year. In Germany sales hit

$159 million in 1994 and have since dropped to $54 million.

Perhaps aware that the world may not provide an infinite supply of suckers, Hughes
wanted to unload some of his shares. But in March, after Herbalife's stock

collapsed, he put off a plan to dump about a third of his holdings on the public.

Contributing to Hughes' woes, Herbalife's chief counsel and legal attack dog, David

Addis, quit in January. Before packing up, he reportedly bellowed at Hughes, "I

can't protect you anymore." Addis, who says he wants to spend more time with his

family, chuckles and claims attorney-client privilege.

Trouble on the home front, too. On a recent conference call with distributors,

Hughes revealed he's divorcing his wife, Suzan, whose beaming and perky image

adorns much of Herbalife's literature.

Meanwhile, in a lawsuit that's been quietly moving through Arizona's Superior

Court,former Herbalife distributor Daniel Fallow of Sandpoint, Idaho charges that

Herbalife arbitrarily withholds payment to distributors and marks up its products

over seven times the cost of manufacturing. Fallow also claims Hughes wanted to

use the Russian mafia to gain entry to that nation's market.

Fallow himself is no angel, but his lawsuit, which was posted on the Internet,

brought out other complaints. Randy Cox of Lewiston, Idaho says Herbalife

"destroyed my business" after he and his wife complained to the company that they

were being cheated out of their money by higher-ups in the pyramid

organization.Will Hughes survive again? Don't count on it this time.

Daniel Fallow:

1) Was a former attorney for Hughes

2) Was a former distributor of Herbalife

3) Co-founded Herbalife

4) Ran Herbalife's German unit


Correct Option
2
is:
Your Option is: 2
Result: Correct
Timetaken: 0.0 secs
Meanwhile,....former Herbalife distributor Daniel Fallow
Explanation:
of Sandpoint

In each of the following questions, a paragraph or a sentence has been broken up

into different parts. The parts have been scrambled and numbered as given below.
2.
Choose the correct order of these parts from the given alternatives.

1) is 2) at 3) TV 4) film 5) midnight 6) a 7) on 8) great 9) There

1) A) 9, 1, 6, 8, 4, 7, 3, 5, 3

2) B) 9, 1, 6, 8, 4, 7, 3, 2, 5

3) C) 9, 6, 1, 8, 4, 7, 3, 2, 5

4) D) 9, 1, 6, 8, 7, 4, 3, 2, 5

Correct Option is: 2


Your Option is: 2
Result: Correct
Timetaken: 0.0 secs
Explanation: There is a great film on TV at midnight

3.
Read the passage and answer the questions that follow on the basis of the

information provided in the passage.

Few areas of neuron behavioral research seemed more promising is the early sixties

than that investigating the relationship between protein synthesis and learning. The

conceptual framework for the research was derived directly from molecular biology,

which had shown that genetic information is stored in nucleic acids and expressed
in proteins why not acquired information as well.

The first step towards establishing a connection between protein synthesis and

learning seemed to be to block memory (cause adhesion) by interrupting the

production of proteins. We were fortunate in finding a non lethal dosage of

puromycin that could, it first appealed, thoroughly inhibit brain protein synthesis as

well as reliability produce amnesia.

Before the actual connection between protein synthesis and learning could be

established however we began to have douche about whether inhibition of protein

synthesis was in fact the method by which puromycin produced amnesia. First,

ocher drugs, glutavimides themselves potent protein synthesis inhibitors either

failed to cause amnesia in some situations where it could easily be induced by

puromycin or produced an amnesia with a different time course from that of

puromycin. Second, puromycin was found to inhabit protein synthesis by breaking

certain amino acid chaim, and the resulting fragments were suspected of being the

actual cause of amnesia is some eases. Third, puromycin was reported to cause

abnormalities in the train, including seizures. Thus, not only weredecreased protein

synthesis and amnesia dissociated, but alternative mechanism for theamnestic

action of puromycin were readily suggested.

So, puromycin turned out to be a disappointment. It came to be regarded as a poor

agent for amnesia studies, although, of course, it was poor only in the context of

our original paradigm of protein synthesis inhibition. In our frustration, our initial

response was simply to change dregs rather than our conceptual orientation. After

many such disappointments, however, it now appears unlikely, that we will make a

firm connection between protein synthesis and learning merely by pursuing the

approaches of the past our experience with drugs has shown that all the amnestic

agents, often interfere with memory in ways that seem unrelated to their inhibition

of protein synthesis. More importantly, the notion that the interruption or

intensification of protein production in the train can be related in cause and affect
fashion to learning non seems simplistic and unproductive. Remove the battery

from a car and the car will not go Drive the car a long distance at high speed and

the battery will become more highly charged. Neither of these facts proves that the

battery power the car, only knowledge of the overall automotive system will reveal

it mechanism of locomotion and the role of the battery with in the system.

. According to the passage, neuron behaviorists initially based their belief that

protein synthesis was related to learning on which of the following?

1) Specific research into learning on which of the following

2) Traditional theories about learning

3) Historic experiments on the effects puromycin

4) Previous discoveries in molecular biology

5) New technique in protein synthesis.

Correct
4
Option is:
Your Option
4
is:
Result: Correct
Timetaken: 0.0 secs
From the passage, it is clear that, neuron behaviorists
initially based their belief that protein synthesis was related
Explanation:
to learning on previous discoveries in molecular
biology.Therefore,option 4 is the correct answer.

4.
Read the passage and answer the questions that follow on the basis of the

information provided in the passage.

Few areas of neuron behavioral research seemed more promising is the early sixties

than that investigating the relationship between protein synthesis and learning. The

conceptual framework for the research was derived directly from molecular biology,
which had shown that genetic information is stored in nucleic acids and expressed

in proteins why not acquired information as well.

The first step towards establishing a connection between protein synthesis and

learning seemed to be to block memory (cause adhesion) by interrupting the

production of proteins. We were fortunate in finding a non lethal dosage of

puromycin that could, it first appealed, thoroughly inhibit brain protein synthesis as

well as reliability produce amnesia.

Before the actual connection between protein synthesis and learning could be

established however we began to have douche about whether inhibition of protein

synthesis was in fact the method by which puromycin produced amnesia. First,

ocher drugs, glutavimides themselves potent protein synthesis inhibitors either

failed to cause amnesia in some situations where it could easily be induced by

puromycin or produced an amnesia with a different time course from that of

puromycin. Second, puromycin was found to inhabit protein synthesis by breaking

certain amino acid chaim, and the resulting fragments were suspected of being the

actual cause of amnesia is some eases. Third, puromycin was reported to cause

abnormalities in the train, including seizures. Thus, not only weredecreased protein

synthesis and amnesia dissociated, but alternative mechanism for theamnestic

action of puromycin were readily suggested.

So, puromycin turned out to be a disappointment. It came to be regarded as a poor

agent for amnesia studies, although, of course, it was poor only in the context of

our original paradigm of protein synthesis inhibition. In our frustration, our initial

response was simply to change dregs rather than our conceptual orientation. After

many such disappointments, however, it now appears unlikely, that we will make a

firm connection between protein synthesis and learning merely by pursuing the

approaches of the past our experience with drugs has shown that all the amnestic

agents, often interfere with memory in ways that seem unrelated to their inhibition

of protein synthesis. More importantly, the notion that the interruption or


intensification of protein production in the train can be related in cause and affect

fashion to learning non seems simplistic and unproductive. Remove the battery

from a car and the car will not go Drive the car a long distance at high speed and

the battery will become more highly charged. Neither of these facts proves that the

battery power the car, only knowledge of the overall automotive system will reveal

it mechanism of locomotion and the role of the battery with in the system.

In the example of the car the battery is meant to represent which of the following

elements in the neuron behavioral research program?

1) glutarimides

2) acquired information

3) puromycin

4) amnesia

5) protein synthesis

Correct Option is: 5


Your Option is: 1
Result: Wrong
Timetaken: 0.0 secs
Explanation: from this passage option 5 is suit well

5.
In each of the following questions, some sentence are given which are on the same

theme. decide which sentence is the most preferable with respect to grammar;

meaning and usage, suitable for formal writing in English. Find the correct

sentence.

A) He came in too quickly to avoid waking his father

B) He entered in quickly. so as not wake his father.

C) Having not to wake his father, he came in quickly.


D) He came in very quickly so that he might avoid waking his father.

1) A

2) B

3) C

4) D

Correct
4
Option is:
Your Option
4
is:
Result: Correct
Timetaken: 0.0 secs
A) Confusing, whose father. B) So and As same usage of
Explanation: words. C)Wordy sentence D) Indirect sentence with correct
verb sequence and pronoun usage

In each of the following questions, some sentence are given which are on the same

theme. decide which sentence is the most preferable with respect to grammar;

meaning and usage, suitable for formal writing in English. Find the correct

sentence.
6.
A) By June next year, Ajay will be twenty years working in the office.

B) Being twenty years completed, Ajay will be working in this office till June next

C) Till June next year, Ajay will work in the office for twenty years.

D) Ajay will be working in this office upon completing twenty years by next June.

1) A

2) B

3) C
4) D

Correct Option
4
is:
Your Option is: 1
Result: Wrong
Timetaken: 0.0 secs
The verb sequence and the usage of correct time is
Explanation: indicated with the usage of 'by' with period. Hence D
Correct

Read each sentence to find if there is any grammatical error in it. If there is any

error, it will be only one part of the sentence. The number or alphabet of that part is

7. your answer.( Disregard punctuation errors if any)

A.Modern film techniques /B.are far superior / C.than that /D.employed in the

past. / E.No error

1) A

2) B

3) C

4) D

5) No error

Correct Option is: 3


Your Option is: 2
Result: Wrong
Timetaken: 0.0 secs
Explanation: superior to not 'than'

8.
Read each sentence to find if there is any grammatical error in it. If there is any
error, it will be only one part of the sentence. The number or alphabet of that part is

your answer.( Disregard punctuation errors if any)

A.In order to save patrol, /B.motorists must have to/C.be very cautious/ D.while

driving along the highways/E.no error.

1) A

2) B

3) C

4) D

5) E

Correct Option
2
is:
Your Option is: 4
Result: Wrong
Timetaken: 0.0 secs
redundant- must and have to. Meaning is same hence
Explanation:
remove either of the words

9.
Read the passage and answer the questions that follow on the basis of the

information provided in the passage.

MARK HUGHES is a master of the fine art of survival. His Los Angeles-based

Herbalife International Inc. is a pyramid outfit that peddles weight-loss and

nutrition concoctions of dubious value. Bad publicity and regulatory crackdowns

hurt his U.S. business in the late 1980s. But Hughes, 41, continues to enjoy a

luxurious lifestyle in a $20 million Beverly Hills mansion. He has been sharing the

pad and a yacht with his third wife, a former Miss Petite U.S.A. He can finance this

lavish lifestyle just on his salary and bonus, which last year came to $7.3 million.

He survived his troubles in the U.S. by moving overseas, where regulators are less
zealous and consumers even more naive, at least initially. Today 77% of Herbalife

retail sales derive from overseas. Its new prowling grounds: Asia and Russia. Last

year Herbalife's net earnings doubled, to $45 million, on net sales of $632million.

Based on Herbalife's Nasdaq-traded stock, the company has a market capitalization

of $790 million, making Hughes 58% worth $454 million.

There's a worm, though, in Hughes apple. Foreigners aren't stupid. In the end they

know when they've been had. In France, for instance, retail sales rose to $97

million by 1993 and then plunged to $12 million last year. In Germany sales hit

$159 million in 1994 and have since dropped to $54 million.

Perhaps aware that the world may not provide an infinite supply of suckers, Hughes

wanted to unload some of his shares. But in March, after Herbalife's stock

collapsed, he put off a plan to dump about a third of his holdings on the public.

Contributing to Hughes' woes, Herbalife's chief counsel and legal attack dog, David

Addis, quit in January. Before packing up, he reportedly bellowed at Hughes, "I

can't protect you anymore." Addis, who says he wants to spend more time with his

family, chuckles and claims attorney-client privilege.

Trouble on the home front, too. On a recent conference call with distributors,

Hughes revealed he's divorcing his wife, Suzan, whose beaming and perky image

adorns much of Herbalife's literature.

Meanwhile, in a lawsuit that's been quietly moving through Arizona's Superior

Court,former Herbalife distributor Daniel Fallow of Sandpoint, Idaho charges that

Herbalife arbitrarily withholds payment to distributors and marks up its products

over seven times the cost of manufacturing. Fallow also claims Hughes wanted to

use the Russian mafia to gain entry to that nation's market.

Fallow himself is no angel, but his lawsuit, which was posted on the Internet,

brought out other complaints. Randy Cox of Lewiston, Idaho says Herbalife

"destroyed my business" after he and his wife complained to the company that they

were being cheated out of their money by higher-ups in the pyramid


organization.Will Hughes survive again? Don't count on it this time.

At the time when this article was written, if Herbalife had had a market

capitalisation of $ 1billion, what would have been Hughes' share?

1) $420 million

2) $580 million

3) $125 million

4) $500 million

Correct Option
2
is:
Your Option is: 1
Result: Wrong
Timetaken: 0.0 secs
Based on Herbalife's Nasdaq-traded stock......making
Explanation: Hughes 58% worth $454 million. Hence for $ 1 billion, $
580 million

In each of the following questions, a paragraph or a sentence has been broken up

into different parts.The parts have been scrambled and numbered as given below.
10.
Choose the correct order of these parts from the given alternatives.

1) quickly 2) hills 3) weather 4) change 5) the 6)can 7)the 8)in 9) very

1) A) 7, 3, 8, 5, 2, 6, 4, 9, 1

2) B) 1, 7, 3, 8, 5, 2, 6, 4, 9,

3) C) 1, 3, 8, 5, 2, 6, 4, 9, 7

4) D) 7, 3, 5, 8 2, 6, 4, 9, 1
Correct Option is: 1
Your Option is: 1
Result: Correct
Timetaken: 0.0 secs
Explanation: The weather in the hills can change very quickly

In each of the following questions, some sentence are given which are on the same

theme. decide which sentence is the most preferable with respect to grammar;

meaning and usage, suitable for formal writing in English. Find the correct

sentence.

A) Common people are rather impressed by the style of a speech than by its

11. substance

B) Common people are impressed rather by the style of a speech than by its

substance

C) Rather common people are impressed by the style of a speech than by its

substance

D) Common people are impressed by the style of a speech than by its substance.

1) A

2) B

3) C

4) D

Correct Option
4
is:
Your Option is: 4
Result: Correct
Timetaken: 0.0 secs
Rather than will come together. Hence A, B, and C are
Explanation:
incorrect

12.
Read the passage and answer the questions that follow on the basis of the

information provided in the passage.

Few areas of neuron behavioral research seemed more promising is the early sixties

than that investigating the relationship between protein synthesis and learning. The

conceptual framework for the research was derived directly from molecular biology,

which had shown that genetic information is stored in nucleic acids and expressed

in proteins why not acquired information as well.

The first step towards establishing a connection between protein synthesis and

learning seemed to be to block memory (cause adhesion) by interrupting the

production of proteins. We were fortunate in finding a non lethal dosage of

puromycin that could, it first appealed, thoroughly inhibit brain protein synthesis as

well as reliability produce amnesia.

Before the actual connection between protein synthesis and learning could be

established however we began to have douche about whether inhibition of protein

synthesis was in fact the method by which puromycin produced amnesia. First,

ocher drugs, glutavimides themselves potent protein synthesis inhibitors either

failed to cause amnesia in some situations where it could easily be induced by

puromycin or produced an amnesia with a different time course from that of

puromycin. Second, puromycin was found to inhabit protein synthesis by breaking

certain amino acid chaim, and the resulting fragments were suspected of being the

actual cause of amnesia is some eases. Third, puromycin was reported to cause

abnormalities in the train, including seizures. Thus, not only weredecreased protein

synthesis and amnesia dissociated, but alternative mechanism for the amnestic

action of puromycin were readily suggested.

So, puromycin turned out to be a disappointment. It came to be regarded as a poor

agent for amnesia studies, although, of course, it was poor only in the context of
our original paradigm of protein synthesis inhibition. In our frustration, our initial

response was simply to change dregs rather than our conceptual orientation. After

many such disappointments, however, it now appears unlikely, that we will make a

firm connection between protein synthesis and learning merely by pursuing the

approaches of the past our experience with drugs has shown that all the amnestic

agents, often interfere with memory in ways that seem unrelated to their inhibition

of protein synthesis. Moreimportantly, the notion that the interruption or

intensification of protein production in the train can be related in cause and affect

fashion to learning non seems simplistic and unproductive. Remove the battery

from a car and the car will not go Drive the car a long distance at high speed and

the battery will become more highly charged. Neither of these facts proves that the

battery power the car, only knowledge of the overall automotive system will reveal

it mechanism of locomotion and the role of the battery with in the system.

. The primary purpose of the passage is to show that extensive experimentation

has

1) Not supported the hypothesis that learning is directly dependent on protein


synthesis

2) Cast doubt on the value of puromycin in the newer behavioral study of learning

3) Revealed the importance of amnesia in the neuron behavioral study of learning

4) Demonstrated the importance of amino acid fragmentation in the induction of


amnesia

5) Not yet demonstrated the applicability of molecular biology to behavioral


research.

Correct
1
Option is:
Your Option
2
is:
Result: Wrong
Timetaken: 0.0 secs
From the passage,it is clear that,the primary purpose of the
given passage is to show that extensive experimentation has
Explanation:
not supported the hypothesis that learning is directly
dependent on protein synthesis.

In each of the following questions, some sentence are given which are on the same

theme. decide which sentence is the most preferable with respect to grammar;

meaning and usage,suitable for formal writing in English. Find the correct sentence.

A) I have read such a lot about him that I am looking forward to seeing him very

much

13. B) I am reading such a lot about him that I will be looking forward to seeing him

very much

C) Having read such a lot about him that I will be looking forward to seeing him

very much

D) I had read such a lot about him that I am looking forward to seeing him very

much.

1) A

2) B

3) C

4) D

Correct
1
Option is:
Your Option
1
is:
Result: Correct
Timetaken: 0.0 secs
Explanation: Satisfaction of a condition, leading to another action.B)
Condition not yet satisfied, action in progress. Incorrect
C)Wordy sentence. D) Verb sequence not parallel past and
present, incorrect. Hence,A

14.
Read the passage and answer the questions that follow on the basis of the

information provided in the passage.

MARK HUGHES is a master of the fine art of survival. His Los Angeles-based

Herbalife International Inc. is a pyramid outfit that peddles weight-loss and

nutrition concoctions of dubious value. Bad publicity and regulatory crackdowns

hurt his U.S. business in the late 1980s. But Hughes, 41, continues to enjoy a

luxurious lifestyle in a $20 million Beverly Hills mansion. He has been sharing the

pad and a yacht with his third wife, a former Miss Petite U.S.A. He can finance this

lavish lifestyle just on his salary and bonus, which last year came to $7.3 million.

He survived his troubles in the U.S. by moving overseas, where regulators are less

zealous and consumers even more naive, at least initially. Today 77% of Herbalife

retail sales derive from overseas. Its new prowling grounds: Asia and Russia. Last

year Herbalife's net earnings doubled, to $45 million, on net sales of $632million.

Based on Herbalife's Nasdaq-traded stock, the company has a market capitalization

of $790 million, making Hughes 58% worth $454 million.

There's a worm, though, in Hughes apple. Foreigners aren't stupid. In the end they

know when they've been had. In France, for instance, retail sales rose to $97

million by 1993 and then plunged to $12 million last year. In Germany sales hit

$159 million in 1994 and have since dropped to $54 million.

Perhaps aware that the world may not provide an infinite supply of suckers, Hughes

wanted to unload some of his shares. But in March, after Herbalife's stock

collapsed, he put off a plan to dump about a third of his holdings on the public.

Contributing to Hughes' woes, Herbalife's chief counsel and legal attack dog, David

Addis, quit in January. Before packing up, he reportedly bellowed at Hughes, "I

can't protect you anymore." Addis, who says he wants to spend more time with his
family, chuckles and claims attorney-client privilege.

Trouble on the home front, too. On a recent conference call with distributors,

Hughes revealed he's divorcing his wife, Suzan, whose beaming and perky image

adorns much of Herbalife's literature.

Meanwhile, in a lawsuit that's been quietly moving through Arizona's Superior

Court,former Herbalife distributor Daniel Fallow of Sandpoint, Idaho charges that

Herbalife arbitrarily withholds payment to distributors and marks up its products

over seven times the cost of manufacturing. Fallow also claims Hughes wanted to

use the Russian mafia to gain entry to that nation's market.

Fallow himself is no angel, but his lawsuit, which was posted on the Internet,

brought out other complaints. Randy Cox of Lewiston, Idaho says Herbalife

"destroyed my business" after he and his wife complained to the company that they

were being cheated out of their money by higher-ups in the pyramid

organization.Will Hughes survive again? Don't count on it this time.

The complaint of Randy Cox of Lewiston, Idaho, against Herbalife was:>

1) The company did not pay them their dues

2) The products supplied by Hughes were inferior

3) Their higher-ups in the pyramid cheated them

4) Hughes had connections with the Russian mafia

Correct Option is: 3


Your Option is: 2
Result: Wrong
Timetaken: 0.0 secs
Last Para-Randy Cox of Lewiston.....pyramid
Explanation:
organization
In each of the following questions, some sentence are given which are on the same

theme. decide which sentence is the most preferable with respect to grammar;

meaning and usage, suitable for formal writing in English. Find the correct

sentence.
15.
A) The teacher asked the student with a frown on his face, to leave the room

B)The teacher asked with a frown on his face the student to leave the room

C)With a frown on his face,the teacher asked the student to leave the room.

D)The teacher asked the student to leave the room with a frown on his face.

1) A

2) B

3) C

4) D

Correct Option
3
is:
Your Option is: 2
Result: Wrong
Timetaken: 0.0 secs
Misplaced modifier, who was with a frown on face,
Explanation:
teacher,student or room? Ans: Teacher hence, C

Read each sentence to find if there is any grammatical error in it. If there is any

error, it will be only one part of the sentence. The number or alphabet of that part is
16.
your answer.( Disregard punctuation errors if any)

A.I believe /B. that respect /C.is more preferable than /D. money.

1) A

2) B
3) C

4) D

5) No error

Correct Option is: 3


Your Option is: 3
Result: Correct
Timetaken: 0.0 secs
Explanation: preferable 'to' not 'than'

In each of the following questions, a paragraph or a sentence has been broken up

into different parts.The parts have beenscrambled and numbered as given


17.
below.Choose the correct order of these parts from the given alternatives.

1) fifty 2) I 3)a 4)cheque 5) for 6)pounds 7)him 8)wrote

1) A) 2, 8, 7, 5, 4, 3, 1, 6

2) B) 2, 8, 7, 3, 4, 1, 5, 6

3) C) 2, 8, 7, 3, 4, 5, 1, 6

4) D) 2, 8, 7, 3, 4, 6, 5, 1

Correct Option is: 3


Your Option is: 3
Result: Correct
Timetaken: 0.0 secs
Explanation: I wrote him a cheque of fifty pounds, Hence 3

18.
In each of the following questions, a paragraph or a sentence has been broken up
into different parts. The parts have been scrambled and numbered as given below.

Choose the correct order of these parts from the given alternatives.

1) her 2) she 3) to 4) cancel 5) dental 6) appointment 7) has

1) A) 2, 7, 3, 4, 1, 5, 6

2) B) 2, 7, 3, 1, 4, 5, 6

3) C) 1, 7, 3, 4, 5, 2, 6

4) D) 2, 1, 3, 4, 7, 5, 6

Correct Option is: 1


Your Option is: 1
Result: Correct
Timetaken: 0.0 secs
Explanation: She has to cancel her dental appointment

19.
Read the passage and answer the questions that follow on the basis of the

information provided in the passage.

MARK HUGHES is a master of the fine art of survival. His Los Angeles-based

Herbalife International Inc. is a pyramid outfit that peddles weight-loss and

nutrition concoctions of dubious value. Bad publicity and regulatory crackdowns

hurt his U.S. business in the late 1980s. But Hughes, 41, continues to enjoy a

luxurious lifestyle in a $20 million Beverly Hills mansion. He has been sharing the

pad and a yacht with his third wife, a former Miss Petite U.S.A. He can finance this

lavish lifestyle just on his salary and bonus, which last year came to $7.3 million.

He survived his troubles in the U.S. by moving overseas, where regulators are less

zealous and consumers even more naive, at least initially. Today 77% of Herbalife

retail sales derive from overseas. Its new prowling grounds: Asia and Russia. Last

year Herbalife's net earnings doubled, to $45 million, on net sales of $632million.
Based on Herbalife's Nasdaq-traded stock, the company has a market capitalization

of $790 million, making Hughes 58% worth $454 million.

There's a worm, though, in Hughes apple. Foreigners aren't stupid. In the end they

know when they've been had. In France, for instance, retail sales rose to $97

million by 1993 and then plunged to $12 million last year. In Germany sales hit

$159 million in 1994 and have since dropped to $54 million.

Perhaps aware that the world may not provide an infinite supply of suckers, Hughes

wanted to unload some of his shares. But in March, after Herbalife's stock

collapsed, he put off a plan to dump about a third of his holdings on the public.

Contributing to Hughes' woes, Herbalife's chief counsel and legal attack dog, David

Addis, quit in January. Before packing up, he reportedly bellowed at Hughes, "I

can't protect you anymore." Addis, who says he wants to spend more time with his

family, chuckles and claims attorney-client privilege.

Trouble on the home front, too. On a recent conference call with distributors,

Hughes revealed he's divorcing his wife, Suzan, whose beaming and perky image

adorns much of Herbalife's literature.

Meanwhile, in a lawsuit that's been quietly moving through Arizona's Superior

Court,former Herbalife distributor Daniel Fallow of Sandpoint, Idaho charges that

Herbalife arbitrarily withholds payment to distributors and marks up its products

over seven times the cost of manufacturing. Fallow also claims Hughes wanted to

use the Russian mafia to gain entry to that nation's market.

Fallow himself is no angel, but his lawsuit, which was posted on the Internet,

brought out other complaints. Randy Cox of Lewiston, Idaho says Herbalife

"destroyed my business" after he and his wife complained to the company that they

were being cheated out of their money by higher-ups in the pyramid

organization.Will Hughes survive again? Don't count on it this time.

In the year in which Hughes' salary and bonuses came to US$ 7.3 million, what was
the retail sales for Herbalife in France?

1) $12 million

2) $159 million

3) $54 million

4) $97 million

Correct Option is: 1


Your Option is: 4
Result: Wrong
Timetaken: 0.0 secs
Explanation: In France, for instance......$12 million last year.

20.
Read the passage and answer the questions that follow on the basis of the

information provided in the passage.

Few areas of neuron behavioral research seemed more promising is the early sixties

than that investigating the relationship between protein synthesis and learning. The

conceptual framework for the research was derived directly from molecular biology,

which had shown that genetic information is stored in nucleic acids and expressed

in proteins why not acquired information as well.

The first step towards establishing a connection between protein synthesis and

learning seemed to be to block memory (cause adhesion) by interrupting the

production of proteins. We were fortunate in finding a non lethal dosage of

puromycin that could, it first appealed, thoroughly inhibit brain protein synthesis as

well as reliability produce amnesia.

Before the actual connection between protein synthesis and learning could be

established however we began to have douche about whether inhibition of protein

synthesis was in fact the method by which puromycin produced amnesia. First,
ocher drugs, glutavimides themselves potent protein synthesis inhibitors either

failed to cause amnesia in some situations where it could easily be induced by

puromycin or produced an amnesia with a different time course from that of

puromycin. Second, puromycin was found to inhabit protein synthesis by breaking

certain amino acid chaim, and the resulting fragments were suspected of being the

actual cause of amnesia is some eases. Third, puromycin was reported to cause

abnormalities in the train, including seizures. Thus, not only weredecreased protein

synthesis and amnesia dissociated, but alternative mechanism for theamnestic

action of puromycin were readily suggested.

So, puromycin turned out to be a disappointment. It came to be regarded as a poor

agent for amnesia studies, although, of course, it was poor only in the context of

our original paradigm of protein synthesis inhibition. In our frustration, our initial

response was simply to change dregs rather than our conceptual orientation. After

many such disappointments, however, it now appears unlikely, that we will make a

firm connection between protein synthesis and learning merely by pursuing the

approaches of the past our experience with drugs has shown that all the amnestic

agents, often interfere with memory in ways that seem unrelated to their inhibition

of protein synthesis. More importantly, the notion that the interruption or

intensification of protein production in the train can be related in cause and affect

fashion to learning non seems simplistic and unproductive. Remove the battery

from a car and the car will not go Drive the car a long distance at high speed and

the battery will become more highly charged. Neither of these facts proves that the

battery power the car, only knowledge of the overall automotive system will reveal

it mechanism of locomotion and the role of the battery with in the system.

This passage was most likely excepted from

1) A book review in a leading journal devoted to genetic research.

2) A diary kept by a practicing neuron behavioral research


3) An article summarizing a series of scientific investigations in neuron behavioral
research.

4) A news paper article on recent advances in the biochemistry of learning

5) A technical article on experimental techniques in the field of molecular biology.

Correct Option is: 3


Your Option is: 1
Result: Wrong
Timetaken: 0.0 secs
Explanation: Nil

21.
Read the passage and answer the questions that follow on the basis of the

information provided in the passage.

MARK HUGHES is a master of the fine art of survival. His Los Angeles-based

Herbalife International Inc. is a pyramid outfit that peddles weight-loss and

nutrition concoctions of dubious value. Bad publicity and regulatory crackdowns

hurt his U.S. business in the late 1980s. But Hughes, 41, continues to enjoy a

luxurious lifestyle in a $20 million Beverly Hills mansion. He has been sharing the

pad and a yacht with his third wife, a former Miss Petite U.S.A. He can finance this

lavish lifestyle just on his salary and bonus, which last year came to $7.3 million.

He survived his troubles in the U.S. by moving overseas, where regulators are less

zealous and consumers even more naive, at least initially. Today 77% of Herbalife

retail sales derive from overseas. Its new prowling grounds: Asia and Russia. Last

year Herbalife's net earnings doubled, to $45 million, on net sales of $632million.

Based on Herbalife's Nasdaq-traded stock, the company has a market capitalization

of $790 million, making Hughes 58% worth $454 million.

There's a worm, though, in Hughes apple. Foreigners aren't stupid. In the end they

know when they've been had. In France, for instance, retail sales rose to $97

million by 1993 and then plunged to $12 million last year. In Germany sales hit
$159 million in 1994 and have since dropped to $54 million.

Perhaps aware that the world may not provide an infinite supply of suckers, Hughes

wanted to unload some of his shares. But in March, after Herbalife's stock

collapsed, he put off a plan to dump about a third of his holdings on the public.

Contributing to Hughes' woes, Herbalife's chief counsel and legal attack dog, David

Addis, quit in January. Before packing up, he reportedly bellowed at Hughes, "I

can't protect you anymore." Addis, who says he wants to spend more time with his

family, chuckles and claims attorney-client privilege.

Trouble on the home front, too. On a recent conference call with distributors,

Hughes revealed he's divorcing his wife, Suzan, whose beaming and perky image

adorns much of Herbalife's literature.

Meanwhile, in a lawsuit that's been quietly moving through Arizona's Superior

Court,former Herbalife distributor Daniel Fallow of Sandpoint, Idaho charges that

Herbalife arbitrarily withholds payment to distributors and marks up its products

over seven times the cost of manufacturing. Fallow also claims Hughes wanted to

use the Russian mafia to gain entry to that nation's market.

Fallow himself is no angel, but his lawsuit, which was posted on the Internet,

brought out other complaints. Randy Cox of Lewiston, Idaho says Herbalife

"destroyed my business" after he and his wife complained to the company that they

were being cheated out of their money by higher-ups in the pyramid

organization.Will Hughes survive again? Don't count on it this time.

Herbalife Inc is based in:

1) Los Angeles

2) Columbus

3) New York

4) Austin
Correct Option is: 1
Your Option is: 4
Result: Wrong
Timetaken: 0.0 secs
Explanation: 2nd sentence of the first para.Los Angeles based

In each of the following questions, a paragraph or a sentence has been broken up

into different parts.The parts have been scrambled and numbered as given
22.
below.Choose the correct order of these parts from the given alternatives.

1) all 2) I 3) the 4) keep 5) sneezing 6) time

1) A) 2, 4, 5, 3, 1, 6

2) B) 2, 4, 6, 5, 3, 1

3) C) 1, 4, 5, 2, 3, 6

4) D) 2, 4, 5, 1, 3, 6

Correct Option is: 4


Your Option is: 4
Result: Correct
Timetaken: 0.0 secs
Explanation: I keep sneezing all the time

Read each sentence to find if there is any grammatical error in it. If there is any

error, it will be only one part of the sentence. The number or alphabet of that part is

23. your answer.( Disregard punctuation errors if any)

A.Not one of the children / B.has ever sang/C.on any occasion /D.in public

before/E.no error
1) A

2) B

3) C

4) D

5) E

Correct Option is: 2


Your Option is: 2
Result: Correct
Timetaken: 0.0 secs
Explanation: has ever sung (3rd form of verb-sing)

24.
Read the passage and answer the questions that follow on the basis of the

information provided in the passage.

Few areas of neuron behavioral research seemed more promising is the early sixties

than that investigating the relationship between protein synthesis and learning. The

conceptual framework for the research was derived directly from molecular biology,

which had shown that genetic information is stored in nucleic acids and expressed

in proteins why not acquired information as well.

The first step towards establishing a connection between protein synthesis and

learning seemed to be to block memory (cause adhesion) by interrupting the

production of proteins. We were fortunate in finding a non lethal dosage of

puromycin that could, it first appealed, thoroughly inhibit brain protein synthesis as

well as reliability produce amnesia.

Before the actual connection between protein synthesis and learning could be

established however we began to have douche about whether inhibition of protein

synthesis was in fact the method by which puromycin produced amnesia. First,

ocher drugs, glutavimides themselves potent protein synthesis inhibitors either


failed to cause amnesia in some situations where it could easily be induced by

puromycin or produced an amnesia with a different time course from that of

puromycin. Second, puromycin was found to inhabit protein synthesis by breaking

certain amino acid chaim, and the resulting fragments were suspected of being the

actual cause of amnesia is some eases. Third, puromycin was reported to cause

abnormalities in the train, including seizures. Thus, not only weredecreased protein

synthesis and amnesia dissociated, but alternative mechanism for theamnestic

action of puromycin were readily suggested.

So, puromycin turned out to be a disappointment. It came to be regarded as a poor

agent for amnesia studies, although, of course, it was poor only in the context of

our original paradigm of protein synthesis inhibition. In our frustration, our initial

response was simply to change dregs rather than our conceptual orientation. After

many such disappointments, however, it now appears unlikely, that we will make a

firm connection between protein synthesis and learning merely by pursuing the

approaches of the past our experience with drugs has shown that all the amnestic

agents, often interfere with memory in ways that seem unrelated to their inhibition

of protein synthesis. More importantly, the notion that the interruption or

intensification of protein production in the train can be related in cause and affect

fashion to learning non seems simplistic and unproductive. Remove the battery

from a car and the car will not go Drive the car a long distance at high speed and

the battery will become more highly charged. Neither of these facts proves that the

battery power the car, only knowledge of the overall automotive system will reveal

it mechanism of locomotion and the role of the battery with in the system.

In the example of the car the battery is meant to represent which of the following

elements in the neuron behavioral research program?

1) glutarimides

2) acquired information
3) puromycin

4) amnesia

5) protein synthesis

Correct Option is: 5


Your Option is: 1
Result: Wrong
Timetaken: 0.0 secs
Explanation: according to this passage option 5 is apt well

Read each sentence to find if there is any grammatical error in it. If there is any

error, it will be only one part of the sentence. The number or alphabet of that part is

25. your answer.( Disregard punctuation errors if any)

A.The principals of equal justice /B. for all is one of / C.the corner stones of our

/D.democratic way of life. / E.no error

1) A

2) B

3) C

4) D

5) No error

Correct Option
1
is:
Your Option is: 2
Result: Wrong
Timetaken: 0.0 secs
Principal:One who holds a rank or important position
Explanation:
Principle:Basic truth law,assumption, rule etc
In each of the following questions, a paragraph or a sentence has been broken up

into different parts. The parts have been scrambled and numbered as given below.

Choose the correct order of these parts from the given alternatives.

1) rent 2)a 3) has 4) room 5)in 6) house 7) she 8) to 9) rent A. 7, 3, 2, 4, 1, 8, 5,


1.
9, 6

B. 7, 3, 1, 4, 8, 2, 5, 9, 6

C. 7, 3, 2, 4, 8, 1, 5, 9, 6

D. 7, 3, 2, 4, 8, 1, 5, 6, 9

1) A

2) B

3) C

4) D

Correct Option is: 3


Your Option is: 1
Result: Wrong
Timetaken: 0.0 secs
"she has a room to rent in rent house"this is correct
Explanation:
order

2.
Read the passage and answer the questions that follow on the basis of the

information provided in the passage.

According to Albert Einstein the non mathematician, is seized by a mysterious

shudderingwhen he hears of 'four-dimensional' things, he is seized by a feeling,

which is very similar to the thoughts awakened by the occult. And at the same time

the statement that the world in which we live is a four-dimensional space - time

continuum is quite a common place statement.


This might lead to an argument regarding the use of the term ''commonplace'' by

Einstein.Yet the difficulty lies more in the wording than the ideas. Einstein's concept

of the universe as a four-dimensional space-time continuum becomes plain and

clear, when what he means by ''continuum'' becomes clear. A continuum is

something that is continuous, A ruler, for example, is a one-dimensional space

continuum. Most rulers are divided into inches and frWASLions, scaled down to one-

sixteenth of an inch. Will it be possible to conceive a ruler, which is calibrated to a

millionth or billionth of an inch. In theory there is no reason why the steps from

point to point should not be even smaller. What distinguishes a continuum is the

fWASL that the space between any two points can be sub-divided into an infinite

number of smaller divisions.

A railroad track is a one-dimensional space continuum and on it the engineer of a

traincan describe his position at any time by citing a single co-ordinate point - i.e., a

stationor a milestone. A sea captain, however, has to worry about two dimensions.

The surface of the sea is a two-dimensional continuum and the co-ordinate points

by which sailor fixes his positions in his two dimensional continuum are latitude and

longitude. An airplane pilot guides his plane through a three - dimensional

continuum, hence he has to consider not only latitude and longitude, but also his

height above the ground. The continuum of an airplane pilot constitutes space as

we perceive it. In other words, the space of our world is a three-dimensional

continuum.

Just indicating its position in space is not enough while describing any physical

event, which involves motion. How position changes in time also needs to be

mentioned. Thus to give an accurate picture of the operation of a New York -

Chicago express, one must mention not only that it goes from New - York to Albany

to Syracuse to Cleveland to Toledo to Chicago, but also the times at which it

touches each of those points. This can be done either by means of a timetable or a

visual chart. If the miles between New York and Chicago are plotted horizontally on
a piece of ruled paper and the hours and minutes are plotted vertically, then a

diagonal line properly drawn across the page illustrates the progress of the train in

two-dimensional space - time continuum. This type of graphic representation is

familiar to most newspaper readers; a stock market chart,

For example, pictures financial events in a two - dimensional dollar - time

continuum. Similarly for the best picturisation of the flight of an airplane from New

York to Los Angeles a four - dimensional space - time continuum is essential. The

latitude, longitude and altitude will only make sense to the traffic manager of the

airline if the time co -ordinate is also mentioned. Therefore time is the fourth

dimension. If a flight has to be looked at, perceived as a whole, it wouldn't work if it

is broken down into a series of disconnected take - offs, climbs, glides, and landing,

it needs to be looked at and perceived as a continuous four - dimensional space -

time continuum curve.

The purpose of this passage is to highlight the point that

1) Stock market charts may be helpful to physicists

2) The fourth dimension is time.

3) Plots and sea captain have something in common

4) Non-mathematician's are often afraid of the commonplace

5) There is a marked quality to distance

Correct Option is: 3


Your Option is: 2
Result: Wrong
Timetaken: 0.0 secs
Explanation: Nil

3.
One of the four sentences given in each question is grammatically wrong .Find the
incorrect sentence.

A) She had finished her work when I met her.

B) Do you believe in God?

C) He cut his hand with a knife.

D) He challenged me for a duel.

1) A

2) B

3) C

4) D

Correct Option is: 4


Your Option is: 4
Result: Correct
Timetaken: 0.0 secs
Explanation: Nil

One of the four sentences given in each question is grammatically wrong .Find the

incorrect sentence.

A) Sumit is my elder brother.


4.
B) He is two years younger to me.

C) He is the eldest man of this village.

D) Ravi is five years older than me.

1) A

2) B

3) C
4) D

Correct Option is: 3


Your Option is: 1
Result: Wrong
Timetaken: 0.0 secs
Explanation: Nil

One of the four sentences given in each question is grammatically wrong .Find the

incorrect sentence.

A) Our followers are but a handful.


5.
B) Neither he nor I was there.

C) Many a glorious deeds were done.

D) Everyone of the boys loves to ride.

1) A

2) B

3) C

4) D

Correct Option is: 3


Your Option is: 2
Result: Wrong
Timetaken: 0.0 secs
Explanation: Nil

6.
Read the passage and answer the questions that follow on the basis of the

information provided in the passage.


According to Albert Einstein the non mathematician, is seized by a mysterious

shudderingwhen he hears of 'four-dimensional' things, he is seized by a feeling,

which is very similar to the thoughts awakened by the occult. And at the same time

the statement that the world in which we live is a four-dimensional space - time

continuum is quite a common place statement.

This might lead to an argument regarding the use of the term ''commonplace'' by

Einstein.Yet the difficulty lies more in the wording than the ideas. Einstein's concept

of the universe as a four-dimensional space-time continuum becomes plain and

clear, when what he means by ''continuum'' becomes clear. A continuum is

something that is continuous, A ruler, for example, is a one-dimensional space

continuum. Most rulers are divided into inches and frWASLions, scaled down to one-

sixteenth of an inch. Will it be possible to conceive a ruler, which is calibrated to a

millionth or billionth of an inch. In theory there is no reason why the steps from

point to point should not be even smaller. What distinguishes a continuum is the

fWASL that the space between any two points can be sub-divided into an infinite

number of smaller divisions.

A railroad track is a one-dimensional space continuum and on it the engineer of a

traincan describe his position at any time by citing a single co-ordinate point - i.e., a

stationor a milestone. A sea captain, however, has to worry about two dimensions.

The surface of the sea is a two-dimensional continuum and the co-ordinate points

by which sailor fixes his positions in his two dimensional continuum are latitude and

longitude. An airplane pilot guides his plane through a three - dimensional

continuum, hence he has to consider not only latitude and longitude, but also his

height above the ground. The continuum of an airplane pilot constitutes space as

we perceive it. In other words, the space of our world is a three-dimensional

continuum.

Just indicating its position in space is not enough while describing any physical

event, which involves motion. How position changes in time also needs to be
mentioned. Thus to give an accurate picture of the operation of a New York -

Chicago express, one must mention not only that it goes from New - York to Albany

to Syracuse to Cleveland to Toledo to Chicago, but also the times at which it

touches each of those points. This can be done either by means of a timetable or a

visual chart. If the miles between New York and Chicago are plotted horizontally on

a piece of ruled paper and the hours and minutes are plotted vertically, then a

diagonal line properly drawn across the page illustrates the progress of the train in

two-dimensional space - time continuum. This type of graphic representation is

familiar to most newspaper readers; a stock market chart,

For example, pictures financial events in a two - dimensional dollar - time

continuum. Similarly for the best picturisation of the flight of an airplane from New

York to Los Angeles a four - dimensional space - time continuum is essential. The

latitude, longitude and altitude will only make sense to the traffic manager of the

airline if the time co -ordinate is also mentioned. Therefore time is the fourth

dimension. If a flight has to be looked at, perceived as a whole, it wouldn't work if it

is broken down into a series of disconnected take - offs, climbs, glides, and landing,

it needs to be looked at and perceived as a continuous four - dimensional space -

time continuum curve.

According to the passage, an airlines traffic manager depends upon all of the

following EXCEPT

1) Latitude, altitude

2) The time co-ordinate

3) Longitude

4) The continuous curve in co four

Correct Option is: 5


Your Option is: 4
Result: Wrong
Timetaken: 0.0 secs
Explanation: Nil

In each of the following questions, some sentence are given which are on the same

theme. decide which sentence is the most preferable with respect to grammar;

meaning and usage, suitable for formal writing in English. Find the correct

sentence.
7.
A) Hoping to be hearing from you, I remain yours sincerely.

B) Hoping to hear from you, I remain yours sincerely.

C) Hoping to have heard from you, I sincerely remain yours.

D) Sincerely I remain yours hoping to be hearing from you.

1) A

2) B

3) C

4) D

Correct Option is: 2


Your Option is: 2
Result: Correct
Timetaken: 0.0 secs
Explanation: Nil

8.
Read each sentence to find if there is any grammatical error in it. If there is any

error, it will be only one part of the sentence. The number or alphabet of that part is

your answer.(Disregard punctuation errors if any)


A.The clothes / B.were neatly / C.hanged /D.on the cloth line.E.No error

1) A

2) B

3) C

4) D

5) No error

Correct Option is: 3


Your Option is: 3
Result: Correct
Timetaken: 0.0 secs
Explanation: Nil

In each of the following questions, a paragraph or a sentence has been broken up

into different parts. The parts have been scrambled and numbered as given below.
9.
Choose the correct order of these parts from the given alternatives.

1) I 2)do 3) a 4) of 5) in 6) lot 7) reading 8) my 9) time 10) free

1) A. 1, 2, 3, 6, 4, 7, 5, 10, 8, 9

2) B. 1, 2, 6, 3, 4, 7, 5, 8, 10, 9

3) C. 1, 2, 3, 6, 7, 4, 5, 8, 10, 9

4) D. 1, 2, 3, 6, 4, 7, 5, 8, 10, 9

Correct Option
4
is:
Your Option is: 2
Result: Wrong
Timetaken: 0.0 secs
"i do a lot of reading in my free time " option d is suit
Explanation:
grammatically

Read each sentence to find if there is any grammatical error in it. If there is any

error, it will be only one part of the sentence. The number or alphabet of that part is
10.
your answer.(Disregard punctuation errors if any)

A.I never have / B.visited / C.or intend to visit /D. foreign countries /E.No error

1) A

2) B

3) C

4) D

5) No error

Correct Option is: 1


Your Option is: 1
Result: Correct
Timetaken: 0.0 secs
Explanation: Nil

In each of the following questions, some sentence are given which are on the same

theme. decide which sentence is the most preferable with respect to grammar;

meaning and usage, suitable for formal writing in English. Find the correct

sentence.
11.
A) They have placed order for books.

B) He has applied for lectureship.

C) The river has overflown its bank.

D) Give me rupees two and a half.


1) A

2) B

3) C

4) D

Correct Option is: 4


Your Option is: 3
Result: Wrong
Timetaken: 0.0 secs
Explanation: Nil

Read the passage and answer the questions that follow on the basis of the

information provided in the passage.

Work expands so as to fill the time available for its completion. The general

recognition of this fact is shown in the proverbial phrase,' It is the busiest man who

has time to spare'.Thus, an elderly lady at leisure can spend the entire day writing

a postcard to her niece. An hour will be spent in writing a postcard , another


12.
hunting for spectacles, half an hour to search for the address , an hour and a

quarter in composition and twenty minutes in deciding whether or not to take an

umbrella when going to the pillar box in the street. The total effort that could

occupy a busy man for three minutes, all told may in this fashion leave another

person completely exhausted after a day of doubt, anxiety and toil.

What is the total time spent by the elderly lady in writing a postcard?

1) A) Three minutes.

2) B) four hours and five minutes.

3) C) half day
4) D)the entire day.

Correct Option is: 4


Your Option is: 1
Result: Wrong
Timetaken: 0.0 secs
Explanation: Nil

In each of the following questions, some sentence are given which are on the same

theme. decide which sentence is the most preferable with respect to grammar;

meaning and usage, suitable for formal writing in English. Find the correct

sentence.
13.
A) The proceeds of the charity show are for riot victims

B) He asked Ajay and I to go.

C) The weather of this place does not suit me.

D) Either Rajesh or his friends has done it.

1) A

2) B

3) C

4) D

Correct Option is: 1


Your Option is: 2
Result: Wrong
Timetaken: 0.0 secs
Explanation: Nil
In each of the following questions, a paragraph or a sentence has been broken up

into different parts. The parts have been scrambled and numbered as given below.

14. Choose the correct order of these parts from the given alternatives.

1) on to 2) seat 3) evening 4) Edinburgh 5)the 6) booked 7) flight 8) a 9) time

10)to

1) A. 9, 6, 8, 2, 1, 5, 3, 7, 10, 4

2) B. 9, 6, 8, 1, 2, 5, 3, 7, 10, 4

3) C. 9, 3, 8, 2, 1, 5, 6, 7, 10, 4

4) D. 9, 6, 1, 2, 5, 8, 3, 7, 10, 4

Correct Option
1
is:
Your Option is: 4
Result: Wrong
Timetaken: 0.0 secs
"time booked a seat on the evening flight to edinburgh"
Explanation:
option 1 only connect grammatically

In each of the following questions, a paragraph or a sentence has been broken up

into different parts. The parts have been scrambled andnumbered as given below.
15.
Choose the correct order of these parts from the given alternatives.

1) I 2) my 3) leg 4) to 5)hire 6) gardener 7) when 8) a 9) had 10) I 11) broke

1) A. 1, 9, 4, 5, 8, 6, 7, 10, 11, 3, 2

2) B. 1, 9, 4, 5, 11, 6, 8, 10, 7, 2, 3

3) C. 1, 9, 4, 5, 8, 6, 7, 10, 11, 2, 3

4) D. 1, 4, 9, 5, 8, 6, 7, 10, 11, 2, 3
Correct Option
3
is:
Your Option is: 2
Result: Wrong
Timetaken: 0.0 secs
"i had to hire a gardener when t broke my leg "this is
Explanation:
grammatically correct order

16.
Read the passage and answer the questions that follow on the basis of the

information provided in the passage.

According to Albert Einstein the non mathematician, is seized by a mysterious

shudderingwhen he hears of 'four-dimensional' things, he is seized by a feeling,

which is very similar to the thoughts awakened by the occult. And at the same time

the statement that the world in which we live is a four-dimensional space - time

continuum is quite a common place statement.

This might lead to an argument regarding the use of the term ''commonplace'' by

Einstein.Yet the difficulty lies more in the wording than the ideas. Einstein's concept

of the universe as a four-dimensional space-time continuum becomes plain and

clear, when what he means by ''continuum'' becomes clear. A continuum is

something that is continuous, A ruler, for example, is a one-dimensional space

continuum. Most rulers are divided into inches and frWASLions, scaled down to one-

sixteenth of an inch. Will it be possible to conceive a ruler, which is calibrated to a

millionth or billionth of an inch. In theory there is no reason why the steps from

point to point should not be even smaller. What distinguishes a continuum is the

fWASL that the space between any two points can be sub-divided into an infinite

number of smaller divisions.

A railroad track is a one-dimensional space continuum and on it the engineer of a

traincan describe his position at any time by citing a single co-ordinate point - i.e., a

stationor a milestone. A sea captain, however, has to worry about two dimensions.
The surface of the sea is a two-dimensional continuum and the co-ordinate points

by which sailor fixes his positions in his two dimensional continuum are latitude and

longitude. An airplane pilot guides his plane through a three - dimensional

continuum, hence he has to consider not only latitude and longitude, but also his

height above the ground. The continuum of an airplane pilot constitutes space as

we perceive it. In other words, the space of our world is a three-dimensional

continuum.

Just indicating its position in space is not enough while describing any physical

event, which involves motion. How position changes in time also needs to be

mentioned. Thus to give an accurate picture of the operation of a New York -

Chicago express, one must mention not only that it goes from New - York to Albany

to Syracuse to Cleveland to Toledo to Chicago, but also the times at which it

touches each of those points. This can be done either by means of a timetable or a

visual chart. If the miles between New York and Chicago are plotted horizontally on

a piece of ruled paper and the hours and minutes are plotted vertically, then a

diagonal line properly drawn across the page illustrates the progress of the train in

two-dimensional space - time continuum. This type of graphic representation is

familiar to most newspaper readers; a stock market chart,

For example, pictures financial events in a two - dimensional dollar - time

continuum. Similarly for the best picturisation of the flight of an airplane from New

York to Los Angeles a four - dimensional space - time continuum is essential. The

latitude, longitude and altitude will only make sense to the traffic manager of the

airline if the time co -ordinate is also mentioned. Therefore time is the fourth

dimension. If a flight has to be looked at, perceived as a whole, it wouldn't work if it

is broken down into a series of disconnected take - offs, climbs, glides, and landing,

it needs to be looked at and perceived as a continuous four - dimensional space -

time continuum curve.

According to the author if on wishes portray a physical event in which motion plays
a role- one has to -

1) Make use of a time-table

2) Indicate how position changes in time

3) Be conversant with the scientist's theories

4) Describe it graphically

5) Be aware of altitude, latitude and longitude

Correct Option is: 2


Your Option is: 4
Result: Wrong
Timetaken: 0.0 secs
Explanation: Nil

Read the passage and answer the questions that follow on the basis of the

information provided in the passage.

Work expands so as to fill the time available for its completion. The general

recognition of this fact is shown in the proverbial phrase,' It is the busiest man who

has time to spare'.Thus, an elderly lady at leisure can spend the entire day writing

a postcard to her niece. An hour will be spent in writing a postcard , another

17. hunting for spectacles, half an hour to search for the address , an hour and a

quarter in composition and twenty minutes in deciding whether or not to take an

umbrella when going to the pillar box in the street. The total effort that could

occupy a busy man for three minutes, all told may in this fashion leave another

person completely exhausted after a day of doubt, anxiety and toil.

Explain the sentence : work expands so as to fill the time available for its

completion?.

1) A) The more work there is to be done , the more time needed.


2) B) whatever time is available for a given amount of work, all of it will be used.

3) C) If you have more time you can do some work.

4) D) If you have some important work to do , you should always have some
additional time.

Correct Option is: 2


Your Option is: 2
Result: Correct
Timetaken: 0.0 secs
Explanation: Nil

In each of the following questions, some sentence are given which are on the same

theme. decide which sentence is the most preferable with respect to grammar;

meaning and usage, suitable for formal writing in English. Find the correct

sentence.
18.
A) From which train did you come?

B) A series of incidents have taken place.

C) It is a five--men committee.

D) This pronunciation is peculiar to Bengalis.

1) A

2) B

3) C

4) D

Correct Option is: 4


Your Option is: 1
Result: Wrong
Timetaken: 0.0 secs
Explanation: Nil

Read each sentence to find if there is any grammatical error in it. If there is any

error, it will be only one part of the sentence. The number or alphabet of that part is
19.
your answer.(Disregard punctuation errors if any)

A.I never have / B.visited / C.or intend to visit /D. foreign countries /E.No error

1) A

2) B

3) C

4) D

5) No error

Correct Option is: 1


Your Option is: 3
Result: Wrong
Timetaken: 0.0 secs
Explanation: Nil

20.
Read the passage and answer the questions that follow on the basis of the

information provided in the passage.

According to Albert Einstein the non mathematician, is seized by a mysterious

shudderingwhen he hears of 'four-dimensional' things, he is seized by a feeling,

which is very similar to the thoughts awakened by the occult. And at the same time

the statement that the world in which we live is a four-dimensional space - time

continuum is quite a common place statement.

This might lead to an argument regarding the use of the term ''commonplace'' by

Einstein.Yet the difficulty lies more in the wording than the ideas. Einstein's concept
of the universe as a four-dimensional space-time continuum becomes plain and

clear, when what he means by ''continuum'' becomes clear. A continuum is

something that is continuous, A ruler, for example, is a one-dimensional space

continuum. Most rulers are divided into inches and frWASLions, scaled down to one-

sixteenth of an inch. Will it be possible to conceive a ruler, which is calibrated to a

millionth or billionth of an inch. In theory there is no reason why the steps from

point to point should not be even smaller. What distinguishes a continuum is the

fWASL that the space between any two points can be sub-divided into an infinite

number of smaller divisions.

A railroad track is a one-dimensional space continuum and on it the engineer of a

traincan describe his position at any time by citing a single co-ordinate point - i.e., a

stationor a milestone. A sea captain, however, has to worry about two dimensions.

The surface of the sea is a two-dimensional continuum and the co-ordinate points

by which sailor fixes his positions in his two dimensional continuum are latitude and

longitude. An airplane pilot guides his plane through a three - dimensional

continuum, hence he has to consider not only latitude and longitude, but also his

height above the ground. The continuum of an airplane pilot constitutes space as

we perceive it. In other words, the space of our world is a three-dimensional

continuum.

Just indicating its position in space is not enough while describing any physical

event, which involves motion. How position changes in time also needs to be

mentioned. Thus to give an accurate picture of the operation of a New York -

Chicago express, one must mention not only that it goes from New - York to Albany

to Syracuse to Cleveland to Toledo to Chicago, but also the times at which it

touches each of those points. This can be done either by means of a timetable or a

visual chart. If the miles between New York and Chicago are plotted horizontally on

a piece of ruled paper and the hours and minutes are plotted vertically, then a

diagonal line properly drawn across the page illustrates the progress of the train in
two-dimensional space - time continuum. This type of graphic representation is

familiar to most newspaper readers; a stock market chart,

For example, pictures financial events in a two - dimensional dollar - time

continuum. Similarly for the best picturisation of the flight of an airplane from New

York to Los Angeles a four - dimensional space - time continuum is essential. The

latitude, longitude and altitude will only make sense to the traffic manager of the

airline if the time co -ordinate is also mentioned. Therefore time is the fourth

dimension. If a flight has to be looked at, perceived as a whole, it wouldn't work if it

is broken down into a series of disconnected take - offs, climbs, glides, and landing,

it needs to be looked at and perceived as a continuous four - dimensional space -

time continuum curve.

The underlying tone of this selection is

1) Persuasive

2) Deferential

3) Candid

4) Instructive

5) Gently condescending

Correct Option is: 4


Your Option is: 4
Result: Correct
Timetaken: 0.0 secs
Explanation: Nil

21.
Read the passage and answer the questions that follow on the basis of the

information provided in the passage.

Work expands so as to fill the time available for its completion. The general
recognition of this fact is shown in the proverbial phrase,' It is the busiest man who

has time to spare'.Thus, an elderly lady at leisure can spend the entire day writing

a postcard to her niece. An hour will be spent in writing a postcard , another

hunting for spectacles, half an hour to search for the address , an hour and a

quarter in composition and twenty minutes in deciding whether or not to take an

umbrella when going to the pillar box in the street. The total effort that could

occupy a busy man for three minutes, all told may in this fashion leave another

person completely exhausted after a day of doubt, anxiety and toil.

What does the expression ?pillar box? stand for?

1) A) a box attached to the pillar.

2) B) a box in the pillar

3) C) box office.

4) D) a pillar type postbox.

Correct Option is: 4


Your Option is: 2
Result: Wrong
Timetaken: 0.0 secs
Explanation: Nil

22.
In each of the following questions, a paragraph or a sentence has been broken up

into different parts. The parts have been scrambled and numbered as given below.

Choose the correct order of these parts from the given alternatives.

. 1) She 2) trust 3) Don't 4) because 5)is 6) her 7) lying

A. 1, 5, 2, 3, 4, 6, 7

B. 3, 2, 6, 4, 1, 5, 7

C. 3, 2, 6, 4, 1, 5, 7
D. 3, 2, 6, 1, 4, 5, 7

1) A

2) B

3) C

4) D

Correct Option is: 2


Your Option is: 4
Result: Wrong
Timetaken: 0.0 secs
Explanation: "don't trust her because she is lying"

Read the passage and answer the questions that follow on the basis of the

information provided in the passage.

Work expands so as to fill the time available for its completion. The general

recognition of this fact is shown in the proverbial phrase,' It is the busiest man who

has time to spare'.Thus, an elderly lady at leisure can spend the entire day writing

a postcard to her niece. An hour will be spent in writing a postcard , another


23.
hunting for spectacles, half an hour to search for the address , an hour and a

quarter in composition and twenty minutes in deciding whether or not to take an

umbrella when going to the pillar box in the street. The total effort that could

occupy a busy man for three minutes, all told may in this fashion leave another

person completely exhausted after a day of doubt, anxiety and toil.

Who is the person likely to take more time to do work.:

1) A) a busy man.

2) B) a man of leisure.
3) C) an elderly person.

4) D)an exhausted person

Correct Option is: 2


Your Option is: 2
Result: Correct
Timetaken: 0.0 secs
Explanation: Nil

In each of the following questions, some sentence are given which are on the same

theme. decide which sentence is the most preferable with respect to grammar;

meaning and usage, suitable for formal writing in English. Find the correct

sentence.
24.
A) It is far too hard an essay for me to attempt.

B) It is too far hard an essay to attempt for me.

C) Too far it is an essay hard for me to attempt.

D) It is too hard an essay for me to far attempt

1) A

2) B

3) C

4) D

Correct Option is: 4


Your Option is: 4
Result: Correct
Timetaken: 0.0 secs
Explanation: Nil

Read the passage and answer the questions that follow on the basis of the

information provided in the passage.

Work expands so as to fill the time available for its completion. The general

recognition of this fact is shown in the proverbial phrase,' It is the busiest man who

has time to spare'. Thus, an elderly lady at leisure can spend the entire day writing

a postcard to her niece. An hour will be spent in writing a postcard , another


25.
hunting for spectacles, half an hour to search for the address , an hour and a

quarter in composition and twenty minutes in deciding whether or not to take an

umbrella when going to the pillar box in the street. The total effort that could

occupy a busy man for three minutes, all told may in this fashion leave another

person completely exhausted after a day of doubt, anxiety and toil.

What happens when the time to be spent on some work increases?

1) A) the work is done smoothly.

2) B) the work is done leisurely.

3) C) work consumes all the time.

4) D) The work needs additional time

Correct Option is: 3


Your Option is: 1
Result: Wrong
Timetaken: 0.0 secs
Explanation: Nil

26.
Read the passage and answer the questions that follow on the basis of the

information provided in the passage.

According to Albert Einstein the non mathematician, is seized by a mysterious


shudderingwhen he hears of 'four-dimensional' things, he is seized by a feeling,

which is very similar to the thoughts awakened by the occult. And at the same time

the statement that the world in which we live is a four-dimensional space - time

continuum is quite a common place statement.

This might lead to an argument regarding the use of the term ''commonplace'' by

Einstein.Yet the difficulty lies more in the wording than the ideas. Einstein's concept

of the universe as a four-dimensional space-time continuum becomes plain and

clear, when what he means by ''continuum'' becomes clear. A continuum is

something that is continuous, A ruler, for example, is a one-dimensional space

continuum. Most rulers are divided into inches and frWASLions, scaled down to one-

sixteenth of an inch. Will it be possible to conceive a ruler, which is calibrated to a

millionth or billionth of an inch. In theory there is no reason why the steps from

point to point should not be even smaller. What distinguishes a continuum is the

fWASL that the space between any two points can be sub-divided into an infinite

number of smaller divisions.

A railroad track is a one-dimensional space continuum and on it the engineer of a

traincan describe his position at any time by citing a single co-ordinate point - i.e., a

stationor a milestone. A sea captain, however, has to worry about two dimensions.

The surface of the sea is a two-dimensional continuum and the co-ordinate points

by which sailor fixes his positions in his two dimensional continuum are latitude and

longitude. An airplane pilot guides his plane through a three - dimensional

continuum, hence he has to consider not only latitude and longitude, but also his

height above the ground. The continuum of an airplane pilot constitutes space as

we perceive it. In other words, the space of our world is a three-dimensional

continuum.

Just indicating its position in space is not enough while describing any physical

event, which involves motion. How position changes in time also needs to be

mentioned. Thus to give an accurate picture of the operation of a New York -


Chicago express, one must mention not only that it goes from New - York to Albany

to Syracuse to Cleveland to Toledo to Chicago, but also the times at which it

touches each of those points. This can be done either by means of a timetable or a

visual chart. If the miles between New York and Chicago are plotted horizontally on

a piece of ruled paper and the hours and minutes are plotted vertically, then a

diagonal line properly drawn across the page illustrates the progress of the train in

two-dimensional space - time continuum. This type of graphic representation is

familiar to most newspaper readers; a stock market chart,

For example, pictures financial events in a two - dimensional dollar - time

continuum. Similarly for the best picturisation of the flight of an airplane from New

York to Los Angeles a four - dimensional space - time continuum is essential. The

latitude, longitude and altitude will only make sense to the traffic manager of the

airline if the time co -ordinate is also mentioned. Therefore time is the fourth

dimension. If a flight has to be looked at, perceived as a whole, it wouldn't work if it

is broken down into a series of disconnected take - offs, climbs, glides, and landing,

it needs to be looked at and perceived as a continuous four - dimensional space -

time continuum curve.

The significant feature of a continuum, according to the passage, revolves around

1) The divisibility of the interval between any two points

2) An ordinary ruler's caliber for marking

3) Its is unending curve

4) Its lucid by providing comprehensibility to the non-scientists as well

5) Its variety of co-ordinates

Correct Option is: 1


Your Option is: 1
Result: Correct
Timetaken: 0.0 secs
Explanation: Nil

Directions for Questions: Read each sentence to find if there is any grammatical

error in it. If there is any error, it will be only one part of the sentence. The number

1. or alphabet of that part is your answer.(Disregard punctuation errors if any)

I never have / visited / or intend to visit / foreign countries.

ABCD

1) A

2) B

3) C

4) D

Correct Option is: 3


Your Option is: 3
Result: Correct
Timetaken: 0.0 secs
Explanation: 'Nor' in place of 'or.'nor' is used after 'neither'.

2.
Directions for Question:

Read the passage and answer the questions that follow on the basis of the

information provided in the passage. According to Albert Einstein the non

mathematician, is seized by a mysterious shuddering when he hears of 'four-

dimensional' things, he is seized by a feeling, which is very similar to the thoughts

awakened by the occult. And at the same time the statement that the world in

which we live is a four-dimensional space - time continuum is quite a common place

statement. This might lead to an argument regarding the use of the term
''commonplace'' by Einstein. Yet the difficulty lies more in the wording than the

ideas. Einstein's concept of the universe as a four-dimensional space-time

continuum becomes plain and clear, when what he means by ''continuum'' becomes

clear. A continuum is something that is continuous, A ruler, for example, is a one-

dimensional space continuum. Most rulers are divided into inches and frWASLions,

scaled down to one-sixteenth of an inch. Will it be possible to conceive a ruler,

which is calibrated to a millionth or billionth of an inch. In theory there is no reason

why the steps from point to point should not be even smaller. What distinguishes a

continuum is the fWASL that the space between any two points can be sub-divided

into an infinite number of smaller divisions. A railroad track is a one-dimensional

space continuum and on it the engineer of a train can describe his position at any

time by citing a single co-ordinate point - i.e., a station or a milestone. A sea

captain, however, has to worry about two dimensions. The surface of the sea is a

two-dimensional continuum and the co-ordinate points by which sailor fixes his

positions in his two dimensional continuum are latitude and longitude. An airplane

pilot guides his plane through a three - dimensional continuum, hence he has to

consider not only latitude and longitude, but also his height above the ground. The

continuum of an airplane pilot constitutes space as we perceive it. In other words,

the space of our world is a three-dimensional continuum. http://www.ChetanaS.org

Just indicating its position in space is not enough while describing any physical

event, which involves motion. How position changes in time also needs to be

mentioned. Thus to give an accurate picture of the operation of a New York -

Chicago express, one must mention not only that it goes from New - York to Albany

to Syracuse to Cleveland to Toledo to Chicago, but also the times at which it

touches each of those points. This can be done either by means of a timetable or a

visual chart. If the miles between New York and Chicago are plotted horizontally on

a piece of ruled paper and the hours and minutes are plotted vertically, then a

diagonal line properly drawn across the page illustrates the progress of the train in
two - dimensional space - time continuum. This type of graphic representation is

familiar to most newspaper readers; a stock market chart, for example, pictures

financial events in a two - dimensional dollar - time continuum. Similarly for the

best picturisation of the flight of an airplane from New York to Los Angeles a four -

dimensional space - time continuum is essential. The latitude, longitude and altitude

will only make sense to the traffic manager of the airline if the time co - ordinate is

also mentioned. Therefore time is the fourth dimension. If a flight has to be looked

at, perceived as a whole, it wouldn't work if it is broken down into a series of

disconnected take - offs, climbs, glides, and landing, it needs to be looked at and

perceived as a continuous four - dimensional space - time continuum curve.

Following are some sample questions on this passage:

The purpose of this passage is to highlight the point that

1) Plots and sea captains have something in common

2) Stock market charts may be helpful to physicists

3) The fourth dimension is time.

4) Non - mathematician's are often afraid of the commonplace

5) There is a marked quality to distance

Correct Option
3
is:
Your Option is: 2
Result: Wrong
Timetaken: 0.0 secs
According to the passage,the purpose of this passage is to
Explanation: highlight the point that the fourth dimension is time.So the
correct option is 3.

3.
S1: For some time in his youth, Abraham Lincoln was manager for a shop.

P : Then a chance Customer would come.


Q : Young Lincoln way of keeping shop was entirely unlike anyone else's.

R : Lincoln would jump up and attend to his needs and then revert to his reading.

S : He used to lie full length on the counter of the shop eagerly reading a book.

S6: Never before had Lincoln had so much time for reading as had then. The Proper

sequence should be:

1) SRQP

2) QSPR

3) SQRP

4) QPSR

Correct
2
Option is:
Your Option
3
is:
Result: Wrong
Timetaken: 0.0 secs
Sentence S1 and Q is a pair.And the information about him
Explanation: is continued in the sentence S.Therefore options,1, 2 and 4
are easily eliminated.Correct answer is 2.

4.
Directions for Questions: In each of the following questions, some sentence are

given which are on the same theme. decide which sentence is the most preferable

with respect to grammar; meaning and usage, suitable for formal writing in English.

Find the correct sentence.

A) They have placed order for books.

B) He has applied for lectureship.

C) The river has overflown its bank.


D) Give me rupees two and a half.

1) A

2) B

3) C

4) D

Correct Option
4
is:
Your Option
1
is:
Result: Wrong
Timetaken: 0.0 secs
In A,'an' should come infront of order. In B, 'the' should
Explanation: come infront of lectureship. In C, overflowed must be used
instead of overflown.

5.
Directions for Question: Read the passage and answer the questions that follow on

the basis of the information provided in the passage.

According to Albert Einstein the non mathematician, is seized by a mysterious

shuddering when he hears of 'four-dimensional' things, he is seized by a feeling,

which is very similar to the thoughts awakened by the occult. And at the same time

the statement that the world in which we live is a four-dimensional space - time

continuum is quite a common place statement. This might lead to an argument

regarding the use of the term ''commonplace'' by Einstein. Yet the difficulty lies

more in the wording than the ideas. Einstein's concept of the universe as a four-

dimensional space-time continuum becomes plain and clear, when what he means

by ''continuum'' becomes clear. A continuum is something that is continuous, A

ruler, for example, is a one-dimensional space continuum. Most rulers are divided
into inches and fractions, scaled down to one-sixteenth of an inch. Will it be

possible to conceive a ruler, which is calibrated to a millionth or billionth of an inch.

In theory there is no reason why the steps from point to point should not be even

smaller. What distinguishes a continuum is the fact that the space between any two

points can be sub-divided into an infinite number of smaller divisions. A railroad

track is a one-dimensional space continuum and on it the engineer of a train can

describe his position at any time by citing a single co-ordinate point - i.e., a station

or a milestone. A sea captain, however, has to worry about two dimensions. The

surface of the sea is a two-dimensional continuum and the co-ordinate points by

which sailor fixes his positions in his two dimensional continuum are latitude and

longitude. An airplane pilot guides his plane through a three - dimensional

continuum, hence he has to consider not only latitude and longitude, but also his

height above the ground. The continuum of an airplane pilot constitutes space as

we perceive it. In other words, the space of our world is a three-dimensional

continuum. Just indicating its position in space is not enough while describing any

physical event, which involves motion. How position changes in time also needs to

be mentioned. Thus to give an accurate picture of the operation of a New York -

Chicago express, one must mention not only that it goes from New - York to Albany

to Syracuse to Cleveland to Toledo to Chicago, but also the times at which it

touches each of those points. This can be done either by means of a timetable or a

visual chart. If the miles between New York and Chicago are plotted horizontally on

a piece of ruled paper and the hours and minutes are plotted vertically, then a

diagonal line properly drawn across the page illustrates the progress of the train in

two - dimensional space - time continuum. This type of graphic representation is

familiar to most newspaper readers; a stock market chart, for example, pictures

financial events in a two - dimensional dollar - time continuum. Similarly for the

best picturization of the flight of an airplane from New York to Los Angeles a four -

dimensional space - time continuum is essential. The latitude, longitude and altitude
will only make sense to the traffic manager of the airline if the time co - ordinate is

also mentioned. Therefore time is the fourth dimension. If a flight has to be looked

at, perceived as a whole, it wouldn't work if it is broken down into a series of

disconnected take - offs, climbs, glides, and landing, it needs to be looked at and

perceived as a continuous four - dimensional space - time continuum curve.

Following are some sample questions on this passage:

The underlying tone of this selection is

1) persuasive

2) deferential

3) candid

4) instructive

5) gently condescending

Correct Option
4
is:
Your Option is: 3
Result: Wrong
Timetaken: 0.0 secs
According to the passage,it is very clear that the underlying
Explanation: tone of the selection is instructive.So the correct option is
4.

6.
Directions for Question: Read the passage and answer the questions that follow on

the basis of the information provided in the passage. According to Albert Einstein

the non mathematician, is seized by a mysterious shuddering when he hears of

'four-dimensional' things, he is seized by a feeling, which is very similar to the

thoughts awakened by the occult. And at the same time the statement that the

world in which we live is a four-dimensional space - time continuum is quite a

common place statement. This might lead to an argument regarding the use of the
term ''commonplace'' by Einstein. Yet the difficulty lies more in the wording than

the ideas. Einstein's concept of the universe as a four-dimensional space-time

continuum becomes plain and clear, when what he means by ''continuum'' becomes

clear. A continuum is something that is continuous, A ruler, for example, is a one-

dimensional space continuum. Most rulers are divided into inches and fractions,

scaled down to one-sixteenth of an inch. Will it be possible to conceive a ruler,

which is calibrated to a millionth or billionth of an inch. In theory there is no reason

why the steps from point to point should not be even smaller. What distinguishes a

continuum is the fact that the space between any two points can be sub-divided

into an infinite number of smaller divisions. A railroad track is a one-dimensional

space continuum and on it the engineer of a train can describe his position at any

time by citing a single co-ordinate point - i.e., a station or a milestone. A sea

captain, however, has to worry about two dimensions. The surface of the sea is a

two-dimensional continuum and the co-ordinate points by which sailor fixes his

positions in his two dimensional continuum are latitude and longitude. An airplane

pilot guides his plane through a three - dimensional continuum, hence he has to

consider not only latitude and longitude, but also his height above the ground. The

continuum of an airplane pilot constitutes space as we perceive it. In other words,

the space of our world is a three-dimensional continuum.Just indicating its position

in space is not enough while describing any physical event, which involves motion.

How position changes in time also needs to be mentioned. Thus to give an accurate

picture of the operation of a New York - Chicago express, one must mention not

only that it goes from New - York to Albany to Syracuse to Cleveland to Toledo to

Chicago, but also the times at which it touches each of those points. This can be

done either by means of a timetable or a visual chart. If the miles between New

York and Chicago are plotted horizontally on a piece of ruled paper and the hours

and minutes are plotted vertically, then a diagonal line properly drawn across the

page illustrates the progress of the train in two - dimensional space - time
continuum. This type of graphic representation is familiar to most newspaper

readers; a stock market chart, for example, pictures financial events in a two -

dimensional dollar - time continuum. Similarly for the best picturization of the flight

of an airplane from New York to Los Angeles a four - dimensional space - time

continuum is essential. The latitude, longitude and altitude will only make sense to

the traffic manager of the airline if the time co - ordinate is also mentioned.

Therefore time is the fourth dimension. If a flight has to be looked at, perceived as

a whole, it wouldn't work if it is broken down into a series of disconnected take -

offs, climbs, glides, and landing, it needs to be looked at and perceived as a

continuous four - dimensional space - time continuum curve. Following are some

sample questions on this passage:

According to the passage, an airlines traffic manager depends upon all of the

following EXCEPT

1) latitude

2) altitude

3) the time co - ordinate

4) longitude

5) the continuous curve in time cotinuum

Correct Option
5
is:
Your Option is: 4
Result: Wrong
Timetaken: 0.0 secs
From the last passage,'The latitude .... also mentioned',It is
Explanation:
very clear that the correct answer is option 5.
S1: A force of exists between everybody in the universe.

P : Normally it is very small but when the one of the bodies is a planet, like earth,

the force is considerable.

Q: It has been investigated by many scientists including Galileo and Newton.


7.
R : Everything on or near the surface of the earth is attracted by the mass of earth.

S : This gravitational force depends on the mass of the bodies involved.

S6: The greater the mass, the greater is the earth's force of attraction on it. We can

call this force of attraction gravity. The Proper sequence should be:

1) PRQS

2) PRSQ

3) QSRP

4) QSPR

Correct
4
Option is:
Your Option
2
is:
Result: Wrong
Timetaken: 0.0 secs
S1 is about a force,so the continuation must be Q.And S and
Explanation: P is a pair of sentence.Therefore R follows the sentence P.So
the correct answer is option 4

8.
Read the passage and answer the questions that follow on the basis of the

information provided in the passage. Work expands so as to fill the time available

for its completion. The general recognition of this fact is shown in the proverbial

phrase,' It is the busiest man who has time to spare'. Thus, an elderly lady at

leisure can spend the entire day writing a postcard to her niece. An hour will be
spent in writing a postcard , another hunting for spectacles, half an hour to search

for the address, an hour and a quarter in composition and twenty minutes in

deciding whether or not to take an umbrella when going to the pillar box in the

street. The total effort that could occupy a busy man for three minutes, all told may

in this fashion leave another person completely exhausted after a day of doubt,

anxiety and toil. Explain the sentence : work expands so as to fill the time available

for its completion?.

1) The more work there is to be done, the more time needed.

2) whatever time is available for a given amount of work, all of it will be used.

3) If you have more time you can do some work.

4) If you have some important work to do , you should always have some additional
time.

Correct
2
Option is:
Your Option
1
is:
Result: Wrong
Timetaken: 0.0 secs
The answer is 2.This can be found through out simple
inference.A statement is made right in the beginning of the
Explanation: passage and the sort of the lady illustrates the fact that
whatever time is available for a work,people tend to use all
of it.

9.
Read the passage and answer the questions that follow on the basis of the

information provided in the passage. Work expands so as to fill the time available

for its completion. The general recognition of this fact is shown in the proverbial

phrase,' It is the busiest man who has time to spare'. Thus, an elderly lady at

leisure can spend the entire day writing a postcard to her niece. An hour will be
spent in writing a postcard , another hunting for spectacles, half an hour to search

for the address, an hour and a quarter in composition and twenty minutes in

deciding whether or not to take an umbrella when going to the pillar box in the

street. The total effort that could occupy a busy man for three minutes, all told may

in this fashion leave another person completely exhausted after a day of doubt,

anxiety and toil. What happens when the time to be spent on some work increases?

1) the work is done smoothly

2) the work is done leisurely

3) work consumes all the time

4) The work needs additional time

Correct
3
Option is:
Your Option
2
is:
Result: Wrong
Timetaken: 0.0 secs
Here the method of elimination applies and simple inference
confirms it.(1) and (2) are eliminated at the first reading.The
Explanation: description that the lady who has enough leisure time takes
the entire day in writting the post card gives us the clue that
the correct answer is 3.

10.
Read the passage and answer the questions that follow on the basis of the

information provided in the passage. Work expands so as to fill the time available

for its completion. The general recognition of this fact is shown in the proverbial

phrase,' It is the busiest man who has time to spare'. Thus, an elderly lady at

leisure can spend the entire day writing a postcard to her niece. An hour will be

spent in writing a postcard , another hunting for spectacles, half an hour to search
for the address, an hour and a quarter in composition and twenty minutes in

deciding whether or not to take an umbrella when goingto the pillar box in the

street. The total effort that could occupy a busy man for three minutes, all told may

in this fashion leave another person completely exhausted after a day of doubt,

anxiety and toil.

Who is the person likely to take more time to do work?

1) a busy man

2) a man of leisure

3) an exhausted person

4) an elderly person

Correct
2
Option is:
Your Option
3
is:
Result: Wrong
Timetaken: 0.0 secs
Here, the answer is 2.It requires inference.The answer is to
be inferred from the factsgiven in the passage that the more
Explanation:
time you have,the more you will need.Therefore,the answer
is arrived at through complex inference.

Directions for Questions: Read each sentence to find if there is any grammatical

error in it. If there is any error, it will be only one part of the sentence. The number
11.
or alphabet of that part is your answer.(Disregard punctuation errors if any)

The clothes / were neatly / hanged / on the cloth line. A B C D

1) A

2) B
3) C

4) D

Correct Option
3
is:
Your Option is: 2
Result: Wrong
Timetaken: 0.0 secs
'Hung' in place of 'Hanged'.'Hanged' is used for for living
Explanation:
beings as hung is used for other objects.

Arrange the sentences into proper order:

S1: In the middle of one side of the square sits the Chairman of the committee, the

most important person in the room.

P : For a committee is not just a mere collection of individuals.

Q: On him rests much of the responsibility for the success or failure of the

committee.
12.
R : While this is happening we have an opportunity to get the 'feel' of this

committe.

S : As the meeting opens, he runs briskly through a number of formalities.

S6: From the moment its members meet, it begins to have a sort nebulous life of

its own.

1) QSRP

2) SQPR

3) PQRS

4) RSQP
Correct
1
Option is:
Your Option
2
is:
Result: Wrong
Timetaken: 0.0 secs
Sentence S1 gives the information about a person,So the
continuation of that sentence will in the sentence Q.And the
Explanation:
sentence R follows S,therefore sentence Q will comes after
R.So the correct answer is option is 1

Directions for Questions: In each of the following questions, some sentence are

given which are on the same theme. decide which sentence is the most preferable

with respect to grammar; meaning and usage, suitable for formal writing in English.

Find the correct sentence.


13.
A) From which train did you come?

B) A series of incidents have taken place.

C) It is a five--men committee.

D) This pronunciation is peculiar to Bengalis.

1) A

2) B

3) C

4) D

Correct Option is: 4


Your Option is: 2
Result: Wrong
Timetaken: 0.0 secs
Correct formofthe sentences are,
A)In which train did u come?
Explanation:
B)A series of incidents has taken place.
C)It is a five-man committee.

S1: Calcutta unlike other cities kepts its trams.

P : As a result there horrendous congestion.

Q : It was going to be the first in South Asia.

14. R : They run down the centre of the road

S : To ease in the city decided to build an underground railway line.

S6: The foundation stone was laid in 1972. The Proper sequence should be:

1) PRSQ

2) PSQR

3) RPSQ

4) SQRP

Correct
3
Option is:
Your Option
2
is:
Result: Wrong
Timetaken: 0.0 secs
The sentence R is about the trams,so it follows S1.P and R is
a pair of sentences.Solution for the problem in the sentence
Explanation:
P is discussed in the sentences S and Q.So the Correct option
is 3.

15.
S1: While talking to a group, one should feel self-confident and courageous.

P: Nor is it a gift bestowed by Providence on only a few.

Q: One should also learn how to think calmly and clearly.


R: It is like the ability to play golf.

S: It is not as difficult as most men imagine.

S2: Any man can develop his capacity if he has the desire to do so.

The proper sequence should be :

1) SQPR

2) QSPR

3) QRSP

4) RSQP

Correct
2
Option is:
Your Option
1
is:
Result: Wrong
Timetaken: 0.0 secs
Sentences S1 and Q is a pair.And the solution for the
problem discussed in the sentence Q is discussed in the
Explanation:
sentence S.Therefore,options 1,3 and 4 is eliminated.So the
correct option is 2

Directions for Questions: one of the four sentences given in each question is

grammatically wrong. Find the incorrect sentence.

A) She had finished her work when I met her.


16.
B) Do you believe in God?

C) He cuts his hand with a knife.

D) He challenged me for a duel.

1) A
2) B

3) C

4) D

Correct Option is: 4


Your Option is: 1
Result: Wrong
Timetaken: 0.0 secs
Explanation: In the sentence D,instead of 'for', 'to' must be used.

17.
Directions for Question: Read the passage and answer the questions that follow on

the basis of the information provided in the passage.

According to Albert Einstein the non mathematician, is seized by a mysterious

shuddering when he hears of 'four-dimensional' things, he is seized by a feeling,

which is very similar to the thoughts awakened by the occult. And at the same time

the statement that the world in which we live is a four-dimensional space - time

continuum is quite a common place statement. This might lead to an argument

regarding the use of the term ''commonplace'' by Einstein. Yet the difficulty lies

more in the wording than the ideas. Einstein's concept of the universe as a four-

dimensional space-time continuum becomes plain and clear, when what he means

by ''continuum'' becomes clear. A continuum is something that is continuous, A

ruler, for example, is a one-dimensional space continuum. Most rulers are divided

into inches and frWASLions, scaled down to one-sixteenth of an inch. Will it be

possible to conceive a ruler, which is calibrated to a millionth or billionth of an inch.

In theory there is no reason why the steps from point to point should not be even

smaller. What distinguishes a continuum is the fWASL that the space between any

two points can be sub-divided into an infinite number of smaller divisions.

A railroad track is a one-dimensional space continuum and on it the engineer of a


train can describe his position at any time by citing a single co-ordinate point - i.e.,

a station or a milestone. A sea captain, however, has to worry about two

dimensions. The surface of the sea is a two-dimensional continuum and the co-

ordinate points by which sailor fixes his positions in his two dimensional continuum

are latitude and longitude. An airplane pilot guides his plane through a three -

dimensional continuum, hence he has to consider not only latitude and longitude,

but also his height above the ground. The continuum of an airplane pilot constitutes

space as we perceive it. In other words, the space of our world is a three-

dimensional continuum. Just indicating its position in space is not enough while

describing any physical event, which involves motion. How position changes in time

also needs to be mentioned. Thus to give an accurate picture of the operation of a

New York - Chicago express, one must mention not only that it goes from New -

York to Albany to Syracuse to Cleveland to Toledo to Chicago, but also the times at

which it touches each of those points. This can be done either by means of a

timetable or a visual chart. If the miles between New York and Chicago are plotted

horizontally on a piece of ruled paper and the hours and minutes are plotted

vertically, then a diagonal line properly drawn across the page illustrates the

progress of the train in two - dimensional space - time continuum. This type of

graphic representation is familiar to most newspaper readers; a stock market chart,

for example, pictures financial events in a two - dimensional dollar - time

continuum. Similarly for the best picturisation of the flight of an airplane from New

York to Los Angeles a four - dimensional space - time continuum is essential. The

latitude, longitude and altitude will only make sense to the traffic manager of the

airline if the time co - ordinate is also mentioned. Therefore time is the fourth

dimension. If a flight has to be looked at, perceived as a whole, it wouldn't work if it

is broken down into a series of disconnected take - offs, climbs, glides, and landing,

it needs to be looked at and perceived as a continuous four - dimensional space -

time continuum curve. Following are some sample questions on this passage:
According to the author if one wishes portray a physical event in which motion plays

a role

1) one has to Make use of a time-table

2) Indicate how position changes in time

3) Be conversant with the scientist's theories

4) Describe it graphically

5) Be aware of altitude, latitude and longitude

Correct Option
2
is:
Your Option is: 4
Result: Wrong
Timetaken: 0.0 secs
From the last paragraph,it is very clear tghat the correct
Explanation:
answer is option 2.

Directions for Questions:

one of the four sentences given in each question is grammatically wrong. Find the

incorrect sentence.

18. A) Sumit is my elder brother.

B) He is two years younger to me.

C) He is the eldest man of this village.

D) Ravi is five years older than me.

1) A

2) B

3) C

4) D
Correct Option
3
is:
Your Option is: 3
Result: Correct
Timetaken: 0.0 secs
In statement C,instead of 'of','in' must be used.So the
Explanation:
correct answer is option 3.

19.
Directions:Read the passage and answer the questions that follow on the basis of

the information provided in the passage.

According to Albert Einstein the non mathematician, is seized by a mysterious

shuddering when he hears of 'four-dimensional' things, he is seized by a feeling,

which is very similar to the thoughts awakened by the occult. And at the same time

the statement that the world in which we live is a four-dimensional space - time

continuum is quite a common place statement. This might lead to an argument

regarding the use of the term ''commonplace'' by Einstein. Yet the difficulty lies

more in the wording than the ideas. Einstein's concept of the universe as a four-

dimensional space-time continuum becomes plain and clear, when what he means

by ''continuum'' becomes clear. A continuum is something that is continuous, A

ruler, for example, is a one-dimensional space continuum. Most rulers are divided

into inches and frWASLions, scaled down to one-sixteenth of an inch. Will it be

possible to conceive a ruler, which is calibrated to a millionth or billionth of an inch.

In theory there is no reason why the steps from point to point should not be even

smaller. What distinguishes a continuum is the fWASL that the space between any

two points can be sub-divided into an infinite number of smaller divisions. A railroad

track is a one-dimensional space continuum and on it the engineer of a train can

describe his position at any time by citing a single co-ordinate point - i.e., a station
or a milestone. A sea captain, however, has to worry about two dimensions. The

surface of the sea is a two-dimensional continuum and the co-ordinate points by

which sailor fixes his positions in his two dimensional continuum are latitude and

longitude. An airplane pilot guides his plane through a three - dimensional

continuum, hence he has to consider not only latitude and longitude, but also his

height above the ground. The continuum of an airplane pilot constitutes space as

we perceive it. In other words, the space of our world is a three-dimensional

continuum. http://www.ChetanaS.org Just indicating its position in space is not

enough while describing any physical event, which involves motion. How position

changes in time also needs to be mentioned. Thus to give an accurate picture of the

operation of a New York - Chicago express, one must mention not only that it goes

from New - York to Albany to Syracuse to Cleveland to Toledo to Chicago, but also

the times at which it touches each of those points. This can be done either by

means of a timetable or a visual chart. If the miles between New York and Chicago

are plotted horizontally on a piece of ruled paper and the hours and minutes are

plotted vertically, then a diagonal line properly drawn across the page illustrates the

progress of the train in two - dimensional space - time continuum. This type of

graphic representation is familiar to most newspaper readers; a stock market chart,

for example, pictures financial events in a two - dimensional dollar - time

continuum. Similarly for the best picturisation of the flight of an airplane from New

York to Los Angeles a four - dimensional space - time continuum is essential. The

latitude, longitude and altitude will only make sense to the traffic manager of the

airline if the time co - ordinate is also mentioned. Therefore time is the fourth

dimension. If a flight has to be looked at, perceived as a whole, it wouldn't work if it

is broken down into a series of disconnected take - offs, climbs, glides, and landing,

it needs to be looked at and perceived as a continuous four - dimensional space -

time continuum curve. Following are some sample questions on this passage:
The significant feature of a continuum, according to the passage, revolves around

1) The divisibility of the interval between any two points.

2) An ordinary ruler's caliber for marking

3) Its unending curve

4) Its lucid from providing comprehensibility to the non - scientists as well

5) Its variety of co - ordinates

Correct Option
1
is:
Your Option is: 1
Result: Correct
Timetaken: 0.0 secs
From the third paragraph,it is very clear that the correct
Explanation:
answer is option 1.

Read the passage and answer the questions that follow on the basis of the

information provided in the passage. Work expands so as to fill the time available

for its completion. The general recognition of this fact is shown in the proverbial

phrase,' It is the busiest man who has time to spare'. Thus, an elderly lady at

leisure can spend the entire day writing a postcard to her niece. An hour will be

20. spent in writing a postcard , another hunting for spectacles, half an hour to search

for the address, an hour and a quarter in composition and twenty minutes in

deciding whether or not to take an umbrella when goingto the pillar box in the

street. The total effort that could occupy a busy man for three minutes, all told may

in this fashion leave another person completely exhausted after a day of doubt,

anxiety and toil. What does the expression 'pillar box' stand for?

1) a box attached to the pillar


2) a box in the pillar

3) box office

4) a pillar type postbox

Correct Option
4
is:
Your Option is: 4
Result: Correct
Timetaken: 0.0 secs
The answer is D.It can be derived through implied
Explanation: information.The lady has to go to the pillar box to drop her
letter.

Directions for Questions: In each of the following questions, some sentence are

given which are on the same theme. decide which sentence is the most preferable

with respect to grammar; meaning and usage, suitable for formal writing in English.

Find the correct sentence.

21. A) It is far too hard an essay for me to attempt.

B) It is too far hard an essay to attempt for me.

C) Too far it is an essay hard for me to attempt.

D) It is too hard an essay for me to far attempt.

1) A

2) B

3) C

4) D
Correct Option
4
is:
Your Option is: 4
Result: Correct
Timetaken: 0.0 secs
According to the superlative rule,Only option D is the
Explanation:
correct answer.

Directions for Question: one of the four sentences given in each question is

grammatically wrong. Find the incorrect sentence.

A) Our followers are but a handful.


22.
B) Neither he nor I was there.

C) Many a glorious deeds were done.

D) Everyone of the boys loves to ride.

1) A

2) B

3) C

4) D

Correct Option
3
is:
Your Option is: 3
Result: Correct
Timetaken: 0.0 secs
In sentence C, as the sentence is in plural form,'a' should
Explanation:
not be used with 'many'.So the answer option is C.

23.
Read the passage and answer the questions that follow on the basis of the

information provided in the passage. Work expands so as to fill the time available
for its completion. The general recognition of this fact is shown in the proverbial

phrase,' It is the busiest man who has time to spare'. Thus, an elderly lady at

leisure can spend the entire day writing a postcard to her niece. An hour will be

spent in writing a postcard , another hunting for spectacles, half an hour to search

for the address, an hour and a quarter in composition and twenty minutes in

deciding whether or not to take an umbrella when goingto the pillar box in the

street. The total effort that could occupy a busy man for three minutes, all told may

in this fashion leave another person completely exhausted after a day of doubt,

anxiety and toil.

What is the total time spent by the elderly lady in writing a postcard?

1) sixty minutes

2) four hours and five minutes

3) half day

4) the entire day

Correct Option
1
is:
Your Option
1
is:
Result: Correct
Timetaken: 0.0 secs
From the passage,it is very clear that the elder lady spent
Explanation: the entire day in writting a postcard.Therefore, the correct
answer is 4.

24.
Directions for Questions: In each of the following questions, some sentence are

given which are on the same theme. decide which sentence is the most preferable

with respect to grammar; meaning and usage, suitable for formal writing in English.
Find the correct sentence.

A) The proceeds of the charity show are for riot victims .

B) He asked Ajay and I to go.

C) The weather of this place does not suit me.

D) Either Rajesh or his friends has done it.

1) A

2) B

3) C

4) D

Correct Option is: 1


Your Option is: 3
Result: Wrong
Timetaken: 0.0 secs
In sentence B,'Me' must be used instead of 'I'.
Explanation: In C,instead of 'of','in' should be used.
In D, have should be used.

25.
Directions for Questions: In each of the following questions, some sentence are

given which are on the same theme. decide which sentence is the most preferable

with respect to grammar; meaning and usage, suitable for formal writing in English.

Find the correct sentence.

A) Hoping to be hearing from you, I remain yours sincerely.

B) Hoping to hear from you, I remain yours sincerely.

C) Hoping to have heard from you, I sincerely remain yours.

D) Sincerely I remain yours hoping to be hearing from you.


1) A

2) B

3) C

4) D

Correct
2
Option is:
Your Option
4
is:
Result: Wrong
Timetaken: 0.0 secs
"hearing' and 'have' cannot be used with 'hoping'.So first and
Explanation: third statement is eliminated.In the last sentence,words are
not arranged in the meaningful order

Which of phrases given below each sentence should replace the phrase printed in

bold type to make the grammatically correct? If the sentence is correct as it is,

mark 'E' as the answer.

He is too important for tolerating any delay.


1.
A.to tolerate

B.to tolerating

C.at tolerating

D.with tolerating

1) A

2) B

3) C
4) D

Correct Option
1
is:
Your Option is: 1
Result: Correct
Timetaken: 0.0 secs
As the complete sentence is in simple present form,the verb
Explanation: to be change must be also simple.So the answer option is
A.

Spot the error :

2. Many of the young people (1) / studying abroad agreed that (2) / returning home

was always (3) / an attractive option (4) / No error (5)

1) 1

2) 2

3) 3

4) 4

5) 5

Correct Option is: 5


Your Option is: 2
Result: Wrong
Timetaken: 0.0 secs
Explanation: this sentence will be corect

Spot the error :

3. My friend lived at the top (1)/ of an old house (2) / which attic had been (3)/

converted into a flat. (4)/ No error (5)


1) 1

2) 2

3) 3

4) 4

5) 5

Correct Option
3
is:
Your Option is: 4
Result: Wrong
Timetaken: 0.0 secs
Instead of 'which attic had been','whose attic had been'
Explanation:
should be used.

4.
Questions are based on the following passage

"I want to criticize the social system, and to show it at work, at its most intense."

Virginia Woolf's provocative statement about her intentions in writing Mrs. Dalloway

has regularly been ignored by the critics, since it highlights an aspect of her literary

interests very different from the traditional picture of the "poetic" novelist

concerned with examining states of reverie and vision and with following the

intricate pathways of individual consciousness. But Virginia Woolf was are ballistic

as well as a poetic novelist satirist and social critic as well as a visionary: literary

critics' cavalier dismissal of Woolf's social vision will not withstand scrutiny. In her

novels, Woolf is deeply engaged by the questions of how individuals are shaped (or

deformed) by their social Environments, how historical forces impinge on people's

lives, how class, wealth, and gender help to determine people's fates. Most of her

novels are rooted in a realistically rendered social setting and in a precise historical

time. Woolf's focus on society has not been generally recognized because of her

intense antipathy to propaganda in art. The pictures of reformers in her novels are
usually satiric or sharply critical. Even when Woolf is fundamentally sympathetic to

their causes, she portray people anxious to reform their society and possessed of a

message or program as arrogant or dishonest, unaware of how their political ideas

serve their own psychological needs own psychological needs. (Her Writer's Diary

notes: "the only honest people are the artists," whereas "these social reformers and

philanthropists...harbor...discreditable desires under the disguise of loving their

kind....") Woolf detested what she called "preaching" in fiction ,too, and criticized

novelist D.H.Lawrence (among others) for working by this method. Woolf's own

social criticism is expressed in the language of observation rather than in direct

commentary, since for her, fiction is a contemplative, not an active art. She

describes phenomena and provides materials for a judgment about society and

social issues; it is the reader's work to put the observation together and understand

the coherent point of view behind them. As a moralist, Woolf works by indirection,

subtly undermining officially accepted mores, mocking, suggesting, calling into

question, rather than asserting, advocating, bearing witness: hers is the satirist's

art .Woolf's literary models were acute social observers like Checkhov and Chaucer.

As she put it in The Common Reader. "It is safe to say that not a single law has

been framed or one stone set upon another because of anything Chaucer said or

wrote; and yet, as we read him, we are absorbing morality at every pore." Like

Chaucer, Woolf chose to understand as well as to judge, to know her society root

and branch-decision crucial in order to produce art rather than polemic.

In the first paragraph of the passage, the author's attitude toward the literary critics

mentioned can best be described as?

1) disparaging

2) ironic

3) factious

4) skeptical but resigned


Correct Option is: 1
Your Option is: 3
Result: Wrong
Timetaken: 0.0 secs
Explanation: according to the line 5 option 1 is the correct answer

In the following question choose the word which best expresses the meaning of the

5. given word.

PREAMBLE

1) rules

2) introduction

3) epilogue

4) instruction

Correct Option
2
is:
Your Option is: 1
Result: Wrong
Timetaken: 0.0 secs
preamble means introduction.So,the correct answer is
Explanation:
option B

6.
Questions are based on the following passage

It is a popular misconception that nuclear fusion power is free of radioactivity; in

fact, the deuterium-tritium reaction hat nuclear scientists are currently exploring

with such zeal produces both alpha particles and neutrons, (The neutrons are used
to produce tritium from a lithium blanket surrounding the reactor.) Another common

is conception is that nuclear fusion power is a virtually unlimited source of energy

because of the enormous quantity of deuterium in the sea. Actually, its limits are

set by the amount of available lithium, which is about as plentiful as uranium in the

Earth's crust. Research should certainly continue on controlled nuclear fusion ,but

no energy program should be premised on its existence until it has proven practical.

For the immediate future, we must continue to use hydroelectric power, nuclear

fission, and fossil fuels to meet our energy needs. The energy sources already in

major use are in major use for good reason.

Which of the following statements concerning nuclear scientists is most directly

suggested in the passage

1) Nuclear scientists exploring the decuterium-tritium reaction have overlooked key


facts in their eagerness to prove nuclear fusion practical

2) Nuclear scientists may have overestimated the amount of lithium actually


available in the Earth's crust.

3) Nuclear scientists have not been entirely dispassionate in their investigation of


the deuterium-tritium reaction.

4) Nuclear scientists have insufficiently investigated the lithium-to-tritium reaction


in nuclear fusion.

Correct Option is: 3


Your Option is: 4
Result: Wrong
Timetaken: 0.0 secs
Explanation: according to the line 2 option 3 is the correct answer

7.
Questions are based on the following passage

It is a popular misconception that nuclear fusion power is free of radioactivity; in

fact, the deuterium-tritium reaction hat nuclear scientists are currently exploring
with such zeal produces both alpha particles and neutrons, (The neutrons are used

to produce tritium from a lithium blanket surrounding the reactor.) Another common

is conception is that nuclear fusion power is a virtually unlimited source of energy

because of the enormous quantity of deuterium in the sea. Actually, its limits are

set by the amount of available lithium, which is about as plentiful as uranium in the

Earth's crust. Research should certainly continue on controlled nuclear fusion ,but

no energy program should be premised on its existence until it has proven practical.

For the immediate future, we must continue to use hydroelectric power, nuclear

fission, and fossil fuels to meet our energy needs. The energy sources already in

major use are in major use for good reason.

The primary purpose of the passage is to .

1) admonish scientists who have failed to correctly calculate the amount of lithium

2) defend the continued short-term use of fossil fuels as a major energy source

3) caution against uncritical embrace of nuclear fusion power as a major energy


source

4) correct the misconception that nuclear fusion power is entirely free of


radioactivity

Correct Option
3
is:
Your Option is: 1
Result: Wrong
Timetaken: 0.0 secs
primary purpose will be according to the passage option 3
Explanation:
is the correct answer

8.
Questions are based on the following passage

"I want to criticize the social system, and to show it at work, at its most intense."

Virginia Woolf's provocative statement about her intentions in writing Mrs. Dalloway
has regularly been ignored by the critics, since it highlights an aspect of her literary

interests very different from the traditional picture of the "poetic" novelist

concerned with examining states of reverie and vision and with following the

intricate pathways of individual consciousness. But Virginia Woolf was are ballistic

as well as a poetic novelist satirist and social critic as well as a visionary: literary

critics' cavalier dismissal of Woolf's social vision will not withstand scrutiny. In her

novels, Woolf is deeply engaged by the questions of how individuals are shaped (or

deformed) by their social Environments, how historical forces impinge on people's

lives, how class, wealth, and gender help to determine people's fates. Most of her

novels are rooted in a realistically rendered social setting and in a precise historical

time. Woolf's focus on society has not been generally recognized because of her

intense antipathy to propaganda in art. The pictures of reformers in her novels are

usually satiric or sharply critical. Even when Woolf is fundamentally sympathetic to

their causes, she portray people anxious to reform their society and possessed of a

message or program as arrogant or dishonest, unaware of how their political ideas

serve their own psychological needs own psychological needs. (Her Writer's Diary

notes: "the only honest people are the artists," whereas "these social reformers and

philanthropists...harbor...discreditable desires under the disguise of loving their

kind....") Woolf detested what she called "preaching" in fiction ,too, and criticized

novelist D.H.Lawrence (among others) for working by this method. Woolf's own

social criticism is expressed in the language of observation rather than in direct

commentary, since for her, fiction is a contemplative, not an active art. She

describes phenomena and provides materials for a judgment about society and

social issues; it is the reader's work to put the observation together and understand

the coherent point of view behind them. As a moralist, Woolf works by indirection,

subtly undermining officially accepted mores, mocking, suggesting, calling into

question, rather than asserting, advocating, bearing witness: hers is the satirist's

art .Woolf's literary models were acute social observers like Checkhov and Chaucer.
As she put it in The Common Reader. "It is safe to say that not a single law has

been framed or one stone set upon another because of anything Chaucer said or

wrote; and yet, as we read him, we are absorbing morality at every pore." Like

Chaucer, Woolf chose to understand as well as to judge, to know her society root

and branch-decision crucial in order to produce art rather than polemic.

It can be inferred from the passage that Woolf chose Chaucer as a literary model

because she believed that

1) Chaucer was an honest and forthright author, whereas novelists like D, H,


Lawrence did not sincerely wish to change society

2) Chaucer was more concerned with understanding his society than with calling its
accepted mores into question

3) Chaucer's writing was greatly, if subtly, effective in influencing the moral


attitudes of his readers

4) Her own novels would be more widely read if, like Chaucer, she did not overtly
and vehemently criticize contemporary society

Correct Option is: 3


Your Option is: 4
Result: Wrong
Timetaken: 0.0 secs
Explanation: according to the line 20 option 3 is the correct ansewer

In the following question choose the word which is the exact OPPOSITE of the given

9. word.

ENCHANT

1) gratify

2) annoy

3) voodoo
4) please

Correct Option
2
is:
Your Option is: 1
Result: Wrong
Timetaken: 0.0 secs
enchant means fill with great delight,charm gratify means
Explanation: give pleasure or satisfaction annoy means irritate voodoo
means

In the following question choose the word which is the exact OPPOSITE of the given

10. word.

set off

1) appease

2) offset

3) spoil

4) reconcile

Correct Option is: 3


Your Option is: 3
Result: Correct
Timetaken: 0.0 secs
Explanation: this word directy opposite to option 3

11.
Questions are based on the following passage

It is a popular misconception that nuclear fusion power is free of radioactivity; in


fact, the deuterium-tritium reaction hat nuclear scientists are currently exploring

with such zeal produces both alpha particles and neutrons, (The neutrons are used

to produce tritium from a lithium blanket surrounding the reactor.) Another common

is conception is that nuclear fusion power is a virtually unlimited source of energy

because of the enormous quantity of deuterium in the sea. Actually, its limits are

set by the amount of available lithium, which is about as plentiful as uranium in the

Earth's crust. Research should certainly continue on controlled nuclear fusion ,but

no energy program should be premised on its existence until it has proven practical.

For the immediate future, we must continue to use hydroelectric power, nuclear

fission, and fossil fuels to meet our energy needs. The energy sources already in

major use are in major use for good reason.

what is the limits are setted by reasearch?

1) lithium

2) fission

3) energy source

4) radioactivity

Correct Option is: 1


Your Option is: 3
Result: Wrong
Timetaken: 0.0 secs
Explanation: according to line 5 option 1 is the correct answer

12.
Pick the correct sentence

(A)Either Ramesh or I are to do this work.

(B)Either Ramesh or I am to do this work.

(C) Neither he nor his friends is reading.


(D)Neither he nor his friends are reading.

1) A and B

2) B and C

3) C and D

4) B and D

Correct Option
4
is:
Your Option is: 4
Result: Correct
Timetaken: 0.0 secs
If twosubjects are joined by 'either or', 'neither nor' the
Explanation:
verb agrees with the the subject that is near.

Which of phrases given below each sentence should replace the phrase printed in

bold type to make the grammatically correct? If the sentence is correct as it is,

mark 'E' as the answer.

We demonstrated to them how we were prepared the artistic patterns.


13.
A.are prepared

B.have prepared

C.are preparing

D.had prepared

1) A

2) B

3) C
4) D

Correct Option
4
is:
Your Option is: 2
Result: Wrong
Timetaken: 0.0 secs
Demonstrated implies pastperfect.So the answer is 'had
Explanation:
prepared'

In the following question choose the word which is the exact OPPOSITE of the given

14. word.

REMONSTRATE

1) criticise

2) demur

3) flatter

4) resist

Correct
3
Option is:
Your Option
4
is:
Result: Wrong
Timetaken: 0.0 secs
remonstrate means protest criticise means indicate the faults
of someone in a disapproving way demur means raise
Explanation:
objections flatter means praise resist means withstand So,
the correct answer is option C

15.
In the following question choose the word which is the exact OPPOSITE of the given
word.

VETERAN

1) wise

2) disciplined

3) exercised

4) amateur

Correct Option
4
is:
Your Option is: 1
Result: Wrong
Timetaken: 0.0 secs
Veteran means expert amateur means non-specialist So,the
Explanation:
correct answer is Option D

Which of phrases given below each sentence should replace the phrase printed in

bold type to make the grammatically correct? If the sentence is correct as it is,

mark 'E' as the answer.

The man who has committed such a serious crime must get the mostly

16. severe punishment.

A.be getting the mostly severely

B.get the most severe

C.have got the most severely

D.have been getting the severe most

1) A

2) B
3) C

4) D

Correct
2
Option is:
Your Option
2
is:
Result: Correct
Timetaken: 0.0 secs
In the options, 'severely' and 'mostly' cannot be used
Explanation: together.First part of the sentence is in past tense, so option
D is ruled out. Therefore, correct option is C.

Which of phrases given below each sentence should replace the phrase printed in

bold type to make the grammatically correct? If the sentence is correct as it is,

mark 'E' as the answer.

We don't know how did the thief made an escape.


17.
A.how the thief did make

B.how the thief does make

C.how the thief made

D.how was the thief made

1) A

2) B

3) C

4) D

Correct Option
3
is:
Your Option is: 3
Result: Correct
Timetaken: 0.0 secs
As the make is verb,there is no need of did or does or
Explanation:
was.So the correct option is 3.

Questions are based on the following passage

It is a popular misconception that nuclear fusion power is free of radioactivity; in

fact, the deuterium-tritium reaction hat nuclear scientists are currently exploring

with such zeal produces both alpha particles and neutrons, (The neutrons are used

to produce tritium from a lithium blanket surrounding the reactor.) Another common

is conception is that nuclear fusion power is a virtually unlimited source of energy

because of the enormous quantity of deuterium in the sea. Actually, its limits are

18. set by the amount of available lithium, which is about as plentiful as uranium in the

Earth's crust. Research should certainly continue on controlled nuclear fusion ,but

no energy program should be premised on its existence until it has proven practical.

For the immediate future, we must continue to use hydroelectric power, nuclear

fission, and fossil fuels to meet our energy needs. The energy sources already in

major use are in major use for good reason.

The passage provides information that would answer which of the following

questions?

1) What is likely to be the principal source of deuterium for nuclear fusion power?

2) How much incidental radiation is produced in the deuterium-tritium fusion


reaction?

3) Why are scientists exploring the deuterium-tritium fusion reaction with such
zeal?

4) Why must the tritium for nuclear fusion be synthesized from lithium?
Correct Option
1
is:
Your Option is: 2
Result: Wrong
Timetaken: 0.0 secs
provide the passage will be according to the passage
Explanation:
option 1 is the correct answer

19.
Questions are based on the following passage

"I want to criticize the social system, and to show it at work, at its most intense."

Virginia Woolf's provocative statement about her intentions in writing Mrs. Dalloway

has regularly been ignored by the critics, since it highlights an aspect of her literary

interests very different from the traditional picture of the "poetic" novelist

concerned with examining states of reverie and vision and with following the

intricate pathways of individual consciousness. But Virginia Woolf was are ballistic

as well as a poetic novelist satirist and social critic as well as a visionary: literary

critics' cavalier dismissal of Woolf's social vision will not withstand scrutiny. In her

novels, Woolf is deeply engaged by the questions of how individuals are shaped (or

deformed) by their social Environments, how historical forces impinge on people's

lives, how class, wealth, and gender help to determine people's fates. Most of her

novels are rooted in a realistically rendered social setting and in a precise historical

time. Woolf's focus on society has not been generally recognized because of her

intense antipathy to propaganda in art. The pictures of reformers in her novels are

usually satiric or sharply critical. Even when Woolf is fundamentally sympathetic to

their causes, she portray people anxious to reform their society and possessed of a

message or program as arrogant or dishonest, unaware of how their political ideas

serve their own psychological needs own psychological needs. (Her Writer's Diary

notes: "the only honest people are the artists," whereas "these social reformers and

philanthropists...harbor...discreditable desires under the disguise of loving their

kind....") Woolf detested what she called "preaching" in fiction ,too, and criticized
novelist D.H.Lawrence (among others) for working by this method. Woolf's own

social criticism is expressed in the language of observation rather than in direct

commentary, since for her, fiction is a contemplative, not an active art. She

describes phenomena and provides materials for a judgment about society and

social issues; it is the reader's work to put the observation together and understand

the coherent point of view behind them. As a moralist, Woolf works by indirection,

subtly undermining officially accepted mores, mocking, suggesting, calling into

question, rather than asserting, advocating, bearing witness: hers is the satirist's

art .Woolf's literary models were acute social observers like Checkhov and Chaucer.

As she put it in The Common Reader. "It is safe to say that not a single law has

been framed or one stone set upon another because of anything Chaucer said or

wrote; and yet, as we read him, we are absorbing morality at every pore." Like

Chaucer, Woolf chose to understand as well as to judge, to know her society root

and branch-decision crucial in order to produce art rather than polemic.

It can be inferred from the passage that the most probable reason Woolf

realistically described the social setting in the majority of her novels was that she?

1) was interested in the effect of a person's social milieu on his or her character and
actions

2) needed to be as attentive to detail as possible in her novels in order to support


the arguments she advanced in them

3) wanted to show that a painstaking fidelity in the representation of reality did not
in any way hamper the artist

4) wished to prevent critics from charging that her novels were written in an
ambiguous and inexact style

Correct Option is: 1


Your Option is: 3
Result: Wrong
Timetaken: 0.0 secs
Explanation: according to the line 9 option 1 is the correct answer

In the following question choose the word which best expresses the meaning of the

20. given word.

TREACHEROUS

1) loyal

2) deceptive

3) reliable

4) forthright

Correct
2
Option is:
Your Option
3
is:
Result: Wrong
Timetaken: 0.0 secs
treacherous means deception i.e., disloyal loyal means
true,faithful deceptive means misleading reliable means
Explanation:
genuine forthright means proceeding directly forwards
So,option B is the correct answer.

21.
Questions are based on the following passage

"I want to criticize the social system, and to show it at work, at its most intense."

Virginia Woolf's provocative statement about her intentions in writing Mrs. Dalloway

has regularly been ignored by the critics, since it highlights an aspect of her literary

interests very different from the traditional picture of the "poetic" novelist

concerned with examining states of reverie and vision and with following the

intricate pathways of individual consciousness. But Virginia Woolf was are ballistic

as well as a poetic novelist satirist and social critic as well as a visionary: literary
critics' cavalier dismissal of Woolf's social vision will not withstand scrutiny. In her

novels, Woolf is deeply engaged by the questions of how individuals are shaped (or

deformed) by their social Environments, how historical forces impinge on people's

lives, how class, wealth, and gender help to determine people's fates. Most of her

novels are rooted in a realistically rendered social setting and in a precise historical

time. Woolf's focus on society has not been generally recognized because of her

intense antipathy to propaganda in art. The pictures of reformers in her novels are

usually satiric or sharply critical. Even when Woolf is fundamentally sympathetic to

their causes, she portray people anxious to reform their society and possessed of a

message or program as arrogant or dishonest, unaware of how their political ideas

serve their own psychological needs own psychological needs. (Her Writer's Diary

notes: "the only honest people are the artists," whereas "these social reformers and

philanthropists...harbor...discreditable desires under the disguise of loving their

kind....") Woolf detested what she called "preaching" in fiction ,too, and criticized

novelist D.H.Lawrence (among others) for working by this method. Woolf's own

social criticism is expressed in the language of observation rather than in direct

commentary, since for her, fiction is a contemplative, not an active art. She

describes phenomena and provides materials for a judgment about society and

social issues; it is the reader's work to put the observation together and understand

the coherent point of view behind them. As a moralist, Woolf works by indirection,

subtly undermining officially accepted mores, mocking, suggesting, calling into

question, rather than asserting, advocating, bearing witness: hers is the satirist's

art .Woolf's literary models were acute social observers like Checkhov and Chaucer.

As she put it in The Common Reader. "It is safe to say that not a single law has

been framed or one stone set upon another because of anything Chaucer said or

wrote; and yet, as we read him, we are absorbing morality at every pore." Like

Chaucer, Woolf chose to understand as well as to judge, to know her society root

and branch-decision crucial in order to produce art rather than polemic.


The author implies that a major element of the satirist's art is the satirist's

1) consistent adherence to a position of loft(B) insistence on the helplessness of


individuals against the social forces that seek to determine an individual's fate

2) cynical disbelief that visionaries can either enlighten or improve their societies

3) fundamental assumption that some ambiguity must remain in a work of art in


order for it to reflect society and social mores accurately

4) refusal to indulge in polemic when presenting social mores to readers for their
scrutiny

Correct Option is: 4


Your Option is: 1
Result: Wrong
Timetaken: 0.0 secs
Explanation: according to the line 20 option 4 is the correct answer

22.
Questions are based on the following passage

"I want to criticize the social system, and to show it at work, at its most intense."

Virginia Woolf's provocative statement about her intentions in writing Mrs. Dalloway

has regularly been ignored by the critics, since it highlights an aspect of her literary

interests very different from the traditional picture of the "poetic" novelist

concerned with examining states of reverie and vision and with following the

intricate pathways of individual consciousness. But Virginia Woolf was are ballistic

as well as a poetic novelist satirist and social critic as well as a visionary: literary

critics' cavalier dismissal of Woolf's social vision will not withstand scrutiny. In her

novels, Woolf is deeply engaged by the questions of how individuals are shaped (or

deformed) by their social Environments, how historical forces impinge on people's

lives, how class, wealth, and gender help to determine people's fates. Most of her

novels are rooted in a realistically rendered social setting and in a precise historical
time. Woolf's focus on society has not been generally recognized because of her

intense antipathy to propaganda in art. The pictures of reformers in her novels are

usually satiric or sharply critical. Even when Woolf is fundamentally sympathetic to

their causes, she portray people anxious to reform their society and possessed of a

message or program as arrogant or dishonest, unaware of how their political ideas

serve their own psychological needs own psychological needs. (Her Writer's Diary

notes: "the only honest people are the artists," whereas "these social reformers and

philanthropists...harbor...discreditable desires under the disguise of loving their

kind....") Woolf detested what she called "preaching" in fiction ,too, and criticized

novelist D.H.Lawrence (among others) for working by this method. Woolf's own

social criticism is expressed in the language of observation rather than in direct

commentary, since for her, fiction is a contemplative, not an active art. She

describes phenomena and provides materials for a judgment about society and

social issues; it is the reader's work to put the observation together and understand

the coherent point of view behind them. As a moralist, Woolf works by indirection,

subtly undermining officially accepted mores, mocking, suggesting, calling into

question, rather than asserting, advocating, bearing witness: hers is the satirist's

art .Woolf's literary models were acute social observers like Checkhov and Chaucer.

As she put it in The Common Reader. "It is safe to say that not a single law has

been framed or one stone set upon another because of anything Chaucer said or

wrote; and yet, as we read him, we are absorbing morality at every pore." Like

Chaucer, Woolf chose to understand as well as to judge, to know her society root

and branch-decision crucial in order to produce art rather than polemic.

Which of the following would be the most appropriate title for the passage?

1) Virginia Woolf: Critic and Commentator on the Twentieth-Century Novel

2) Trends in Contemporary Reform Movements as a Key to Understanding Virginia


Woolf's Novels
3) Society as Allegory for the Individual in the Novels of Virginia Woolf

4) Virginia Woolf's Novels: Critical Reflections on the Individual and on Society

Correct Option
4
is:
Your Option is: 2
Result: Wrong
Timetaken: 0.0 secs
according to the passage and according to the line 1 option
Explanation:
4 is the appropriate title for the passage

In the following question choose the word which best expresses the meaning of the

23. given word.

AVERT

1) largesse

2) assistance

3) abet

4) impede

Correct
4
Option is:
Your Option
4
is:
Result: Correct
Timetaken: 0.0 secs
avert means turn away or prevent largesse means money or
gifts given generously assistance means help or aid abet
Explanation: means assisting wrongly impede means hinder i.e.,
preventing something by obstructing them So,option D is the
correct answer.
In the following question choose the word which best expresses the meaning of the

24. given word.

CONCEITED

1) arrogant

2) humble

3) meek

4) timid

5) modest

Correct Option
1
is:
Your Option is: 4
Result: Wrong
Timetaken: 0.0 secs
Conceited means excessively proud of oneself i.e.,vain or
Explanation:
arrogant.So,Option A is the correct answer

In the following question choose the word which best expresses the meaning of the

25. given word

BIFURCATE

1) abridge

2) gather

3) ramify

4) shrink

Correct
3
Option is:
Your Option
3
is:
Result: Correct
Timetaken: 0.0 secs
bifurcate means dividing into branches. abridge means
shorten. gather means assemble. ramify means forming
Explanation:
branches branches or offshoots. shrink means becoming
smaller in size. So,the correct answer is option C

You might also like